Location via proxy:   [ UP ]  
[Report a bug]   [Manage cookies]                

Business Mathematics and Statistics: Fundamentals of

Download as pdf or txt
Download as pdf or txt
You are on page 1of 468

Foundation

Fundamentals of
Business Mathematics and
Statistics
Paper

3
The Institute of Cost Accountants of India
Statutory Body under an Act of Parliament
www.icmai.in
About the Institute

T
he Institute of Cost Accountants of India is a Statutory Body set up under an Act of Parliament in the
year 1959. The Institute as a part of its obligation, regulates the profession of Cost and Management
Accountancy, enrols students for its courses, provides coaching facilities to the students, organizes
professional development programmes for the members and undertakes research programmes in
the ield of Cost and Management Accountancy. The Institute pursues the vision of cost competitiveness, cost
management, ef icient use of resources and structured approach to cost accounting as the key drivers of the
profession.

With the current emphasis on management of resources, the specialized knowledge of evaluating operating
ef iciency and strategic management the professionals are known as ''Cost and Management Accountants
(CMAs)''. The Institute is the 2ⁿ largest Cost & Management Accounting body in the world and the largest in
Asia, having more than 5,00,000 students and 90,000 members all over the globe. The Institute operates
through four regional councils at Kolkata, Delhi, Mumbai and Chennai and 113 Chapters situated at important
cities in the country as well as 11 Overseas Centres, headquartered at Kolkata. It is under the administrative
control of the Ministry of Corporate Affairs, Government of India.

Vision Statement

T he Institute of Cost Accountants of India would be the preferred source of resources and
professionals for the inancial leadership of enterprises globally.”

Mission Statement

T he Cost and Management Accountant professionals would ethically drive enterprises globally by
creating value to stakeholders in the socio-economic context through competencies drawn from
the integration of strategy, management and accounting.”

Motto

From ignorance, lead me to truth


From darkness, lead me to light
From death, lead me to immortality
Peace, Peace, Peace

Cover Image Source: https://online.york.ac.uk/everything-you-need-to-know-about-data-analytics/

Behind Every Successful Business Decision, there is always a CMA


FOUNDATION
Paper 3

Fundamentals of Business Mathematics


and Statistics
Study Notes
SYLLABUS 2022

The Institute of Cost Accountants of India


CMA Bhawan, 12, Sudder Street, Kolkata - 700 016
www.icmai.in
First Edition : August 2022

Price: ` 400.00

Published by :
Directorate of Studies
The Institute of Cost Accountants of India
CMA Bhawan, 12, Sudder Street, Kolkata - 700 016
studies@icmai.in

Printed at :
M/s. Print Plus Pvt. Ltd.
212, Swastik Chambers
S. T. Road, Chembur
Mumbai - 400 071

Copyright of these Study Notes is reserved by the Institute of Cost Accountants of India and prior permission
from the Institute is necessary for reproduction of the whole or any part thereof.
Copyright © 2022 by The Institute of Cost Accountants of India
PAPER 3: FUNDAMENTALS OF BUSINESS MATHEMATICS
AND STATISTICS
Syllabus Structure:
The syllabus in this paper comprises the following topics and study weightage:

Module No. Module Description Weight


Section A: Fundamentals of Business Mathematics 40%
1 Arithmetic 15%
2 Algebra 20%
3 Calculus - Application in Business 5%
Section B: Fundamentals of Business Statistics 60%
4 Statistical Representation of Data 5%
5 Measures of Central Tendency and Dispersion 15%
6 Correlation and Regression 15%
7 Probability 15%
8 Index Numbers and Time Series 10%

A
40%
B
60%
Contents as per Syllabus
SECTION A: FUNDAMENTALS OF BUSINESS MATHEMATICS 01 - 234

Module 1 : Arithmetic 01 - 82
1.1 Ratios, Variations and Proportions
1.2 Time Value of Money and Annuity – Simple and Compound Interest
1.3 Arithmetic Progression and Geometric Progression
1.4 Time and Distance

Module 2 : Algebra 83 - 176


2.1 Set Theory, Venn Diagram
2.2 Indices and Logarithms (Basic Concepts)
2.3 Permutation and Combinations (Basic Concepts)
2.4 Quadratic Equation (Basic Concepts)

Module 3 : Calculus – Application in Business 177 - 234


3.1 Concept of Calculus and its Application in Business
3.2 Revenue and Cost Function
3.3 Optimization Techniques (Basic Concepts)

SECTION B: FUNDAMENTALS OF BUSINESS STATISTICS 235 - 454

Module 4 : Statistical Representation of Data 235 - 268


4.1 Diagrammatic Representation of Data
4.2 Frequency Distribution
4.3 Graphical representation of Frequency Distribution – Histogram, Frequency
Polygon Curve, Ogive, Pie-chart
Contents as per Syllabus
Module 5 : Measures of Central Tendency and Dispersion 269 - 344
5.1 Mean, Median, Mode, Mean Deviation
5.2 Range, Quartiles and Quartile Deviation
5.3 Standard Deviation
5.4 Co-efficient of Variation
5.5 Karl Pearson and Bowley’s Co-efficient of Skewness

Module 6 : Correlation and Regression 345 - 388


6.1 Scatter Diagram
6.2 Karl Pearson’s Co-efficient of Correlation
6.3 Regression Analysis

Module 7 : Probability 389 - 418


7.1 Concepts and Terminologies
7.2 Primary applications of Probability Theorems

Module 8 : Index Numbers and Time Series 419 - 454


8.1 Uses of Index Numbers, Methods of Construction of Index Number
8.2 Components of Time Series and Calculation of Trend by Moving
Average Method
SECTION - A
FUNDAMENTALS OF BUSINESS MATHEMATICS
ARITHMETIC 1
This Module includes:

1.1 Ratios, Variations and Proportions


1.2 Time Value of Money and Annuity – Simple and Compound Interest
1.3 Arithmetic Progression and Geometric Progression
1.4 Time and Distance

The Institute of Cost Accountants of India 1


ARITHMETIC

Module Learning Objectives:


Upon completion of study this module, students will be able to understand –
 The concepts of Ratios and Proportions as well as their different compositions
 The different pattern of Variations among variables
 The application areas of Ratios, Proportions and Variations in real life situations
 What is actually meant by Time value of Money
 The concept of Simple and Compound interest
 What is meant by Annuity
 How to calculate Simple and Compound interests and Annuities under different time scales
 How to apply time value of money practical situations
 What is actually meant by Arithmetic & Geometric progression
 The properties of AP and GP
 The relation between time and distance and application of the same

2 The Institute of Cost Accountants of India


Ratios, Variations and
Proportions 1.1
1.1.1 Ratios:
Ratio is the comparative relation between two quantities of same kind expressed in the same units.
Example: In a class test A secured 80 marks and B secured 40 marks out of 100 then we can compare that A secures
double that of B.
i.e. Ratio is 80/40 = 2, is a pure natural number, and no unit is associated with it.

Note:
1. In the ratio a:b, a & b are called the terms of the ratio. Here ‘a’ is called Antecedent and ‘b’ is called Consequent.

1.1.1.1 Properties of Ratio:


1. The value of ratio remains unchanged when the terms of the ratio are multiplied or divided by the same
number.
Ex: 2:4, Multiplied by 2, 4:8; 9:27, divided by 9, 1:3

2. Two or more ratios can be compared by reducing them to the same denominator.
Ex: In the ratios 3:4 and 8:12 which is greater?
multiplying the terms of 3:4 by 3, we get 9:12
multiplying the terms of 8:12 by 1, we get 8:12
As 9:12 > 8:12, we conclude 3:4 > 8:12

1.1.1.2 Ratio of equality and inequality:


1. If a = b then the ratio a:b is called equal ratio.
Ex: 3:3, 4:4 etc. are equal ratios.

2. If a > b then the ratio a:b is called greater inequality.


Ex: 4:3, 9:7 etc. are the ratios of greater inequalities.

3. If a < b then the ratio a:b is called lesser inequality.


Ex: 3:4, 7:9 etc. are the ratios of lesser inequalities.

The Institute of Cost Accountants of India 3


Fundamentals of Business Mathematics and Statistics

Inverse ratio or reciprocal ratio:


The inverse ratio of a:b is b:a

1.1.1.3 Different kinds of ratios:


1. Compound ratio:
If two or more ratios are multiplied together then the resultant ratio is called Compounded ratio.

Ex: For the ratios a : b, c : d , e : f the compounded ratio is ace : dbf


Note:

The compounded ratio of two reciprocal ratios is unity. i.e. a : b is the reciprocal ratio of b : a

Then the compounded ratio is ab : ba = 1

2. Duplicate ratio:
If two equal ratios are compounded together then the resulting ratio is called Duplicate ratio of the given ratio i.e.
the duplicate ratio of a:b is a:b × a:b = a 2 : b 2 .

3. Triplicate ratio:
If three equal ratios are compounded together then the resulting ratio is called Triplicate ratio of the given ratio i.e.
the triplicate ratio of a:b is a:b × a:b × a:b = a3 : b3.

4. Sub duplicate ratio and Sub triplicate ratios:

a : b is the Sub Duplicate ratio of a : b and 3


a : 3 b is the Sub Triplicate ratio of a : b

5. Continued ratio:
The continued ratio is the relation between the magnitudes of two or more ratios and is denoted by a : b : c
Ex: The continued ratio of 2:3 and 4:10 is 8:12:30 or, 4:6:15

Points to remember:

1. Bring the quantities to the same units while finding a Ratio.

Ex: if A = `2 and B = 50 p

Then A : B = 200 : 50 = 4 :1

2. When the quantity is increased by given ratio multiply the quantity by greater ratio.

3. When the quantity is decreased by given ratio multiply the quantity by lesser ratio.

4. When both increasing and decreasing of quantities are present in a problem multiply the quantity by greater
ratio to increase and multiply the result by lesser ratio to obtain the final result.

4 The Institute of Cost Accountants of India


Arithmetic

1.1.2 Proportions (OR) proportional:


If two ratios are equal then we say that the two ratios are in proportion. In other words the four quantities a, b, c
and d are said to be in proportion if a : b = c : d Or, a = c
b d
Here the first and last quantities i.e. a & d are extremes and the two middle terms b & c are called means.

Property:

The four quantities a, b, c, d are in proportion if a = c Or, ad = bc. Converse is also true.
b d
1.1.2.1 Continued proportion:
If 3 quantities a, b, and c are such that a : b = b : c then we say that these 3 quantities are in continued proportion.
If 3 quantities are in continued proportion then we get b2 = ac.
Ex: 3, 6, 12 are in continued proportion.

So 62 = 3 × 12 = 36
Ex: 2, –4, 8 are in continued proportion
So (−4) 2 = 2 × 8 = 16

1.1.2.1 Mean Proportional, Third Proportional and Fourth Proportional:


If x, y, z are in continued proportion then y is called the Mean Proportional (or Geometric Mean) of x & z. Thus
x y or, y2 = xz or y =
= xz
y z
This is the formula to obtain Mean Proportional (y) of the numbers x & z

Here z is called the Third Proportional of x and y. Thus the same formula as above can be used to find at 3rd
proportinal of x & y.

Similarly if a, b, c and d are in proportion i.e. a:b = c:d then d is called the Fourth Proportional of a, b and c.
Example:
(i) Mean proportional between 0.08 and 0.18 is 0.08 × 0.18 = .0144 = 0.12
16 36
(ii) Third Proportional to 16 & 36 is obtained as 16:36: :36: x, where x is the 3rd Proportional. So =
36 x
36 × 36
or 16x = 36 × 36 or, x = = 81 . Hence 81 is the 3rd Proportional to 16 & 36
16
(iii) Fourth Proportional to 4, 9 and 12 is calculated as 4:9: : 12: X, where X is the 4th Proportional.
So 4x = 9 × 12 or, X = 27. Hence 27 is the 4th proportional to 4, 9 & 12.

1.1.2.3 Basic rules of proportions:


1. Invertendo:
If a : b = c : d which implies b : a = d : c then we say that the operation of Invertendo is applied on the given
proportion.

The Institute of Cost Accountants of India 5


Fundamentals of Business Mathematics and Statistics

2. Alternendo:
If a : b = c : d which implies a : c = b : d then we say that the operation of Alternendo is performed on the given
proportion.

3. Componendo:
If a : b = c : d which implies a + b : b = c + d : d then we say that the operation of Componendo is performed on
the given proportion.

4. Dividendo:
If a : b = c : d which implies a − b : b = c − d : d then we say that the operation of Dividendo is performed on
the give proportion.

5. Componendo and dividendo:


If a : b = c : d which implies a + b : a − b = c + d : c − d then we say that the operation of Componendo and
Dividendo has been performed on the given proportion.

6. Important theorem:
1
a c e  pa n + qc n + re n   n
If b = d = f  then each ratio =  n n n 
 pb + qd + rf  

Where p, q, r…… are quantities

Proof:
a c e
Let = = ...... = k (Say)
b d f

a
So = k , Or a = bk
b

c e
Similarly = k , Or, c = dk & = k , Or e = fk ....... So on
d f
RHS
1 1
 pa n + qc n + re n   n  p(bk ) n + q(dk ) n + r ( fk ) n   n
= n n n  = 
 pb + qd + rf    pb n + qd n + rf n  

1
 pb nk n + qd nk n + rf nk n   n
= 
 pb n + qd n + rf n  
1
  pb n + qd n + rf n   n
= k n  n 
  pb + qd + rf  
n n

= {k n } = k
1/n

6 The Institute of Cost Accountants of India


Arithmetic

Note:
1. Put n = 1

pa + qc + re + 
So = each ratio = k
pb + qd + rf + 

2. Put p = q = r =  = 1
1/n
 a n + c n + e n + 
So  n n n  = each ratio
 b + d + f +  

3. The continued ratio

x: y : z = a :b:c
y
Can be written as x = = z
a b c
4. If x : y = a : b it does not mean that x = a & y = b

But x = ka, y = bk , where k is a constant

Illustration 1.
4 x − 3z 4 z − 3 y 4 y − 3x x+ y+ z
If = = , show that each ratio =
4c 3b 2a 2a + 3b + 4c

Answer:
4 x − 3z 4 z − 3 y 4 y − 3x 4 x − 3 y + 4 z − 3 y + 4 y − 3x x + y + z (Proved)
= = = =
4c 3b 2a 4c + 3b + 2a 2a + 3b + 4c

Illustration 2.
a c e g aceg a 4 + c 4 + e f + g 4
If = = = Show =
b d f h bdfh b 4 + d 4 + f 4 + h 4

Answer:
a c e g
= = = = k (say)
b d f h

So a = bk , c = dk , e = fk , g = hk
aceg bk × dk × fk × hk
LHS = bdfh = = k4
bdfh

RHS
a 4 + c 4 + e 4 + g 4 b 4k 4 + d 4k 4 + f 4k 4 + h 4k 4
= = k4
b4 + d 4 + f 4 + h4 b4 + d 4 + f 4 + h4

So LHS = RHS (Proved)

The Institute of Cost Accountants of India 7


Fundamentals of Business Mathematics and Statistics

Illustration 3.
n −1
a1  a1 
If a1, a2  an be continued proportion, show that =
an  a2 
Answer:
a1 a2 a3 a
= = =  ... ... n−1 =  .k (say)
a2 a3 a4 an

a1 a2 a3 a a
k n−1 = × × =  ... ... × n−1 = 1
a2 a3 a4 an an
n −1
a
Again k n −1 =  1 
 a2 

n −1
a1  a1 
So = (Proved)
an  a2 

Illustration 4.

x y z x 2 − yz y 2 − zx z 2 − xy
= = Prove that 2 = =
a b c a − bc b 2 − ca c 2 − ab

Answer:
x y z
= = = k ( Say )
a b c
So x = ak , y = bk , z = ck

x 2 − yz ( ak ) − bk.ck k ( a − bc )
2 2 2

= = = k2
a 2 − bc a 2 − bc a 2 − bc

y 2 − zx z 2 − xy
Similarly = =k
b 2 − ca c 2 − ab

x 2 − yz y 2 − zx z 2 − xy
So = = (Proved)
a 2 − bc b 2 − ca c 2 − ab

Illustration 5.
p q r
If b − c = c − a = a − b Prove that p + q + r = 0 = pa + qb + rc

Answer:
p q r
= = = k ( say )
b−c c−a a −b

So p = k (b − c ) , q = k ( c − a ) , r = k ( a − b )

8 The Institute of Cost Accountants of India


Arithmetic

So p + q + r = k (b − c ) + k ( c − a ) + k ( a − b ) = 0

So pa + qb + rc = k (b − c ) a + k ( c − a ) b + k ( a − b ) c = 0
Hence the result (Proved)

Illustration 6.
The marks obtained by four examinees are as follows: A: B = 2:3, B: C = 4:5, C: D = 7:9, find the continued ratio.
Answer:
A:B = 2:3
3 3 15
So B: C = 4:5 = 4 × : 5 × = 3 :
4 4 4

So C: D = 7:9 = 7 × 15 : 9 × 15 = 15 : 135
28 28 4 28
15 135
So A : B : C : D = 2 : 3 : : = 56 : 84 :105 :135
4 28
Illustration 7.
Two numbers are in the ratio of 3:5 and if 10 be subtracted from each of them, the remainders are in the ratio of
1:5, find the numbers.

Answer:
x 3
Let the numbers be x and y, so that = Or, 5x = 3 y ... ... ...(1)
y 5
x − 10 1
Again =
y − 10 5
Or, 5x − 50 = y − 10

Or, 3 y − 50 = y − 10 [As 5x = 3y from (1)]

Or, 2 y = 40, So y = 20
And 5x = 3 y = 60, So x = 12 and y = 20

Illustration 8.
The ratio of annual incomes of A and B is 4:3 and their annual expenditure is 3:2. If each of them saves `1000 a
year, find their annual income.

Answer:
Let the incomes be 4x and 3x (in `)
4 x − 1000 3
=
3x − 1000 2
From above x = 1000
So income of A = `4,000 & that of B = `3,000

The Institute of Cost Accountants of India 9


Fundamentals of Business Mathematics and Statistics

Illustration 9.
The prime cost of an article was three times the value of material used. The cost of raw materials was increased in
the ratio 3:4 and the productive wage was increased in the ratio 4:5. Find the present prime cost of an article, which
could formerly be made for `180.
Answer:
Prime cost = x + y, where x = productive wage, y = material used
Now prime cost = 180 = 3y or, y = 60, again x + y =180, x = 180 – y = 180 – 60 = 120
Present material cost = 4y/3, present wage 5x/4
Present prime cost = 4 ×60/3 + 5×120/4 = `230

PRACTICE PROBLEMS
1. The ratio of the present age of a father to that of his son is 5:3. Ten years hence the ratio would be 3:2. Find
their present ages. (Ans. 50, 30)

2. The monthly salaries of two persons are in the ratio of 3:5. If each receives an increase of `20 in salary, the
ratio is altered to 13:21. Find the respective salaries. (Ans. `240, `400)

3. What must be subtracted from each of the numbers 17, 25, 31, 47 so that the remainders may be in
proportion? (Ans. 3)

x y z
4. If = = show that (b − c ) x + ( c − a ) y + ( a − b ) z = 0
b+c c+a a+b
x y z 1
5. If = = = k prove that k = , if x + y + z ≠ 0
y+ z z+ x x+ y 2

a− b 1 a 2 + ab + b 2 91
6. If = prove that 2 =
a+ b 2 a − ab + b 2 73
a b c a+b+c
7. If = = prove that =2
4 5 9 9
b+c c+a a+b
8. If = = and a + b + c ≠ 0 then show that each of these ratios is equal to 2. Also prove that
a b c
2 2 2
a + b + c = ab + bc + ca

9. If a : b = c : d Show that xa + yb : aα − bβ = xc + yd : cα − d β

α β γ
10. If = = Prove that α + β + γ = 0, pα + qβ + r γ = 0
q−r r− p p−q

11. Monthly incomes of two persons Ram and Rahim are in the ratio 5:7 and their monthly expenditures are in the
ratio 7:11. If each of them saves `60 per month. Find their monthly income. (Ans. `200, `280)

12. There has been increment in the wages of labourers in a factory in the ratio of 22:25, but there has also been
a reduction in the number of labourers in the ratio of 15:11. Find out in what ratio the total wage bill of the
factory would be increased or decreased. (Ans. 6:5 decrease)

10 The Institute of Cost Accountants of India


Arithmetic

1.1.3. Variations:

1.1.3.1 Direct Variation:


If two variable quantities A and B be so related that as A changes B also changes in the same ratio, then A is said to
vary directly as (or simply vary) as B. This is symbolically denoted as A∝B (read as A varies as B)
Example:
The circumference of a circle = 2πr, so circumference of a circle varies directly as the radius, for if the radius
increase (or decrease), circumference also increases or decreases.
From the above definition, it follows that:
If A varies as B, then A = KB, where K is constant (≠ 0)
Cor: A ∝ B then B ∝ A if A ∝ B , then A = k B. or B = A/k i.e., B ∝ A .
Example: The area of a circle is directly proportional to radius of circle (i.e. if radius increases the area of a circle
increases in the same ratio as radius) i.e.
A ( Area ) ∝ r ( radius )
A = Kr
Or, K= A/r
K is nothing but constant of proportionality

1.1.3.2 Inverse variation:


When two variable quantities A and B are such that the increase or decrease of A causes decrease or increase of B
in a costant ratio then. A is said to be varies inversely as B. or A varies directly as the reciprocal of B.

i.e. A ∝ 1/ B
Or, A = K/B [K = Constant of proportionality]
Or, K = AB
Example: Speed (S) is inversely proportional to time (t) i.e.
S ∝ 1/ t
Or, S = K / t
Or, K = St

1.1.3.3 Joint variation:


A is said to vary jointly as B, C, and D…. if A varies directly as the product of (B, C, D….) i.e.

A ∝ ( BCD ……)

Or, A = K ( BCD ……)


Example: The volume of cuboid varies directly as the product of length (l), breadth (b), height (h)

i.e. V ∝ lbh

The Institute of Cost Accountants of India 11


Fundamentals of Business Mathematics and Statistics

Or, V = Klbh
Or, K = V / lbh

1.1.3.4 Some Elementary Results:


(i) If A ∝ B , then B ∝ A
(ii) If A ∝ B and B ∝ C , then A ∝ C
(iii) If A ∝ B and B ∝ C , then A − B ∝ C
(iv) If A ∝ C and B ∝ C , then A − B ∝ C

(v) If A ∝ C and B ∝ C , then AB ∝ C

(vi) If A ∝ B , then An ∝ B n .
(vii) If A ∝ B and C ∝ D , then AC ∝ BD and A/C ∝ B/D

(viii) If A ∝ BC , then B ∝ A / C and C ∝ A / B

Illustration 10.

If a + b ∝ a − b prove that

(i) a ∝ b
(ii) a 2 + b 2 ∝ a 2 − b 2

(iii) a 2 + b 2 ∝ ab

Answer:
(i) Since a + b ∝ a − b , then a + b = k ( a − b ) , k is constant of variation. Or, ( k − 1) a = ( k + 1) b
a k +1
Or, =
b k −1

a k +1 k +1
Or, = × b = mb , where m = , a constant, Hence a ∝ b
b k −1 k −1

a 2 + b 2 m 2b 2 + b 2 m 2 + 1
(ii) = = = a constant. So a 2 + b 2 ∝ a 2 − b 2
a 2 − b 2 m 2b 2 − b 2 m 2 − 1

a 2 + b 2 m 2b 2 + b 2 ( m + 1)
2

(iii) = = = a constant. So a 2 + b 2 ∝ ab
ab mb 2
m2

Illustration 11.
If x + y ∝ x − y , prove that ax + by ∝ px + qy , where a, b, p q are constants

12 The Institute of Cost Accountants of India


Arithmetic

Answer:

As x + y ∝ x − y, So x + y = k (constant)
x− y
x k +1
Or, x + y = k ( x − y ) Or, ( k − 1) x = ( k + 1) y Or, y = k − 1 = m (a constant), Or, x = my
ax + by amy + by am + b constant. So ax + by ∝ px + qy
= = =
px + qy pmy + qy pm + q

Illustration 12.

If ( a + b ) ∝ ( a − b ) show that ( a 2 + b 2 ) ∝ ab
Answer:
a k +1
As ( a + b ) ∝ ( a − b ) , So ( a + b ) = k ( a − b ) Or a ( k − 1) = b ( k + 1) , Or, = = m (a constant)
b k −1
a 2 + b2 a b k + 1 k − 1
= + = + = Constant. So ( a 2 + b 2 ) ∝ ab
ab b a k −1 k +1

Illustration 13.
If the cost price of 12 kg. of rice is `10, what will be the cost of 15 kg. of rice?
Answer:
Let Cost and Quantity be denoted by A and B.
Given: A = `10, B = 12 kg. Now A∝ B i.e., A = KB or, 10 = K. 12 or, K = 10/12
Now, we are to find A, when B = 15 kg.
Again from A = KB, we have A =10/12 × 15 = `12.50

Illustration 14.
A man can finish a piece of work, working 8 hours a day in 5 days. If he works now 10 hours daily, in how many
days can he finish the same work?
Answer:
Let A = No. of days required to finish the work and B = Working hours per day
Given: A = 5, B = 8, it is clear that A∝ 1/B
i.e. A = k. 1/B or 5 = k. 1/8 or k = 40
To find A when B = 10, we have A = 40 ×1/10 = 4 days

Illustration 15.
One part of the total expenditure in a hostel is constant and the other part directly varies with the number of
boarders. Per head expenditure is `100 when the number of boarders is 35 and `80 when the number of boarders
is 45. If the number of boarders is 150 find the total expenditure of the boarders.
Answer:

The Institute of Cost Accountants of India 13


Fundamentals of Business Mathematics and Statistics

Let n be the number of boarders and E be the total expenditure of the hostel. Part of the total expenditure is constant
= F (let) and the other part of the total expenditure is V (let)
By question
V∝ n, then V = Kn, where K is constant
So, E = F + V = F + Kn … … … … (1)
When n = 35, then E = 35 ×100 = ` 3500
When n = 45, then E = 45 × 80 = ` 3600
From (1) 3500 = F + K35 … … … … (2)
From (2) 3600 = F + k 45 … … … … (3)

By (3) – (2) we have 100 = 10K. So K = 10


In Equation (2) by putting K = 10, we have 3500 = F + 35 ×10, So F = ` 3150
So from equation (1) we have E = F + V = F + Kn = 3150 + 10n
So if there are 150 boarders then total expenditure will be E = 3150 + 10 ×150 = `4650

Illustration 16.
If (5a – 2b) : (2a + b) = (6a – b) : (8a – b) find a : b

Answer:
Given (5a – 2b) : (2a + b) = (6a – b) : (8a – b)

5a − 2b 6a − b
Or, =
2a + b 8a − b
Or, (5a − 2b )(8a − b ) = ( 6a − b )( 2a + b )

Or, 40a 2 − 5ab − 16ab + 2b 2 = 12a 2 − 2ab + 6ab − b 2

Or, 40a 2 − 21ab + 2b 2 = 12a 2 + 4ab − b 2

Or, 28a 2 − 25ab + 3b 2 = 0

Or, ( 4a − 3b )( 7a − b ) = 0

So, 4a − 3b = 0 and 7a − b = 0
Therefore 4a = 3b i.e. a = 3
b 4
a 1
and 7a = b i.e. =
b 7

14 The Institute of Cost Accountants of India


Time Value of Money and Annuity – Simple
and Compound Interest 1.2
1.2.1 Simple Interest

1.2.1.1 Interest:
Interest is the additional money which is paid by the borrower to the lender on the principal borrowed. The additional
money (or) interest is paid for the use of money by the borrower. Interest is usually denoted by I.

For example:
Y borrowed ` 500 from Z for a year and returned `550. Here `50 is paid additionally. This `50 is the interest.

1.2.1.2 Rate of interest per annum:


R r
Rate of interest per annum is the interest paid yearly for every `100 It is denoted by or .
100 100
1.2.1.3 Amount:
The sum of principal and interest paid is called as an Amount. It is denoted by A. Thus A = P + I

1.2.1.4 Simple Interest:


If the interest is calculated uniformly on the original principal throughout the loan period it is called Simple
Interest. It is denoted by S.I.
Formula:
Simple interest on the principal ‘P’ borrowed at the rate of  r % p.a for a period of “t” years is usually given by
S.I = P×r×t/100
For example:
Gopi borrowed `1200 from Siva Reddy at 9% simple interest p.a for 3 years. How much Siva should get from
Gopi as S.I.?
Sol: P = 1200, r = 9%, t = 3
S.I = P × r × t/100 = 1200 × (9/100) × 3 = `324

1.2.1.5 Important Relations to remember:


1. S.I = P × r × t/100

2. A = P + S.I

The Institute of Cost Accountants of India 15


Fundamentals of Business Mathematics and Statistics

3. r = S.I ×100/Pt

4. t = S.I ×100/Pr

5. P = S.I × 100/rt

6. P = A – S.I

7. S.I = A – P

(
8. A = P 1 +
rt
100 )
Illustration 17.
Amit deposited `1,200 to a bank at 9% simple interest p.a. find the total interest that he will get at the end of 3
years.

Answer:
Here P = 1200, r = 9/100 = 0.09, t = 3, I =?
I= P × r × t = 1200 × 0.09 × 3 = 324.
Amit will get `324 as interest.

Illustration 18.
Sumit borrowed `7,500 are 14.5% p.a S.I. for 2-1/2 year. Find the amount he had to pay after that period

Answer:
P = 7500, r = 14.5/100 = 0.145, t = 2-1/2 = 2.5, A =?

( rt
)
A = P 1 + 100 = 7500 (1+ 0.145 × 2.5) = 7500 (1+ 0.3625)= 7500 × 1.3625 = 10218.75

Reqd. amount = `10,218.75

Illustration 19.
What sum of money will yield `1407 as interest in 1-1/2 year at 14% p.a simple interest?

Answer:
Here S.I = 1407, t = 1.5, r = 0.14, P =?
S.I = P × r × t or, 1407 = P × 0.14 × 1.5
Or, P = 1407/(0.14 × 1.5) = 6700
Required amount = `6,700

Illustration 20.
A sum of `1,200 was lent out for 2 years at S.I. The lender got `1,536 in all. Find the rate of interest.

16 The Institute of Cost Accountants of India


Arithmetic

Answer:
P = 1200, A = 1536, t = 2, r = ?
A = P (1+ rt/100) or, 1536 = 1200 (1 + r × 2/100) = 1200 + 24r
Or, 1536 – 1200 = 24r or, r = 336/24 = 14
Required rate = 14%

Illustration 21.
At what rate percent of Simple Interest will a sum, become double of itself in 5-1/2 years?

Answer:
A = 2P, P = principal, t = 5.5, r = ?
A = P (1 + rt/100) or, 2P = P + P × r × t/100
1 = r × 5.5 /100
Required. Rate = r = 18.18%

Illustration 22.
In a certain time `1200 becomes `1560 at 10% p.a simple interest. Find the principal that will become ` 2,232 at
8% p.a in the same time.

Answer:
In 1st case: P = 1200, A = 1560, r = 0.10, t = ?
1560 = 1200 {(1 + t × (.10)} = 1200 + 120t
Or, 120t = 360 or, t = 3

In 2nd case: A = 2232, t = 3, r = 0.08, P = ?


2232 = P {(1+ 3 × (0.08)} = P (1+ 0.24)
Or, P = 2232/1.24 = 1800
Required Principal is ` 1,800

Illustration 23.
Divide ` 2,760 in two parts such that simple interest on one part at 12.5% p.a for 2 years is equal to the simple
interest on the other part at 12.5% p.a for 3 years.

Answer:
Let Investment in 1st case = X (say)
Investment in 2nd case = ` (2,760 – X)
Interest in 1st case = X×2×0.125
Interest in 2nd case = (2760–X)×3×0.125

The Institute of Cost Accountants of India 17


Fundamentals of Business Mathematics and Statistics

By question (2760 – X)×3×0.125 = X×2×0.125


Or, 2760×3×0.125 – 3×0.125×X = 2×0.125×X
Or, 1035 = 0.625X
X = 1035/0.625 = 1656
So first part is `1656 & second part is (2760 – 1656) = `1104

Illustration 24.
A person borrowed `8,000 at a certain rate of simple interest for 2 years and then `10,000 at 1% lower than the first
for 1 year. In all he paid `2,500 as interest in 3 years. Find the two rates at which he borrowed theamount.

Answer:
Let the rate of interest in the 1st cast = r, so that in the 2nd case, rate of interest will be (r–1). Now
8000×r×2/100 + 10000×(r–1)×1/100 = 2500
Or, 160r +100r-100 = 2500
Or, 260r = 2600
Or, r = 2600/260 = 10%
So in first case interest is 10% and in second case it is (10–1) = 9%
Calculation of interest on deposits in a bank: Bank allows interest at a fixed rate on deposits from a fixed day of
each month up to the last day of the month. Again interest may also be calculated by days.

1.2.2 Time Value of Money


A rupee today will be more valuable than the same in future. Because a receipt today generate more preferable
current consumption whereas a receipt tomorrow causes less preferable future consumption. To entice an individual
for future receipt something extra is to be given with the present receipt.

This something extra is the compensation for foregoing current consumption – in financial term called INTEREST.

The interest can be used to express the time value of the money.

The time value of money analysis involves:

� What is `1 worth n years from today ⇒ Future value of a single flow?

� What is `1 to be deposited per year for n years’ worth at the end of nth year from today ⇒ Future value of an
annuity

� What is `1 to be received in nth year from today worth today ⇒ Present value of a single flow

� What is `1 to be received per year for n years’ worth today ⇒ Present value of an annuity

Applications

Time value of money analysis is applied

18 The Institute of Cost Accountants of India


Arithmetic

(i) For pricing of Bonds.

(ii) For pricing of stocks/firms.

(iii) For finding out the value a new project contributes to the organization.

(iv) For finding out the value of a particular division within an organization.

(v) For loan amortization.

(vi) For comparative study between rebate and low financing.

1.2.3 Types of Valuation


1.2.3.1 Future value of a single flow:
If an individual deposits `100 into a savings account at an interest rate of 10%, compounded annually then the
deposit would grow as follows:
At the end of one year: Principal at the beginning of the period ⇒ `100
Interest for the one year period @ 10% ⇒ `10
----------------------------------------------------------
Value of the original amount at the end of the year ⇒ `110
At the end of 2nd year: Principal at the beginning of the period ⇒ `110
Interest for the one year period @ 10% ⇒ ` 11
-----------------------------------------------------------
Value of the original amount at the end of the 2nd year ⇒ `121

As shown above the interest amounts to `11 in the second year composed of `10 as interest on the original principal
of `100 in the first year and (10/100)×10 i.e. `1 as interest on the interest of the first year in the second year .This
interest on interest principle is called interest compounded annually.
So if ‘P’ is deposited for ‘n’years at an interest rate ‘r% ’ , compounded annually , then the amount P would be
• At the end of 1st year
= FV1 = P × (1 + r ) , where FV1 is the future value of P at end of year 1
1

• At the end of 2nd year
= FV2 = P × (1 + r ) where FV2 is the future value of P at end of year 2
2

• At the end of 3rd year
= FV3 = P × (1 + r ) where FV3 is the future value of P at end of year 3
3

…………………….

• At the end of nth year


= FVn = P × (1 + r )
n

where FVn is the future value of P at end of year …………..(1)

The Institute of Cost Accountants of India 19


Fundamentals of Business Mathematics and Statistics

In general equation (1) gives FVn — the future value of a deposit at the end of nth period if r is the interest rate
per annum.
Equation (1) is the basic equation in compounding analysis.

The factor (1 + r ) n is termed as


� Compounding factor or
� The Future Value Interest Factor (FVIF) or
� Terminal value Interest Factor.

So equation (1) can be alternatively written as

FVn = P × FVIFr ,n , where FVIFr , n is the future value interest factor at r% for n periods

Tables have been constructed for FVIFr , n for wide ranges of r & n. Such table is given at the end of all finance
book.
So if the interest is added after certain period of time (say yearly, half yearly, quarterly and monthly etc) to the
principal so that the amount at the end of the period becomes the principal for the next time period then the total
interest paid over all the time period is called the Compound Interest. It is usually denoted by C.I.

Compound Interest = Amount – Principal = P × FVIFr ,n − P = P × (1 + r ) − P


n

Illustration 25.
If Mr. X deposits `5,000 today in a bank which pays 8% interest compounded annually, how much the deposit
will grow at the end of 10th year ?

Answer:
Here we are going to find FVn = P × FVIFr ,n , where P = `5,000, n = 10 and FV8%,10 = 2.159. So FV10 = 5000 × 2.159
= 10795.

1.2.3.2 Future value of an Annuity


Annuity is a periodical flow of money, where at equal interval of time equal quantum of money flows. So a flow of
money will be considered as annuity if two factors are satisfied-
� Interval between any two successive flows is same;
� Quantum of money flow in each interval is same;
These two factors are to be satisfied simultaneously. Breaking of any one of these or both, will constitute a multiple
flow but not an annuity.
e.g.
(a) `5,000 is deposited at the end of every month is an annuity – interval between successive deposit is unchanged
and amount of deposit is also remained same.
(b) `10,000 is deposited at the beginning of every quarter is an annuity – interval between successive deposit is
unchanged and amount of deposit is also remained same.
(c) `5,000 is deposited at the end of every month from January to March, `8,000 is deposited at the end of every

20 The Institute of Cost Accountants of India


Arithmetic

month from April to September and `10,000 is deposited at the end of every month from October to December.
It is a multiple flow but not an annuity – although interval between successive deposits is unchanged but there
are changes in amount of deposit.
(d) `10,000 is deposited at the end of first year, `10,000 is deposited at the end of 4th year and `10,000 is deposited
at the end of 9th year. It is a multiple flow but not an annuity – although there is no change in the amount of
deposit but interval between successive deposits is changed.
(e) `10,000 is deposited at the end of first quarter, `20,000 is deposited at the end of 3rd quarter and `18,000 is
deposited at the end of 4th quarter. It is a multiple flow but not an annuity – because here interval between
successive flows as well as quantum, both changed. No deposit is made at the end of 2nd quarter and every time
deposit amount varied.
Annuities are of two types:
(i) Regular annuity or deferred annuity when flow of money occurs at the end of a period;
(ii) Annuity due when flow of money occurs at the beginning of a period;
Suppose an individual will deposit `1,000 in a bank at the end of each year for next five years. Also suppose bank
gives 10% interest p.a. What will be the matured amount at the end of the fifth year?
Here there will be an annuity because interval between successive deposits is same (one year) and quantum of
deposit in each case is also same (`1000). Moreover since deposit is made at the end of a period, therefore the
annuity is Regular annuity.
Total matured amount comprises of –

1st year 2nd year 3rd year 4th year 5th year
` 1000 ` 1000 ` 1000 ` 1000 ` 1000

1. Future value of `1,000, deposited at the end of 1st year matured at the end of 5th year FVn = P × FVIF(r, n) i.e.
FV4 = 1000 × FVIF (10%, 4) = 1000×1.464 (from table) = ` 1464;
2. Future value of `1,000, deposited at the end of 2nd year matured at the end of 5th year FVn = P × FVIF(r, n) i.e.
FV3 = 1000 × FVIF (10%, 3) = 1000×1.331 (from table) = ` 1331;
3. Future value of `1,000, deposited at the end of 3rd year matured at the end of 5th year FVn = P × FVIF(r, n) i.e.
FV2 = 1000 × FVIF (10%, 2) = 1000×1.210 (from table) = ` 1210;
4. Future value of `1,000, deposited at the end of 4th year matured at the end of 5th year FVn = P × FVIF(r, n) ie.
FV1 = 1000 × FVIF (10%, 1) = 1000×1.100 (from table) = ` 1100;
5. Future value of `1,000, deposited at the end of 5th year matured at the end of 5th year FVn = P × FVIF(r, n) i.e.
FV0 = 1000 × FVIF (10%, 0) = 1000×1.000 (from table) = ` 1000;
Sum total of (1) to (5) will be the matured amount of the annuity at the end of 5th year i.e. `6105
But instead of computing the matured amount of a regular annuity in the aforesaid manner, general formula is made
and that is

FVn = P ×
(1 + r )n − 1 … … … … … … (2)
r

The Institute of Cost Accountants of India 21


Fundamentals of Business Mathematics and Statistics

Where,
FVn = Future value;
P = Amount deposited at the end of each period;
r = Rate of interest per period;
n = Duration of the annuity;

(1 + r )n − 1 : This factor is called Future Value Interest Factor for Annuity (Regular) for period n at an interest rate
r
of r p.a or FVIFr,n
Tables have been constructed for FVIF(r, n) for wide ranges of r & n.
So in our illustrative problem P = `1000, r = 10% p.a, n = 5 yrs;

Therefore FVn = P ×
(1 + r )n − 1 = P × FVIF = 1000 × 6.105 (from table) = `6105, same as in our previous
r 10%,5

calculation.

In case of Annuity due equation (2) will be slightly modified as follows:

(1 + r )n − 1 × (1 + r )
FVn = P ×   … … … … … … (2A)
r

1.2.3.3 Present value of a single flow


Suppose an individual will receive `1,000 from a bank deposit three years hence for his deposit today at an interest
rate of 10 per cent, compounded annually.
From our knowledge on FUTURE VALUE of a single flow we can safely conclude that `1000 in our example is
the future value of a single flow i.e. the deposit amount of today. If this deposit amount we consider as P then we
can write:
1000 = P × (1+ 0.10)3
Or, 1000 = P × FVIF (10%, 3)
Or, P = 1000/ (1+.10)3 OR 1000/FVIF (10%, 3)
The ‘P’ in the above case is present day deposit .In business term it is called PRESENT VALUE OF A SINGLE
FLOW occurring in future. The factor 1/ (1+.10)3 or 1/FVIF (10%,3) or reciprocal of future value interest factor is
called PRESENT VALUE INTEREST FACTOR or the DISCOUNTING FACTOR.
Present value calculations of future cash flows allow us to place all the future cash flows on a current footing so
that comparisons can be made in terms of today’s flows.
So general formula for present value of a single flow can be obtained by manipulating the future value formula
and it is:
FV … … … … … … (3)
PV =
(1 + r )n

22 The Institute of Cost Accountants of India


Arithmetic

The factor 1 is called Present value interest factor of a single flow or PVIF at r percent for n years.
(1 + r ) n
Interest rate in the above formula is also called discount rate or the capitalization rate.
A present value table containing PVIFs for wide ranges of ‘r’ and ‘n’ is available.
Present value bears the following characteristics in contrast to future value:
� Greater the interest rate, lower is the present value.
� Interest compounding period more, lower is the present value.

1.2.3.4 Present value of an annuity


Suppose an individual is expecting to receive `1000 annually for 3 years starting from 1 year hence. What is the
value of such annuity receipts right now i.e. today (time period 0). The following diagrammatic representation of
the above case will be helpful for better understanding:

0 1 2 3

1000 1000 1000


From the above it is clear that finding the present value of an annuity is nothing but finding the sum of a series of
individual present values. Therefore we can write the general formula for the present value of an ordinary annuity
of magnitude ‘A’ for n period at a discounting rate (rate of interest) ‘r’ as:

A A A A
PVA = 1 + 2 + 3 + ……………+
(1 + r ) (1 + r ) (1 + r ) (1 + r )n

PVA = A
(1 + r )n − 1 … … … … … … (4)
r (1 + r )
n

The factor
(1 + r )n − 1 is called Present value interest factor of an (ordinary) annuity or PVIFA in short.
r (1 + r )
n

A present value table containing PVIFAs for wide ranges of ‘r’ and ‘n’ is available

PVIFA =
(1 + r )n − 1
r (1 + r )
n

Or, PVIFA =
(1 + r )n − 1 × 1
r ( r )n
1 +

Or, PVIFA = FVIFAr ,n × PVIFr ,n

The Institute of Cost Accountants of India 23


Fundamentals of Business Mathematics and Statistics

Illustration 26.

Mr. X will receive `1500.00 at the end of every year starting from today and this will continue for next 5 years.
How much deposit he has to make today if the interest rate is 10% p.a..?

Answer:
This is clearly a problem of present value. Since the receipts in the hands of Mr. X are of same magnitude and also
occurring in equal intervals of time – at the end of every year. So receipts form regular annuity. Present value of
such annuities as per formula 4 is

PVA = A
(1 + r )n − 1
r (1 + r )
n

Where A = 1500, r = 10% = 0.10 & n = 5

Or, PVA = 1500 ×


(1 + 0.1)5 − 1 =≅ ` 5686.18
5686.18
0.1 × (1 + 0.1)
5

OR

PVA = A × FVIFAr ,n × PVIFr ,n

= 1500 × FVIFA10%,5 × PVIF10%,5


= 1500 ×6.105 × 0.621 = 5686.801

Formula 4 is applicable in case of regular annuity.

In case of Annuity Due the formula for present value is given by

PVA = A
(1 + r )n − 1 × (1 + r ) … … … … … … (4A)
r (1 + r )
n

So from formula (1), (2), (2A), (3), (4) & (4A) we can summarize the following formula for unit amount (i.e. A = 1),
for interest rate ‘r p.a.’ and for ‘n years’ for ready reference and comparative study.

FLOW FUTURE VALUE PRESENT VALUE


Single
(1 + r )n 1
(1 + r )n
Regular Annuity
(1 + r )n − 1 (1 + r )n − 1
r r (1 + r )
n

Annuity Due (1 + r )n − 1 × (1 + r )


  (1 + r )n − 1 × (1 + r )
r (1 + r )
n
r

In all the above cases we have considered Compounded Annually. But if compounding i.e. interest on interest is

24 The Institute of Cost Accountants of India


Arithmetic

given more than once in a year then formulae need modification. More than once compounding will be treated in
next note.

1.2.3.5 Shorter compounding Period


Till now we have our all computations assuming interest rate ‘r’ on per annum basis i.e. all flows are compounded
annually. If compounding is done for shorter period then different results will arrive. Suppose Mr y deposited
`1000 in a company deposit which gives 5% interest on the balance amount at the end of every six months i.e.
compounding is done semi-annually. So

Principal at the beginning = `1000.00


Interest for the first 6 month = `1000 × 5/100× 6/12 = `25.00
Total amount at the end of 6 month = `1025.00
This `1025 will be the principal amount at the beginning of the next 6 months. Result will be
Principal at the beginning = `1025.00
Interest for the first 6 month = `1025 × 5/100× 6/12 = `25.625
Total amount at the end of next 6 month = `1050.625

If compounding is done annually total amount at the end of one year will be `1050.00 instead of `1050.625. Extra
amount is `0.625.This is nothing but the interest for the last six months on interest for the first six months i.e.
Interest for the first six months = ` 25.00
Interest on this amount @ 5% p.a. for the last six months = 25×5/100×1/2 = `0.625. So in one year if compounding
is done every six months the maturity value will be
`1050.625 = 1000 × (1.050625)

= 1000 × (1 + 0.05625)

( )
2
5
= 1000 × 1 +
200

5
= 1000 × {1 + ) 2×1 … … … … … … (7)
2 × 100

In formula (7), first ‘2’ represents no of times compounding is done in one year for semi-annual compounding.
Therefore general formula for future value of a single flow ` P, deposited for ‘n’ years at an interest rate of ‘r %
p.a’ with compounding done ‘m’ times in a year is

( )
m× n
r
FV = P × 1 + … … … … … … (8)
m

Following the same principle the general formula for different types of flows for ‘n’ years at an interest rate of ‘r
% p.a’ with compounding done ‘m’ times in a year are given below for an unit amount (i.e. A = 1):

The Institute of Cost Accountants of India 25


Fundamentals of Business Mathematics and Statistics

FLOW FUTURE VALUE PRESENT VALUE


Single
(1 + r / m) mn
1
(1 + r / m)mn
Regular Annuity
(1 + r / m)mn − 1 (1 + r / m)mn − 1
r/m r / m (1 + r / m )
mn

Annuity Due (1 + r / m )mn − 1 × (1 + r / m )


  (1 + r / m)mn − 1 × (1 + r / m)
r / m (1 + r / m )
mn
r/m

Illustration 27.
Mr. P opened a recurring deposit account in a bank, started depositing semi-annual instalments in arrear amounting
` 5000 for 5 years. If the rate of compound interest is 6% p.a. what will be the maturity amount.
Answer:
It is a problem of future value of regular annuity when compounding is done more than once in a year – in the given
problem it is twice in a year. So the matured amount will be:
5000 × FVIFA r = 5000 × FVIFA 6 = 5000 × FVIFA(3%,10)
Amount = ( m ,mn) ( 2,5×2)
= 5000 × 11.464 (from table) = `57320

1.2.3.6 Doubling Period


A frequently asked question in finance is “how long it would take to double the invested amount at a given interest
rate?”
A rule of thumb called “Rule of 72’’ is available to answer this question. This rule states that the doubling period
will be obtained by dividing 72 by the given interest rate.
For example if interest rate is 6 percent then doubling period will be 72/6 i.e. 12 years.
Another thumb rule of doubling period is named “Rule of 69”. As per this rule the doubling period will be = 0.35
+ (69 / interest rate)

1.2.3.7 Effective versus Nominal Interest Rate


While discussing the shorter compounding period it is found that `1000 if deposited for one year @5% p.a.
compounded semi-annually, the maturity amount will be `1050.625.This means that `1000 grows at the rate of
5.0625% p.a. This 5.0625% is called the effective rate of interest.
So effective rate of interest is defined as that interest rate which under annual compounding produces the same
result as that produced by an interest rate under shorter compounding period. The general relationship between
effective interest rate and nominal interest rate is as follows:

( )
m
r
Effective rate = 1 + m −1

m, the no of times compounding is done in a period

26 The Institute of Cost Accountants of India


Arithmetic

Illustration 28.
Find the compound interest on `1000 for 4 years at 5% p.a

Answer:
Here P = `1000, n = 4, r = 0.05, A = ?

We have FVn = A = P × (1 + r ) = 1000 × (1 + 0.05) = 1000 × 1.216 = 1216


n 4

So Compound interest = A – P = 1216 – 1000 = `216

Illustration 29.
In what time will a sum of money double itself at 5% p.a C.I.

Answer:
Here, P = P, A = 2P, r = 0.05, n = ?

We have FVn = A = P × (1 + r )
n

Or, 2P = P × (1 + 0.05)
n

Or, 2 = (1 + 0.05)
n

log 2
Or, log 2 = n log 1.05 Or, n = = 14.2 years
log1.05
72
From rule of 72 approximate doubling period = = 14.4 years
5
Illustration 30.
The difference between simple and compound interest on a sum put out for 5 years at 3% was `46.80. Find the sum.

Answer:
Let the sum is X
Simple Interest = X × 0.03 × 5

Compound interest = X (1 + 0.03) − X


5

So by question we can write [ X (1 + 0.03)5 – X  ] – [ X × 0.03 × 5 ] = 46.80

Or, X (1 + 0.03) − 1 − 0.15 = 46.80


5

Or, X [1.16 − 1 − 0.15] = 46.80

Or, X [1.16 − 1.15] = 46.80

Or, X [0.01] = 46.80

Or, X = 46.80 = ` 4,680 (approx.)


0.01

The Institute of Cost Accountants of India 27


Fundamentals of Business Mathematics and Statistics

Illustration 31.
What is the present value of `1,000 due in 2 years at 5% compound interest p.a, according as the interest is paid
(a) yearly (b) half-yearly?
Answer:
FV
(a) We know present value P = , where interest is paid yearly
(1 + r )n
1000 1000
Or, P = = = ` 907 (approx)
(1 + 0.05)2 1.1025
FV
(b) We know present value P = , where interest is paid m times in a year
(1 + r / m)mn
In the given case interest is paid semiannually i.e. m = 2

FV 1000
P= = = ` 906 (approx)
(1 + r / m)mn
( )
2× 2
0.05
1+
2
Illustration 32.
A sum of money invested at C.I. payable yearly amounts to `10,816 at the end of the second year and to `11,248.64
at the end of the third year. Find the rate of interest and the sum.

Answer:

We have FVn = A = P × (1 + r )
n

In First case: FVn = A = 10816 = P × (1 + r ) … … … .. (1)


2

In Second case: FVn = A = 11248.64 = P × (1 + r ) … … … .. (2)


3

11248.64 P × (1 + r )
3

Dividing (2) by (1): = 2 = (1 + r )


10816 P × (1 + r )

Or, 1.04 = 1 + r Or, r = 0.04 Or, r = 4%. This is the required rate of interest.

From (1) we have A = P × (1 + r ) Or, 10816 = P (1.04) . So P = `10,000. This is the required sum
n 2

Illustration 33.
Calculate the value 5 years hence of a deposit of `1,000 made today when the interest rate is 8% p.a. if (i) simple
interest is given (ii) compound interest is given

Answer:
(i) On simple interest
Interest amount at the end of 5 years = 1000 × 0.08 × 5 = ` 400

28 The Institute of Cost Accountants of India


Arithmetic

So the maturity amount will be = 1000 + 400 = ` 1,400


(ii) On compound interest
The maturity amount A = 1000 × (1+0.08)5 = 1000 × 1.469 = ` 1,469

Illustration 34.
Find the present value of ` 1,000 receivable 6 years hence if the rate of compound interest is 6% p.a.

Answer:
Let the present value = P
So by question we can write 1000 = P × (1 + 0.06)6 = P × (1.06)6
1000
So, P = = ` 704.96
(1.06)6

Illustration 35.
A finance company offers to give `8000 after 12 years in return for `1000 deposited today. Find out the approx
rate of interest offered p.a?

Answer:
By question we can write 8000 = 1000 × (1 + r )
12

We have to find out the value of r


We can write

8000
= (1 + r )
12
1000
Or, 8 = (1 + r )
12

Or, Log 8 = 12 Log (1 + r )

Or, 0.90309 = 12 Log (1 + r )

Or, 0.0752 = Log (1 + r ) Or, 100.0752 = 1+ r

Or, 1.19 = 1 + r , Or, r = 1.19 – 1 = 0.19

Or, r = 19%

Illustration 36.
A bank offers 8% nominal rate of interest with quarterly compounding. What is the effective rate of interest?

Answer:

( )
m
k
We know that effective rate of interest r = 1 + −1
m

The Institute of Cost Accountants of India 29


Fundamentals of Business Mathematics and Statistics

Where
r = effective rate of interest
k = nominal rate of interest
m = no of times interest is given in a year

( )
4
0.08
So by question we can write r = 1 + −1
4
Or, r = (1.02) − 1 ≅ 0.082 = 8.2%
4

Illustration 37.
A man invests `100 at the end of 2001, `200 at the end of 2002, `300 at the end of 2003, `400 at the end of 2004 &
`500 at the end of 2005. If all interest accumulates at 5% p.a. what do his investments amount to at the end of 2006?

Answer:
By question we can write
A1 = 100 × (1.05) = 127.6282
5

A2 = 200 × (1.05) = 243.1013


4

A3 = 300 × (1.05) = 347.2875


3

A4 = 400 × (1.05) = 441


2

A5 = 500 × (1.05) = 525


1

So at the end of 2006 total investment amounts to A1 + A2 + A3 +A4 + A5 = ` 1,684.02

Illustration 38.
Mr S invests in a bank ` X on the first day of 2006. In the subsequent years on the 1st of January he deposits money
double that of the money deposited in the previous year after withdrawing the interest only on the same day. It was
found that balance in his account on 2nd January 2015 was `2046. Find out the amount he deposited on 1st January
2006.
Answer:
Let the money invested on 1st January 2006 be X.
Then by question we can write

X + 2 X + 4 X +10 terms = 2046 [As, time period between 1st January 2006 to 1st January 2015 is 10 years.]

Or, X ( 20 + 21 + ………. + 29 ) = 2046

Or, X
(2− 1)
10

= 2046
2 −1
Or, 1023 X = 2046
Or, X = ` 2

30 The Institute of Cost Accountants of India


Arithmetic

Illustration 39.
Find the interest on `1000 for 10 years at 4% p.a., the compound interest being paid quarterly
Answer:
By question we can write

( )
4×10
0.04
A = 1000 × 1 +
4
Or, A = 1000 × (1.01)
40

Or, A = 1488.864 = 1488.86 (approx.)

So Interest = 1488.86 – 1000 = ` 488.86

Illustration 40.
Someone promises to give you `5,000 after 10 years in exchange for `1,000 today. What interest rate compounded
annually is implicit in the offer?
Answer:
By question we can write
A = P × (1 + r )
10

Or, 5000 = 1000 × (1 + r )


10

Or, 5 = (1 + r )
10

Or, Log 5 = 10 Log (1 + r )


Or, 0.69897 = 10 Log (1 + r )
Or, 0.0699 = Log (1 + r ) Or, 1+r = 100.069
Or, 1.175 = 1+ r
Or, r = 0.175 = 17.5%

Illustration 41.
If `1 was deposited in the post office savings bank on 1st April, 2003, what would it amount to on 1st April 2033 if
compound interest at 3% p.a. was allowed?

Answer:
By question we can write
A = 1 × (1 + 0.03)30
Or, A = (1.03)30 = ` 2.43 (approx.)

Illustration 42.
A sum of money invested at compound interest amounts to `21632 at the end of 2nd year and `22497.28 at the end
of third year. Find the rate of interest and sum invested.

The Institute of Cost Accountants of India 31


Fundamentals of Business Mathematics and Statistics

Answer:
By question we can write
21632 = P × (1 + r ) … … … … … (1)
2

22497.28 = P × (1 + r ) … … … … … (2)
3

By (2) / (1) we get

22497.28 (1 + r )
3

=
21632 (1 + r )2
Or, 1.04 = 1 + r Or, r = 0.04 = 4%
Now we can write
21632 = P (1.04)2 Or, P = ` 20,000

Illustration 43.
What is the difference between the effective rate of interest and nominal rate of interest in the following case?
“Nominal rate of interest is 24% and the frequency of compounding is twelve times a year”

Answer:

( )
m
k
We know that effective rate of interest r = 1 + −1
m
Where
r = effective rate of interest
k = nominal rate of interest
m = no of times interest is given in a year

( )
12
0.24
So by question we can write r = 1 + −1
12
Or, r = (1.02) − 1 ≅ 0.268 = 26.8%
12

This is the required effective rate of interest.

Illustration 44.
Mr. Raghu deposits `10,000 in a bank now. The interest rate is 10% and compounding is done semi-annually. What
will the deposit grow to after 10 years?
Answer:

( )
m ×t
r
By question we can write A = P × 1 + m

Here P = 10000, r = 0.10, m = no. times of interest is given in year = 2, t = 10

( )
2×10
0.1
So we can write A = 10000 × 1 +
2
Or, A = 10000 × (1.05)
20
Or, A = ` 26532.98

32 The Institute of Cost Accountants of India


)XWXUH9DOXH,QWHUHVW)DFWRU )9,)  DWUIRUQSHULRGV 
)9,)  U Q)9 39 )9,)UQ 
                       
       
       
       
       
       
       

The Institute of Cost Accountants of India


       
       
       
       
       
       
       
       
        
        
        
        
         
         
         
            
            
             
             
              
                    
                        
                       
                       


33
Arithmetic
34
3UHVHQW9DOXH,QWHUHVW)DFWRU 39,)  DWUIRUQSHULRGV 
39,)  U Q39 )9 39,)UQ 
                       
                       
                       
                       
                       
                       
                       
                       
                       
                       
                       
                       
Fundamentals of Business Mathematics and Statistics

                       
                       
                       
                       
                       
                       
                       
                       
                       
                       
                       
                       
                       
                       
                       
                       
                       
                       
                       


The Institute of Cost Accountants of India


Arithmetic

  
    

 
  
 
            
            
            
            
            
            
            
            
            
            
            
            
            
            
            
            
            
            
            
            
            
            
            
            
            
            
            
            
            
            


           
           
           
           
           
           
           
           
           
           
           
           
           
           
           
           
           
           
           
           
           
           
           
           
           
           
           
           
           
           


The Institute of Cost Accountants of India 35
Fundamentals of Business Mathematics and Statistics

   


 
     
 
            
            
            
            
            
            
            
            
            
            
            
            
            
            
            
            
            
            
            
            
            
            
            
            
            
            
            
            
            
            


           
           
           
           
           
           
           
           
           
           
           
           
           
           
           
           
           
           
           
           
           
           
           
           
           
           
           
           
           
           


36 The Institute of Cost Accountants of India
Arithmetic

Arithmetic Progression and Geometric


Progression 1.3
1.3.1 Arithmetic Progression:
For any series with ‘n’ terms, if the difference between two consecutive terms is constant throughout the series,
the series is termed as Arithmetic Progression (A. P.). The Difference between two consecutive terms can be either
positive or negative.
Difference between two consecutive terms is represented by “d” and known as Common Difference.

1.3.1.1 Formula for calculating the value Common Difference (D) of an A.P
We have “d” = 2nd Term – 1st Term or (Any term – its preceding term)
The value of “d” is constant throughout the Series. In other words, “d” calculated from any two consecutive terms
will be equal throughout
For example,
d1 = 5th term – 4th term
d2 = 8th term – 7th term
Both, d1 and d2 will be equal for any given A. P.
If the common difference is Positive (d > 0), the terms of the series will be in ascending order. In other words, the
1st term will be smaller than the 2nd term and so on.
For example,
Series: 1, 3, 5, 7, 9 has “d” = 2 {5–3 = 2; 3–1 = 2}

Illustration 45.
A given series is: 26, 29, 32, 35, 38, 41 Identify the type of Series. Also calculate the values of “d” and ‘a’.

Answer:
From the given series it can be inferred that any term of the series is obtained by adding a constant to the preceding
term. So the type of Series is A. P.
Values of “d” and ‘a’:
‘a’ – First Term of the Series = 26.
“d” – Common Difference between any two consecutive terms of the series
“d” = 2nd term – 1st term = 29 – 26 = 3

The Institute of Cost Accountants of India 37


Fundamentals of Business Mathematics and Statistics

So Type of Series- A. P. with 1st Term (a) = 26 and Common Difference (d) = 3.

Illustration 46.
Form an A.P. with the value of “d” = 6, ‘a’ = – 13. Determine first 5 terms.

Answer:
The following values of “d” and ‘a’ are given,
‘a’ – First Term of the Series = – 13.
‘d’ – Common Difference between any two consecutive terms of the series = 6
The first 5 terms of the Series would be determined as follows:
– 13, (– 13+6), (– 13+6+6), (– 13+6+6+6), (– 13+6+6+6+6)
A. P. Series with first 5 terms: – 13, – 7, – 1, 5, 11

1.3.1.2 Formula for determining the value of nth term (tn) of an A.P.
t n = a + (n − 1) × d
Where,
tn= Value of the nth term
a = First Term of the Series
n = Number of terms in the Series
d = Common Difference between any two consecutive terms of the Series
The above mentioned formula can be taken into consideration for the following:
(i) To find the number of terms of the Series.
(ii) To Find the Common Difference between two consecutive terms of the Series.
(iii) To Find the First Term of the Series.
(iv) To Find the Value of the nth term of the Series.
(v) To Find the Value of the Last Term of the Series, if value of ‘n’ is given.
For the above illustration a = –13 and d = 6
1st term = a = – 13 (given)
Using formula tn = a + (n – 1) d
2nd term = t2 = – 13 + (2 – 1) × 6 = – 13 + 1 × 6 = – 13 + 6 = – 7
3rd term = t3 = – 13 + (3 – 1) × 6 = – 13 + 2 × 6 = – 13 + 12 = – 1
4th term = t4 = – 13 + (4 – 1) × 6 = – 13 + 3 × 6 = – 13 + 18 = 5
5th term = t5 = – 13 + (5 – 1) × 6 = – 13 + 4 × 6 = – 13 + 24 = 11

A. P. Series with first 5 terms:  – 13,  – 7,  – 1, 5, 11.

38 The Institute of Cost Accountants of India


Arithmetic

If the Common Difference is Negative (d < 0), the terms of the series will be in descending order. In other words,
the 1st term will be larger than the 2nd next term and so on.
For example,
Series: 9, 7, 5, 3, 1 has d =  – 2 {5 – 7 =  – 2; 7 – 9 =  – 2}

Illustration 47.
A given series is: a + 14, a + 12, a + 10, a + 8, a + 6 Identify the type of Series. Also calculate the values of ‘a’
and d.
Answer:
Since from the Series it is observed that Subtraction of an constant being made to obtain the consecutive terms, the
type of Series is A. P.
Values of a & d:
‘a’ – First Term of the Series = a +14.
‘d’ – Common Difference between two consecutive terms of the series
‘d’ = 2nd term – 1st term = (a +12) – (a+14) = a+12–a–14 = 12–14 = – 2
So Type of Series- A. P. with 1st Term (a) = a +14 and Common Difference (d) = –2.

1.3.1.3 Formula for Calculating the Sum of the terms (Sn) for any A.P.
n
Sn = × {2a + (n − 1) × d }
2
Where
Sn = Sum upto n terms of the series
Number of terms in the series = n
1st Term of the series = a
Common Difference between any two consecutive terms of the series = d

Illustration 48.
Find the sum of the first 10 terms for the series: 12, 15, 18, 21, 24.

Answer:
For calculating the sum of first 10 terms of the given series: 12, 15, 18, 21, 24, the following formula is used.
n
Sn = × {2a + (n − 1) × d }
2
n = Number of terms in the series = 10
a = 1st Term of the series = 12
d = Common Difference between any two consecutive terms of the series = 2nd term – 1st term = 15 – 12 = 3
S10 = (10/2) × {2 × 12 + (10-1) × 3}

The Institute of Cost Accountants of India 39


Fundamentals of Business Mathematics and Statistics

Or, S10 = 5 × {24 + 9 × 3}


Or, S10 = 5 × {24 + 27} = 5 × 51 = 255.

1.3.2 Geometric Progression


For any series with ‘n’ terms, if the ratio between two consecutive terms is constant throughout the series, the
series is termed as Geometric Progression (G. P.). The Common Ratio between two consecutive terms can be either
positive or negative.
Common Ratio between two consecutive terms is represented by ‘r’.

1.3.2.1 Formula for calculating the value of Common Ratio (r) of a G.P.
2nd Term Any Term
We have r = Or
st
1 Term Previous Term

The value of r is constant throughout the Series. In other words, ‘r’ calculated from any two consecutive terms will
be throughout.
For example,
r1 = 6th term / 5th term
r2 = 9th term / 8th term
Both, r1 and r2 will be equal for any given G. P.
If the Common Ratio is greater than 1 the terms of the series will be in ascending order.
For example,
Series: 1, 3, 9, 27, 81 has ‘r’ = 3 {9/3 = 3; 3/1 = 3}

Illustration 49.
A series is given as: 7, 14, 28, 56, 112 Identify the type of Series. Also calculate the values of ‘a’ and ‘r’.

Answer:

Since from the Series it is observed that there exist a Common Ratio between the consecutive terms, the type of
Series is G. P.

Values of ‘a’ and ‘r’:

‘a’ – First Term of the Series = 7.

‘r’ – Common Ratio between any two consecutive terms of the series

‘r’ = 2nd term / 1st term = 14 / 7 = 2

So the type of Series is G. P. with 1st Term (a) = 7 and Common Ratio (r) = 2.

Illustration 50.
For the given series: 1875,  – 375, 75,  – 15, 3, find out the values of ‘a’ and ‘r’. Also identify the type of Series.

40 The Institute of Cost Accountants of India


Arithmetic

Answer:
Since the Series has a Common Ratio beween the consecutive terms, the type of Series is G. P.
Values of ‘a’ and ‘r’:
‘a’ – First Term of the Series = 1875.
‘r’ – Common Ratio between any two consecutive terms of the series
‘r’ = 2nd term / 1st term =  – 375 / 1875 =  – 1/5
Type of Series- G. P. with 1st Term (a) = 1875 and Common Ratio (r) =  – 1/5.

1.3.2.2 Formula for determining the value of nth term (tn) of a G.P
t n = ar n-1

Where,
tn = Value of the nth term
a = First Term of the Series
n = Number of terms in the Series
r = Common Ratio between two consecutive terms of the Series

The above mentioned formula can be taken into consideration for the following:
(i) To find the number of terms of the Series.
(ii) To Find the Common Ratio between two consecutive terms of the Series.
(iii) To Find the First Term of the Series.
(iv) To Find the Value of the nth term of the Series.
(v) To Find the Value of the Last Term of the Series, if value of ‘n’ is given.

Illustration 51.
Determine 9th term for the series: 11, 33, 99, 297, 891.

Answer:
We know that tn = ar n −1
a = First Term of the Series = 11
n = Number of terms in the Series = 9
r = Common Ratio between two consecutive terms of the Series
r = 2nd term / 1st term = 33/11 = 3
tn = a × r (n-1)
Or, t9 = 11 × 3(9-1)
Or, t9 = 11 × 3(8) = 72171

The Institute of Cost Accountants of India 41


Fundamentals of Business Mathematics and Statistics

1.3.2.3 Formula for Calculating the Sum of the terms (Sn) for any G. P.
rn − 1
Sn = a , When r > 1
r −1
1 − rn
Sn = a , When r < 1
1− r
Where
Sn = Sum upto n terms of the series
n = number of terms in the series
a = 1st Term of the series
r = Common Ratio between any two consecutive terms of the series

For a G.P with infinite terms, the cormula for Summation is S = when r < 1

Illustration 52.
Find the sum of the first 9 terms for the series: 5, 10, 20, 40, 80.

Answer:
n = number of terms in the series = 9
a = 1st Term of the series = 5
r = Common Ratio between any two consecutive terms of the series
r = 2nd term / 1st term = 10/5 = 2 > 1

We know that S a Since r > 1

S9 = 5 × {(29  – 1) / (2 – 1)}


= 5 × {(29  – 1) / 1}
= 5 × (512  – 1)
= 5 × 511 = 2555

Illustration 53.
Find the sum of the first 10 terms for the series: 48, 24, 12, 6, 3.

Answer:
n = number of terms in the series = 10
a = 1st Term of the series = 48
r = Common Ratio between any two consecutive terms of the series
r = 2nd term / 1st term = 24/48 = 1/2 < 1

42 The Institute of Cost Accountants of India


Arithmetic

We know that S a Since r < 1

S10 = 48 × {1 – (1/2)10} / (1 – 1/2)


= 48 × {1 – (1/2)10 }/ (1/2)
= 48 × {1 – (1/2)10}× 2
= 96 × {1 – (1/2)10}
= 96 × (1 – 0.00098)
= 96 × 0.99902 = 95.9059

EXERCISE WITH Hints:


1 1
1. If mth term of an AP is and nth term is , and m ≠ n then show that the first term is equal to the common
n m
difference

Hints:
Let a denote the first term and d the common difference. So by question we can write
1 1
tm = = a + (m − 1) d & tn = = a + (n − 1) d [a = 1st term and d = Common difference]
n m
1 1
Now − = (m − 1) d − (n − 1) d
n m
m−n
Or, = md − d − nd + d
mn
m−n
Or, = (m − n) d
mn
1
Or, d =
mn
1 1 1 1 m 1 1
Now from the expression for tm we can write a = − (m − 1) d = − (m − 1) = − + = = d (Proved)
n n mn n mn mn mn
2. If a′ = 1 + c a + c 2a + c 3a +  ∞ (0 < c < 1)
b 2b 3b
And b′ = 1 + c + c + c +  ∞ (0 < c < 1)
1 1
(a′ − 1) a (b′ − 1) b
Then prove that c = =
a′ b′
Hints:
1
By question we can write a′ = 1 + c a + c 2a + c 3a +  ∞ = [from sum of infinite GP series]
1 − ca
b 2b 3b
Similarly b′ = + c + c + c +  ∞ =
1

( )
1
1 a  1 − 1 + ca  a
1
−1  1 − ca  1
(a′ − 1) 1a
− ca   (c a ) a
Now = = = c
=C
a′ 1 1 1
1 − ca 1− c a

The Institute of Cost Accountants of India 43


Fundamentals of Business Mathematics and Statistics

Similarly (b′ − 1) = c
b

b′
3. If pth, qth, rth terms of an AP are a, b, c respectively. Show that (q – r) a + (r – p) b + (p – q) c = 0

Hints:
Let 1st term = A and Common difference = d
By question we can write
a = A + (p – 1) d … … … (i)
b = A + (q – 1) d … … … (i)
c = A + (r – 1) d … … … (i)
By (i) × (q – r), (ii) × (r – p) & (iii) × (p – q) and subsequently adding we get

a (q – r) + b (r – p) + c (p – q)

= A (q – r + r – p + p – q)

+ d [(p – 1) (q – r) + (q – 1) (r – p) + (r – 1) (p – q)]

= 0 + d [pq – q – pr + r + qr – r – pq + p + pr – p – qr + q]

=0+0=0
a a b b c c
4. If x = a + + +  .. ∞ , y = b − + 2 −  .. ∞ , & z = c + 2 + 4 −  .. ∞
r r2 r r r r
xy ab
Show that =
z c
Hints:
By applying sum of infinite G.P series formula
ara
By question x = =
1 r −1
1−
r
br cr 2
Similarly y = &z= 2
r +1 r −1
a br 2
xy r 2 − 1 ab
Now = =
z cr 2 c
r2 −1

5. The rate of monthly salary for a person is increased annually in AP. It is known that he was drawing `400 a
month during the 11th year of his service and ` 760 during the 29th year. Find his starting salary and the rate of
annual increment. What should be his salary at the time of retirement just on the completion of 36th years of
service?
Hints:
Let the salary per month in the 1st year is x and in every year it increased by an amount d.

44 The Institute of Cost Accountants of India


Arithmetic

So by question we can write:


t11 = x + (11 − 1) d = 400 … … … … (1)
t29 = x + (29 − 1) d = 760 … … … … (2)
So from (1) & (2) above we can write
x + 10d = 400 … … … .. (3)
and
x + 28d = 760 … … … (4)
By (4) –(3) we get 18d = 360.So d = ` 20.
Putting this value of d in equation (3) we get x = 400 – 10 × 20 = ` 200

We have to find out t36 = x + (36 − 1) d = 200 + 35 × 20 = ` 900

6. A man secures an interest free loan of `14,500 from a friend and agrees to repay it in 10 instalments. He pays
`1,000 as first instalment and then increases each instalment by equal amount over the preceding instalment.
What will be his last instalment?
Hints:
Let the first instalment is A and equal increased amount is d
So payment of instalments will form a series like
A + ( A + d ) + ( A + 2d ) +  10 terms = 14500
Now by question A = 1000

We can write S10 = 14500 = 10 [2 × 1000 + (10 − 1) d ]


2
Or, 2900 = [2000 + 9d]
Or, 2900 – 2000 = 9d
So, d = 100
We have to find out the last instalment. That is we have to find the last term of the series.
So we can write t10 = A + 9d = 1000 + 9 × 100 = ` 1900

7. Find the sum of the first 15 terms of a GP given by 1,  – 1/2, 1/4,  – 1/8…………………..
Hints:
−1/ 2 1/ 4 −1/ 8 1
Here a = 1 , r = = = = − , n = 15
1 −1/ 2 1/ 4 2

a (1 − r n ) 1[1 − ( −1 / 2 )15 ] (215 + 1)15 215 + 1 10923


So Sn = = = = =
1− r 1 − ( −1 / 2 ) 1+1/ 2 3 × 214 16384

The Institute of Cost Accountants of India 45


Fundamentals of Business Mathematics and Statistics

8. Evaluate the recurring decimal 0.19

Hints:
Now 0.19 = 0.1+ 0.09 + 0.009 + 0.0009 +…………

= 1 + 9 2 + 9 3 + 9 4 +
10 10 10 10

1 9  1 1
= + × 1+ + ... ... ..
10 102  10 102 
 
1 9  1  a
= + (Since the sum of the terms of an infinite G.P = , when r < 1)
10 102  1 1− r
1 −
 10 
1 9 10 2 1
= + × = =
10 102 9 10 5

9. If a, b, c, d are in GP then
(a) a 2 + b 2 , b 2 + c 2 , c 2 + d 2 are in GP
(b) a 2 − b 2 , b 2 − c 2 , c 2 − d 2 are in GP
(c) a 2b 2 , b 2c 2 , c 2d 2 are in GP
a 2 b2 c 2
(d) , , are in GP
b2 c 2 d 2

Hints:
We have b = ak, c = bk, d = ck. i.e. b = ak, c = ak2 & d = ak3. Hence the answer is (b)

10. MR X arranges to pay off a debt of `9600 in 48 annual instalments which form an AP. When 40 of these
instalments are paid, MR X becomes insolvent with Rs.2400 still remains unpaid. Then value of second
instalment is
(a) ` 97.50
(b) ` 92.50
(c) ` 82.50
(d) ` 87.50

Hints:
48 40
We can write 9600 = [2a + (48 − 1) d ] & (9600 − 2400) = [2a + (40 − 1) d ] & hence the answer is (d)
2 2
11. An enterprise produced 600 units in the 3rd year of its existence and 700 units in its 7th year. If production is
maintained as an AP then production in the first year is
(a) 580

46 The Institute of Cost Accountants of India


Arithmetic

(b) 575
(c) 550
(d) 590

Hints:
By question 600 = a + 2d & 700 = a + 6d and hence the answer is (c)

12. The sum of first 6 terms of a GP is 9 times the sum of first three terms. The common ratio is
(a) 3
(b) 1
3
(c) 1
(d) 2

Hints:

a (r 6 − 1) a (r 3 − 1)
=9 . Hence the answer is (d)
r −1 r −1

13. Find sum of all natural numbers from 100 to 300 which are divisible by 4
(a) 10900
(b) 10200
(c) 10000
(d) 11000

Hints:
Series is 100 + 104 + 108 +………. 300. So 300 = 100 + (n –1) 4, Hence the answer is (b)

1 39
14. If you insert 9 arithmetic means between and (−) , then 1st arithmetic mean is
4 4
1
(a) − 5
1
(b) −
8
(c) 0
3
(d) −
4

Hints:
1 −39 39 1
9 arithmetic means are inserted between & 4 i.e. we have 11 terms on the whole. So − = + (11 − 1) d .
4 4 4
Hence the answer is (d)

The Institute of Cost Accountants of India 47


Fundamentals of Business Mathematics and Statistics

15. A man saved ` 16500 in 10 years. In each year after the first he saved ` 100 more than he did in the preceding
year. In the first year his savings was
(a) ` 1000
(b) ` 1000
(c) ` 1200
(d) ` 800

Hints:
10
16500 = [2 × a + 9 × 100], Hence the answer is (c)
2

16. A moneylender lends `1000 and charges an overall interest of `140. He recovers the loan and interest by 12
monthly instalments each less by `10 than the preceding one. The amount of the first instalment is
(a) ` 175
(b) ` 300
(c) ` 150
(d) ` 400

Hints:
12
1000 + 140 = [2 × a + 11 × (−)10] , Find ‘a’ to get the answer (c)
2

17. Sum of the series r 2 − 2r + 2 , r 2 − 2r + 3 , r 2 − 2r + 4 up to (2r – 1) terms is

(a) 4r 3 + 2r 2 + r + 3

(b) r 3 + 5r 2 − 7r − 1
(c) 1
(d) 2r 3 − 3r 2 + 3r − 1

Hints:
The given series is AP with c.d. = 1 and hence the answer is (d)

18. If (1 + 1 + 3 + 5 + up to k terms) − tk = 0 , then tk is equal to


(a) k 2 − 2k + 2
(b) k 2
(c) 2k 2 − 5k + 3
(d) k

48 The Institute of Cost Accountants of India


Arithmetic

Hints:

(1 + 1 + 3 + 5 + upto k terms) – tk = 0. So tk = (1 + 1 + 3 + 5 + upto k terms) Or, tk = 1+ [1 + 3 + 5 + upto (k – 1)


terms] and the bracketed portion is an A.P. Hence the answer is (a)

3 4
19. The sum of , , 5 ... ... ... to 25 terms is
5 5
1
(a)
5
(b) 5
15
(c)
5
75
(d)
5

Hints:
1
AP series where c.d. is and hence the answer is (d)
5

The Institute of Cost Accountants of India 49


Time and Distance 1.4
Introduction:
Time and Distance have a mathematical relationship with Speed. There exists a dependence of speed, time and
distance on each other.
For example,
In case, the distance is specified in km, the time taken to cover would be in hrs then the unit of speed would be
given in km/hr. While, on the other hand, if the distance is specified in m and the time taken to cover is given in
sec, the unit of speed would be m/sec.

Relation between Time and Distance with Speed


1. Distance = Speed × Time
Unit of Distance is denoted as km or m.
Distance
2. Time =
Speed

Unit of Time is denoted as hr, min or sec.


Distance
3. Speed =
Time

Unit of Speed is denoted as km/hr or m/sec.

50 The Institute of Cost Accountants of India


Arithmetic

Exercise:
Theoretical Questions
 Multiple Choice Questions (MCQ)
[Ratio, Variation and Proportion]
1. If Q: W = 2:3, W:T = 4:5 and T: Y = 6:7, find Q: Y.
(a) 35: 51
(b) 16: 35
(c) 16: 51
(d) 35: 51
2. Two numbers are in the ratio 7: 9, if the sum of the numbers is 288, then the smaller number is
(a) 126
(b) 288
(c) 162
(d) 144
3. When the number 1580 is increased in the ratio 5: 9, the new number is
(a) 788
(b) 2488
(c) 878
(d) 2844
4. The ratio of the number of faces to the number of edges of a box is
(a) 3: 8
(b) 8: 3
(c) 1: 2
(d) 2: 1
5. Find the fourth proportional to 6, 8, 9 is
(a) 18
(b) 12
(c) 7
(d) 13
6. Find the compounded ratio of: 15: 8 and 18: 5
(a) 27: 4
(b) 4: 27
(c) 27: 8
(d) 8: 27

The Institute of Cost Accountants of India 51


Fundamentals of Business Mathematics and Statistics

1
7. Find the third proportional to 6 and 5
4
(a) 9 1
2
(b) 3 1
5
(c) 4
(d) 8

8. Find the Duplicate ratio of : 8a : 4b

(a) 64a 2 : 2b 2

(b) 16a 2 : b 2

(c) 32a 2 : 4b 2

(d) 32a 2 : 8b 2

1 1
9. If A ∝ and A = 7 when B = 3, then when B = 2 , A is
B 3

3
(a)
7
(b) 3
(c) 9
(d) 1

A B X Y
10. If AB ∝ XY and + = + then
B A Y X

(a) A2 + B 2 ∝ X 2 + Y 2
(b) A + B ∝ XY
(c) AB ∝ X + Y

(d) A + B ∝ 1
X +Y
341 y 3 + 3 y
11. Using the properties of proportion, solve the following equation for y, given =
91 3 y 2 + 1
(a) 14
(b) 11
(c) 12
(d) 10

52 The Institute of Cost Accountants of India


Arithmetic

12. The ratio of 45 seconds to 5 3 minutes is


4
(a) 5: 21
(b) 3:23
(c) 2: 19
(d) 1: 27
13. A class consists of 48 male students and 23 female students. Find the ratio of female students to total
strength of the class.
(a) 23: 48
(b) 48: 23
(c) 48: 71
(d) 23: 71
1
14. The mean proportional between 80 and 5 is
(a) 16
(b) 4
(c) 8
(d) 32
15. Find the Triplicate Ratio of: 1 : 1
3 2
(a) 8: 27
(b) 27: 8
(c) 4: 9
(d) 9: 4
16. If 4, 6, p, 27, q are in continued proportion, find the values of p and q.
(a) p = 9, q = 9
(b) p = 9, q = 81
(c) p = 81, q = 9
(d) p = 81, q = 81
17. If a Stadium having spectators divided into groups of Young Generation and Old Generation and the
number of young spectators are 1525 and old spectators are 1875. Find the ratio of Total Spectators to
Young Spectators.
(a) 136:75
(b) 136:85
(c) 75:61
(d) 136:61

The Institute of Cost Accountants of India 53


Fundamentals of Business Mathematics and Statistics

18. Find two numbers whose mean proportional is 8 and the 1st number is square of the 2nd number.
(a) 8, 8
(b) 4, 2
(c) 16, 4
(d) 64, 8
19. What number must be subtracted from each of the numbers 12, 13, 24 and 29 to make them proportional?
(a) 9
(b) 8
(c) 35
(d) 36
20. The ratio of the pocket money saved by Rakesh and his sister is 5:6. If the sister saves `60 more, how much
more the brother should save in order to keep the ratio of their savings unchanged?
(a) ` 60
(b) ` 40
(c) ` 50
(d) ` 70

[Time value of Money and Annuity – Simple and Compound Interest]

21. AMS & Co. borrows `1,20,000 for one year at 15% annual interest, compounded monthly. Find their
monthly payment.
(a) ` 11,380
(b) ` 10,830
(c) ` 11,430
(d) ` 10,740

22. A certain sum of money invested at a certain rate of compound interest doubles in 8 years. In how many
years will it become 16 times?
(a) 31 years
(b) 28 years
(c) 30 years
(d) 32 years

23. Calculate the present value of an annuity of `3,000 received at the beginning of each year for 5 years at a
discount factor of 6%
(a) ` 13,998
(b) ` 13,398

54 The Institute of Cost Accountants of India


Arithmetic

(c) ` 14,398
(d) ` 14,898
24. AB LLP is expecting to receive a certain sum of money five years from now. If the present value of this
sum is `38,400 at current market interest rate of 6% when the interest is compounded monthly, then how
much amount they will receive after 5 years?
(a) ` 52,684
(b) ` 52,884
(c) ` 51,904
(d) ` 51,794
25. What sum will amount to `5480 in 6 years at 10% p.a. compound interest payable half-yearly?
(a) ` 3,051
(b) ` 2,051
(c) ` 3,501
(d) ` 2,501
26. Mr. Singh deposits `2,000 at the beginning of each year for 5 years. How much do these accumulate at the
end of 5th year at an interest rate of 6%?
(a) ` 10,272
(b) ` 11,591
(c) ` 10,727
(d) ` 11,951
27. A buys a shop for `48,750 down payment and `60,000 after one year. If the money is worth 10% per year
compounded half-yearly. Find the purchase price of the shop.
(a) ` 94,280
(b) ` 1,03,170
(c) ` 45,530
(d) ` 54,420
28. A loan of `20,000 has been issued for 5 years. Compute the amount to be repaid to the lender if simple
interest is charged @ 8% per year.
(a) ` 28,000
(b) ` 25,000
(c) ` 27,000
(d) ` 24,000

The Institute of Cost Accountants of India 55


Fundamentals of Business Mathematics and Statistics

29. A sum of `4895 borrowed from a money lender at 5% p.a. compounded annually. Find the annual
instalment, if amount borrowed is to be paid back in three equal annual instalments.
(a) ` 1,597
(b) ` 1,997
(c) ` 1,397
(d) ` 1,797
30. AMS & Co. makes a monthly payment for `11,350 for one year at 11% annual interest, compounded
monthly. Find the amount borrowed by them.
(a) 1,31,889
(b) 1,38,284
(c) 1,21,542
(d) 1,22,802
31. Calculate the present value of an annuity of `5,000 received annually for 4 years at a discount rate of 5%.
(a) ` 17,720
(b) ` 12,250
(c) ` 22,250
(d) ` 27,720
32. Find the Principal, if the compound value is `9,750 at the interest rate of 6.25% p.a. compounded annually
for 16 years.
(a) ` 4,287
(b) ` 3,857
(c) ` 3,696
(d) ` 4,857
33. Find the amount and the compound interest of `9, 350 at the rate of 8% p.a. compounded half-yearly for
four years.
(a) ` 12,795 and ` 3,445
(b) ` 12,720 and ` 3,370
(c) ` 12,758 and ` 3,408
(d) ` 12,835 and ` 3,485
34. When the effective rate of interest is 7.82% payable quarterly, what would be the nominal rate of interest?
(a) 8.00%
(b) 7.60%
(c) 7.00%
(d) 8.20%

56 The Institute of Cost Accountants of India


Arithmetic

35. Find the present value of perpetuity of `2,725 at 0.50% per month compound interest.
(a) ` 45,167
(b) ` 45,417
(c) ` 44,517
(d) ` 44,157
36. A sum of money double itself at 7% p.a. compound interest in
(a) 10.27 years
(b) 10.00 years
(c) 12.07 years
(d) 12.00 years
37. A Laptop depreciated in value each year at 15% of its previous value and at the end of fourth year, the
value was `36,331. Find its original value.
(a) ` 69,600
(b) ` 69,200
(c) ` 69,300
(d) ` 69,500
38. A wants to create an endowment fund of `51,750 to provide for a prize of every year. If the fund can be
invested at 11% p.a. compound interest, find the amount of prize.
(a) ` 5,933
(b) ` 5,963
(c) ` 5,993
(d) ` 5,693
39. Accumulated series of deposits as future sum money is classified as –
(a) Annuity Fund
(b) Sinking Fund
(c) Marginal Fund
(d) Nominal Fund
40. The compound interest on `27,500 at 8% p.a. is `7,142. Calculate the required time.
(a) 3 years
(b) 2.5 years
(c) 3.5 years
(d) 4 years

The Institute of Cost Accountants of India 57


Fundamentals of Business Mathematics and Statistics

[Arithmetic Progression and Geometric Progression]


Solve Q. Nos. 41 to 50 using the following information.
For the series 5, 12, 19, 26, 33, 40 .........

41. Identify the type of given series.


(a) A.P.
(b) G.P.
(c) H.P.
(d) None of the Above
42. Find the value of the 12th term of the series.
(a) 89
(b) 92
(c) 82
(d) 75
43. Which term of the series would be 68?
(a) 10
(b) 9
(c) 12
(d) 8
44. What would be the value of the 9th term?
(a) 66
(b) 68
(c) 59
(d) 61
45. What would be the sum of the first 11 terms of the series?
(a) 419
(b) 440
(c) 420
(d) 426
46. Which term of the series would be 82?
(a) 12
(b) 11
(c) 10
(d) 14

58 The Institute of Cost Accountants of India


Arithmetic

47. If the sum of the series is 297, how many terms are there in the series?
(a) 8
(b) 9
(c) 10
(d) 7
48. What is the value of ‘a’?
(a) 40
(b) 12
(c) 5
(d) 7
49. Find the value of ‘d’?
(a) 7
(b) 5
(c) -7
(d) -5
50. Value of‘d’ is ____?
(a) Zero
(b) Negative
(c) Positive
(d) Undefined
51. Find the next 5 terms for the series: 2, -4, 8, -16, 32
(a) -64, 128, -256, 512, -1024
(b) 64, -128, 256, -512, 1024
(c) 64, 128, 256, 512, 1024
(d) -64, -128, -256, -512, -1024
52. Find the value of ‘r’ for the series: 729, 243, 81, 27, and 9
(a) 3
(b) -3
(c) 1/9
(d) 1/3
53. For any series having 50 terms forming A.P. with first term equal to 25, what will be the value of ‘n’?
(a) 50
(b) 25

The Institute of Cost Accountants of India 59


Fundamentals of Business Mathematics and Statistics

(c) 75
(d) 100
54. Find the next 6 terms for the series: 128, 139, 150, 161, 172, 183.
(a) 194, 205, 216, 228, 240, 252
(b) 194, 205, 217, 229, 242, 256
(c) 194, 205, 218, 231, 245, 259
(d) 194, 205, 216, 227, 238, 249
55. For the given series: 66, 71, 76, 81, 86, 91, 96 ….666, 671, 676.
With ‘n’ terms in the series, what will be the value of ‘d’ ?
(a) -5
(b) 15
(c) -3
(d) 5
56. Identify the type of series and the value of ‘a’: -85, -83, -81, -79, -77
(a) A.P. , a = -85
(b) A.P. , a = -2
(c) G.P. , a = -2
(d) G.P. , a = -85
57. For any G.P. starting with 28, if each term is obtained by dividing the preceding one by 2, what would be
value of ‘r’?
(a) 2
(b) -2
(c) 1/2
(d) -1/2
58. If the First term is 748, Last Term is 28, and the value between two consecutive terms is 9 deducted, find
the number of terms in the series.
(a) 9
(b) 8
(c) 10
(d) None of the above
59. If the Last term is 187, First Term is 371, value between two consecutive terms is 8 deducted, find the
number of terms in the series.
(a) 25
(b) 22

60 The Institute of Cost Accountants of India


Arithmetic

(c) 23
(d) 24
60. Identify the type of series: 1+2+3+4+5
(a) H.P.
(b) G.P.
(c) Either of ‘a’ or ‘b’
(d) A.P.

[Time and Distance]

61. When a Bike had travelled for 78 km in 3 hours 45 minutes in the evening, how much distance would the
Bike travelled in 2 hours?
(a) 41.60 km
(b) 48.88 km
(c) 52 km
(d) 55 km
62. A cycle travels a distance of 300 m in every second. What is the distance covered in an hour by the cycle?\
(a) 3000 km
(b) 1080 km
(c) 1800 km
(d) 2160 km
63. A Car takes 4 hours to travel 90 km. If time is reduced by 15 minutes, how much distance would be covered
in the given time?
(a) 84.375 km
(b) 80 km
(c) 95.625 km
(d) 90 km
64. A Train takes 35 hours to reach Punjab from Kolkata (1940 km) and takes 42 hours to reach Kolkata from
Gujarat (2160 km). But it took 25 hours from Punjab to Gujarat (1420 km). How many days it take for a
trip from Kolkata-Punjab-Gujarat-Kolkata and what is the distance covered?
(a) 4 days 6 hours and 5520 km
(b) 3 days 18 hours and 5520 km
(c) 4 days and 2680 km
(d) 5 days and 2680 km

The Institute of Cost Accountants of India 61


Fundamentals of Business Mathematics and Statistics

65. If Bike had travelled at the speed of 40km/hr for 2 hours and scooty had travelled at the speed of 25km/hr
for 3 hours, which one of them covered more distance and how much more was covered?
(a) Scooty, 30km
(b) Bike, 80 km
(c) Bike, 5 km
(d) Scooty, 75km
66. If Bike covers 165 km in 3 hours and scooty covers 100 km in 2 hours, in order to cover 1025 km, how
much time would be required by bike and scooty?
(a) 12.42 hours and 12.42 hours
(b) 21.50 hours and 19.63 hours
(c) 19.52 hours and 19.52 hours
(d) 18.63 hours and 20.50 hours
67. y is the yardstick to measure the performance of two vehicles, where y = Speed × Time × Distance. If Time
taken by one of the vehicle (1st Vehicle) is increased by 10%, what would be the impact on the yardstick?
(a) No change
(b) 1st vehicle would be better than 2nd Vehicle
(c) 2nd Vehicle would be better than 1st Vehicle
(d) None of the Above
68. If a student is travelling at a speed of 45 km/hr, how long should it take to travel 90 km?
(a) 2 hours
(b) 3 hours
(c) 100 minutes
(d) 90 minutes
69. With 5% increase in Distance and 2.50% increase in Time, what would be the impact on Speed?
(a) Decrease by 2.44%
(b) Increase by 2.44%
(c) Increase by 2.38%
(d) Decrease by 2.38%
70. If a Car covered 200 km at a speed of 25kmph and a Bike covered 150 km at a speed of 30kmph, which
one of them took more time to cover the distance and what was the time taken?
(a) Car, 3hrs
(b) Bike, 8 hrs
(c) Car, 8 hrs
(d) Bike, 5 hrs

62 The Institute of Cost Accountants of India


Arithmetic

71. If Toto had travelled at the speed of 15km/hr for 11 hours and Car had travelled at the speed of 45km/hr
for 3.5 hours, which one of them covered shorter distance and how much short ?
(a) Car, 7.50 km
(b) Toto, 7.50 km
(c) Car, 15 km
(d) Toto, 15 km

72. If ratio between Speed(km/hr.) and Time(Hrs) is 7: 5, what will be the distance(km)?

(a) 30 km

(b) 35 km

(c) 36 km

(d) 42 km

73. The distance between two terminal stations of Metro is 240 km. A metro rail takes 2 hours to cover the
distance. Calculate the total distance covered in 5 days, if 10 trips to and fro takes place between the two
stations in a day?

(a) 6,000 km

(b) 12,000 km

(c) 24,000 km

(d) 18,000 km

74. If A takes 3 hours to cover a distance of 60 km, B takes 2 hours to cover a distance of 50 km, how much
time would A take more than B to cover 300 km?

(a) 6 hours

(b) 12 hours

(c) 15 hours

(d) 3 hours

75. If Toto covered 240 km at a speed of 40kmph while Scooty covered 180 km at a speed of 60 kmph, which
one of them took less time to cover the distance and what was the time taken ?

(a) Scooty, 3 hrs.

(b) Toto, 6 hrs

(c) Toto, 3 hrs.

(d) Scooty, 6 hrs.

The Institute of Cost Accountants of India 63


Fundamentals of Business Mathematics and Statistics

76. With 10% decrease in Distance and 5% increase in Time, what would be the impact on Speed?

(a) Decrease by 16.67%

(b) Increase by 16.67%

(c) Increase by 14.28%

(d) Decrease by 14.28%

77. The distance between two terminal metro stations A & B is 300 km. A metro rail takes 1.50 hours to cover
the distance. Calculate the total time worked by the driver (in terms of days) in a week, if 6 trips from
Station A to Station B takes place in a day?

(a) 3.50 days

(b) 5.25 days

(c) 2.50 days

(d) 4.25 days

78. y is the yardstick to measure the performance of two vehicles, where y = Speed × Time × Distance. If
Distance travelled by one of the vehicle (2nd Vehicle) is increased by 2%, what would be the impact on
the yardstick?

(a) 1st Vehicle would travel 4% more distance

(b) 2nd Vehicle would travel 4% more distance

(c) No change in the distance travelled by any of the vehicle

(d) None of the above

79. If a Toto travels for 6 hours on a working day to cover 1.5 times the distance travelled on holiday and on a
holiday it takes 4 hours to cover 110 km. than for a span of 7 days: (5 Working and 2 Holidays), how much
distance is covered?

(a) 1054 km

(b) 1045 km

(c) 880 km

(d) 990 km

80. If ratio between Time (seconds) and Speed (m/s) is 12:13, what will be the distance (m)?

(a) 156 m
(b) 1.56 km
(c) 0.0156 m
(d) 15.6 km

64 The Institute of Cost Accountants of India


Arithmetic

 State True or False

1. Mean Proportional of p and q is same as their Mean.

2. If 3x + 4y : 5x – 3y = 5 : 3 then x : y = 27 : 16

3. The ratio of two numbers is 12:5. If antecedent is 45 then the consequent is 108

4. If the ratio of two positive numbers is 4:5 and their L.C.M is 140 then the number are 35, 45

5. The compound ratio of sub- duplicate ratio and sub-triplicate ratio of 729:64 is 81:8

6. The ratio of two numbers is 11:15. The sum of 3 times the first number and twice the second numbers is
630. The H.C.F of the numbers is 10

7. The mean proportional of 4x and 16x3 is 12x2

8. The third proportional of 1 hr 20 minutes, 1hr 40 minutes is 2 hrs.

9. The fourth proportional of `5, `3.50, and 150 gm. is 125 gm.

10. If A: B = B: C = C: D = 5:6 then A: B: C: D = 125:150:180:216

11. If the first and third numbers of four positive number is continued proportion be 3 and 12 respectively then
fourth number is 36

12. A sum of money amounts to `720 in 2 years and `789 in 3 years. The rate of simple interest is 12%

13. The S.I at x % for x years will be `X on a sum of x

14. The S.I on a sum of money at 8% p.a for 6years is half the sum. The sum is `640

15. If the interest on `1,200 more than the interest on `1,000 by `50 in 3 years then the rate of Interest is 8 13 %

16. The rate of S.I p.a when a sum of money grows to one and a half times itself in 8 years is 6 14 %

17. The C.I on a certain sum of money for 1year at 8% p.a compounded quarterly is `824 then the sum is
`10,033

18. In 3 years `1,600 amount to `1,936 at 10% p.a C.I

19. The C.I on `5,000 for 2 years at a certain rate of interest p.a amounts to `1,050 then the rate of interest is
10%

20. The difference between S.I and C.I on `1,000 for 1 year at 4% payable quarterly is Re. 0.40

The Institute of Cost Accountants of India 65


Fundamentals of Business Mathematics and Statistics

 Fill in the Blanks

1. `2530 is distributed between Ram and Hari such that Ram gets 11/12 part of that Hari gets. Then Hari
gets______

2. Some amount of money is distributed among Rama, Mitra and Shipra such that twice the money that Rama
gets = thrice the amount of money that Mitra gets = four times the amount of money that Shipra gets. Then
the continued ratio of their money is ______

3. In a map 2cm denotes a distance of 3 km., then the scale in the map is_________

4. The ratio of two numbers is 2:3. If 6 is subtracted from the second number then the number which is
subtracted from the first number so that the new ratio becomes the same as that of the previous, is ______

5. The sub-duplicate ratio of 49:81 is ________

6. ( )( )
1 1 1 1
+ : × : ______
2 3 2 3
7. The compound ratio of 1.2:2.5, 2.1:3.2 and 5:3 is: ________

8. If A: B = 3:4, B: C = 2:5, then A: B: C: __________

9. Two numbers are in the ratio is 5:8 and if 6 be subtracted from each of them then the reminders are in the
ratio 1:2, then the numbers are______

10. If 3, x, 27 are in continued proportion then x =_________

11. For the ratio 8:27, the Triplicate Ratio of its Subtriplicate Ratio is ______

12. A certain sum of money at S.I amounts to `500 in 3 years and to `600 in 5 years then the principal is ____

13. In ___ time the Simple interest on `900 at 6% will be equal to S.I on `540 for 8 years at 5%

14. Due to fall in the rate of simple interest from 12% to 10½% p.a money lender’s yearly income diminished
by `90. The capital is _____

15. A sum was put at S.I at a certain rate for 2 years. Had it been put at 2% higher rate, it would have fetched
`100 more. The sum is _____

16. At ____ percent rate, the Simple interest will be equal to the principal amount in 10 years.

17. I lent `4,000 for 9 months at 12% p.a. If C.I is reckoned quarterly what will I get after 9 months is _____

18. The C.I on `5,000 for 3 years, when the rate of interest is 5%, 6% and 7% for the 1st, 2nd and 3rd years
respectively, is ____

19. The difference between S.I and C.I of `4,000 for 2 ½ years @ 10% p.a is _____

20. The sum of money will amount to `6,050 in 2 years at 10% p.a. C.I is _____

21. The C.I on a certain sum of money for 2 years at 8% p.a. compounded annually is `1,040. The sum is ____

66 The Institute of Cost Accountants of India


Arithmetic

Answer:

 Multiple Choice Questions (MCQ)

Q. No. Answer Remarks


1 (b) Q : W = 2:3 , W : T = 4:5 and T : Y = 6:7
2 4 6
× ×
3 5 7
48
105
16 or16 : 35
35
2 (a) Ratio = 7:9
Sum of numbers = 288
Sum of ratios = 7+9 = 16
Smaller number = 288/16×7 = 126
3 (d) 1580 increased in 5 : 9
1580/5×9 = 2844
4 (c) No. of faces : 6
No. of edges : 12
Ratio of no. of faces to edges = 6 : 12 = 1 : 2
5 (b) Let Fourth proportional to 6, 8, 9 be x
6 9 9 × 8 72
= or, x = = = 12
8 x 6 6
6 (a) 15 : 8 and 18 : 5
15 18
×
8 5
3
×9
4
27 or 27:4
4
7 (c) Third proportional to 6 1 and 5 is
1 4
6 : 5 :: 5 : Y
4
25
: 5 :: 5 : Y
4
25
×Y = 5×5
4
Y
=1
4
Y =4

The Institute of Cost Accountants of India 67


Fundamentals of Business Mathematics and Statistics

8 (b)
8a : 4b is the given ratio
(8a) 2 : ( 4b ) 2 = Duplicate Ratio of the given one
= 64a 2 : 4b 2
= 16a 2 : b 2
9 (c) 1 K
A∝ ∴A=
B B
Given A = 7 when B = 3 ∴ 7 = K or, K = 21
3
1 7 21
So when B = 2 = then A = or, A = 9
3 3 7/3
10 (a) AB ∝ XY ∴ AB = K .XY
A B X Y
+ = +
B A Y X

A2 + B 2 X 2 + Y 2 A2 + B 2 X 2 + Y 2
Or, = Or, =
AB XY K .XY XY

Or, A2 + B 2 = K ( X 2 + Y 2 )

∴ A2 + B 2 ∝ X 2 + Y 2
11 (b) 341 y 3 + 3 y
=
91 3 y 2 + 1
341 + 91 ( y + 3 y) + (3 y + 1)
3 2

or, = 3 [By Componendo and Dividendo]


341 − 91 ( y + 3 y) − (3 y 2 + 1)

432 ( y 3 + 3 y + 3 y 2 + 1)
or, =
250 ( y 3 + 3 y − 3 y 2 − 1)

216 ( y + 1)3
or, =
125 ( y − 1)3

or,
( y + 1)3 = 63
( y − 1)3 53
or,
( )
( )
or, 5 × ( y + 1) = 6 × ( y − 1)
or, 5 y + 5 = 6 y − 6
or, 6 y − 5 y = 5 + 6
or, Y = 11

68 The Institute of Cost Accountants of India


Arithmetic

12 (b) 45 Seconds 45 Seconds


45 Seconds : 5 34 Minutes = =
(5 34 × 60) Seconds 345 Seconds
5
=
23
13 (d) Male Students – 48 , Female students – 23,
Total strength of the class : Male students + Female students
Total strength : (48+23) = 71 students.
Ratio of female students to Total strength = 23 : 71
14 (b) 1
Mean Proportional between 80 and is
5
1
80 ×
5
= 16 = 4
15 (a) 1 1
: = Given Ratio
3 2

() ()
3 3
1 1
: = Triplicate Ratio of the given ratio
3 2
1 1
= 27 : 8

1 8
= 27 × 1
8
= 27 or 8:27

16 (b) 4, 6, p, 27, q are in continued proportion


4/6 = 6/p and p/27 = 27/q
or, 4×p = 6×6
or, 4×p = 36
or, p = 36/4 = 9
p/27 = 27/q
or, p×q = 27×27
or, 9×q = 729
or, q = 729/9 = 81
p = 9 and q =81
17 (d) Young Spectators – 1525 , Old Spectators – 1875,
Total strength of the stadium = Young Spectators + Old Spectators
Total strength = (1525+1875) = 3400 spectators.
Ratio of Total Strength to Young Spectators = 3400 : 1525 = 136 : 61

The Institute of Cost Accountants of India 69


Fundamentals of Business Mathematics and Statistics

18 (c) Mean proportional is 8


1st number is square of 2nd number ,
Let two numbers be x 2 and x,
Mean proportional of the numbers = x × x 2

8 = x 3 (Given)
or, 82 = ( x 3 ) 2
or, 64 = x 3
or, 43 = x 3
or, x = 4
or, x 2 = 16
Two numbers are 16 and 4.
19 (a) Number must be subtracted from 12, 13, 24 and 29 to make them proportional be y,
(12–y) : (13–y) : : (24–y) : (29–y)

(12–y)×(29–y) = (24–y)×(13–y)

or, 348 − 12 y − 29 y + y 2 = 312 − 24 y − 13 y + y 2


or, 348 – 41y + y2 = 312 – 37y + y2
or, 41y – 37y = 348 – 312
or, 4y = 36 or, y = 9
20 (c) Let the savings of Rakesh and his sister be 5x and 6x,
And Rakesh would save ` y more,
(5x + y) 5
=
(6 x + 60) 6

or, (5x + y) × 6 = 5 × (6 x + 60)

or, 30 x + 6 y = 30 x + 300

or, 6 y = 300
300
or, y = = 50
6

70 The Institute of Cost Accountants of India


Arithmetic

21 (b) 0.15
V = 120000, r % = 15 and monthly interest rate (r) = 12 = 0.0125
V ×r
Monthly payment (P) =
1 − (1 + r )
−n

120000 × 0.0125
P=
1 − (1 + 0.0125)
−12

1500
P=
1 − 0.8615
1500
P=
0.1385
P = 10830.32 or 10,830 (Approx.)
22 (d) If amount doubles in 8 years,
2P = P (1 + r )
8

2 = (1 + r )
8

Amount would be 16 times,


16P = P(1 + r ) n

16 = (1 + r ) n

24 = (1 + r ) n

Replacing 2 with (1 + r )8 , we get


(1 + r )8×4 = (1 + r )n
(1 + r )32 = (1 + r )n
n = 32 years
23 (b)
P = `3,000, r = 6 = 0.06 , n = 5
100
P × (1 + r )  1 
V= × 1 − n
r  (1 + r ) 

3000 × (1 + 0.06)  1 
V= × 1 − 5
0.06  (1 + 0.06) 

1 
V = 53000 × 1 −
 1.3382 

V = 53000 × [1 − 0.7472]

V = 53000 × 0.2528 = 13,398.40. or 13,398 (Approx.)

The Institute of Cost Accountants of India 71


Fundamentals of Business Mathematics and Statistics

24 (d) P = `38,400, r = 6 = 0.06 , n = 5


100
A
P=
( )
12 × n
r
1+
12
A
38400 =
( )
12 × 5
0.06
1+
12
A
38400 =
(1 + 0.005)60
A
38400 =
1.3488
A = 38400 × 1.3488 = 51,793.92 or 51,794 (Approx.)
25 (a) 10
A = 5480, n = 6, m = 2, r = or 0.10
100
( )
mn
r
A = P × 1+
m

( )
2× 6
0.10
5480 = P × 1 +
2
5480 = P × (1.05)
12

5480 = P × 1.7959
5480
P= = 3051.39 or 3,051 (Approx.)
1.7959
26 (d) P = 2000, r = 10 or 0.06, n = 5
100
P × (1 + r )
[(1 + r ) − 1]
n
A=
r
2000 × (1 + 0.06)
[(1 + 0.06) − 1]
5
A=
0.06
2120
A= [1.3382 − 1]
0.06
A = 35333.33 × 0.3382
A = 11949.73 or 11950 (Approx.)
27 (b)
A (Present Value) = 60000, r = 10 or 0.10
100

( )
−2n
r
P = A 1+
2

( )
−2 ×1
0.10
P = 60,000 × 1 +
2
P = 60,000 × (1.05)
−2

P = 60,000 × 0.9070 = 54, 420


Purchase price of shop = 48750 + 54420 = 103170

72 The Institute of Cost Accountants of India


Arithmetic

28 (a) P = 20,000, n = 5 years, r = 8% or 0.08


A=P+P×n×r
A = 20,000 + 20,000 × 5 × 0.08
A = 20,000 + 8,000
A = 28,000
29 (d) V = 4895, r = 0.05, n = 3
P 
× 1 − (1 + r ) 
−n
V=
r 
P
× 1 − (1 + 0.05) 
−3
4895 =
0.05 
4895 × 0.05 = P × 1 − (1.05) 
−3

244.75 = P × [1– 0.8638]


244.75 = P × 0.1362
244.75
P= = 1796.99 or 1,797 (Approx.)
0.1362
30 (c)
A = 11,305 × 12 = 1,35,660, r % = 11 and monthly interest rate = 0.11 = 0.0092 , n = 1 year
12
12 n
A = P ×  1 + r 
 12 
135660 = P (1 + 0.0092)12
P = 1,21,541.59 = 1,21,542 (approx.)
31 (a)
A = 5,000, r = 5 = 0.05, n = 4
100
A 1
P = [1 − ]
r (1 + r )n
5,000 1
P= [1 − ]
0.05 (1 + 0.05)4
1
P = 100000 [1 − ]
1.2155
P = 100000 [1 – 0.8227]
P = 100000 × 0.1772
P = 17720
32 (c) A = 9750, r = 6.25% p.a. or 0.0625, n = 16
A = P (1+ r)n
Or, 9750 = P (1+ 0.0625)16
Or, 9750 = P × 2.63793
Or, P = 3696.08

The Institute of Cost Accountants of India 73


Fundamentals of Business Mathematics and Statistics

33 (a)
P = `9,350, r = 8 = 0.08, n =4
100

( )
2n
r
A = P 1+
2
A = 9350 (1 + 0.04)
2× 4

A = 9350 (1.04)
8

A = 9350 × 1.3685
A = 12795
CI = A – P = 12795 – 9350 = 3445

( )
34 (b) 1× 4
r
0.0782 = 1 + −1
4

( )
4
r
0.0782 = 1 + −1
4

( )
4
r
0.0782 + 1 = 1 +
4

( )
4
r
1+ = 1.0782
4
Taking logarithms on both sides, we get

( )
4 × log 1 +
r
4
= log 1.0782

( )
log 1 +
r
4
=
0.0327
4

( )
log 1 +
r
4
= log 0.008175

Anti-logarithms on both sides, we get

( )
1+
r
4
= 1.0190

r
= 1.0190 − 1
4
r
= 0.0190
4
r = 0.0190 × 4 = 7.60%
35 (b) P = 2,725, r = 0.50% per month = 0.50 × 12 = 0.06 = 6% p.a.
P
V=
r
2725
V= = 45416.66 or 45,417 (Approx.)
0.06

74 The Institute of Cost Accountants of India


Arithmetic

36 (a) 2P = P (1 + 0.07) n
2 = (1.07) n
log 2 = log (1.07) n
0.3010 = n × 0.0293
0.3010
n= = 10.27 years
0.0293
37 (a)
A = `36,331, r = 15 = 0.15, n = 4
100
A = P (1 − r ) n
36331 = P (1 − 0.15) 4
36331 = P (0.85) 4
36331 = P × 0.5220
36331
P= = 69599.62 or 69,600 (Approx.)
0.5220
38 (d)
V = 51750, r = 11 or 0.11
100
P
V=
r
P
51750 =
0.11
P = 51750 × 0.11 = 5,692.50 or 5,693 (Approx.)
39 (b) Accumulated series of deposits as future sum money is classified as -Sinking Fund
40 (a) A = 27500 + 7142 = 34642, P = 27500, r = 8% p.a. = 0.08
n
A = P × (1 + r )
34642 = 27500 × (1 + 0.08) n
34642
= (1 + 0.08) n
27500
1.2597 = (1.08) n
1.833 = (1.08) n
n = 3 years.
41 (a) Type of Series – A.P.
42 (c) Value of the 12th the term = a + (n – 1) × d = 5 + (12–1)×7
= 5 + 11×7 = 5+77 = 82
43 (a) Term of the series would be 68
Value = a + (n–1)×d
68 = 5 + (n–1)×7 or, 68–5 = (n–1)×7 or, 63 = (n–1)×7
or, 63/7 = (n–1) or, 9 = n–1 or, n = 9+1 = 10

The Institute of Cost Accountants of India 75


Fundamentals of Business Mathematics and Statistics

44 (b) Value of 9th term = a + (n–1) × d = 5 + (9–1) ×7


= 5 + 9×7 = 5 + 63 = 68
45 (b) Sum of first 11 terms of the series
n/2 × {2a + (n–1) × d } = (11/2) × {2×5 + (11–1)×7}
= (11/2) × { 10 + 10×7} = (11/2) × {10+70}
= (11/2) × {80} = 11/2×80 = 11×40 = 440
46 (a) term of the series would be 82
Value = a+(n-1)×d
82 = 5 + (n–1)×7 or, 82–5 = (n–1)×7 or, 77 = (n–1)×7
or, 77/7 = (n–1) or, 11 = n–1 or, n = 11+1 = 12
47 (b) sum of the series is 297
n/2 × {2a + (n–1)×d} = 297
or, n × {2×5 + (n–1)×7} = 297×2
or, n × { 10 + 7n –7} = 594
or, n (7n + 3) = 594
or, 7n2 +3n –594 = 0
or, 7n2 + 66n – 63n –594 = 0
or, n (7n + 66)  – 9(7n + 66) = 0
or, (7n + 66) = 0 & (n –9) = 0
or, n = – 66/7 & n = 9
Since, n cannot be negative, there are 9 terms (n = 9).
48 (c) Value of ‘a’ = 1st term of the Series = 5
49 (a) Value of ‘d’ = 2nd Term – 1st Term = 12 –5 = 7
50 (c) Value of ‘d’ = 2nd Term – 1st Term = 12 –5 = 7
Since 7 > 0, the value of ‘d’ is Positive
51 (a) Next 5 terms for the series : 2,  – 4, 8,  – 16, 32
Since, type of Series is G.P.
‘r’ = 2nd term / 1st term = – 4/2 = – 2
Next 5 terms would be
[32 × ( −2)], [32 × ( −2) 2 ], [32 × ( −2)3 ], [32 × ( −2) 4 ], [32 × ( −2)5 ]

 – 64, 128,  – 256, 512,  – 1024


52 (d) value of ‘r’ for the series : 729, 243, 81, 27, 9
r = 2nd Term / 1st Term = 243/729 = 1/3
53 (a) The series is having 50 terms,
Therefore, value of ‘n’ would be 50

76 The Institute of Cost Accountants of India


Arithmetic

54 (d) Next 6 terms for the series :


128, 139, 150, 161, 172, 183
Since, type of Series – A.P.
‘d’ = 2nd term – 1st term = 139 – 128 = 11
Next 6 terms would be
(183+11) , (183+11+11), (183+11+11+11), (183+11+11+11+11), (183+11+11+11+11+11),
(183+11+11+11+11+11+11)
194, 205, 216, 227, 238, 249
55 (d) Series : 66, 71, 76, 81, 86, 91, 96, …….666, 671, 676
Type of Series – A.P.
‘d’ = 2nd term – 1st term = 71 – 66 = 5
56 (a) Type of series and the Value of ‘a’ :
 – 85,  – 83,  – 81,  – 79,  – 77
Type of series : A.P.
‘a’ : First Term =  – 85
57 (c) G.P. series which is starting with 28, each term divided by 2, what would be value of ‘r’
Since the Common Ratio is 1/2, value of ‘r’ = 1/2
58 (a) If the First term is 748, Last Term is 28, value between two consecutive terms is 9 deducted
tn = a + (n – 1) × d
or, 28 = 748 + (n – 1) × (– 9)
or, 28 – 748 = (n – 1) × (– 9)
or,  – 720 = (n – 1) ×  (– 9)
or,  – 720 / (– 9) = n – 1
or, 8 = n – 1
or, n = 8 + 1 = 9
59 (d) If the Last term is 187, First Term is 371, value between two consecutive terms is obtained
by deducting 8, find the number of terms in the series
tn = a + (n – 1)×d
or, 187 = 371 + (n – 1) × (– 8)
or, 187 – 371 = (n – 1) × (– 8)
or,  – 184 = (n – 1) × (– 8)
or,  – 184 / (– 8) = n – 1
or, 23 = n – 1
or, n = 23+1 = 24

The Institute of Cost Accountants of India 77


Fundamentals of Business Mathematics and Statistics

60 (d) series : 1 + 2 + 3 + 4 + 5


Type of Series – A.P.
61 (a) In the Evening, Bike covered 78 km in 3 hours 45 minutes,
45 minutes in an hour = 45/60 = 0.75
Total time the bike travelled in the evening = 3+0.75 = 3.75 hrs
If 78 km covered in 3.75 hrs, then in 1 hr = 78/3.75 = 20.80 km
If travelled for 2 hours then it would cover = 20.80×2 = 41.60 km
62 (b) Distance = 300 m
Time taken = 1 second for every 300 m
Total time = 1 hour = 60 minutes = 60×60 seconds = 3600 seconds
Total Distance to be covered in an hour
= 300 m × 3600 seconds = 1080000 m
We know, 1000 m = 1 km
So, 1080000 m = 1080 km
63 (a) Ideal Situation :
Distance – 90 km in 4 hours
When time is reduced by 15 minutes (15/60 = 0.25 hr)
Time available = 4 – 0.25 = 3.75 hrs
Distance can be covered = 90/4 × 3.75 = 84.375 km
64 (a) Punjab from Kolkata (1940 km) – 35 hours
Kolkata from Gujarat (2160 km)- 42 hours
Punjab to Gujarat (1420 km)-25 hours
Trip from Kolkata-Punjab-Gujarat-Kolkata:
Total Distance = 1940 + 2160 + 1420 = 5520 km
Total Time taken (in Hrs.) = 35 + 42 + 25=102 hours
= 102/24 (As 1 Day = 24 hours) = 4.25 days or 4 days and 6 hours
65 (c) Bike :
Speed – 40km/hr and Time -2 hrs
Distance (km) = Speed × Time = 40 × 2 = 80 km
Scooty :
Speed – 25km/hr and Time -3 hrs
Distance (km) = Speed × Time = 25×3 = 75 km
Therefore, Bike travelled 5 km more than Scooty (80–75 km).

78 The Institute of Cost Accountants of India


Arithmetic

66 (d) Bike covers 165 km in 3 hours : In an hour = 165/3 = 55 km


Scooty covers 100 km in 2 hours : In an hour = 100/2 = 50 km
In order to cover 1025 km,
Bike = 1025/55 = 18.63 hours
Scooty = 1025/50 = 20.50 hours
67 (a) Yard stick (y) = Speed × Time × Distance
Where, speed = Distance / Time
So, y = Distance / Time × Time × Distance
y = (Distance) 2
Therefore, any change in Time, yardstick would not have any impact. So, no change in the
yardstick due to increase in time.
68 (a) Travelling Speed - 45 kmph
Distance to be covered - 90 km
Speed (kmph) = Distance (km) / Time(hours)
Time = Distance / Speed
Time (hours) = 90 / 45 = 2 hours
69 (b) Let Distance be 100 – Increase by 5% - New Distance = 105
Time be 50 – Increase by 2.5% - New Time = 51.25
Speed = 100/50 = 2,
New Speed = 105/51.25 = 2.0488
Change in Speed = 2.0488–2 = 0.0488
Increase (%) = 0.0488/2×100 = 2.44%
70 (c) For Car :
Distance - 200 km, Speed – 25 kmph
Time = Distance / Speed = 200/25 = 8 hrs
For Bike :
Distance – 150 km, Speed – 30 kmph
Time = Distance / Speed = 150/30 = 5 hrs
Car took (8 – 5) = 3 hours more than Bike.
71 (a) Toto :
Speed – 15km/hr and Time-11 hrs
Distance (km) = Speed × Time = 15×11 = 165 km
Car :
Speed – 45km/hr and Time-3.50 hrs
Distance (km) = Speed × Time = 45×3.50 = 157.50 km
Therefore, Car travelled 7.50 km less than Toto (165-157.50 km).

The Institute of Cost Accountants of India 79


Fundamentals of Business Mathematics and Statistics

72 (b) Speed 7 km/hr, Time – 5hrs


Speed (km/hr) = Distance (km) / Time(hr)
Distance (km) = Speed (km/hr) × Time(hr)
= 7 × 5 = 35 km
73 (c) Distance between two terminal stations of Metro is 240 km.
Total distance covered in 5 days,
if 10 trips to and fro takes place between the two terminal stations
Distance in single trip = 240×2 = 480 km
In a Day, 10 trips (to and fro) = 480×10 = 4800 km
For 5 days, 4800 km × 5 = 24,000 km
74 (d) A takes 3 hours to cover a distance of 60 km,
To cover 300 km, A would take = 3/60×300 = 15 hours
B takes 2 hours to cover a distance of 50 km,
To cover 300 km, B would take = 2/50×300 = 12 hours
A would take 3 hours more than B (15–12 hours)
75 (a) For Toto :
Distance - 240 km, Speed – 40 kmph
Time = Distance / Speed = 240/40 = 6 hrs
For Scooty :
Distance – 180 km, Speed – 60 kmph
Time = Distance / Speed = 180/60 = 3 hrs
Scooty took 6 – 3 = 3 hours less time than Toto.
76 (d) Let Distance be 100 – Decrease by 10% - New Distance = 90
Time be 50 – Increase by 5% - New Time = 52.50
Speed = 100/50 = 2,
New Speed = 90/52.50 = 1.7143
Change in Speed = 1.7143 – 2 = – 0.2857
Decrease (%) = – 0.2857/2×100 = 14.28%
77 (a) Distance between two terminal metro stations is 300 km.
Total distance covered in a week (7 days),
6 trips from A to B takes place
In a Day, 6 trips = 300×6×2 = 1800 km × 2
For 7 days, 2 × 1800 km × 7 = 25,200 km
Time taken for a week = 2/300×12600 = 84 hours = 3.50 days

80 The Institute of Cost Accountants of India


Arithmetic

78 (b) Yardstick (y) = Speed × Time × Distance


Where, speed = Distance / Time
So, y = Distance / Time × Time × Distance
y = (Distance) 2
Therefore, with Increase in Distance of 2nd Vehicle by 2%, the distance would be increased
by 4%.
79 (b) Toto travels for 6 hours on a working day to cover 1.5 times the distance travelled on holiday
On a holiday it takes 4 hours to cover 110 km
Holiday – 110 km in 4 hours
Working day – 110×1.5 = 165 km in 6 hours
For a span of 7 days : 5 Working and 2 Holidays
Distance = 5×165 + 2×110 = 825 + 220 = 1045 km
80 (a) Let Time(s) 12 seconds, Speed (m/s) – 13 m/s
Speed (m/s) = Distance(m) / Time(s)
Distance (m) = Speed(m/s)×Time(s)
= 12 × 13 = 156 m

The Institute of Cost Accountants of India 81


Fundamentals of Business Mathematics and Statistics

 State True or False

1 2 3 4 5 6 7 8 9 10
F T F F F T F F F T
11 12 13 14 15 16 17 18 19 20
F T F F T T T F T F

 Fill in the Blanks

1 `1,320 2 6:4:3
3 1:150000 4 4
5 7:9 6 5:1
7 21:40 8 3:4:10
9 15, 24 10 ± 9
11 8:27 12 `350
13 4 year 14 ` 6,000
15 ` 2,500 16 10%
17 `4,370.91 18 ` 954.55
19 ` 76.23 20 `5,000
21 `6,250

82 The Institute of Cost Accountants of India


ALGEBRA 2
This Module includes

2.1 Set Theory, Venn Diagram


2.2 Indices and Logarithms (Basic Concepts)
2.3 Permutation and Combinations (Basic Concepts)
2.4 Quadratic Equations (Basic Concepts)

The Institute of Cost Accountants of India 83


Fundamentals of Business Mathematics and Statistics

ALGEBRA

Module Learning Objectives:


After studying this Module, the students will be able to understamd –
 The concepts of Set alongwith it’s representation by Venn diagram
 What is meant by Indices and it’s relationship with Logarithm
 The basic concepts of Permutation and Combination
 The theories of Quadratic Equations

84 The Institute of Cost Accountants of India


Set Theory, Venn Diagram 2.1
Definition of Set
Any collection of things or objects can be defined as set when the things or objects of the collection are distinctly
different and well-defined. By ‘well-defined’, we understand that all the things or objects of that collection must
belong to the collection.
Each object or thing in the collection is termed as Element of the Set.

For example,
On a table, 5 journals, 5 magazines and 1 laptop are placed. In this scenario, 5 journals and 5 magazines can be part
of the “Books and Periodicals” Collection while the laptop cannot be part of such collection. So, 5 journals and 5
magazines are be part of the Set “Books and Periodicals” while the Laptop is not a part of the Set.

Methods for Representation of Sets


1. Tabular or Roster Method
When the elements can be identified and are explicitly given, the method of representation called is Tabular or
Roster Method. In this method, the elements are separated by comma and are written one after another. Such
Sets are denoted within brackets [ ] or braces { }.
For example,
The set of odd numbers between: 11 to 30.
Let A be the set of odd numbers between 11 to 30,
A = {11, 13, 15, 17, 19, 21, 23, 25, 27, 29}

The set of even numbers between: 1 to 25.


Let N be the set of even numbers between 1 to 25,
N = [2, 4, 6, 8, 10, 12, 14, 16, 18, 20, 22, 24}

2. Set-Builder Method or Property Method


Most general method of representing set is Property or Set-Builder Method. In this method, all the elements
of the set are not listed or mentioned rather the common properties shared by all the elements are stated in the
Set.

The Institute of Cost Accountants of India 85


Fundamentals of Business Mathematics and Statistics

For example,
If W be the set of all even numbers between: 81 to 100. For selecting any arbitrary number within the range as
t, we write as follows:
W = {t: t is even and 81 < t <100}
Understanding the expression:
W is a set of numbers t where t is an even number and lies between: 81 to 100.

Types of Set
1. Set of Sets
When all the elements of a set are sets in themselves, that set is called set of sets.
For Example,
T = {a, b, c, d}
Where, a = {1, 2, 3, 4}; b = {1, 3, 5, 7}; c = {2, 4, 6, 8}; d = {1, 3, 4, 6}
Here, T would be termed as Set of Sets.
2. Singleton Set
If any set has only one element, it is termed as Singleton Set. In simple terms, set with just one element is called
unit set or singleton set.
For example,
Given: Set L = {33}
Meaning: Set L has only one element which is 33.
3. Null Set
Void Set or Empty Set or Null Set is a set which do not have any element and is denoted by { } or ø.
It may be noted that the Null Set ø is distinctly different from the Set {0}. The former contains no element at
all, whereas the latter contains only are element i.e. 0.
4. Finite Set
If the number of elements can be counted for a given set, that set is termed as Finite Set. Finite Set comprises
of specific number of elements.
5. Infinite Set
When the number of elements to be counted in a given set is infinite, the set is termed as Infinite Set. So,
Infinite set does not have specific number of elements.
6. Equal Set
For any two given sets, if each of the elements of one set are same as each of the elements of the other set, then
the sets are called Equal Set.
For Example,
If all elements of set Y are same to all the elements of set W, Y = W can be written to signify that every element
of Set Y is equal to every element of Set W.

86 The Institute of Cost Accountants of India


Algebra

7. Subset
For any two given sets, if one set has all the elements of the other set, the other set is termed as subset of the
first one.
For example,
Let A = { 2,4,5,6,7,8,9} and B ={2,4,6,8}
Since, B has all the elements present in A, Set B is called a subset of Set A.
Denoted as B ⊆ A or A ⊇ B
Here A is called a Superset of B.
Inportant points to note:
(i) Every Set is a Subset of itself
(ii) Empty Set is a Subset of every set i.e. ø ⊆ A.
(iii) Every Set has at least two Subsets – the Empty Set and the Set itself.
8. Proper Subset and Superset
For any given two sets, if one set has all the elements of the other set and also at least one more element is
present in it, the other set is termed as proper subset of the first one. While, the first one is termed as Superset
of the other one.
For example,
Let A = { 2,4,5,6,7,8,9} and B ={2,4,6,8}
Since, A has all the elements present in B and some other elements as well, Set B is called a proper subset of
Set A. While, Set A is termed as Superset of Set B.
Denoted as B ⊂ A and A ⊃ B.
Important points to note :
(i) Set A is not a Proper Subset of itself
(ii) A is a Proper Subset of B, A ⊂ B if all the clements of Set A are also the elements of Set B as well as A ≠ B
9. Equivalent sets [0]
Two finite sets A and B are said to be equivalent if there is one to one correspondence between the elements of
A and B and we write A ≡ B.
From the definition it can be said that two finite sets A & B are equivalent if and only if they have same number
of elements.
i.e. n (A) = n (B).
The symbol ‘0’ is used to denote equivalence of sets
Let A = {a, e, i, o, u} then n (A) = 5
And B = {1, 2, 3, 4, 5} then n (B) = 5
So A 0 B as n (A) = n (B)

The Institute of Cost Accountants of India 87


Fundamentals of Business Mathematics and Statistics

10. Universal Set


Universe of Discourse or Universal Set is a fixed set. If all the sets which are under consideration are subsets of
a Fixed Set then the fixed set is called as Universal Set. The Universal Set is also calleds Space and its elements
are called Points. The symbol U or S is used to denote the Universal Set.
For Example,
Q = {-10,-9,-8,-7.....-1}; W = {-100,-80,-60,-40 ...40, 60}
E = { 15,30,45,60,75,90} ; R = { -90,-60,-30,........60,90}
S = {-100, -99... 99, 100}
The elements of each of the sets (Q, W, E, R) are present in the Set S, that means, all the sets (Q, W, E, R) are
subsets of S. Hence, S is the Universal Set.
11. Power Set
Set comprises of all the subsets of a given set, is termed as Power Set of the given set. It is denoted by P (V),
considering set V is the given set. Let V = {0, 1, 2}. P(V) = [ø, {0}, {1}, {2}, {0,1}, {0,2}, (1,2}, {0,1,2}]
The number of elements of Power set can be determined as follows:
For any finite set M with n elements, the power set of M would have 2n elements. In the above example, given
set V has 3 slements. So no of elements to of P(V) is 23 = 8.
12. Product Set
The Product Set of two given sets A & B is the set whose elements are the Ordered Pairs (a, b) where ‘a’ is an
element of set A & B ‘b’ is an element of Set B. It is written as A X B & read as “A cross B”. It can be noted
that A X B & B X A are different.
Let A = {1,2} & B = {a,b,c} then A × B = {1,a), (1,b), (1,c), (2,a), (2,b), (2,c)} and B X A = {(a,1), (a,2), (b,1),
(b,2), (c,1), (c,2)}

Cardinality (or) order of a set


The no. of distinct elements in a set is called the cardinality (or) order of the set. If a finite set A has n distinct
elements, then the cardinality of the set A is n and is denoted by 0 (A) or n (A)
If A = {a, b, c} then n (A) = 3
B = {1, 2, 3} then 0 (B) = 3
∴ n (A) = 0 (B)
Note: The order of empty set is zero.

Important points to be kept in mind


1. Every Set would be a Subset of its’ own Set.
Denoted as Q ⊆ Q.
2. Null Set would be a Subset of every Set.
Denoted as ∅ ⊆ Q.
3. When there are 3 sets – Q, W and R and Q ⊆ W and W ⊆ R, then Q ⊆ R.

88 The Institute of Cost Accountants of India


Algebra

In simple terms,
If Q is a Subset of W and W is a Subset of R then Q is a Subset of R.

Set Operations
1. Joint or Union
When two sets are joined, it is termed as Union of Sets. In simple terms, the union of two different sets A & B
results in a new set with all the elements of the A & B appearing only once. It is denoted as A  B.
Let A = {1,3,5,7} and B = {5,7,9} So A  B = {1,3,5,7,9}. This operation in possible for more than two sets
also.
2. Meet or Intersection
When a set is formed with the common elements of two or more sets, the new set is said to be the result of
Intersection of sets. If A & B are the two given sets then their Intersection is denoted as A∩B. When A =
{2,3,4} and B = {3,4,6} then A∩B = {3,4}
3. Disjoint
When two sets do not have any elements in common, both the sets are termed as disjoint to each other.
Two sets A and B are said to be disjoint sets if they have no element in common i.e., A∩B= Ø.
If A = {1, 3, 5}, B = {2, 4}
Then A∩B = Ø. Hence A & B are disjoint sets
4. Difference
Of the two sets, a new set formed by the elements of the first which do not form part of the other is, the end
result of the operation Difference of sets.
In other words, the Difference of two sets A & B is the set of all elements of A which do not belong to B. It is
written as A – B.
Let A = {5,7,8,9} and B = {9} then A – B = {5,7,8}
5. Complement or Negation
When A is the sub-set of the universal set U, the elements of Complement of A would comprise of all the
elements of U except the elements of A. It is denoted as A′ or Ac or A .
If A = {1,2,3,4} and U = {1,2,3,4,5,6,7} then A′ = U – A ={5,6,7}.

Laws of Algebra of Sets


1. Idempotent :
(i) A  A = A
(ii) A  A = A
While consedring two or more sets, the term idempotent means the sets would be same and identical even after
the operation.

The Institute of Cost Accountants of India 89


Fundamentals of Business Mathematics and Statistics

2. Commutative :
(i) A  B = B  A
(ii) A  B = B  A
While solving two or more sets, the commutative law states that the elements of both the sets would be added.

3. Associative :
(i) A  (B  C) = (A  B)  C
(ii) A  (B  C) = (A  B)  C
When two or more sets are to be combined, the order of the sets would not matter since the result would be
same if the sets are associative to each other.

4. Distributive :
(i) A  (B  C) = (A  B)  (A  C)
(ii) A  (B  C) = (A  B)  (A  C)
When two or more sets are to be distributed, the order of the sets would not matter since the result would be
same if the sets are distributive.

5. De Morgan’s Law :
(i) ( A  B)′ = A′  B′
Complement of the Union of two sets is the Intersection of the Complement of both the sets.
(ii) ( A  B)′ = A′  B ′
Complement of the Intersection of two sets is the Union of the Complement of both the sets.

6. Identity Law :
(i) A  U = U
(ii) A  φ = A
(iii) A  φ = φ
(iv) A  U = A
For any set A in relation to Universal Set U and Null Set, the above mentioned laws are applicable.

7. Complement Law :
(i) A  A′ = U
(ii) Φ′ = U
(iii) U ′ = Φ
(iv) A  A′ = Φ
(v) ( A′)′ = A
For any set A in relation to Universal Set U and Null Set, the above mentioned laws are applicable.

90 The Institute of Cost Accountants of India


Algebra

VENN DIAGRAM
Pictorial representation of the relationship between two or more sets, is known as Venn diagram. It was introduced
by Euler and Developed by John Venn. In this diagram, sets are represented in closed curves by points. Each sets
are represented by Circles and form a part of Rectangle (which is representing the Universal Set).

Set operation using Venn diagram


1. Joint or Union (A  B)

A B

Either A or B or both AB=

Fig. 1: Venn Diagram representing A union B

2. Disjoint
U

A B

Fig. 2: Venn Diagram showing A and B are disjoint sets

3. Meet or Intersection (A 3 B)
U

A B

Both A and B A3B=

Fig. 3: Venn Diagram representing A intersection B

The Institute of Cost Accountants of India 91


Fundamentals of Business Mathematics and Statistics

4. Proper Subset
U

B is proper B_C
Fig. 4: Venn Diagram representing B is proper subset of A

5. Complement or Negation (A′)

U A′

A ′ the complement of A =
Fig. 5: Venn Diagram representing complement of Set A

Important results related to Set Oeprations


1. n (A  B) = n (A) + n (B) – n (A 3 B)
2. n (A  B) = n (A) + n (B) when A & B are disjoint sets
3. n (A  B  C) = n (A) + n (B) + n (C) – n (A 3 B) – n (B 3 C) – n (C 3 A) + n (A 3 B 3 C)
4. n (A  B  C) = n (A) + n (B) + n (C) when A, B & C are disjoint sets

Illustration 1.
Write the set-builder form of set for: M = {1, 8, 27, 64, 125 .....................}
Answer:
The elements of the set M are cube of all natural numbers.
Hence, M in set-builder form would be expressed as follows:
M = {x: x = n3, n ∈ N}

Illustration 2.
Write the roster form of set for: B = {x:x is a letter in the word ‘LANGUAGE’}
Answer:
The word ‘LANGUAGE’ has 8 letters, while the distinct letters are: L, A, N, G, U, E

92 The Institute of Cost Accountants of India


Algebra

B = {L, A, N, G, U, E}

Illustration 3.
The Set with their elements are mentioned below :
A = {All numbers}
B = {2, 4, 6, 8, 10}
C = {1, 3, 5, 7, 9}
D={ }
Identify the different types of sets.

Answer:
A - Universal Set - It comprises of All Numbers.
B and C - Sub-sets of A that is Sub-sets of U.
D – Null Set – It comprises of no elements.

Illustration 4.

If Set A = { 2, 4, 6, 8, 10 }; Set B = { 1, 3, 5, 7, 9 }; Set C = { 1, 2, 3, 4, 5 } and Set D = { 6,7,8,9,10 }.


Find:
(i) A  B (ii) A  C
(iii) B  D (iv) A  D

Answer:
(i) A  B = { 1,2,3,4,5,6,7,8,9,10 }
(ii) A  C = {2, 4}
(iii) B  D = {7, 9}
(iv) A  D = {6, 8, 10}

Illustration 5.
If Q = {1, 2, 3, 5} and W = {3, 4, 5, 7}. Find the Union of Q and W.

Answer:
Q - Contains 4 elements: 1,2,3,5
W – Contains 4 elements: 3,4,5,7
Q  W – Contains 6 elements since no elements should repeat in case of union of sets.
Q  W = {1, 2, 3, 4, 5, 7}

Illustration 6.
If Q = {-1, -2, 8, 10} and W = {6, 9, -2, -1}. Find the Intersection of Q and W.

The Institute of Cost Accountants of India 93


Fundamentals of Business Mathematics and Statistics

Answer:
Q - Contains 4 elements: -1, -2, 8, 10
W – Contains 4 elements: 6, 9, -2, -1
Q  W = {-1, -2}

Illustration 7.
If U = { -5,-4,-3,-2,-1,0,1,2,3,4,5} and A = {0,1,2,-1,-2}, find A′.
Answer:
All the elements which are part of U but not part of A, would be part of A′.
A′ = {-5,-4,-3, 3, 4, 5}

Illustration 8.
If T = {q, w, e, r, t, y} and Y= {w, r, y, u, p, o}, find the difference (Y-T).
Answer:
Set (Y-T) – All the elements of Set Y which are not part of Set T.
Set (Y-T) = {u, p, o}.

Illustration 9.
If U = { -2,1,2,3,4,5} and A = { -2 , 5 }, find the elements of set (U - A′).
Answer:
We know that, U – A = A′, hence, U – A′ = A.
So, set (U–A′) = set A = {–2, 5}

Illustration 10.
Rewrite the following examples using set notation
(i) First ten even natural numbers.
(ii) Set of days of a week.
(iii) Set of months in a year which have 30 days.
(iv) The numbers 3, 6, 9, 12, 15
(v) The letters m, a, t, h, e, m, a, t, I, c, s
Answer:
(i) A = {2, 4, 6, 8, 10, 12, 14, 16, 18, 20}
= {x: x is an even integer and 2 ≤ x ≤ 20}
(ii) A = {Sunday, Monday ……. Saturday}
= {x: x is a day in a week}
(iii) A = {April, June, September, November}
= {x: x is a month of a year which has 30 days}

94 The Institute of Cost Accountants of India


Algebra

(iv) A = {x: x is a positive number multiple of 3 and 3 ≤ × ≤ 15}


(v) A = {x: x is a letter in the word mathematics}

Illustration 11.
Write the following set in roster form.
(i) A = {x: x is an integer, - 3 < x < 7}
(ii) B = {x: x is an integer, 4 < x < 12}

Answer:
(i) A = {-3, -2, -1, 0, 1, 2, 3, 4, 5, 6}
(ii) B = {6, 8, 10, 12}

Illustration 12.
Represent the following sets in a selector method:
(i) All numbers less than 15
(ii) All even numbers
Answer:
Taking R to be the set of all real numbers in every case:
(i) {x: x∈ R and x < 15}
(ii) {x: x ∈ R and x is a multiple of 2}

Illustration 13.
A = {1, 2, 3, 4, 6, 7, 12, 17, 21, 35, 52, 56}, B and C are subsets of A such that B = {odd numbers},C = {prime
numbers}. List the elements of the set {x: x ∈ B ∩ C}

Answer:
B ∩ C = {1, 3, 7, 17, 21, 35} ∩ {2, 3, 7, 17} = {3, 7, 17}
Therefore, reqd. list = {3, 7, 17}

Illustration 14.
If A = {1, 2, 3, 4, 5}, B = {2, 4, 5, 8}, C = {3, 4, 5, 6, 7}, find A (BC).

Answer:
B  C = {2, 3, 4, 5, 6, 7, 8}, A  (B  C) = {1,2,3,4,5,6,7,8}

Illustration 15.
If A = {1, 2, 3} and B = {2, 3, 4}; find (A – B)  (B – A)

Answer:
A – B = {1}, B – A = {4}, (A- B)  (B – A) = {1, 4}

The Institute of Cost Accountants of India 95


Fundamentals of Business Mathematics and Statistics

Illustration 16.
If S is the set of all prime numbers, M = {x : x is an integer, 0 ≤ x ≤ 9} exhibit
(i) M – (S  M)
(ii) M  N, where N = {0, 1, 2 …….. 20}

Answer:
S = {2, 3, 5, 7, 11, 13…}, M = {0, 1, 2… 8, 9}
(i) S  M = {2, 3, 5, 7}, M – (S  M) = {0,1,4,6,8,9}
(ii) M  N = {0, 1… 20}

Illustration 17.
In a class of 100 students, 45 students read Physics, 52 students read Chemistry and 15 students read both the
subjects. Find the number of students who study neither Physics nor Chemistry.

Answer:
We know n (AB) = n (A) + n (B) – n (A∩B). Let A indicates the set of students who read Physics and B for
Chemistry.
Given:
n (A) = 45, n (B) = 52, n (A ∩ B) = 15
So, n (AB) = 45 + 52 – 15 = 82
We are to find n (A′ ∩ B′) = 100 – n (AB) = 100 – 82 = 18

Illustration 18.
In a survey of 1000 families it is found that 454 use electricity, 502 use gas, 448 use kerosene, 158 use gas and
electricity, 160 use gas and kerosene and 134 use electricity and kerosene for cooking. If all of them use at least
one of the three, find how many use all the three fuels.

Answer:
Let us take E for set of families use electricity, G for gas, and K for kerosene.
Now n (E) = 454, n (G) = 502, n (K) = 448
n (G ∩ E) = 158, n (G ∩ K) = 160, n (E ∩ K) = 134, n (E ∩ G ∩ K) =?
n (E  G  K) = 1000
Again n (E  G  K) = n (E) + n (G) + n (K) – n (E ∩ G) – n (G ∩ K) – n (K ∩ E) + n (E ∩ G ∩ K)
Or, 1000 = 454 + 502 + 448 – 158 – 160 – 134 + n (E ∩ G ∩ K) = 952 + n (E ∩ G ∩ K)
Or, n (E ∩ G ∩ K) = 1000 – 952 = 48.

96 The Institute of Cost Accountants of India


Indices and Logarithm 2.2
INDICES
The power or exponent of any number or a variable is known as Index. Plural of Index is Indices, Indices provide
a compact algebraic notation for repeated multiplication. For solving expression with Indices, the law of exponents
is applied. In the expression x5, the power 5 is known as Index and x is known as Base.
For Example,
(a) 23 = 2×2×2 = 8
(b) (22)2 = (2×2)2 = 42 = 4×4 = 16

1. Find the value of 1.25 to the power of 5.

Answer:
1.25 to the power of 5 is written as
(1.25)5
= (1.25) × (1.25) × (1.25) × (1.25) × (1.25)
= 3.05175
Thus, 1.25 to the power of 5 = 3.05. (Approx).

2. Find the value of (2.5)5

Answer:
(2.5)5 = (2.5) × (2.5) × (2.5) × (2.5) × (2.5)
= 244.14062

Thus, (2.5)6 = 244.14. (Approx).

Root of a number:

If x n = a then, x is called nth root of a

x = n a Or, x = (a)1/n

For example: 24 = 16 or 2 = 4 16

The Institute of Cost Accountants of India 97


Fundamentals of Business Mathematics and Statistics

Basic rules of Indices:

1. a m × a n = a m + n
For example: 53 × 52 = 53+ 2 = 55

m
2. a n = a m − n
a
53
For example: = 53− 2 = 5
52

a am
3. ( ) m = m
b b
6 62
For Example: ( ) 2 = 2
5 5

4. (ab) m = a m × b m
For example: (5 × 4) = 52 × 42 = 25 × 16
2

1
5. = a −m
am
1
For example: = 5−2
52

1
6. n
a=a n

1
For example: 5
6 = 65

7. a 0 = 1
For example: 60 = 1

8. If in an equation bases on both sides are same, powers can be equated.


For example: If 22 = 2x then x = 2

9. If in an equation powers or indices on both sides are same, bases can be equated.
For example: If x2 = 22 then x = 2

Illustration 19.

Evaluate: 133.
Answer:
133
= 13 × 13 × 13 = 2197.

98 The Institute of Cost Accountants of India


Algebra

Illustration 20.

() 1 1 1
y
a
If a x = = k m and a ≠ 1, prove that x − y = m
k
Answer:

Or, a x k y = a y
Or, k y = a y − x
1
Or, k = (a y − x ) y
y−x
Or, k = a y

m
 y−x 
Or, k m =  a y 
 
( y − x)m
Or, a x = a y

( y − x)m
Or, x =
y
x x
Or, = 1−
m y
1 1 1
Or, − = (Proved)
x y m

Illustration 21.
x z x m + y m + z m + wm m
If y = w show that − m − m = ( xyzw)
2
−m −m
x + y +z +w
Answer:
x z
Let = = k , then x = yk , z = wk
y w
x m + y m + z m + wm y m k m + y m + wm k m + wm
=
x − m + y − m + z − m + w− m y − m k − m + y − m + w− m k − m + w− m

=
(y m
+ wm )( k m + 1)
(y −m
+ w− m )( k − m + 1)

(y m
+ wm )( k m + 1)

( )
=
 1 1  1
 y m + wm  k m + 1

= k m y m wm

The Institute of Cost Accountants of India 99


Fundamentals of Business Mathematics and Statistics

m m
Also ( xyzw) 2 = ( yk.y.wk.w)m / 2 = ( y 2k 2 w2 ) 2 = y mk m wm

x m + y m + z m + wm m
So − m = ( xyzw) (Proved)
2
−m −m −m
x + y +z +w

Illustration 22.
Solve: 62 x + 4 = 33x ⋅ 2 x +8 .
Answer:
We have 62 x + 4 = 33x ⋅ 2 x +8
Or, 22 x + 4.32 x + 4 = 33x.2 x +8
22 x + 4 33x
Or, x +8 = 2 x + 4
2 3
Or, 22 x + 4 − x −8 = 33x − 2 x − 4
Or, 2 x − 4 = 3x − 4

() ()
x 4
Or, 2 =
2 Or, x = 4
3 3

Illustration 23.
m + 2n 3m −8n
Simplify x 5m⋅ −x6n
x
Answer:
x m + 2n ⋅ x 3m −8n
= x m + 2n + 3m −8n −5m + 6n
x 5m − 6 n
= x 4 m − 6 n − 5m + 6 n
−m 1
= x = m
x
Illustration 24.

()
y
2
Solve: 6 x − 3.2 x + y − 8.3x − y + 24 = 0, xy = 2
3
Answer:

()
y
2
6x − 3.2 x + y − 8.3x − y + 24 = 0
3

()
y
2
Or, ( 2.3)
x
− 3.2 x + y − 8.3x − y + 24 = 0
3

()
y
2
Or, 2 x.3x − 3.2 x + y − 8.3x − y + 24 = 0
3

100 The Institute of Cost Accountants of India


Algebra

1
Or, 2 x.2 y.3x. − 3.2 x + y − 8.3x − y + 24 = 0
3y
Or, 2 x + y.3x − y − 3.2 x + y − 8.3x − y + 24 = 0
Or, 2 x + y.3x − y − 3.2 x + y = 8.3x − y − 24
Or, 2 x + y (3x − y − 3) = 8 (3x − y − 3)
Or, 2 x + y = 8 [Assuming 3x–y – 3 ≠ 0]
Or, 2 x + y = 23
Or, x + y = 3
Or, x = 3 – y
Putting this value of x in xy = 2 we get
(3 – y)y = 2
2
Or, 3 y − y = 2
2
Or, y − 3 y + 2 = 0
Or, y = 2 or 1
So x = 1 or 2

Illustration 25.
1
3 −1 −1 −
Obtain the simplest value of [1 − {1 − (1 − x ) } ] 3
when x = 1
Answer:
1

[1 − {1 − (1 − x 3 ) −1}−1] 3

1

 1  3

= 1 − 
 1 − (1 − x ) 
3 −1

1

1 − (1 − x 3 )−1 − 1 3

=  
 1 − (1 − x 3 ) 
−1

1

 − 1  3
3
=  1− x 
 1 
 1 − 
1 − x3 

( )
1

1 3
= 3
x
1
= ( x 3 ) 3 = x = 1 [As x = 1 (given)]

The Institute of Cost Accountants of India 101


Fundamentals of Business Mathematics and Statistics

Illustration 26.

If x = 31/3 + 3−1/3 prove that 3x 3 − 9 x = 10


Answer:

x = 31/3 + 3−1/3
Cubing both sides we get

x 3 = (31/3 ) + (3−1/3 ) + 3.31/3.3−1/3 (31/3 + 3−13 )


3 3

1 1 1 1

Or, x 3 = 3 + 3−1 + 3.33 − 3 ( x ) [As x = 33 + 3 3
(Given)]

1
Or, x 3 = 3 + + 3.1.x
3
Or, 3 ⋅ x 3 = 9 + 1 + 9 x
Or, 3x 3 − 9 x = 10 (Proved)

Illustration 27.

1 1 1
If ( 2.381) = ( 0.2381) = 10 z prove that
x y
= +
x y z
Answer:
Let (2.381)x = (0.2381)y = 10z = K
1 1 1
So 2.381 = k x & 0.2381 = k y & 10 = K z

2381 1
Or, = Kx
1000
1
Or, 2381 = 103 K x ... ... ... ... (1)
1
0.2381 = K y
1
2381
Or, =Ky
10000
1
Or, 2381 = 104 K y ... ... ... ... (2)
(2) ÷ (1) gives
1
2381 104 K y
=
2381 1
103 K x
1

Or, 1 = 10.K1
y

Kx

102 The Institute of Cost Accountants of India


Algebra

1 1
Or, K x = 10K y
1 1 1
Or K x = k z ⋅ K y
1 1 1
So = + (Proved)
x y z

Illustration 28.

If x = 31/ 4 + 3−1/ 4 & y = 31/4 − 3−1/4 prove that 3( x 2 + y 2 ) 2 = 64


Answer:

x = 31/ 4 + 3−1/ 4
Squaring both sides we have

x 2 = (31/ 4 + 3−1/ 4 ) = 31/ 2 + 3−1/ 2 + 2.31/ 4.3−1/ 4


2

Or, x 2 = 31/ 2 + 3−1/ 2 + 2 ... ... ... ... ... (1)


Also
1 1

y = 34 − 3 4

Squaring both sides we have

y 2 = (31/ 4 − 3−1/ 4 ) = 31/ 2 + 3−1/ 2 − 2.31/ 4.3−1/ 4


2

Or, y 2 = 31/2 + 3−1/2 − 2 ... ... ... ... ... (2)


(1) + (2) gives

x 2 + y 2 = 31/2 + 3−1/2 + 31/2 + 3−1/2

Or, x 2 + y 2 = 2(31/2 + 3−1/2 )


Squaring both sides we get
( x 2 + y 2 ) 2 = 4(3 + 3−1 + 2.3.3−1)
2 2 2 1
Or, ( x + y ) = 4 3 + 3 + 2 = 4
9 +1+ 6
3 ( ) ( )
Or, 3 ( x 2 + y 2 ) = 64 (Proved)
2

Illustration 29.
2 −1 −1 −1 2 −1 −1 −1 2
If a + b + c = 0, simplify x a b c
xa bc
xa b c

Answer:
2 −1 −1 −1 2 −1 −1 −1 2
xa b c
xa bc
xa b c

The Institute of Cost Accountants of India 103


Fundamentals of Business Mathematics and Statistics

= xa
2 2 2
/ bc
. x b / ac. x c / ab

2
/ bc + b2 / ac + c 2 / ab
= xa
3 3 3
= x a + b + c / abc
3 3 3
= x[a + b + c −3abc + 3abc]/ abc

Now a + b + c − 3abc = ( a + b + c ) ( a + b + c − ab − bc − ca ) = 0 [As a + b + c = 0 (given)]


3 3 3 3 3 3

3 3 3
So x[a + b + c −3abc + 3abc]/ abc = x 3abc / abc = x 3

Illustration 30.
Express as positive indices –
1

(i) x 3

2 1
− −
(ii) 2x 5 ⋅ x 3

2 1

(iii) x 3 ÷ a 5

Answer:

1
1
(i) x 3
= 1 ;
x3

2

1
2 1
(ii) 2 x 5 ⋅ x 3
= 2 × 1 ;
x 5
x3
2

(iii) x 3 ÷ a
2

1
x3 2 1
5
= 1 = x3 × a5

a 5

Illustration 31.

( )
3

Find the value of (i) 1 4 −
1
; (ii) (243) 5
81
Answer:

( )
3

(i) 1 4 3 3
= (81) 4 = (34 ) 4 = 33 = 27
81
1
1
= (35 )
1 −
(ii) ( 243)

5 5
= 3−1 =
3

Illustration 32.

2 (3n +1 ) + 7 (3n −1 )
Show that =1
()
1− n
1
3n + 2 − 2
3

104 The Institute of Cost Accountants of India


Algebra

Answer:

2 (3n +1 ) + 7 (3n −1 ) = 2.(3n.3) + 7.(3n.3−1 ) = 3n 2.3 + 7. ( 1


3)= 3n.
25
3

() ( )
1− n
1 2 25
= 3n.32 − 2.(3−1 )
1− n
3n + 2 − 2 = 3n.32 − 2.3n.3−1 = 3n 9 − = 3n
3 3 3

25
2 (3n +1 ) + 7 (3n −1 ) 3n.
3 = 1 (Proved)
So =
() 25
1− n
1 3n
3n + 2 − 2 3
3

Illustration 33.

p q
 p + 1  . p − 1 
 q  q
Simplify p q
q + 1  .q − 1 
 p  p

Answer:
p
 p + 1  . p − 1 
p
 pq + 1 . pq − 1
qq
( pq + 1) ( pq − 1) p q

 q  q  q   q  qp qq
= = .
p q p
q + 1  .q − 1   pq + 1 . pq − 1
q
( pq + 1) p ( pq − 1)q
 p  p  p   p  pp pq

( pq + 1) p ⋅ pp ( pq − 1) ⋅ p q q

= p ⋅
qp ( pq + 1) qq ( pq − 1)q
p+q
 p
=
 q

Illustration 34.
a 2 + ab + b 2 b 2 + bc + c 2 c 2 + ca + a 2
 xa   xb   xc 
Show that  b  × c × a =1
x  x  x 

Answer:
a 2 + ab + b 2 b 2 + bc + c 2 c 2 + ca + a 2
 xa   xb   xc 
 x b  × c × a
x  x 

= x ( a − b )( a ) × x(b − c)(b2 + bc + c2 ) × x(c − a)(c2 + ca + a2 )


2
+ ab + b 2

3 3 3 3 3 3
= ( x )a −b ⋅ ( x )b − c ⋅ ( x )c − a = x a −b +b −c +c −a
3 3 3 3 3 3

= x0 = 1

The Institute of Cost Accountants of India 105


Fundamentals of Business Mathematics and Statistics

Illustration 35.
1 1 1
Find the value of a + b + c, if x a = y b = z c is satisfying xyz = 1
Answer:
1 1 1
Let x a = y b = z c = k

So x = k a , y = k b , z = k c

So xyz = k a ⋅ k b ⋅ k c = 1

Or, k a + b + c = k 0 Or a + b + c = 0

Illustration 36.

Solve 2 x + 3 + 2 x +1 = 320
Answer:

2 x + 3 + 2 x +1 = 320

Or, 2 x+1 ( 22 + 1) = 320

320
Or, 2 x+1 = = 64 = 26
5
Or, x + 1 = 6. So x = 5

LOGARITHM
Definition
If ax = N and a > 0, N > 0 then x = loga N and read as logarithm of N to the base ‘a’ or in other words log N to the
base ‘a’.
So logarithm of a positive number N signifies that for a given base ‘a’ it is the power to which ‘a’ must be raised
to produce the positive number ‘N’.
For example,
Since 53 = 125 we express the corresponding logarithm as log5 125 = 3. This signifies when 5 is raised to the power
of 3 the value would be 125.

Understanding Common and Natural Logarithms


Common Logarithms are the logarithms for which the base is considered as 10. For any calculation with logarithms,
if base is taken as 10, the logarithms are considered to be Common Logarithms. In such scenarios, the base to a
logarithm is not generally mentioned but is assumed to be 10.
When the base of Logarithm is e (where e means exponential and having value 2.718) then it is called Natural
Logarithm. Its extensive use is seen in Calculus.

106 The Institute of Cost Accountants of India


Algebra

For example,
If log 25 is given, we consider it to be log10 25.
Also loge 25 is written as ln 25.

Laws followed for Logarithms


1. Logarithm of the product of the numbers would be equal to the sum of their logarithms.
loga(mn) = logam + logan
For Example,
log 5 (14) = log 5 ( 2 × 7 ) = log 5 2 + log 5 7

2. Logarithms of the quotient of two numbers would be equal to the difference of their logarithms.
log a
m
n ( )
= log nm − log nn
For Example,
32
log 5 (3.2) = log 5 = log 5 32 − log 5 10
10
3. Logarithm of any number raised to a power would be equal to the product of the logarithm of the number and
the power.
log a (m) n = n × log a m
For example,
log 2 (3) 4 = 4 × log 2 3
4. For any logarithm where base is same as the number, the value of logarithm would always be one (1). The
value of base or number should not be equal to 1 and should be a positive number (>0).
log p p = 1
For example,
log 2 2 = 1
log10 = log10 10 = 1
5. For any logarithm where number is 1 and the base is a positive number, then value would always be zero.
logp 1 = 0
For example,
log 4 1 = 0
6. The base of a logarithm for a number would be equal to the reciprocal of the logarithm with the number
replaced by base.
1
log r s =
log s r
For example,
1
log 5 8 =
log8 5

The Institute of Cost Accountants of India 107


Fundamentals of Business Mathematics and Statistics

7. If logarithm with base of a number is raised to its’ power, the logarithm of the number raised to the power
would be equal to the number.
s log s r = r
For example,
8log816 = 16
Important points to be kept in mind
Change of Base for Logarithm
The logarithm of a number with a base can be changed with the introduction of the base and number splitted into
two expressions of logarithms. The base and the number would be same with a new variable introduced both as
base and number to the new logarithmic expressions.
log r s = log t s × log r t
Illustration 37.
Express in simplified form: log 2 49
Answer:
log 2 49 = log 2 ( 7 × 7 )

= log 2 7 + log 2 7
= 2 × log 2 7

Illustration 38.
97
Express in simplified form: log 2
7
Answer:
97
log 2 = log 2 97 − log 2 7

Illustration 39.
Express as sum of two logarithms: log 2 (15 × 3)
Answer:
log 2 (15 × 3) = log 2 15 + log 2 3

Illustration 40.

Express as difference of two logarithms: log ( )


50
2
with base 2.
Answer:
log ( )
50
2
with base 2 would be written as : log 2
50
2( )
log 2 ( )
50
2
= log 2 50 − log 2 2

= log 2 50 − 1

108 The Institute of Cost Accountants of India


Algebra

Illustration 41.
Find the value of: log 2 64
Answer:
log 2 64 = log 2 26
= 6 × log 2 2
=6×1=6

Illustration 42.
Find the simplified form: log 3 4 Write logarithms by following the way it is written here
Answer:
2
log 34 = log 32 = 2 log 32

Illustration 43.
Express as quotiet of two logarithms with base 10: log8 83

Answer:
log10 83
log8 83 =
log10 8

Illustration 44.
Express with base value as 3: log 2 3
Answer:
1
log2 3 =
log 32

Illustration 45.
Express with base value as 3 after simplification : log12 3

Answer:
1
log12 3 =
log 3 12
1 1
= log 12 = log (3 × 4)
3 3

1 1
= =
log 3 (3 × 4) log 3 3 + log 3 4
1 1
=
3 3 + log 3 4 1 + log 34

Illustration 46.
Find the value of: 3log3 9

The Institute of Cost Accountants of India 109


Fundamentals of Business Mathematics and Statistics

Answer:
3log3 9 = 9

Illustration 47.

Find the logarithm of 2025 to the base 3 5


Answer:
( ) ( )
x 4
Let x be the required number; then 3 5 = 2025 = 3452 = 3 5 ∴ x = 4 is the required number

Illustration 48.
The logarithm of a number of the base 2 is k. What is its logarithm to the base 2 2 ?
Answer:

Let ( 2) K = N
1 3
= 2 2 = 2 × 22 = 22
1
1

( )
1
So  3 3 3
2 = 2 =  22  = 2 2
2
 
k
So ( 2) K = (2 2) 3 = N

k
So the required number is
3
Illustration 49.
21
If log 2 x + log 4 x + log16 x = find x
4
Answer:

21
log 2 x + log 4 x + log16 x =
4
21
Or, log16 x log 2 16 + log16 x log 4 16 + log16 x =
4
21
Or, log16 x(log 2 16 + log 4 16 + 1) =
4
21
Or, log16 x(4 + 2 + 1) =
4
3
Or, log16 =
4
3 3
Or, x = (16) 4 = (24 ) 4 = 8

Illustration 50.

If p = log10 20 and q = log10 25, find x and such that 2 log10 ( x + 1) = 2 p − q

110 The Institute of Cost Accountants of India


Algebra

Answer:

2 p − q = 2 log10 20 − log10 25 = log10 202 − log10 25 = log10 ( )


400
25
= log10 16

Or, 2 log10 ( x + 1) = log10 ( x + 1) 2 = log10 16

Or, ( x + 1) 2 = 16
Or, x + 1 = ± 4
Or, x = 3, − 5
If x = –5 then x + 1 = – 4. In that case 2 log10 (x+1) = 2 log10 (– 4) which is impossible. So x ≠ – 5
Required value of x is 3

Illustration 51.

If x = log2a a, y = log3a 2a, z = log4a 3a show that xyz + 1 = 2 yz

Answer:

LHS = xyz + 1 = log 2a a × log 3a 2a × log 4a 3a + 1

= (log10 a × log 2a 10) × (log10 2a × log 3a 10) × (log10 3a × log 4a 10) + 1

log10 a log10 2a log10 3a


= × × +1
log10 2a log10 3a log10 4a

log10 a
= +1
log10 4a
= log 4a a + 1

= log 4a a + log 4a 4a

= log 4a (a × 4a) = log 4a (4a 2 )

RHS = 2 yz = 2 × log 3a 2a × log 4a 3a = 2 log 4a 2a = log 4a ( 2a ) = log 4a 4a 2


2

So LHS = RHS (Proved)

Illustration 52.

Show that

log 3 3 3 3,,,,,,, ∞ = 1

The Institute of Cost Accountants of India 111


Fundamentals of Business Mathematics and Statistics

Answer:

Let x = 3 3 3  .. ∞

Squaring both sides

Or, x 2 = 3 3 3 3  .. ∞

Or, x 2 = 3x
Or, x( x − 3) = 0. So x = 3 & x = 0

So log 3 3 3 3 ,,,,,,, ∞ = log 3 x = log 3 3 = 1

Illustration 53.
1 1
Prove that log (ab) + log (ab) = 1
a b

Answer:

Let x = log a (ab) & y = log b (ab)


Then by definition

a x = ab, b y = ab

1 1
Or, a = (ab) x , b = (ab) y
1 1
( + )
Or, ab = (ab) x y

1 1
+ =1
x y
1 1
Or, + = 1 (Proved)
log a (ab) log b (ab)

112 The Institute of Cost Accountants of India


Permutation and Combinations
(Basic Concepts) 2.3
PERMUTATION
Fundamental Principle of Counting:
Let travelling from city A to city B is possible either by car or by bus or by train i.e. there are 3 ways of going from
city A to city B and with each of these 3 ways there are associated 2 ways (by auto or by rickshaw) of going from
city B to city C. Hence possible ways of going from A to C via B can be 3×2 = 6
Above concept can be generalised as –
If there are ‘m’ ways of doing a job and with each of these ‘m’ ways there are assoiated ‘n’ ways of doing another
job then the two jobs can be done in (m × n) number of ways.

Understanding the meaning of ‘n’ factorial


Taking any number as ‘n’. The Factorial of such number would be ‘n’ factorial and is denoted as n! or n
This means product of all integers from n upto 1. So n! = n (n–1) (n–2) ...... 3,2,1. It can be mentioned that factorials
are possible only for positive integers.
For Example,
Factorial of 3: is expressed as 3! and it is calculate as 3! = 3×2×1
11! = 11×10×9×8×7×6×5×4×3×2×1
Important results related to factorial are –
(i) 0! = 1, (ii) 1! = 1, (iii) 2n! = 2n(2n–1) (2n–2) (2n–3) ... 3,2,1

Illustration 1. Find the value of ‘Factorial of 7’


Answer: Factorial of 7 would be written as 7!
7! = 7 × 6 × 5 × 4 × 3 × 2 × 1 = 5040.
Permutation of n different objects taking r at a time (r ≤ n)
The word permutation means Arrangement. Total number of permutations or arragements that can be made out of
‘n’ different objects taking ‘r’ at a time (when r ≤ n) is given as n pr . The formula for it is given below:

Formula:
n!
n pr =
(n − r )!

The Institute of Cost Accountants of India 113


Fundamentals of Business Mathematics and Statistics

nPr = n × (n − 1) × (n − 2) × (n − 3) ... × (n − r + 1)
Where, ‘n’ and ‘r’ are both positive integers.
Permutations can also be represented as
n P( n, r ), P n, r
Permutations are made for arrangements of objects in linear order or rearrangements of such already ordered set. It
is a type of arrangement made in definite order.

Illustration 54.
With 5,000 hospitals to be built in the state of Goa, only 2,500 hospitals would have ICU Beds. Officials are
required to select 2,500 hospitals for installation of ICU Beds. What would be the value of ‘n’ and ‘r’ in the above
situation?

Answer:
In the given question, the total number of hospitals to build are 5,000 and ICU Beds to be installed in only 2,500
hospitals out of those 5,000 hospitals built.
For the purpose of Permutation, ‘n’ stands for the total number of objects which are available for selection. Hence,
in the given question, total hospitals to be built are 5,000 hospitals. So, the value of ‘n’ would be 5,000.
As 2500 hospitals to be selected, the value of ‘r’ should be 2500.

Classification of Permutations:
(i) When Repetition is not allowed
(ii) When Repetition is allowed
(iii) Permutation in cases where distinction of objects is not possible
(iv) Circular permutation

(i) When Repetition is not allowed


When repetition is not allowed, with every object being selected, the availability of the remaining objects do
decrease with the selection of objects from the availability. Basically, the total number of objects available for
selection decreases after every selection is done.

Illustration 55.
In how many ways can 5 candidates be screened for 3 vacancies?

Answer:
There are 3 vacancies and 5 candidates.
First vacancy can be filled up by selecting 1 out of 5 in 5 ways.
So there remains 2 vacancies to be filled up by selecting 2 from the remaining.
Hence second vacancy can be filled up in 4 ways.
Similarly third vacancy can be filled up by slectly 1 out of the remaining 3 in 3 ways.

114 The Institute of Cost Accountants of India


Algebra

So, the number of ways of filling up 3 vacancies from 5 candidates = 5 × 4 × 3 = 60


Therefore, there are 60 ways in which 5 candidates can be screened for 3 vacancies.

(ii) When Repetition is allowed


When the objects to be selected can have repetition without any restriction. In simple words, ‘n’ different objects
could be taken ‘r’ at a time.
Formula: n r

Illustration 56.
In how many ways 3 cheques be given to 4 employees when each employee is eligible to receive any number of
cheques?

Answer:
All the 3 cheques can be given to all 4 employees. Even after the 1st cheque is given, the rest 2 cheques can also
be given to any employee out of 4 employees. So, the total number of ways in which 3 cheques can be given to 4
employees –
4 × 4 × 4 = 43 = 64 ways.

(iii) Permutation of cases where distinction of objects is not possible


When objects are similar in nature and cannot be distinguished among themselves or the identical distinction of any
object from the available objects is not possible due to objects being identical, in such cases if permutation would
be done considering all alike objects to be one single object. Thus, while finding number of possible permutations,
the denominator would be divided by the number of permutation of alike objects as well.
n
The permutation of ‘n’ objects of which ‘p’ are alike of one kind and ‘q’ are alike of another kind is
p⋅ q
Illustration 57.
Determine the arrangements which could be made from the letters of the word “APPLE”?

Answer:
The word “APPLE” has 5 letters with 2 P’s and 1 each of A, L an E. Thus, required the number of arrangements
would be

5! 5.4.3 2
= = 60
2! 2

(iv) Circular Permuation

When the objects are arranged in a circular formation then that is called Circular Permutation.
� No. of circular permutations of ‘n’ different objects taking all at a fime is (n-1)!
� No. of circular permutations of ‘n’ different taking all at a time is (n-1)!/2 when clockwise or anti clockwise
arrangements are not different.

The Institute of Cost Accountants of India 115


Fundamentals of Business Mathematics and Statistics

Illustration 58.
In how many ways 8 boys can form a ring?
Answer:
Keeping one boy fixed in any position, remaining 7 boys can be arranged in 7! Ways.
Hence, the required no. of ways = 7! = 7.6.5.4.3.2.1 = 5040.

Illustration 59.
In how many ways 8 different beads can be placed in a necklace?
Answer:
8 beads can be arranged in 7! Ways. In this 7! Ways, arrangements counting from clockwise and anticlockwise are
taken different, but necklace obtained by clockwise permutation will be same as that obtained from anticlockwise.
So total arrangement will be half of 7!
Hence, required no. of ways = ½ ×7! = ½ ×5040 = 2520

COMBINATION
Combination means selection or choice or formation of a group.
Each of different groups of selection that can be made by taking some or all of the no. of things at a time is called
combination.

Combination of ‘n’ different objects taking ‘r’ at a time (when r ≤ n)


n!
It is denoted by nCr and is defined as nC r =
( n − r )! r !
Here n = given total no. of things
C = combination or selection
r = no. of things taken at a time

Illustration 60.

In how many ways a committee of 2 boys can be formed from 4 boys.


Answer:
4! 4! 4.3.2.1
4C 2 = = = =6
( 4 − 2)!2! 2!2! 2.1.2.1
Relation between nP r and nC r :
n!
nP r =
( n − r )!
nPr n!
Or, = =n
r ! ( n − r )! r ! C r
Or, nPr = r !nC r

116 The Institute of Cost Accountants of India


Algebra

Properties:
1. nCr = 1
2. nC0 = 1
3. nCr = nCn −r

Proof:
n! n!
R.H.S: nC n − r = = = nC r = L.H .S
( n − n + r )!( n − r )! ( n − r )!r !
4. If nCr = nCS then r = s and also r = n – s

5. Pascal’s law:
If n and r be two non-negative integers such that 1≤ r ≤ n. then
nCr −1 + nCr = n + 1Cr

Formulae:
A) No. of combinations of n dissimilar things taken r at a time when ‘p’ particular things always occur is n − pC r − p;

B) No. of combinations of n dissimilar things taken ‘r’ at a time when ‘p’ particular things never comes is n − pC r ;
C) Total No. of combinations of n different things taken 1, 2, 3……n things at a time is 2n − 1 i.e.
nC + nC + nC +  + nCn = 2n − 1 .
1 2 3

Total no. of combinations of n different things taken one or more things at a time is 2n − 1 .

Grouping:
When it is required to form two groups out of (m + n) things, (m ≠ n) so that one group consists of m things then
formation of one group represents the formation of the other group automatically. Hence the number of ways m
thing can be selected from (m + n) things.

m + nC m =
( m + n )! = ( m + n )!
m !( m + n − m )! m !n !
Note:
(2m)!
1. If m = n, the groups are equal and in this case the number of different ways of subdivision = since two
(m !) 2
groups can be interchanged without getting a new subdivision.
(2m)!
2. When 2m things are divided equally between 2 persons, then the number of ways =
(m !) 2
3. Now (m + n + p) things (m ≠ n ≠ p), to be divided in to three groups containing m, n, p things respectively.
‘m’ things can be selected out of (m + n = p) things in ( m + n + pC m ) ways. Thereafter n things out of the
remaining (n + p) things can be selected in (n+p)Cn ways. Hence total number of ways of selection to form three
(m + n + p)! (n + p)! (m + n + p)!
groups containing m, n and p things = (m + n + p) Cm × (n + p)Cn = m !(n + p)! × n ! p ! = m ! n ! p !

Note:
3m ! 1
1. If now m = n = p, the groups are equal and in this case, the different ways of subdivision = ×
m !m !m !! 3!
since the three groups of subdivision can be arranged in 3! Ways.

The Institute of Cost Accountants of India 117


Fundamentals of Business Mathematics and Statistics

Note 2:
3m !
If 3m things are divided equally amongst three persons, the number of ways =
m !m !m !

Distinction between Permutation and Combination


Permutation:
Number of arrangements which can be made after selecting few from the available objects. The order of selection
does matter.
Formula for Permutation
n!
nP r =
( n − r )!

Combination:
Number of ways in which selection of objects from availability would be done. The order of selection does not
matter.
Formula for Combination
n!
nC r =
r ! × ( n − r )!

Permutation could be termed as Combination in an ordered manner.

Illustration 61.
Find the Factorial of 9.

Answer:
Factorial of 9: 9! = 9 × 8 × 7 × 6 × 5 × 4 × 3 × 2 × 1 = 3, 62,880.
Therefore, Factorial of 9 = 3, 62,880.

Illustration 62.
In how many ways 3 rows be selected for vaccination out of 5 rows of college students?
Answer:
For selecting 3 rows out of 5 rows of college students for vaccination, the following procedure is considered.
Assuming the selection of raws be done are by one, it can be said that the first row be slected in 5 ways.
So row there remains 4 rows of which the second can be slected in 4 wyas. Similarly the third can be selected in
3 ways.
Hence total number of ways of selection of 3 rows of students out of 5 rows is 5 × 4 × 3 = 60
Thus, 60 ways are there for selecting 3 rows for vaccination out of 5 rows of college students.

Illustration 63.
Solve for n: nP 6 = 13P 6

118 The Institute of Cost Accountants of India


Algebra

Answer:
Since, r = 6 in the permutations of both sides of the equation, n = 13.

Illustration 64.

If n + 1P 3 : nP 2 = 27 : 9 , find ‘n’.

Answer:
n + 1P 3 : nP 2 = 27 : 9

 (n + 1)!  :  n !  = 27 : 9
 (n + 1 − 3)!  (n − 2)!

(n + 1)!   n ! 
Or,  : = 27 : 9
 (n − 2)!  (n − 2)!

(n + 1)! 27 (n + 1) (n !) 27
Or, = Or, =
n! 9 n! 9
Or, (n + 1) = 3
Or, n = 3–1 = 2.

Illustration 65.

If 8 × n − 1P 3 = n − 1P 4 , find n

Answer:
8 × n − 1P 3 = n − 1P 4

n − 1P 3 1
Or, =
n − 1P 4 8

( n − 1) × ( n − 2) × ( n − 3) =
1
Or,
( n − 1) × ( n − 2) × ( n − 3) × ( n − 4) 8
1 1
Or, =
( n − 4) 8

Or, 8 = n − 4
Or, n − 4 = 8
Or, n = 8 + 4 = 12
Therefore, n = 12.

Illustration 66.

If n
P5 = 42 × n P3 , find the value of n.

The Institute of Cost Accountants of India 119


Fundamentals of Business Mathematics and Statistics

Answer.
n
P5 = 42 × n P3
n! n!
Or, = × 42
( n − 5)! ( n − 3)!

n × ( n − 1) × ( n − 2) × ( n − 3) × ( n − 4) × ( n − 5)! n × ( n − 1) × ( n − 2) × ( n − 3)!
Or, = × 42
( n − 5 )! ( n − 3)!
Or, n × ( n − 1) × ( n − 2) × ( n − 3) × ( n − 4) = n × ( n − 1) × ( n − 2) × 42

Or, (n − 3) × (n − 4) = 42

Or, n 2 − 3n − 4n + 12 = 42

Or, n 2 − 7n + 12 − 42 = 0

Or, n 2 − 7n − 30 = 0

Or, n 2 − 10n + 3n − 30 = 0

Or, n (n − 10) + 3(n − 10) = 0

Or, (n − 10) (n + 3) = 0
Either n – 10 = 0 or, n + 3 = 0
Either n = 10 or, n = – 3
Since, n is a positive integer, it should be 10.
Thus, ‘n’ = 10.

Illustration 67.
In how many ways 6 mobiles be gifted to 6 employees?

Answer:
The number of employees to be given gifts = 6 employees, the number of mobiles to be gifted = 6 mobiles, so, n
= 6 and r = 6.
Since, n = r, the number of permutations would be n! = 6! = 6×5×4×3×2×1 = 720 ways.

Illustration 68.
In how many ways 4 laptops be purchased from 7 laptops available at the store?

Answer:
In the given question, the number of laptops available in the store - 7 laptops would be ‘n’, while the number of
laptops to be purchased – 4 laptops would be ‘r’.
Number of ways 4 laptops be purchased from 7 laptops would be as follows:

120 The Institute of Cost Accountants of India


Algebra

n 7!
Pr = 7 P4 =
(7 − 4)!
7! 7 × 6 × 5 × 4 × 3!
= =
3! 3!
= 7×6×5×4
= 840 ways.
Therefore, 4 laptops can be purchased from 7 laptops available in the store in 840 ways.

Illustration 69.
In how many ways can 8 articles numbered from 1 to 8 be arranged, if even numbered articles are same?
Answer: Since number of articles to be arranged – 8 articles (n = 8) and given that all even numbered articles
(Article 2,4,6,8) are same, the no. of ways to arrange such articles would be,
8! 8 × 7 × 6 × 5 × 4!
=
4! 4!
= 8 × 7× 6 ×5 = 1680 ways.
Thus, 8 articles where all even numbered articles are same, can be arranged in 1680 ways.

Illustration 70.
Find the number of 5 digit numbers that can be formed with the digits 8, 6,4,3,2,1,5,9.
Answer:
The numbers to be formed would be equal to the permutation of 8 different objects taking 5 at a time.
8 8! 8! 8 × 7 × 6 × 5 × 4 × 3!
Hence, P5 = = = = 8× 7× 6 ×5 ×4 = 6,720.
(8 − 5)! 3! 3!
Therefore, 6,720 numbers can be formed.

Illustration 71.
Find the value of 10C7.
Answer:
n!
Formula: nCr =
r ! × ( n − r )!
10C7 = 10!
7! × (10 − 7 )!
= 10!
7! × 3!
10 × 9 × 8 × 7!
=
7! × (3 × 2 × 1)
10 × 9 × 8
=
(3 × 2 × 1)
720
= 6 = 120.

The Institute of Cost Accountants of India 121


Fundamentals of Business Mathematics and Statistics

Illustration 72.
A bag contains 10 balls. In how many ways can a child take out any 2 balls?

Answer:
The number of ways for selection of 2 different balls:-
10!
10C =
2 2!(10 − 2)!

10! 10 × 9 × 8! 10 × 9 90
= 2! 8! = 2! 8! = 2 × 1 = 2 = 45

Illustration 73.

Find the value of (i) 7 P5 ; (ii) 7 P1 ; (iii) 7 p0 ; (iv) 7 p7 ;

Answer:
7! 7.6.5.4.3.2.1.
(i) 7 P 5 = = = 7.6.5.4.3 = 2520
( 7 − 5 )! 2.1.
7! 7.6.5.4.3.2.1.
(ii) 7 P1 = =
6.5.4.3.2.1. = 7
(7 − 1)!
7! 7.6.5.4.3.2.1.
(iii) 7 P 0 = =
(7 − 0)! 7.6.5.4.3.2.1. = 1
7! 7.6.5.4.3.2.1.
(iv) 7 P 7 = = = 7.6.5.4.3.2.1 = 5040
( 7 − 7 )! 1.

Illustration 74.

If nP 2 = 110, find n

Answer: nP 2 = 110
n!
Or, = 110
( n − 2 )!
n.( n − 1) .( n − 2)!
Or, = 110
( n − 2 )!
Or, n (n – 1) = 110
Or, n (n – 1) = 11.10
Or, n (n – 1) = 11 (11 – 1)
Or, n = 11

Illustration 75.
nP 5
Solve for n, given =2
nP 3

122 The Institute of Cost Accountants of India


Algebra

Answer:
n!
n P 5 ( n − 5 ) ! ( n − 3) !
= = = ( n − 3)( n − 4 ) = 2 (Given)
nP 3 n! ( n − 5 )!
( n − 3) !
Or, n 2 − 7n + 10 = 0
Or, ( n − 5)( n − 2 ) = 0
Or, n = 5, 2
If n = 2 then both nP5 and nP have no meaning. So the required value of n is 5.
3

Illustration 76.
In how many ways 6 books out of 10 different books can be arranged in a book-self so that 3 particular books are
always together?
Answer:
At first 3 particular books are kept outside. Now remaining 3 books out of remaining 7 books can be arranged in
7 P ways. In between these three books there are 2 places and at the two ends there are 2 places i.e. total 4 places
3
where 3 particular books can be placed in 4 P ways. Again 3 particular books can be arranged among themselves
1
in 3! ways.
7! 4!
Hence, required no. of ways 7 P 3 × 4 P1 × 3! = 7 − 3 ! × 4 − 1 ! × 3.2.1 = 7.6.5.4.3.2.1 = 5040
( ) ( )

Illustration 77.
In how many ways can the letters of the word TABLE be arranged so that the vowels are always (i) together (ii)
separated?

Answer:
(i) In the word TABLE there are 2 vowels and 3 consonants which are all different. Taking the 2 vowels (A, E)
as one letter we are to arrange 4 letters (i.e. 3 consonants + 1) which can be done in 4! Ways. Again 2 vowels
can be arranged between themselves in 2! ways.
Hence, required numbers of ways of arranging the letters so that the vowles are always together = 4! × 2! =
(4.3.2.1) × (2.1) = 48.
(ii) Required number of ways of arranging the letters so that the vowles are separated = 5! – 48 = 120 –48 = 72
[Taking all the letters together the no. of possible arrangements = 5!]

Illustration 78.
Find in how many ways can the letters of the word PURPOSE be rearranged –
(i) Keeping the positions of the vowels fixed;
(ii) Without changing the relative order of the vowels and consonants.

The Institute of Cost Accountants of India 123


Fundamentals of Business Mathematics and Statistics

Answer:
(i) In the word, PURPOSE there are 3 vowels and 4 consonants. Since the positions of all the vowels should
remain fixed, we are to rearrange only 4 consonants, of which there are 2Ps. So the arrangement is possible in

4! 4 × 3 × 2!
= = 4 × 3 = 12 ways
2! 2!

(ii) The relative order of vowels and consonants should remain unaltered means that vowel will take place of
vowel and consonant will take place of consonant. Now the 3 vowels can be arranged among themselves in 3!
ways, while 4 consonants with two Ps can be arranged in

4! 4 × 3 × 2!
= = 4 × 3 = 12 ways
2! 2!
So total number of ways of rearrangement in which the given arrangement is included = (3! ×12) – 1 = (6×12)
– 1 = 72 – 1
Hence, required number of arrangements = 72 – 1= 71

Illustration 79.
How many numbers between 5000 and 6000 can be formed with the digits 3, 4, 5,6,7,8?
Answer:
The number to be formed will be of 4 digits. Further the required numbers are between 5000 and 6000. So the digit
5 is to be placed in the 1st place (from left). Now the remaining 3 places can be filled up by the remaining 5 digits
in 5P 3 ways.
5!
Hence, required number of nos. 5P 3 = = 5.4.3 = 60
2!

Illustration 80.
In how many ways can the letters of the word SUNDAY be arranged? How many of them do not begin with S?
How many of them do not begin with S, but end with Y?

Answer:
There are 6 different letters in the word SUNDAY, which can be arranged in 6! = 720 ways.
Now placing S in first position fixed, the other 5 letters can be arranged in (5)! =120 ways.
The number of arrangements of letters that do not begin with S = (6)! – (5)! = 720 –120 = 600
Lastly Placing Y in the last position, we can arrange in (5)! = 120 ways and keeping Y in the last position and S in
the first position, we can arrange in (4)! = 24 ways.
Hence, the required no. of arrangements beginning with S but not ending with Y = (5)! -4! = 120 – 24 = 96.

Illustration 81.
In how many ways 5 boys and 5 girls can take their seats in a round table, so that no two girls will sit side by side.

124 The Institute of Cost Accountants of India


Algebra

Answer:
If one boy takes his seat anywhere in a round table, then remaining 4 boys can take seats in 4! =24 ways. In each of
these 24 ways, between 5 boys, if 5 girls take their seat then no two girls will beside by side. So in this way 5 girls
may be placed in 5 places in 5! = 120 ways.
Again the first boy taking seat, may take any one of the 10 seas. i.e. he may take his seat in 10ways.
Hence, required number ways = 24×120×10 = 28800

Illustration 82.
In how many ways can the college football team of 11 players be selected from 16 players?
Answer:
16! 16.15.14.13.12
The required number of ways = 16C11 = 16 − 11 !11! = = 4368
( ) 5.4.3.2
Illustration 83.
From a group of 15 men, how many selections of 9 men can be made so as to exclude 3particular men?
Answer:
Excluding 3 particular men in each case, we are to select 9 men out of (15-3) men. Hence the number of selection is
12! 12.11.10
equal to the number of combination of 12 men taken 9 at a time which is equal to 12C 9 = = = 220
(12 − 9 )!9! 3.2.1
Illustration 84.
There are seven candidates for a post. In how many ways can a selection of four be made amongst them, so that:
1. 2 persons whose qualifications are below par are excluded?
2. 2 persons with good qualifications are included?
Answer:

1. Excluding 2 persons, we are to select 4 out of 5(=7-2) candidates. Number of possible selections = 5C 4 = 5
ways
2. In this case, 2 persons are fixed, and we are to select only (4-2) = 2 persons out of (7-2), i.e. 5 candidates. Hence
the required number of selection = 5C = 5.4 = 10
2 2.1

Committee from more than one group:

Illustration 85.
In how many ways can a committee of 3 ladies and 4 gentlemen be appointed from a meeting consisting of 8 ladies
and 7 gentlemen? What will be the number of ways if Mrs. X refuses to serve in a committee having Mr. Y as a
member?
Answer:
8.7.6
1st part. 3 ladies can be selected from 8 ladies in 8C 3 = = 56 ways
3.2.1

The Institute of Cost Accountants of India 125


Fundamentals of Business Mathematics and Statistics

7.6.5
4 gentlemen can be selected from 7 gentlemen in 7C 4 = = 35 ways
3.2.1
Now, each way of selecting ladies can be associated with each way of selecting gentlemen. Hence, the required no.
of ways = 56 × 35 = 1960.

2nd part: If both Mrs. X and Mr. Y are members of the committee then we are to select 2 ladies and 3 gentlemen
from 7 ladies and 6 gentlemen respectively. Now 2 ladies can be selected out of 7 ladies in 7C 2 ways, and 3
gentlemen can be selected out of 6 gentlemen in 6C 3 ways.
Since each way of selecting gentlemen can be associated with each way of selecting ladies.
Hence, No. of ways = 7C 2 × 6C 3 = 420 ways
Hence, the required no. of different committees, not including Mrs. X and Mr. Y = 1960 – 420 =1540.

Illustration 86.
From 7 gentlemen and 4 ladies a committee of 5 is to be formed. In how many ways can this be done to include at
least one lady?
Answer:
Committee of 5 with at least one lady means the committee can be formed with
(i) 1 lady and 4 gentlemen or
(ii) 2 ladies and 3 gentlemen or
(iii) 3 ladies and 2 gentlemen or
(iv) 4 ladies and 1 gentleman or

For (i), 1 lady can be selected out of 4 ladies in 4C ways and 4 gentlemen can be selected from7 gentlemen in 7C
1 4
ways. Now each way of selecting lady can be associated with each way of selecting gentlemen. So 1 lady and 4
gentlemen can be selected in 4C1 × 7C 4 ways.
Similarly,

Case (ii) can be selected in 4C 2 × 7C 3 ways

Case (iii) can be selected in 4C 3 × 7C 2 ways

Case (iv) can be selected in 4C 4 × 7C1 ways.


Hence the total number of selections of the committee of 5 so that, at least one lady is included
= 4C 1 × 7C 4 + 4C 2 × 7C 3 + 4C 3 × 7C 2 + 4C 4 × 7C 1
= 4 × 35 + 6 × 35 + 4 × 21 + 1 × 7 = 441

Illustration 87.
In how many ways can a boy invite one or more of his 5 friends?

Answer:
The number of ways = 5C1 + 5C 2 + 5C 3 + 5C 4 + 5C 5 = 5 + 10 + 10 + 5 + 1 = 31

126 The Institute of Cost Accountants of India


Algebra

Illustration 88.
In how many ways 15 things can be divided into three groups of 4, 5, 6 things respectively.
Answer:

15.14.13.12.11!
The first group can be selected in 15C 4 = = 1365 ways:
11!.4.3.2.1

The second group can be selected in (15 − 4)C = 11C = 11.10.9.8.7.6! = 462ways
ways
5 5 6!.5.4.3.2
And lastly the third group in 6C 6 = 1 way

Total number of ways = 1365 × 462 = 630630

Illustration 89.
In how many ways can the letters of the word SIGNAL be arranged so that the vowels occupy only the odd places?
Answer:
Number of letters in the word SIGNAL is 6 and they are all different. So there are 3 odd places namely first, third
and fifth.
Vowels are only 2. The number of ways in which 2 vowels can occupy 3 places is 3P . For each such way the
2
remaining 4 places are to be occupied by 4 remaining consonants. This can be achieved in 4! different ways.

Hence the required arrangements are possible in 3P2 × 4! ×=144


2 = 288
ways

Illustration 90.
How many triangles can be made by joining the vertices of decagon? How many diagonals will it have?
Answer:
A triangle is obtained by joining any three of the 10 vertices of a decagon. The number of triangles can be made by

joining the vertices of a decagon = 10C3 = 10 × 9 × 8 = 120


3× 2
Now a diagonal is obtained by joining any two vertices which are not adjacent
10 × 9
Total number of ways in which two vertices of a decagon can be joined = 10C2 = = 45
2
Number of lines obtained by joining two adjacent vertices = Number of sides of decagon = 10
Hence number of diagonals = 45 –10 = 35

The Institute of Cost Accountants of India 127


Quadratic Equations 2.4
Definition and Form of Equation:
Quadratic Expression:
An expression which is of the form ax2+bx+c is called quadratic expression.
In this expression a, b and c are constants. An important point regarding a quadratic expression is the fact that
the maximum power (or index) of the variable x in the expression is 2. In other words ‘Degree” of a Quadratic
expression is 2.
The term ‘Degree’ refers to the highest power of the variable in the expression. For expressios with single variable,
the highest power of the variable is considered to find out the degree and for expressions involving multiple
variable the highest sum of powers of different variables in any of the terms of the expression is taken as the degree.
For the expression x3 + 3x3y2 + 2xy3 + 4y2 – 2y +1 the Degree is 5 because sum of the powers of x3y2 is (3+4)=5 and
it is the higest among all the terms.

Ex: 2 x 2 + 4 x + 7 is a quadratic expression.

Ex: ( 2 + i ) x 2 − 3ix + 6(2 + i ) is also a quadratic expression.

Quadratic Equation:

Any equation expressed in the form ax 2 + bx + c = 0 , is termed as Quadratic Equation.


For Example,

a. 15 x 2 + 28 x + 13 = 0
Here the equation is expressed in the form of ax 2 + bx + c = 0 with a = 15, b = 28 and c = 13
b. x2 = 30x – 6 Or, x2 – 30x + 6 = 0 Or, 1x2 + (– 30) x + 6 = 0
Here also the equation is expressed in the form of ax2 + bx + c = 0 with a = 1, b = –30 and c = 6

Roots and Nature of Roots:


Each Quadratic Equation has two solutions called roots and normally denoted as α and β

Solution methods of quadratic equation:


1. Factorization method

128 The Institute of Cost Accountants of India


Algebra

2. Completing square method:

Find the roots of the quadratic equation ax 2 + bx + c = 0


Given

Quadratic equation ax 2 + bx + c = 0
Multiplying both sides by 4a we get.

4a 2 x 2 + 4abx + 4ac = 0
Or, 4a 2 x 2 + 4abx = −4ac
Adding b2 on both sides we get
Or, 4a 2 x 2 + 4abx + b2 = b2 − 4ac

Or, ( 2ax ) + 2 ( 2abx ) + b2 = b2 − 4ac


2

Or, ( 2ax + b ) = b − 4ac


2 2

Or, ( 2ax + b ) = ± b − 4ac


2

Or, 2ax = −b ± b2 − 4ac


2
Or, x = b ± b 4ac This formula is also called Sridhar Acharya’s formula
2a
So the roots α and β of the quadratic equation are given as

−b + b2 − 4ac
Value of αx =
2a

−b − b2 − 4ac
Value of βx =
2a

Here b2 − 4ac is called the Discriminant and it describes the nature of the roots of the quadratic equation and is
denoted by ∆ = b2 − 4ac

Discriminant of a Quadratic Equation: b2 − 4ac

Nature of Roots:

1. If b2 − 4ac > 0 then the roots are real and unequal.

2. If b2 − 4ac = 0 then the roots are real and equal.

3. If b2 − 4ac < 0 then the roots are imaginary or conjugate complex numbers.

4. If b2 − 4ac > 0 and perfect square then the roots are rational and unequal.
5. If b – 4ac > 0 but not a perfect square then the roots are irrational numbers.

The Institute of Cost Accountants of India 129


Fundamentals of Business Mathematics and Statistics

6. If one root of a quadratic equation ax 2 + bx + c = 0 , is a + ib then the other root will be a – ib

7. If one root of quadratic equation ax 2 + bx + c = 0 is a + d then the other root is a − d .

To Form a Quadratic Equation


For forming a Quadratic Equation, the following formula needs to be followed:

Formula:
x2 – (Sum of the Roots) × x + (Product of the Roots) = 0

Sum of the Roots: α + β


Product of the Roots :αβ

Case I: Sum of the roots

−b + b2 − 4ac −b − b2 − 4ac 2b b
α+β = + =− =−
2a 2a 2a a

Case II: Product of the roots


2
−b + b2 − 4ac −b − b2 − 4ac  b   b2 − 4ac 
2
b2 b2 4ac c
αβ = × =   −  = 2− 2+ 2 =
2a 2a  2a   2a 
 4a 4a 4a a

Special cases:
(i) If the constant term vanishes then one of the root is zero.
Explanation:
c 0
If c = 0, then α β = = =0
a a
i.e. either α = 0 or β = 0 or both α & β = 0

(ii) If the coefficient of x vanishes then the roots are equal in magnitude and opposite in sign.
Explanation:
b 0
If b = 0, α + β = − =− =0
a a
i.e. either α = – β or β = – α

Methods for Solving Quadratic Equation:


1. Factorisation
2. Using Square Roots
3. Using Quadratic Formula or Sridhar Acharya’s Formula

130 The Institute of Cost Accountants of India


Algebra

1. Factorisation
The Equation is solved by factorising the quadratic expression involved with the equation.
For example:

Equation: x 2 − 5 x + 6 = 0
Solution:
x 2 − 5x + 6 = 0
Or, x 2 − 2 x − 3x + 6 = 0
Or, x (x – 2) – (x – 2) = 0
Or, ( x − 2 )( x − 3) = 0
Or, x = 2 and x = 3
2. Using Square Roots
The value of x would be a square number when solving the equation.
3. Using Quadratic Formula or Sridhar Acharya’s Formula
Formula for Solving Quadratic Equation

−b ± b 2 − 4ac
x=
2a
Illustration 91.

Solve (5 x 2 − 6 x + 8) − (5 x 2 − 6 x − 7) = 1;

Answer:

( )
5 x 2 − 6 x + 8 − 5 x 2 − 6 x − 7 = 1 … … … (1)

Now ( 5 x − 6 x + 8 ) − ( 5 x − 6 x − 7 ) = 15
2 2

Or,
(5x 2
− 6 x + 8) − (5x 2 − 6 x − 7 )
=
15
[Substituting 5 x 2 − 6 x + 8 − 5 x 2 − 6 x − 7 = 1 from equation (1)]
5 x 2 − 6 x + 8 − 5 x 2 − 6 x − 7) 1

Or,
{( 5x 2 − 6 x + 8 −) (
2
) } = 15
5x 2 − 6 x − 7
2

{ 5x − 6x + 8 − 5x − 6x − 7) } 1
2 2

Or,
( 5x − 6x + 8 − 5x − 6x − 7 )( 5x − 6x + 8 +
2 2 2
5x 2 − 6 x − 7 ) = 15
5x 2 − 6 x + 8 − 5x 2 − 6 x − 7) 1

Or, ( )
5 x 2 − 6 x + 8 + 5 x 2 − 6 x − 7 = 15 … … … (2)

The Institute of Cost Accountants of India 131


Fundamentals of Business Mathematics and Statistics

By (1) + (2) we get 2 5 x 2 − 6 x + 8 = 16

Or, 5x 2 − 6 x + 8 = 8

Or, 5 x 2 − 6 x + 8 = 64

Or, 5 x 2 − 6 x − 56 = 0

6 ± 36 + 4 × 5 × 56
Or, x =
2×5

6 + 36 + 4 × 5 × 56 6 − 36 + 4 × 5 × 56
Or, x = and x =
2×5 2×5
Or, x = 4 and x = – 2.8

Illustration 92.

Solve x (2/3) + x (1/3) = 2;

Answer:

Put x1/3 = t we get


t2 + t − 2 = 0
Or, t2 + 2t – t – 2 = 0 Or, t (t + 2) – 1 (t + 2) = 0
Or, ( t + 2 )( t − 1) = 0
Or, t = −2 and 1
Or, x1/3 = −2 and 1
Or, x = −8 and 1

Illustration 93.

Evaluate 6 + 6 + 6 + 6 +……..∞

Answer:

Let x = 6 + 6 + 6 + 6 +……..∞

Or, x 2 = 6 + 6 + 6 + 6 + 6 +……..∞

Or, x 2 = 6 + x

Or, x 2 − x − 6 = 0

132 The Institute of Cost Accountants of India


Algebra

Or, ( x − 3)( x + 2 ) = 0

Or, x = 3 and – 2

Illustration 94.

1
2+
Evaluate 1 ;
2+
1
2+
2 +……….∞
Answer:
1
Let x = 2 +
1
2+
1
2+
2 +……….∞
Or, = +

Or, x 2 − 2 x − 1 = 0

2+ 4+4
Or, x = and 2 − 4 + 4
2 2

Or, x = 1 + 2 and 1 − 2

Illustration 95.

a+x + a−x a x≠0


Solve = ,
a+x − a−x x

Answer:
Rationalising the denominator of L.H.S we get

a+x + a−x a+x + a−x a


× = , x≠0
a+x − a−x a+x + a−x x

( )
2
a+x + a−x a
Or, =
(a + x ) − (a − x ) x

a+ x+a−x+2 a+ x × a−x a
Or, =
2x x

2a + 2 a + x × a − x a
Or, =
2x x
Or, a + a 2 − x 2 = a

The Institute of Cost Accountants of India 133


Fundamentals of Business Mathematics and Statistics

Or, x2 = a2
Or, x = a and – a

Illustration 96.

Solve ( x 2 + 2) 2 + 8 x 2 = 6 x ( x 2 + 2);

Answer:

Let x 2 + 2 = A

Or, A2 + 8 x 2 − 6 Ax = 0

Or, 8 x 2 − 6 Ax + A2 = 0

Or, 8 x 2 − 4 Ax − 2 Ax + A2 = 0

Or, 4 x ( 2 x − A) − A ( 2 x − A) = 0

Or, ( 2 x − A)( 4 x − A) = 0

A A
Or, x = and
2 4
Or, x 2 − 2 x + 2 = 0 and x 2 − 4 x + 2 = 0

2± 4−8 4 ± 16 − 8
Or, x = and
2 2
Or, x = 1 ± i and 2 ± 2

Illustration 97.
A trader bought a number of articles for ` 1200. Ten articles were damaged and he sold each of the rest at ` 2 more
than what he paid for, thus clearing a profit of ` 60 on the whole transaction. Taking the number of articles he
bought as x form an equation in x and solve it.

Answer:
Let x be the number of articles purchased at ` 1200
1200
So purchase price per unit =
x
1200
Therefore sale price per unit = +2
x
Since 10 articles were damaged so no of articles sold = x – 10
Since there is a profit of ` 60, then total sale proceeds will be = 1200 + 60 = ` 1260
1200
Therefore we can write ( x − 10 ) ×  + 2  = 1260
 x 
Or, ( x − 10 )(1200 + 2 x ) = 1260 x

134 The Institute of Cost Accountants of India


Algebra

Or, 1200 x − 12000 + 2 x 2 − 20 x = 1260 x


Or, 2 x 2 − 80 x − 12000 = 0
Or, x 2 − 40 x − 6000 = 0

Or, x = 40 ± 1600 + 4 × 6000


2
Or, x = 100 and – 60
x being physical quantity can never be negatie. So x = 100 units

Illustration 98.
Solve x ( x + 2)( x 2 − 1) = −1;

Answer:

x ( x + 2 ) ( x 2 − 1) = x ( x + 2 )( x + 1)( x − 1) = −1

Or, x ( x + 1)( x + 2 )( x − 1) = −1

Or, ( x 2 + x )( x 2 + x − 2 ) = −1

Let x 2 + x = A

So A ( A − 2 ) = −1

Or, A2 − 2 A + 1 = 0

Or, ( A − 1) = 0
2

Or, A =1
Or, x 2 + x − 1 = 0

Or, x = −1 ± 5
2
Illustration 99.
2 2
If α and β are the roots of the equation x 2 − 4 x + 2 = 0 , find the value of α + β ;

Answer:
As α and β are the roots of the equation x 2 − 4 x + 2 = 0 which is of the form ax2 + bx + c = 0 with a = 1, b = – 4
and c = 2, we can write
−b − ( −4) c 2
α+β = = = 4 and α β = a = 1 = 2
a 1
Therefore α2 + β2 = ( α + β ) − 2αβ = ( 4 ) − 2 × 2 = 16 − 4 = 12
2 2

Illustration 100.
If the roots of the equation x 2 − lx + m = 0 differ by 1, then prove that l 2 = 4m + 1 ;

The Institute of Cost Accountants of India 135


Fundamentals of Business Mathematics and Statistics

Answer:
Given equation x 2 − lx + m = 0 is of the form ax 2 + bx + c = 0 with a = 1, b = – l and c = m
Let the roots be α & α + 1
c
So α + α + 1 = −b = l & α (α + 1) = = m
a a
So 2α = l − 1
l −1
Or, α = & putting this value in α ( α + 1) = m
2
l −1 l −1 
Or, + 1 = m
2  2 
l −1 l +1
Or, × =m
2 2
Or, l 2 − 1 ≡ 4m

Or, l 2 = 4m + 1 (Proved)

Illustration 101.
If the equation x2 – ax + b = 0 and x2 – cx + d = 0 have one root in common and the second equation has equal roots,
then prove that ac = 2(b + d) ;

Answer:
The equation x2 – cx + d = 0 is of the form ax2 + bx + c = 0 with a = 1, b = – c and c = d

Since x 2 − cx + d = 0 has equal roots we can write


Discriminant = 0 i.e. b2 – 4ac = 0
c2 – 4d = 0
2
So d = c .... (1)
4
Putting this value in x 2 − cx + d = 0 we get
c2
x 2 − cx + =0
4
Or, 4 x 2 − 4cx + c 2 = 0

Or, ( 2 x − c ) = 0
2

c
Or, x =
2
Since equations x2 – ax + b = 0 and x2 – cx + d = 0 have one root in common, we can write
2
 c −a c +b = 0
2 2
 
c2 c
Or, −a +b = 0
4 2

136 The Institute of Cost Accountants of India


Algebra

c c2
Or, d − a + b = 0 [ From (1) d = ]
2 4
Or, ac = 2 (b + d)  (Proved)

Illustration 102.
If the roots of the equation 2 x 2 + ( k + 1) x + ( k 2 − 5k + 6 ) = 0 are of opposite sign then show that 2 < k < 3 ;

Answer:
Given equation 2x2 + (k + 1) x + (k2 – 5k + 6) = 0 is o the form ax2 + bx + c = 0 with a = 2, b = k + 1 and c = k2 – 5k + 6
Since the roots are of opposite sign as well as roots are real and distinct
So discriminant > 0 and product of roots < 0
c
Or, b2 – 4ac > 0 and <0
a
k 2 − 5k + 6
Or, (k + 1)2 – 4 × 2 × (k2 – 5k + 6) > 0 and <0
2
First condition is always true when second holds
k 2 − 5k + 6
Hence <0
2
Or ( k − 3)( k − 2 ) < 0
This is possible only if 2 < k < 3 (Proved)

Illustration 103.

If the equations 3x 2 + px + 1 = 0 and 2 x 2 + qx + 1 = 0 have a common root, show that 2 p 2 + 3q 2 − 5 pq + 1 = 0 ;

Answer:
Let α be the common root of the equations given. So we can write

3α2 + pα + 1 = 0 … … … (1) &

2α2 + qα + 1 = 0 … … … (2)
By (1) × 2 and (2) × 3 we get

6α2 + 2 pα + 2 = 0 … … … (3) &

6α2 + 3qα + 3 = 0 … … … (4)


By (4) – (3) we get

α ( 3q − 2 p ) = −1
1
So α = −
3q − 2 p

The Institute of Cost Accountants of India 137


Fundamentals of Business Mathematics and Statistics

Putting this value in (1)


1 p
3× − +1 = 0
( 3q − 2 p )
2
3q − 2p

Or, 3 − p ( 3q − 2 p ) + ( 3q − 2 p ) = 0
2

Or, 3 − 3 pq + 2 p 2 + 9q 2 − 12 pq + 4 p 2 = 0

Or, 6 p 2 + 9q 2 − 15 pq + 3 = 0

Or, 2 p 2 + 3q 2 − 5 pq + 1 = 0 (Proved)

Illustration 104.
α
If α & β be the roots of the equation 2 x 2 − 3x + 1 = 0 find an equation whose roots are & β
2β + 3 2α + 3
Answer:
Given equation 2x2 – 3x +1 = 0 is of the form ax2 + bx + c = 0 with a = 2, b = – 3 and c = 1
−b 3 , α β = c = 1
α+β = =
a 2 a 2
The required equation is x2 – (sum of the roots) x + product of the roots = 0

2α 2 + 3α + 2β 2 + 3β 2 ( α + β ) + 3 ( α + β ) 2 ( α + β ) − 4αβ + 3 ( α + β )
2 2 2
α β
Sum of the roots = + = = =
2β + 3 2α + 3 ( 2β + 3)( 2α + 3) ( 4αβ + 6α + 6β + 9) {4αβ + 6 (α + β) + 9}

2 ( α + β ) − 4αβ + 3 ( α + β )
2 2× ( 94 ) − 4 ( 12 ) + 3( 32 ) = 4.5 − 2 + 4.5 = 7
(4 × 12 + 6 × 23 + 9) 2 + 9 + 9 20
Or, Sum of the roots = =
{4αβ + 6(α + β) + 9}
1 1
α β αβ αβ 2 1
Product of the roots = × = = = = 2 =
2β + 3 2α + 3 ( 4αβ + 6α + 6β + 9) 4αβ + 6 (α + β) + 9 1 3 20 40
4⋅ + 6⋅ + 9
2 2
2 7 1
So the required equation is x − x+ =0
20 40
Or, 40 x 2 − 14 x + 1 = 0

Illustration 105.

x 1− x 1
Solve + =2
1− x x 6

Answer:

x
Put = t2
1− x

138 The Institute of Cost Accountants of India


Algebra

1 13
We get t + =
t 6
Or 6t 2 + 6 = 13t

Or 6t 2 − 13t + 6 = 0

Or 6t 2 − 4t − 9t + 6 = 0
Or 2t (3t – 2) – 3 (3t – 2) = 0

Or ( 2t − 3)( 3t − 2 ) = 0

3 2
Or t = and
2 3
3 x 9 x ). So 4x = 9 – 9x
Now t = Or, = (Substituting t =
2 1− x 4 1− x
9
Or 13x = 9. So x =
13
Also
2 x 4
t= Or, =
3 1− x 9

Or 9 x = 4 − 4 x
4
Or 13x = 4. So x =
13
9 4
Required solution is x = and
13 13
Illustration 106.

For 16a2 + 16a + 1 = 0, prove that the roots are real

Answer:
16a2 + 16a +1 = 0 ≡ ax2 + bx + c = 0
So, a = 16, b = 16, c = 1

Formula for finding the value of Discriminant: b2 − 4ac

= (16)2 – 4 × (16) × (1)

= 256 – 64

= 192 > 0
Since, b2 – 4ac > 0, roots of the equation 16a2 + 16a +1 = 0 are Real. (Proved)

Illustration 107.
For 12a2 + 10a + 2 = 0 prove that the roots are rational and real

The Institute of Cost Accountants of India 139


Fundamentals of Business Mathematics and Statistics

Answer:
12a2 + 10a + 2 = 0 ≡ ax2 + bx + c = 0
So, a = 12, b = 10, c = 2

Formula for finding the value of Discriminant: b2 − 4ac

(10)2 – 4 × (12) × (2) = 100 – 96 = 4 > 0

Since, b2 – 4ac > 0, roots of the equation 12a2 + 10a +2 = 0 are real. Since, b2 – 4ac is a perfect square (22 = 4),
roots are rational and real. (Proved)

Illustration 108.
For 32a2 + 16a + 2 = 0 prove that the roots are real & equal

Answer:
32a2 + 16a +2 = 0 ≡ ax2 + bx + c = 0
So, a = 32, b = 16, c = 2

Formula for finding the value of Discriminant: b2 − 4ac


= (16)2 – 4 × (32) × (2)
= 256 – 256
=0
Since, b2 – 4ac = 0, roots of the equation 16a2 + 16a +1 = 0 are Real and Equal.

Illustration 109.
If sum of roots is -7 and product of roots is -14, form a quadratic equation.

Answer:
For forming a Quadratic Equation, the following formula needs to be used:
Formula:
x2 - (Sum of the Roots) × x + (Product of the Roots) = 0
Sum of the Roots: α + β = –7
Product of the Roots: α × β = –14
Required equation is x2– (–7) × x + (–14) = 0

Or, x 2 + 7 x − 14 = 0
Quadratic Equation is x2 + 7x – 14 = 0 when sum of the roots is -7 and product of the roots is -14.

Illustration 110.

Identify whether the equation 25d 2 + 36e 2 + 5 = 1 is Quadratic Equation or not.

140 The Institute of Cost Accountants of India


Algebra

Answer:
No, the given equation is not a polynomial equation with degree 2 for same variable. The given equation has 2
different variables. Hence, it is not a Quadratic Equation.

Illustration 111.
Determine the nature of roots for the equation: 10y2 + 11y + 3 = 0
Answer:
Quadratic equation: 10y2 + 11y + 3 = 0 ≡ ax2 + bx + c = 0
So, a = 10, b = 11, c = 3
Formula for Discriminant (D): b2 – 4ac
= (11)2 – 4 × (10) × (3)
= 121 – 120 = 1 > 0
Since, value of discriminant (D) > 0, the roots are Real and also D is a perfect square, the roots are rational.
Hence, the roots of the given equation are real and rational.

Illustration 112.
Determine the nature of roots for the equation: 3y2 – 5y + 3 = 0

Answer:
Quadratic equation: 3y2 + (–5)y + 3 = 0 ≡ ax2 + bx + c = 0
So, a = 3, b = –5, c = 3
Formula for Discriminant (D): b2 – 4ac
= (–5)2 – 4 × (3) × (3)
= 25 – 36 = –11 < 0
Since, value of the discriminant (D) < 0, the roots are Imaginary.

Illustration 113.
When two roots are equal and of the value 5 then form the quadratic equation.

Answer:
Let the two roots be α and β,
Given condition, both roots are equal and the value is 5,
To form a Quadratic Equation, the Formula is: x2 – (Sum of the Roots) × x + (Product of the Roots) = 0
Sum of Roots = α + β = 5 +5 = 10
Product of Roots: α × β = 5 × 5 = 25
Required Quadratic Equation is: x2 – (10) × x + (25) = 0
Or, x2 – 10x + 25 = 0.

The Institute of Cost Accountants of India 141


Fundamentals of Business Mathematics and Statistics

Illustration 114.
If the roots of a quadratic equation are reciprocal and one root be 1/2 then form the quadratic equation.

Answer:
Let the two roots be α and β,
Given condition, One root is reciprocal of the other one and the value is ½ ,
Value of other root would be 2.
To form a Quadratic Equation the Formula is: x2 – (Sum of the Roots) × x + (Product of the Roots) = 0
Sum of the Roots: α + β = 1/2 + 2 = 5/2
Product of the Roots: α × β = 1/2 × 2 = 1
Required Quadratic Equation is: x2 – (5/2) × x + (1) = 0 Or, 2x2 – 5x + 2 = 0.

Illustration 115.
Solve using factorisation method: y2 – 14y + 49 = 0

Answer:
Given quadratic equation: y2 – 14y + 49 = 0

or, y2 – 7y – 7y + 49 = 0

or, y (y – 7) –7 (y – 7) = 0

or, (y – 7) (y – 7) = 0

(y – 7) = 0 and (y – 7) = 0

y = 7 and y = 7.

Hence, the value of y in y2 – 14y + 49 = 0 would be 7.

Illustration 116.
Solve using square roots method: y2 – 10y + 25 = 0

Answer:
Given quadratic equation: y2 – 10y + 25 = 0
or, y2 – 2 × 5y + (5)2 ≡ (y – 5)2 = 0
or, (y – 5) = 0
or, y = 5.
Hence, the value of y in y2 – 10y + 25 = 0 would be 5.

Illustration 117.

Solve: 11x2 +12x –5 = 0.

142 The Institute of Cost Accountants of India


Algebra

Answer:
Given Quadratic Equation: 11x2 +12x –5 = 0 cannot be solved using either factorisation method or square roots
method, it would be solved using following formula.
Formula for Solving Quadratic Equation

−b ± b2 − 4ac
x=
2a

Given Equation: 11x2 + 12x – 5 = 0 is of the form ax2 + bx + c = 0 with a = 11, b = 12, c = –5
Replacing values of a, b and c in formula:

(12 ) − 4 × 11 × ( −5)
2
−12 ±
x=
2 × 11

−12 ± 144 + 220


Or, x =
22

Or, x = −12 ± 364


22

Or, x = −12 ± 19.08


22

Or, x = −12 + 19.08 and x = −12 − 19.08


22 22

Or, x = 7.08 and x = −31.08


22 22
Or, x = 0.32 and x = – 1.41
Therefore, the values of x for 11x2 + 12x –5 = 0 are 0.32 and –1.41.

Illustration 118.

Solve x 2 − 7 x + 12 = 0

Answer:
Given equation can be expressed as x 2 − 3x − 4 x + 12 = 0 or, x ( x − 3) − 4 ( x − 3) = 0

Or, ( x − 3)( x − 4 ) = 0 Or, x = 3, 4

Alternatively from the given equation we can write a = 1, b = – 7 & c = 12

−b ± b2 − 4ac − ( −7 ) ± ( −7 )
2
− 4.1.12 7 ± 49 − 48
So x = = =
2a 2.1 2

7 + 49 − 48 7 − 49 − 48
Or, x = and i.e. x = 4 and 3
2 2

The Institute of Cost Accountants of India 143


Fundamentals of Business Mathematics and Statistics

Illustration 119.
Solve x 4 − 10 x 2 + 9 = 0

Answer:
Taking x2 = u we get the transformed form of the given equation as u2 – 10u + 9 = 0
Or, ( u − 9 )( u − 1) = 0. So u = 9 and u = 1
Or, u = x2 = 9. So x = ± 3. & u = x2 = 1, So x = ± 1.
Here the power of x is 4, so we get four values of x.

Illustration 120.
1 1 1
Solve ( x + 1) 3 + (1 − x ) 3
=2 3

Answer:

Cubing both sides we get (1 + x ) + (1 − x ) + 3 ( x + 1) 3 × (1 − x ) 3 ( x + 1) 3 + (1 − x ) 3  = 2


1 1 1 1

 
1 1 1 1 1 1
Or, 2 + 3 ( x + 1) 3 × (1 − x ) 3 2 3
= 2 [Since ( x + 1) 3 + (1 − x) 3 = 2 3 ]

Or, 3. (1 − x 2 ) .21/3 = 0
1/3

Or, (1 − x 2 )
1/3
= 0 , or x2 = 1, Or x = ± 1

Illustration 121.
6− x x
Solve = +2
2
x −4 x + 2
Answer:
6− x x
= +2
x2 − 4 x + 2
x
Or, ( 6 − x ) = × ( x2 − 4) + 2 ( x2 − 4)
x+2
Or, 6 − x = x ( x − 2 ) + 2 ( x − 4 )
2

Or, 6 − x = x 2 − 2 x + 2 x 2 − 8
Or, 6 − x = 3x 2 − 2 x − 8
Or, 3x − x − 14 = 0

Drawing similarity of the above equation with ax2 + bx + c = 0 we can write a = 3, b = – 1 & c = – 14 here.

−b ± b2 − 4ac − ( −1) ± ( −1) − 4.3. ( −14 )


2
1 ± 1 + 168
So x = = =
2a 2.3 6

1 + 169 1 − 169
Or, x = and i.e. x = 7/3 and – 2
6 6

144 The Institute of Cost Accountants of India


Algebra

Alternatively : 3x2 – x – 14 = 0, Or, 3x2 – 7x + 6x – 14 = 0


Or, 3x2 + 6x – 7x – 14 = 0 Or, 3x (x + 2) – 7 (x + 2) = 0
Or, (x + 2) (3x – 7) = 0 Or, x = – 2 and 7/3
Now x = – 2 yields undefined result when substituted in the given equation. Thus x ≠ – 2.
So the required value of x is 7/3

Illustration 122.
Solve 4 x − 3.2 x+2 + 25 = 0

Answer:
4 x − 3 ⋅ 2 x+ 2 + 25 = 0
Or, 22 x − 3.2 x 22 + 32 = 0 Or, (2 x ) 2 − 12.2 x + 32 = 0
So Let 2x = z, we get z2 – 12z + 32 = 0
Or, z2 – 4z – 8z + 32 = 0
Or, z (z – 4) – 8 (z – 4) = 0
Or, (z – 4) (z – 8) = 0
Or, z = 4 and z = 8
Or, z = 2x = 4 = 22 and z = 2x = 8 = 23
Or, x = 2 and x = 3

Illustration 123.
Solve x 2 + 7 x + x 2 + 7 x + 9 = 3

Answer:

x2 + 7x + x2 + 7x + 9 = 3
Adding 9 to both sides, we have x 2 + 7 x + 9 + x 2 + 7 x + 9 = 12

Now substituting x 2 + 7 x + 9 = u we get

u 2 + u − 12 = 0
Or (u + 4) (u – 3) = 0
So u = – 4 and 3.

So x2 + 7 x + 9 = 3
Squaring both sides we get x2 + 7x + 9 = 9 Or, x2 + 7x = 0 Or, x (x + 7) = 0
So x = 0 and – 7

Also x 2 + 7 x + 9 = −4 Or, x2 + 7x + 9 = 16 Or, x2 + 7x – 7 = 0


−7 ± 72 − 4.1.( −7) −7 ± 49 + 28 −7 ± 77
So x = Or, x = So x =
2.1 2 2

The Institute of Cost Accountants of India 145


Fundamentals of Business Mathematics and Statistics

Illustration 124.

Solve: ( x 2 + 3x ) + 2 ( x 2 + 3x ) = 24
2

Answer:

Let x 2 + 3x = A ,

(x + 3x ) + 2 ( x 2 + 3x ) = 24 Or, A2 + 2 A − 24 = 0
2 2

Or, A2 + 6A – 4A – 24 = 0
Or, A(A + 6) – 4(A + 6) = 0
Or, (A + 6) (A – 4) = 0
Either, A = x2 + 3x = – 6 Or A = x2 + 3x = 4
Either, x2 + 3x + 6 = 0 Or, x2 + 3x – 4 = 0
From x2 + 3x + 6 = 0

−b ± b2 − 4ac − ( 3) ± ( 3) − 4.1.6 −3 ± 9 − 24
2

x= = = = Not feasible because 9 − 24 = −15 which is


2a 2.1 2
Imaginery Number.

From x2 + 3x – 4 = 0

−b ± b 2 − 4ac − (3) ± (3)2 − 4.1.( −4) −3 ± 9 + 16


x= = = = −4 and 1
2a 2.1 2

146 The Institute of Cost Accountants of India


Algebra

Exercise:
Theoretical Questions
 Multiple Choice Questions (MCQ)

In a customer feedback survey of 60, it was found Coke was liked by 24, Fanta by 27, Coca Cola by 11, Thums
Up by 9. While, 18 liked both Fanta and Coke and 15 liked both Thums Up and Coca Cola and 11 liked all.
Based on the above information answer Q. Nos. 1 to 10.
1. How many people liked any one of the products?
(a) 33
(b) 18
(c) 11
(d) 27
2. How many people liked Fanta?
(a) 18
(b) 11
(c) 24
(d) 27
3. How many people did not like any of the products?
(a) 33
(b) 13
(c) 9
(d) 15
4. How many people liked either Fanta orThumsUp?
(a) 27
(b) 36
(c) 37
(d) 28
5. How many people liked Fanta, Coke or Thums Up?
(a) 42
(b) 53
(c) 41
(d) 54

The Institute of Cost Accountants of India 147


Fundamentals of Business Mathematics and Statistics

6. How many people liked Mazaa?


(a) 0
(b) Not available as product (None of them)
(c) Zero
(d) All of the above
7. How many people liked any two products?
(a) 18
(b) 15
(c) 33
(d) 22
8. How many people liked Thums Up, Coke, Coca Cola and Fanta?
(a) 11
(b) 49
(c) 60
(d) 50
9. How many people liked all the products?
(a) 60
(b) 11
(c) 09
(d) 12
10. How many people liked either Coke or Thums Up?
(a) 24
(b) 11
(c) 09
(d) 33
11. If Set A = {Q, W, E, R, T, Y} and Set B = {B, G, R, E, O, K}, find (A-B).
(a) Set (A-B) = {Q, W, T, Y}
(b) Set (A-B) = { B, G, O, K}
(c) Set (A-B) = { E, R}
(d) None of the Above
12. If A={W, N, M, I}; B={Q, E, H, J}; C= {I, L, E, R}. Find A  (B  C).
(a) {W, Q, E, H, J}
(b) {Q, I, L, E, R}

148 The Institute of Cost Accountants of India


Algebra

(c) {E, W, N, M, I}
(d) {T, I, L, E, R}
13. If Set B = {B, G, R, E, O, K} is a subset of Set A= {Q, E, R, T, Y, U, I, O, P, K, G, B}, find B′.
(a) Set B′ = {B, G, R, E}
(b) Set B′ = {Q, E, G, B}
(c) Set B′ = {E, R, T, Y}
(d) Set B′ = {Q, T, Y, U, I, P}
14. If Set A={8, 9, 7, 5, 6, 2} and Set B = { 1, 4, 9, 3, 8, 2}, find the union of B and A.
(a) Set (B  A) = { 8, 9, 7, 5, 6, 2, 1, 4, 9, 3, 8, 2 }
(b) Set (B  A) = { 8, 9, 2 }
(c) Set (B  A) = { 1, 2, 3, 4, 5, 6, 7, 8, 9 }
(d) None of the Above
15. If P1= {78, 77, 76, 75}; P2= {71, 72, 73, 74}; P3 = {80, 79, 70, 69}.
Find (P1 P2)  (P2P3).
(a) (P1 P2)  (P2P3) = { }
(b) (P1 P2)  (P2P3) = {}
(c) Both ‘a’ and ‘b’
(d) None of the Above
16. If A= {W, R, Y, I}; B= {Q, E, T, U}; C= {Q, W, E, R}. Find A  (B  C).
(a) A (BC) = {W, R}
(b) A  (BC) = {Q, E}
(c) A (BC) = {Y, I}
(d) A (BC) = {T, U}
17. If Set A = {Q, W, E, R, T, Y} and Set B = {B, G, R, E, O, K}, find (B-A).
(a) Set (B-A) = {Q, W, T, Y}
(b) Set (B-A) = { B, G, O, K}
(c) Set (B-A) = { E, R}
(d) None of the Above
18. If A= {8, 11, 14, 17}; B= {5, 7, 9, 15}; C= {19, 21, 17, 15}. Find A  (BC).
(a) A  (BC) = {8, 11, 14, 15, 17}
(b) A  (B  C) = {8, 5, 7, 9, 15}
(c) A  (B  C) = {11, 19, 21, 17, 15}
(d) A (B  C) = {11, 19, 21, 15, 8}

The Institute of Cost Accountants of India 149


Fundamentals of Business Mathematics and Statistics

19. In Venn diagram, Universal Set is represented by ______.


(a) Stars
(b) Squares
(c) Rectangle
(d) Circles
20. According to De Morgan’s Law: (B C)′ = _________.
(a) B′  C′
(b) B′  C′
(c) B  C′
(d) B′  C
21. Find the value of: 39×33
(a) 312
(b) 531441
(c) 19683×27
(d) All of the above
22. Find the value of (2.85)t, when 0.0016 = (1/5) 4t.
(a) 2.85
(b) (2.85)2
(c) (2.85)–1
(d) Either ‘b’ or ‘c’
23. When the value of p > m, simplify qp × qm.
(a) qm+p
(b) qm-p
(c) qm×p
(d) None of the above
24. What will be the value of (93)2 ?
(a) 59004 × 9
(b) 59049 × 9
(c) 49005 × 9
(d) 49059 × 9
25. Find the value of 1178/1181 ?
(a) 113
(b) 1 / 113

150 The Institute of Cost Accountants of India


Algebra

(c) 1331
(d) –1331
26. What will be the value of 36×34×3-2×3-3×36 ?
(a) 321
(b) 316
(c) 319
(d) 311
27. Find the value of 63×6-2×6-5×64
(a) 0
(b) 1
(c) 6
(d) 216

()
a /b
a
28. Find the value of b when = (a) a /b−1 and a = 2b
b
(a) 1
(b) 2a
(c) 22
(d) a2
29. What will be the value of (73)2 ?
(a) 117649
(b) 196471
(c) 16807
(d) 18607
30. What will be the value of 96×9-4×92×93×96 ?
(a) 913
(b) 921
(c) Both ‘a’ and ‘b’
(d) None of the above
31. Find the value of (92/ 33) × (34/9)?
(a) 34
(b) 35
(c) 33
(d) 36

The Institute of Cost Accountants of India 151


Fundamentals of Business Mathematics and Statistics

32. Find the value of (1.25)q, when (1/3)4q = (6561)–1


(a) 1.5 + 0.0625
(b) 1.5625
(c) (1.25)2
(d) All of the above

33. Simplify 3(a1)2 b1 / 6b2(a3)1


(a) 1/(2ab)
(b) ½ × ab
(c) 2ab
(d) None of the above

21/3 ⋅ 82/3 ⋅ 6 −5/ 4 ⋅ 3−3/ 4 ⋅ 9


34. Evaluate 3
16
1
(a) 4
2
(b) 2
(c) 4
2
(d) None of the above

35. Simplify 77/9 × 72/9 × 74/9 × 7-1 × 7-4/9


(a) 7
(b) 1
(c) 0
(d) None of the above
36. When ax = by = cz and b2 = ac then 1/x, 1/y, 1/z should be in ———
(a) G.P
(b) A.P
(c) H.P
(d) None of the above
37. What will be the value of 7–5 × 7–8 × 74 × 73 × 76 ?
(a) –7
(b) 1
(c) 7
(d) None of the above

152 The Institute of Cost Accountants of India


Algebra

38. Find the value of (33/ 92) × (9/34)?


(a) 3-1
(b) 3-2
(c) 3-3
(d) 3-7
39. What will be the value of (63×62)/(6-3×64) ?
(a) (62)2
(b) 65
(c) 6
(d) None of the above
40. Find the value of 199 / 192
(a) 192
(b) 197
(c) 199
(d) 190

41. Express log 9 1 = 0 in exponential form.

(a) 91 = 0

(b) 90 = 1

(c) 19 = 1

(d) 09 = 0

42. Express 43 = 64 in logarithmic form.

(a) log 4 6 = 3

(b) log 64 3 = 4

(c) log 4 64 = 3

(d) log 3 64 = 4

43. Solve : log10 (8 y + 3) = 3

1003
(a)
8

997
(b)

The Institute of Cost Accountants of India 153


Fundamentals of Business Mathematics and Statistics

1008
(c)
3
992
(d)
3

44. If log 4 (t − 1) + 1 = 2 × log 4 t , find the value of t.


(a) 1
(b) 0
(c) 4
(d) 2

45. Find t when log 3 t = 6


(a) 729
(b) 27
(c) 81
(d) 243

46. Find the logarithm of 324 with the base 3 2 .


(a) – 4
(b) 0
(c) – 2
(d) 4

47. Find the value of z : 100z = z log10 z


(a) 100

1
(b)
10
(c) Both ‘a’ and ‘b’

(d) None of the above

48. For the logarithms to be in A. P. Series, the positive numbers should be following ____.

(a) H. P.

(b) G. P.

(c) A. P.

(d) None of the above.

154 The Institute of Cost Accountants of India


Algebra

1 1
49. If log y = , what will be the value of base?
2 3
1
(a)
2

1
(b)
4

1
(c)
8

1
(d) −
2

50. Find t when log t 3125 = 5


(a) 5
(b) 125
(c) 25
(d) 625

25 81 10
51. State True or False: log 2 + 2 log = 3log + 7 log
24 80 9
(a) True

(b) False

(c) Not possible

(d) None of the above

52. State True or False: log 2 10 + log8 125 = 1


(a) Not possible
(b) True
(c) May be true
(d) None of the above

53. State True or False: log 32 2.15 when log 2 = 0.30103 .


(a) False
(b) True
(c) LHS > RHS
(d) LHS < RHS

The Institute of Cost Accountants of India 155


Fundamentals of Business Mathematics and Statistics

54. Find the value of the logarithm of 2nd number (b) for 3 consecutive numbers (a, b, c).
(a) log (1 + ac)
1
(b) × 2 × log (1 + ac)
2
1
(c) × log (1 + ac)
2
(d) 2 × log (1 + ac)

55. If log y 32 = 10 , then which of the following is the value of y ?


(a) 4
(b) 2
(c) 4
(d) 2
56. Find the base when 3 is the logarithm of 729.
(a) 7
(b) 11
(c) 9
(d) 3

57. If log (7 y − 5) = 2 , find the value of y.


(a) 15
(b) 10
(c) 08
(d) 07
58. For any series of positive numbers which are in G. P., their logarithms would be in ______.
(a) G. P.
(b) A. P.
(c) H. P.
(d) None of the Above

59. Solve: log (9t − 2) = 2


102
(a)
9
98
(b) 9
109
(c) 2
91
(d) 2

156 The Institute of Cost Accountants of India


Algebra

60. Find the logarithm of 125 to the base 5 5


(a) 6
(b) 4
(c) 3
(d) 2

12!
61. Compute
(8! × 4!)
(a) 3960
b. 495
c. 99
d. 440
62. Find the value of v, if (v –1)! × 20 = (v + 1)!
(a) 0
(b) 2
(c) 4
() 5
63. Determine the number of ways in which a digit and a vowel can be selected from 10 digits and 26 letters
of Alphabet?
(a) 50
(b) 210
(c) 105
(d) None of the Above
64. Find the LCM of {12!, 14!, 13!}
(a) 11!
(b) 15!
(c) 12!
(d) 14!
65. When 9 local trains are running between Haldia and Burdwan. In how many ways can a passenger travel
from Burdwan to Haldia and return by a different train?
(a) 72 ways.
(b) 70 ways.
(c) 64 ways.
(d) 60 ways.

The Institute of Cost Accountants of India 157


Fundamentals of Business Mathematics and Statistics

66. If nP6 : nP5 = 1:3, find the value of n.


(a) 3
(b) 4
(c) 7
(d) 9

2 1 1
67. Find the value of − −
36! 35! 35!
(a) –35 /18!
(b) –1 / 18!
(c) –2 / 36!
(d) –70 /36!

68. Find the number of permutations for 15 scooters if 3 scooters are to be considered at a time.
(a) 2730
(b) 2370
(c) 2184
(d) 2814

10!− 9!
69. Find the value of
8!
(a) 81
(b) 80
(c) 10
(d) 9

70. How many ways can 5 drivers refill their tanks from 5 refills, assuming no refills in the fuel station remain
unused?
(a) 24 ways.
(b) 120 ways.
(c) 5 ways.
(d) 60 ways.

71. In how many ways 8 cardholders stand in a queue at fair price shop?
(a) 40320 ways
(b) 40230 ways
(c) 20430 ways
(d) 20340 ways

158 The Institute of Cost Accountants of India


Algebra

72. Find the number of permutations for 11 bikes if 5 bikes are to be taken at a time.
(a) 54540
(b) 55440
(c) 44550
(d) 45450

9 3 12
73. Find the value of − +
7! 6! 7!
(a) 0
(b) 1/ 7!
(c) –1/7!
(d) None of the above

74. If nP7: nP8 = 4:1, find the value of n.


(a) 12
(b) 10
(c) 11
(d) 13

75. Find the value of a, if (a – 2)! × 24 = (a + 1)!


(a) 0
(b) 2
(c) 4
(d) 3

76. What would be the factorial notation for: 11×10×9×8×7


(a) 11! / 6!
(b) 11! / 5!
(c) 10! / 6!
(d) 10! / 5!

77. How many ways can 8 people get vaccinated from 8 vaccinators, assuming no vaccinator is idle?
(a) 40320 ways.
(b) 5040 ways.
(c) 5760 ways.
(d) 35280 ways.

The Institute of Cost Accountants of India 159


Fundamentals of Business Mathematics and Statistics

78. In how many different ways can 4 different cars, one of each of the 4 manufacturers, be parked in a parking
lane?
(a) 20 ways
(b) 22 ways
(c) 24 ways
(d) 26 ways

3 2 2
79. Find the value of − −
13! 12! 12!
(a) – 49 /13!
(b) –30 / 13!
(c) –25 / 12!
(d) –50 /12!
80. In how many ways 6 customers stand in a queue for depositing cash in bank?
(a) 680 ways
(b) 480 ways
(c) 600 ways
(d) 720 ways

81. Examine the nature of the roots for the following equation 16 x 2 − 24 x + 9 = 0
(a) Real and unequal
(b) Real, Irrational and Equal
(c) Real, Rational and Equal
(d) Unreal and Imaginary
82. For what value of c, would the product of roots be zero?
(a) a = 1, b = 1, c = 1
(b) a = 1, b = 0, c = 1
(c) a = 1, b = 1, c = 0
(d) a = 1, b = 2, c = 2
83. Form the equation whose roots are 9, –4

(a) x + 5x − 36 = 0

(b) x 2 − 5x − 36 = 0

(c) x 2 − 5x + 36 = 0

(d) x 2 + 5x + 36 = 0

160 The Institute of Cost Accountants of India


Algebra

84. Form quadratic equation with roots as a − t, a + t

(a) x 2 − 2ax + a 2 − t 2 = 0

(b) x 2 + 2ax + a 2 − t 2 = 0

(c) x 2 − 2ax − a 2 + t 2 = 0

(d) x 2 + 2ax − a 2 + t 2 = 0

Solve Q85 – Q90 using the following information


When α and β are the roots of 5x 2 − 7 x + 9 = 0 then find the values of:
85. 1/β + 1/ α
(a) 7/9
(b) -7/9
(c) 9/7
(d) -9/7

86. β 2 + α 2
(a) 41/25
(b) -41/25
(c) 139/25
(d) -139/25

87. Sum of the Roots


(a) 9/5
(b) 5/9
(c) -7/5
(d) 7/5

88. Product of the Roots


(a) -7/5
(b) 7/5
(c) -9/5
(d) 9/5

89. α / β + β / α
(a) 369/5
(b) -41/45

The Institute of Cost Accountants of India 161


Fundamentals of Business Mathematics and Statistics

(c) -369/5
(d) 41/45

90. α + β + 4
(a) 27/5
(b) 29/5
(c) 28/5
(d) 24/5
91. If b2 – 4ac > 0, is a perfect square, the nature of roots would be
(a) Real and Equal
(b) Imaginary
(c) Unreal
(d) Real and Unequal
92. For what values of a and b, the product of roots would be equal to c?
(a) a = c ≠ b
(b) a ≠ b = c
(c) a = 1
(d) a ≤ 0

93. Choose the correct form (From the alternatives given below) in which quadratic equation is generally
written -
(a) ax 2 + bx + c = 0
(b) x 2 + ax − b = 0
(c) ax 2 − bx + c = 0
(d) x − bx + c =
94. Choose the correct condition for any equation to be Quadratic equation-
(a) 5 constants and all constants > 0.
(b) 5 constants and all constants < 0.
(c) 3 constants and a > 0.
(d) 3 constants and a ≠ 0.

95. Which one of the following is Discriminant of a quadratic equation?


(a) −b + b 2 − 4ac
(b) −b − b 2 − 4ac
(c) b 2 − 4ac

(d) b 2 + 4ac

162 The Institute of Cost Accountants of India


Algebra

96. When are nature of roots real rather than imaginary?


(a) If Discriminant is negative
(b) If Discriminant is less than zero
(c) If Discriminant is not a perfect square
(d) If Discriminant is more than or equal to zero
97. For what values of a and c, value of sum of the roots would be equal to b.
(a) a = 1, c = n
(b) a = −1, c = n
(c) a = n, c = −1
(d) a = n, c = 1
98. For any sum of roots of quadratic equation, ‘b’ represents -
(a) Coefficient of x
(b) Constant term
(c) Coefficient of x2
(d) All of the above

99. Find the square of the difference of the roots of 115 + 5 ( x 2 − 12 x ) = 0


(a) 24
(b) 48
(c) 26
(d) 52
100. For any sum of roots of quadratic equation, ‘a’represents -
(a) Coefficient of x
(b) Coefficient of x2
(c) Constant term
(d) None of the above

The Institute of Cost Accountants of India 163


Fundamentals of Business Mathematics and Statistics

 State True or False

1. The statement (A ∩ B)1 = A1U B1 is true (or) False


2. If the set A has 4 elements, B has 3 elements then the number of elements in A × B is 12 elements
3. The Statement {2} ⊂/ {2, 3, 5} is true (or) False
4. The statement {1} ⊂ {1, 2, 3} is true (or) False
5. The statement “Equivalent sets” are always equal is True (or) False
3
6. If x = 2 + 1 − 3 2 − 1 then the value of x 3 + 3x is 2

1 1
7. If x = 5 + 2 6 and xy = 1 , then + is 89
x2 y2

9 10
8. If 2 x + 2 y = 22 x − y = 8 , then x = and y =
10 3
9. If x = 2 + 5 , then x 3 + 3x 2 − 29 x is 7
10. log 3 + log 5 is log 15
11. The value of log2 log2 log3 81 = 2
1
12. The logarithm of 324 to base is – 4
3 2
13. The logarithms with base 10 are called Natural logarithm
14. The logarithms with base e are called common Logarithm
15. The logarithm of one to any base is zero
16. The logarithm of a number which is not equal to one with itself as base is zero
17. The integral part of the value of logarithm of a number is called characteristic
18. The decimal part of the value of logarithm of a number is called mantissa
19. The logarithm of same number for different bases are different
20. A polygon has 44 diagonals then the number of its sides are 8
21. The total number of 9 digit numbers which have all different digits is 9 × 9
22. There are 8 questions in an examination paper and each question has an alternative. The number of ways
in which a student can give his answer is 6561
23. In a football competition there were 153 matches. A match occurs between two teams. The total numbers
of teams took part in the competition is 18
24. If nC n = 1 , then 0! =1

25. If the roots of the equation 2 x 2 + 8x + c = 0 are equal then c = 8

26. The g. c. d of the equations 2 x 2 − x − 1 = 0 and 4 x 2 + 8x + 3 = 0 is 3x + 1

27. x 2 − 4 x − 1 = 0 is the quadratic equation whose roots are 2 and 5

164 The Institute of Cost Accountants of India


Algebra

28. The roots of the equation ( x − 4) ( x − 2)( x + 4) = 0 are 4, – 4, 2 and –2


2

29. The degree of the expression 3x 5 + xyz 2 + y 3 is 3

 Fill in the Blanks


1. If A = {1, 2, 3, 4}, B = {2, 4, 6}, then A Δ B is ________ [AΔB=(A-B)(B-A) and is knwon as
the symmetric difference of two sets]
2. If A and B are two sets then A ∩ (B-A) is ________

3. If A = {1, 2, 3}, B = {2, 3, 4}, C = {1, 2, 5, 6) then AU (B ∩ C) is ______

4. IF A and B are two sets then A ∩ B = A  B if and only if ______

5. If A and B are two disjoint sets then n (A  B) is equal to ______

6. If A has 2 elements, B has 4 elements and A ⊂ B then A ∩ B has ______ elements

7. If A and B are the two sets of positive and negative integers respectively then A  B is ________

8. If a1/3 + b1/3 + c1/3 = 0 , then ( a + b + c ) = ______


3

9. If y = x1/3 − x −1/3 , then y3 +3y = ______

10. If a = 2 + 3 2 + 3 4 , then a 3 − 6a 2 + 12a = ______

11. If 64 x = 2 2 , then x = ______

12. If x = 8, y = 27 , then the value of ( x 4/3 + y 2/3 )1/ 2 = ______


1
13. If log x 0.1 = then the value of x is: _____
3
14. If log 32 x = 0.8 then x is equal to: __________

15. If log 4 x + log 2 x = 6 then x is equal to: ____________


1 1
16. If log8 x + log8 ( ) = then the value of x is: ___________
6 3
17. If log 2 = 0.30103, then the number of digits in 450 is: ________

18. If log 2 = 0.30103, then the number of digits in 520 is: _____

19. The value of log 1 881 is equal to: ______


(− )
3

20. The value of log 2 3 (1728) is equal to: _____

21. The value of log2 (log5 625) is: ______

1
22. The value of log10 125 − 2 log10 4 + log10 32 is: ______
3
23. These are 10 lamps in a room. Each one of them can be switched on independently. The number of ways
in which the hall can be illuminated is ______

The Institute of Cost Accountants of India 165


Fundamentals of Business Mathematics and Statistics

24. These are 11 questions in an examination paper of mathematics. A candidate has to answer 6 questions of
which the question under 1 is compulsory. The total number of selections of his answering in questions
is ______

25. There are two groups in a question paper, each group contains 7 questions. A candidate has to answer
questions but taking not more than 5 from any group. The total number of selections of 9 questions is ___

26. There are 10 points in plane and among them 4 are collinear. The total number of triangles formed by
joining them is ______

27. Out of 18 points in plane, no three are in the same straight line except 5 points which are collinear. Then
the number of straight lines obtained by joining them is _____

28. If the equations x 2 − 7 x + 12 = 0 and x 2 + mx + 5 = 0 have common roots the value of m is equal to______

29. The least positive value of m for which the equation x 2 + mx + 4 = 0 has real roots ______

30. The value of m for which the difference between the roots of the equation x 2 + mx + 4 = 0 is 2 are ______

31. If p, q are the roots of the equation f ( x ) = 6 x 2 + x − 2 = 0 the value of p/q is ______

166 The Institute of Cost Accountants of India


Algebra

Answer:

 Multiple Choice Questions (MCQ)

Q. No. Answer Remarks


1 (d) Like any of the product :
n (ABCD) = n (A) + n (B) + n (C) + n (D) – n (A  B) – n (A  C) – n (A  D) – n (B  C)
– n (B  D) – n (C  D) + n (A  B  C)+ n (A  B  D) + n (A  C  D) + n (B  C  D)
– n (A  B  C  D) = 24 + 27 + 11 + 9 – 18 – 15 – 11 = 27
2 (d) Like Fanta = B = n(B) = 27
3 (a) Did Not Like any of the products :
n ( A  B  C  D)′ = n (S) – [(n(A) + n(B)+ n(C)+n(D) – n(A  B) – n(C  D) + n(A  B  C
 D)] = 60 – (24 + 27 + 11 + 9 – 18 – 15 – 11) = 60 – 27 = 33
4 (b) Liked either Fanta or Thums Up =
n (B  D) = n(B) + n(D) – n(B  D) = 27 + 9 – 0 = 36
5 (a) Liked Fanta, Coke or Thums Up = n (A  B  D) =
n(A) + n(B) + n(D) – n(B  D) – n(A  B) – n(A  D) + n (A  B  D)
= 24 + 27 + 9 – 0 – 18 – 0 + 0 = 42
6 (d) Liked Mazaa: Since Mazaa is not available as a product, None of them liked Mazaa or it
can be considered as Zero who liked Mazaa.
7 (c) Liked any two products : n( A  B) + n(C  D) = 18 + 15 = 33
(Intersection of B&C, C&A, A&D, B&D & C & D are not taken into account because no
information regarding no. of elements of these sets is provided)
8 (a) Liked Thums Up, Coke, Coca Cola and Fanta :
Comprises of all the products = Liked all the products :
11 people liked all.
9 (b) Comprises of all the products = Liked all the products :
11 people liked all.
10 (d) Either Coke or Thums Up :
n (AD) = n (A) + n (D) – n (A  D) = 24 + 9 – 0 = 33
11 (a) Set A = {Q, W, E, R, T, Y} and Set B = { B, G, R, E, O, K}
Set (A–B) = { Q, W, T, Y}
Since, Q, W, T, Y are present in Set A but not present in Set B.
12 (c) A = {W, N, M, I} ; B = {Q, E, H, J} ; C = {I, L, E, R}
B  C = {E}
A  (B  C ) = {W, N, M, I, E}

The Institute of Cost Accountants of India 167


Fundamentals of Business Mathematics and Statistics

13 (d) Set B = {B, G, R, E, O, K} is a subset of Set A = {Q, E, R, T, Y, U, I, O, P, K, G, B}


B′ = Set (A–B) = {Q, T, Y, U, I, P}
14 (c) Set A = {8, 9, 7, 5, 6, 2} and Set B = {1, 4, 9, 3, 8, 2}
Union of B and A = {1, 2, 3, 4, 5, 6, 7, 8, 9}
15 (c) P1= {78, 77, 76, 75} ; P2={71, 72, 73, 74} ; P3 = {80, 79, 70, 69}.
(P1 P2) (P2 P3) = P2 (P1 P3)
P1 P3 = { 69, 70, 75, 76, 77, 78, 79, 80}
(P1P2)  (P2P3) = { } = {ϕ}
16 (a) A = {W, R, Y, I} ; B = {Q, E, T, U} ; C = {Q, W, E, R}
B C = {Q, W, E, R, T, U}
A  (BC) = {W, R }
17 (b) Set A = {Q, W, E, R, T, Y} and Set B = {B, G, R, E, O, K}
Set (B–A) = {B, G, O, K}
Since, B, G, O, K are present in Set B but not present in Set A.
18 (a) A = {8, 11, 14, 17} ; B = {5, 7, 9, 15} ; C = {19, 21, 17, 15}
B C = {15}
A(BC) = {8, 11, 14, 15, 17 }
19 (c) In Venn Diagram, Universal Set is represented by Rectangles.
20 (a) According to De Morgan’s Law : (B C)′ = B′ C′
21 (d) 39×33 = 39+3 = 312 = 531441 (19683×27) All of the above
22 (a) 0.0016 = (1/5)4t, It as 16/10000 = 1/625 = 1/(54) = (1/5)4
So, (1/5)4 = (1/5)4t or, 4=4t or, t=1
2.85t = 2.851 = 2.85
23 (a) qp x qm = qm+p
24 (b) (93)2 = 96 = 531441 = 59049×9
25 (b) 1178/1181 = 1178-81 = 1/113
26 (d) 36×34×3-2×3-3×36 = 36+4-2-3+6 = 311
27 (b) 63×6-2×6-5×64 = 63+(-2)+(-5)+4 = 67-7 = 60 = 1

()
28 (a) a
a b a
−1
= (a) b
b

( )
2b
or, 2b b 2b
= (2b) b
−1
[As a = 2b]
b
or, 22 = (2b)2–1 or, 22 = 2b or, b = 2
29 (a) (73)2 = 76 = 117649

168 The Institute of Cost Accountants of India


Algebra

30 (a) 96×9-4×92×93×96 = 96-4+2+3+6 = 913


31 (c) (92/ 33) × (34/9) = 32×2 / 33 × 34/32 = 34-3 × 34-2
= 31×32 = 31+2 = 33
32 (d) (1/3)4q = 1/6561
or, (1/3)4q = 1/38 = (1/3)8
So, 4q=8 or, q =2
(1.25)q = (1.25)2 = 1.5625 = (1.50+0.0625)
33 (a) 3(a1)2b1 / 6b2(a3)1 = (3/6) a1*2b1 / a3*1b2
= (1/2) a2b1 / a3b2 = (½) a2-3b1-2 = (½) a-1b-1 = 1/(2ab)
34 (a) 1 2 −5 −3
2 3 ⋅ 8 3 ⋅ 6 4 ⋅ 3 4 ⋅ 9 / 3 16

1 2 −5 −3 1 1 −5 −5 −3 4
= 2 3 ⋅ (23 ) 3 ⋅ (2 ⋅ 3) 4 ⋅ 3 4 ⋅ (32 ) / (24 ) 3 = 2 3 ⋅ 22 ⋅ 2 4 ⋅ 3 4 ⋅ 2 4 ⋅ 32 / 2 3
1 −5 4 5 3
= 23 +2 −
4 3
⋅3
− − +2
4 4

4+ 24 −15−16 −5−3+8
= 2 12
⋅3 4

= 2 − 4 ⋅ 30 =  1  = 1
1 4

2
  4
2
35 (b) 77/9 × 72/9 × 74/9×7-1×7-4/9 = 7 7/9 + 2/9 + 4/9 -1 - 4/9 = 70 = 1
36 (b) 1 1 1
x y z
= a = b = c = K (Let) ∴ a = k x , b = k y , c = k z
1 2 2
b2 = ac or,  k y  = k x , k x or, k y = k x + z
1 1 1 1

 
1 1 2 1 1 1
So x + z = y i.e. , & are in A.P.
x y z

37 (b) 7-5×7-8×74×73×76 =7-5-8+4+3+6 = 70 = 1


38 (c) (33/ 92) × (9/34) = 33 / 32×2 × 32/34 = 33-4 × 32-4
= 3-1×3-2 = 3-1-2 = 3-3
39 (a) (6×62)/(6-3×64) = (63+2)/(6-3+4)
= (65)/( 61) = 65-1 = 64 = (62)2
40 (b) 199 / 192 = 19 9-2 = 197
41 (b) log9 1 = 0 in Exponential Form : 90 = 1
42 (c)
43 = 64 in Logarithmic Form : log 4 64 = 3

The Institute of Cost Accountants of India 169


Fundamentals of Business Mathematics and Statistics

43 (b) log10 (8 y + 3) = 3
(8y + 3) = 103
or, 8y + 3 = 1000
or, 8y = 1000 – 3
or, 8y = 997

or, y = 997
8
44 (d) log 4 (t − 1) + 1 = 2 × log 4 t
or, log 4 (t − 1) + log 4 4 = log 4 t 2
or, (t − 1) × 4 = t 2
or, t 2 − 4t + 4 = 0
or, (t − 2) 2 = 0
or, t=2
45 (a) log 3 t = 6
t = 36
t = 729
46 (d) Logarithm of 324 with the base 3 2
log 3 2 324 = x

(3 2 ) = 324
x

or, (3 2 ) = (3 2 )
x 4

or, x = 4
47 (c)
100 z = z log10 z or, log10 (100 z ) = log10 z ( ) or, log
log 10z
10
z
100 + log10 z
= log10 z
× log10

or, log10 × 10 + log10 = ( log10 )


2 z z

( )
10 z z 2
or, 2 log10 + log10 = log10
z
or, 2+ x = x2 (Let log10 = x )
or, x2 – x – 2 = 0
or, (x – 2) (x + 1) = 0
∴ x=2&–1
1
z
log10 = 2 or, z =102 = 100 & log10z = −1 or, z = (10) −1 or, z =
10
48 (b) For the logarithms to be in A. P. Series, the positive numbers should be following G.P.

170 The Institute of Cost Accountants of India


Algebra

49 (c) 1 1
log y =
2 3
3 3
1 1  1  13 
= y 3 or,  2  =  y 
2  
1
or, y =
8
50 (a) log t 3125 = 5

3125 = t 5
or, 55 = t 5
or, t = 5
51 (a) 25 52
LHS = log 2 + 2 log = log 2 + 2 log = log 2 + 2[log 52 − log (8 × 3)]
24 8×3
= log 2 + 2 [2 log 5 – log 8 – log 3] = log 2 + 2 [2 log 5 – 3 log 2 – log 3]

= log 2 + 4 log 5 – 6 log 2 – 2 log 3 = 4 log 5 – 2 log 3 – 5 log 2


81 10
R.H.S = 3 log + 7 log
80 9
= 3 (log 81 – log 80) + 7 (log 10 – log 9)

= 3 [log 34 – log (16×5)] + 7 [log (5×2) – log 32]

= 3 [4 log 3 – log 16 – log 5] + 7 [log 5 + log 2 – 2 log 3]

= 3 [4 log 3 – 4 log 2 – log 5] + 7 log 5 + 7 log 2 – 14 log 3

= 12 log 3 – 12 log 2 – 3 log 5 + 7 log 5 + 7 log 2 – 14 log 3

= 4 log 5 – 2 log 3 – 5 log 2


So, L.H.S = R.H.S. Hence true
52 (a) Not possible
log 2 10 + log8 125 ≠ 1

53 (b) True
log 5 32 = 2.15 when log 2 = 0.30103
32 2 5 2
log10
32 log10 5log10
L.H.S = log = 5 = =
5
log10 ( ) log10 − log102
10
log10 2
10

5 × 0.30103
= = 2.15 = R.H.S., Hence true.
1 − 6.30103

The Institute of Cost Accountants of India 171


Fundamentals of Business Mathematics and Statistics

54 (c) Let the consecutive numbers be a = x –1, b = x and c = x + 1


1
× log (1 + ac)
2
1 1
= log[1 + ( x − 1)( x + 1)] = log x 2 = log x = log b
2 2
55 (d) log y 32 = 10 or, y10 = 32 or, y10 = 25 or, y = 25/10
or, y = 2
56 (c) base when 3 is the logarithm of 729
logx 729 = 3
729 = x3
or, 93 = x3
∴ x=9
57 (a) log (7y – 5) – 2 or, log10 (7y – 5) = 2
or, (7 y − 5) = 102
or, (7 y − 5) = 100
or, 7 y = 100 + 5
or, 7 y = 105

or, y = 105
7
or, y = 15
58 (b) a
Let us consider a series of 3 positive numbers , a and ar which are in G.P.
r
Logarithm of these numbers will be log
a
()
r , log a and log (ar)
a
()
But log r = log a – log r and log (ar) = log a + log r

Hence logarithm of the 3 positive numbers in G.P. can be rewritten as (log a – log r), log a,
(log a + log r) which are in A.P with common difference log r
59 (a) log (9t − 2) = 2 or, log10 (9t – 2) = 2
or, (9t − 2) = 102
or, (9t − 2) = 100
or, 9t = 100 + 2
or, 9t = 102
or, t = 102
9

172 The Institute of Cost Accountants of India


Algebra

60 (d) logarithm of 125 to the base 5 5


log 5 5 125 = x (let)

(5 5 ) = 125
x

or, (5 5 ) = (5 5 )
x 2

or, x = 2
61 (b) 12! / (8!×4!) = 12×11×10×9×8! / (8!×4×3×2×1)
= 12×11×10×9 / 24 = 11880/24 = 495
62 (c) (v–1)! × 20 = (v + 1) × v × (v–1)! Or, 20 = (v + 1) × v
or, 5×4 = (v+1)×v or, v = 4.
63 (a) 10C × 5C = 10 × 5 = 50
1 1

64 (d) 14! = 14×13×12! ; 12! = 1×12! ; 13! = 13×12! Hence, 14!


65 (a) First event : Burdwan to Haldia – 9 ways.
Second Event : Haldia to Burdwan – (9-1) ways
So, 9×8 ways = 72 ways.
66 (d) n
P6 : nP5 = 1:3 or, {n!/(n–6)!} : {n! / (n–5)!} = 1:3
or, n! /{ (n–6)×(n–5)! }× (n–5)! / n! = 1/3
or, 1/(n–6) = 1/3 or, 3 = n–6 or, n = 3+6 = 9.
67 (d) 2/36!-1/35!-1/35! = 2/36!-2/35! = 2/36!-(36×2)/(36×35!)
= 2/36! – 72/36! = (2-72) / 36! = -70/36!
68 (a) 15
P3 = 15! / (15-3)! = 15!/12!
= 15×14×13×12! / 12! = 15×14×13 = 2730.
69 (a) (10!-9!)/8! = (10×9×8! – 9×8!) / 8!
= 8!(10×9 – 9) / 8! = 90–9 = 81
70 (b) 5 drivers can refill from 5 refills,
Hence, possible ways : 5×4×3×2×1 = 120 ways.
71 (a) 8!/(8-8)! = 8!/0! = 8!/1 = 40320 ways
72 (b) 11
P5 = 11! / (11-5)! = 11! / 6!
= 11×10×9×8×7×6! /6! = 11×10×9×8×7 = 55440

The Institute of Cost Accountants of India 173


Fundamentals of Business Mathematics and Statistics

73 (a) 9/7!-3/6!+12/7! = (9+12)/7! – (7×3) /(7×6!)


= 21/7! – 21/7! = 0
74 (a) n
P7 :nP8 = 4:1 or, {n!/(n-7)!} : {n! / (n-8)!} = 4:1
or, {n! /(n-7)!} × {(n-8) × (n-7)!} / n! = 4/1
or, n-8 = 4 or, n = 4+8= 12.
75 (d) (a-2)!×24 = (a+1)! Or, (a-2)!×24=(a+1)×a×(a-1)×(a-2)!
Or, 24 = (a+1)×a×(a-1) or, 4×3×2 = (a+1)×a×(a-1)
or, a = 3.
76 (a) 11×10×9×8×7×6×5×4×3×2×1 / 6×5×4×3×2×1 = 11!/6!
77 (a) 8 people and 8 vaccinators
Hence, possible ways = 8×7×6×5×4×3×2×1 = 40320 ways
78 (c) 4! / (4-4)! = 4!/0! = 4!/1 = 4! = 4×3×2×1 = 24 ways
79 (a) 3/13!-2/12!-2/12!=3/13!-4/12! = 3/13! – (13×4)/(13×12!)
= 3/13! – 52/13! = (3-52) /13! = -49/13!
80 (d) 6!/(6-6)! = 6!/0! = 6!/1= 6! = 720 ways
81 (c) b2–4ac = (–24)2–4(16)(9) =576–576 = 0. Hence, Roots are Real, Rational and Equal.
82 (c) Product of roots can be expressed as c/a,
Hence, a = 1, product of roots = c/a = c/1 = c
83 (b) Sum of the roots = 9+(–4) = 5, Product of the Roots = 9× (– 4) = –36
x2– (Sum of the Roots)× x + (Product of the Roots) = 0
x2– (5)x + (–36)= x2–5x–36 = 0
84 (a) Sum of the Roots = (a–t)+(a+t) = 2a,
Product of the Roots = (a–t)×(a+t) = a2 – t2
x2– (Sum of the Roots) × x + (Product of the Roots) = 0
x2– (2a)x + (a2 – t2) = x2 – 2ax + a2 – t2 = 0
85 (a) 7
b (−7) 7 c 9 1 1 α+β 5 7
α+β = − = − = and α β = = . So + = = =
a 5 5 a 5 α β αβ 9 9
5
86 (b) (α+β)2–2αβ = (7/5)2 – 2×(9/5) = 49/25 –18/5=
(49–90) / 25= –41/25

174 The Institute of Cost Accountants of India


Algebra

87 (d) 7 b
= (− )
5 a

88 (d) 9
5
=
c
a()
89 (b) – 41/45
90 (a) (7/5) + 4 = (7 + 20)/5 = 27/5
91 (d) Real and Unequal
92 (c) Product of roots can be expressed as c/a,
Hence, a =1, product of roots = c/a = c/1 = c
93 (a) ax2+bx+c=0
94 (d) 3 constants and a (coefficient of x2) ≠ 0.
95 (c) b2–4ac is known as Discriminant of the equation
96 (d) For b2–4ac ≥ 0, roots are real.
97 (b) Sum of the roots is expressed as –b/a,
Hence, for a = –1, Sum of the roots = – b/(–1) = b
98 (a) Cofficient of x (ax2 + bx + c = 0)
99 (d) 5x2 – 60x+115 = 0 or, x2 – 12x + 23 = 0
Sum (α+β): 12, Product (α×β) : 23,
Square of Difference (α–β)2 = α2+β2 – 2αβ
= (α+β)2 – 2αβ – 2αβ = (12)2 – 4 × (32) = 52
100 (b) Coefficient of x2 (ax2 + bx + c = 0)

The Institute of Cost Accountants of India 175


Fundamentals of Business Mathematics and Statistics

 State True or False

1 2 3 4 5 6 7 8 9 10
T T F T F T F F T T
11 12 13 14 15 16 17 18 19 20
F T F F T F T T T F
21 22 23 24 25 26 27 28 29
F F T T T F F F F

 Fill in the Blanks

1 {1,3,6} 2 j
3 {1,2,3} 4 A=B
5 n (A) + n (B) 6 2
7 Set of all integers except zero 8 27 abc
9 x – 1/x 10 20
11 ¼ 12 5
13 1000 14 16
15 16 16 12
17 31 18 14
19 (-4) 20 6
21 2 22 1
23 1023 24 252
25 1470 26 116
27 144 28 (–21/4, –14/3)
29 4
30 (±2 5)

31 3 4
(− and − )
4 3

176 The Institute of Cost Accountants of India


CALCULUS – APPLICATION IN BUSINESS 3
This Modules includes

3.1 Concept of Calculus and its Application in Business


3.2 Revenue and Cost Function
3.3 Optimization Techniques (Basic Concepts)

The Institute of Cost Accountants of India 177


Fundamentals of Business Mathematics and Statistics

CALCULUS – APPLICATION IN BUSINESS

Module Learning Objectives:


After studying this Module, the students will be able to understand –
 The meaning of a Function
 The Concept of Limit of a Function
 The Concept of Differentiation of a Function
 The application of differentiation in business situations with special emphasis to the Cost and
Revenue functions
 The application of differentiation for optimization

178 The Institute of Cost Accountants of India


Concept of Calculus and its
Application in Business 3.1
FUNCTION
If x and y be two real variables related to some rule, such that corresponding to every value of x within a defined
domain or set of values we get a defined value of y, then y is said to be a function of x defined in its domain.
Here the variable x to which we may arbitrarily assign different values in the given domain is known as Independent
variable (or argument) and y is called the Dependent variable (or function).
Notations : Generally we shall represent functions of x by the symbols f(x), F(x), y(x) etc.
Example 1 : A man walks at an uniform rate of 5 km per hour. If s indicates the distances and t be the time in hours
(from start), then we may write, s = 5t.
Here s and t are both variables, s is dependent if t is independent. Now s is a function of t and the domain (value)
of t is 0 ≤ t ≤ ∞.
x2
Example 2 : y = f ( x ) = .
x
For x ≠ 0, y = x and for x = 0, y is not known (undefined). Here the domain of x is the set of real numbers except
zero. (refer worked out problem 2 of limit & continuity)
Constant Function : y = f(x) = 7 for all real values of x. Here y has just one value 7 for all values of x.
Single-valued, Multi-valued Function: For a function sometimes it may so happen that for y = f (x), there exists
a single value of y for every value of x. This type of function is known as single-valued function.
Example 3 : y = f (x) = 2x + 3
For x = 1, y = 2.1 + 3 = 2 + 3 = 5.
x = 2, y = 2.2 + 3 = 4 + 3 = 7
If again we get more than one value of y for a value of x, then y is said to be a multiple-valued (or multi-valued)
function of x.

Example 4 : y2 = x. Here for every x > 0, we find two values of y as y = ± x


Explicit and Implicit Function: A function is said to be explicit when it is expressed directly in terms of the
independent variable; otherwise it is implicit.

Example 5 : y = x2 – x + 1 is an explicit function :


2x2 + 3xy + y2 = 0 an implicit function.

The Institute of Cost Accountants of India 179


Fundamentals of Business Mathematics and Statistics

Parametic Representation of a Function : If the dependent variable x be expressed in terms of a third variable,
say t, i.e., y = f (t), x = F (t), then these two relations together give the parametic representation of the function
between y & x.
Example 6 : y = t2 + 1, x = 2t.
Odd and Even Functions : A function f (x) is an odd function of x if f(– x) = – f(x) and is an even function of x
if f (– x) = = f(x).
Example 7 : f(x) = x. Now f (–x) = – x i.e f (– x) = – f (x), so f (x) = x is an odd function of x.
f(x) = x2, f ( – x) = (– x)2 = x2 = f(x), so f (x) = x2 is an even function of x.
Inverse Function : If from a function y = f(x), we can obtain another function x = F(y), then each function is
known as the inverse of the other.
1
Example 8 : y = 4x – 3 and x = ( y + 3) are inverse to each other.
4
Both are the Functions of single independent variable :
Polynomial Function : A function of the form
F(x) = a0 + a1x + a2x2 + … + an - 1 xn–1 + anxn,
(Where n is a positive integer and a0, a1 …. an are constants) is known as a polynomial function in x.
For n = 0, f(x) = a0, a constant function
= 1, f(x) = a0 + a1x, a linear function in x
= 2, f(x) = a0 + a1x + a2x2, a quadratic function in x.
= 3, f(x) = a0 + a1x + a2x2 + a3x3, a cubic function in x.
Rational function : A function that is expressed as the ratio of two polynomials
a0 + a1 x + a2 x 2 + ...... + an x n
i.e., f ( x ) = .
b0 + b1 x + b2 x 2 + ...... + bn x n
P( x)
i.e., in the form of Q( x)

is called a rational function of x, such function exists for denominator ≠ 0.


x+2
Example 9 : f ( x ) = 2 exists for all values of x, if x2 – 4x + 3 ≠ 0. Now for x2 – 4x + 3 = 0 or
x − 4x + 3
(x – 1) (x – 3) = 0 or, x = 1, 3 Denominator becomes zero and hence the given function does not exist.
Irrational function : On the contrary, if a function f(x) cannot be represented in this form, it is called an irrational
function.
Example 10 : Functions of the form x (where x is not a perfect square number)
Algebraic function : A function in the form of a polynomial with finite number of terms is known as algebraic
function.
2 2
Example 11: x + 2 x − 3, x + 1 etc.

180 The Institute of Cost Accountants of India


Calculus - Application in Business

Domain and Range of a Function :


The set of values of independent variables x is called the ‘Domain’ of the function and the set of corresponding
values of f(x) i.e. the dependent variable y is called the ‘Range’ of the function.
Example 12 : For the squared function of y = x2, we get the ordered pairs (1, 1) (2, 4) (3, 9), … etc. as the values of
x and y respectively, The set of values {... –2, –1, 0, 1, 2, 3, …} is the domain of the Independent Variable, where
as set of values {0, 1, 4, 9, .…} represents the range of the Dependent one.
Example 13 : For the following functions find the domain and range.
x2 − 4 3− x 2x + 1
(i) f ( x) = , x ≠ 2 (ii) f ( x ) = , (iii) f ( x ) =
x−2 x−3 ( x − 1) ( x + 1)
Solution:
−4 −3 5
(i) f ( 0) = = 2, f (1) = = 3, f (3) = = 5, f ( 4) = 6, f ( −1) = 1
−2 −1 1
∴ domain = { .... –2, –1, 0, 1, 3, 4, …..}, range = {1, 2, 3, 5, 6, ….}
= R – {2}, = R – {4}, R = real number,
3
(ii) f ( 0) = = −1, f (1) = −1, f ( 2) = −1, f ( −1) = −1
−3
domain = { – 1, 0, 1,2, 4, ….}, range = { –1, –1, –1, ….} = {–1}
= R – {3}, where R is a real number

(iii) The function of f(x) exists for x ≠ 1, x ≠ – 1.


∴ domain = R – {– 1, 1}, i.e, all real numbers excluding 1 & (–1)
range = R.
4x − 5
Example 14 : Find the domain of definition of the function
2
x − 7 x + 12
Solution:
Denominator = x 2 − 7 x + 12 = x 2 − 4 x − 3 x + 12 = x( x − 4) − 3( x − 4) = ( x − 3) ( x − 4)

From above it can be said that if 3 < x <4 then (x – 3) is positive and (x – 4) is negative.
Thus (x – 3) (x – 4) = (Positive) × (Negative) = Negative

Hence ( x − 3) ( x − 4) = Negative = Imaginary number. Also for x = 3 & 4, denominator is zero.


Thus given function is not defined for 3 ≤ x ≤ 4. So domain is all real values of x except 3 ≤ x ≤ 4.
Absolute Value : A real number “a” may be either a = 0 or, a > 0 or a < 0. The absolute value (or modulus) of a,
denoted by | a | is defined as | a | = a, for a > 0
= – a, for a < 0
Thus | – 4| = – (– 4) = 4, and | 4 | = 4.
Complex No :
A number of the form (a+ib) or (a–ib) [where a & b are real numbers] is called a complex number (where i = −1 )
The complex number has two parts; a real part & an imaginary part. ‘a’ is the real part & ‘ib’ is the imaginary part.

The Institute of Cost Accountants of India 181


Fundamentals of Business Mathematics and Statistics

Example 15 :
If y = x2 + 4 is a function under consideration then solving for y = 0, we get
0 = x2 + 4
or, x2 = – 4
or, x = ± −4
or, x = ± 2 −1 = ± 2i
The number ± 2i is a complex number whose real part is 0 & imaginary part is ± 2
Example 16 : Given f(x) = 2x2 – 3x + 1 ; find f(2), f(0), f (– 3)
Solution:
f(2) = 2.22 – 3.2 + 1 = 2.4 – 6 + 1 = 8 – 6 + 1 = 3
f(0) = 2.02 – 3.0 + 1 = 2.0 – 0 + 1 = 0 – 0 + 1 = 1
f(– 3) = 2 (– 3)2 – 3. (– 3) + 1 = 2.9 + 9 + 1 = 18 + 9 + 1 = 28.
Example 17 : If y = 4x – 1, find the value of y for x = 2. Can y be regarded as a function of x? Also find the domain.
Solution:
For x = 2, y = 4.2 – 1 = 8 – 1 = 7. Again for x = 0, y = – 1 and for x = – 1, y = – 5. So for every value of x in – ∞ <
x < ∞, we find different values of y, So y is a function of x and its domain is -
– ∞ < x < + ∞.
Example 18 : If f (x) = x + | x |, find f (3) and f (– 3) and show also they are not equal.
Solution:
f(3) = 3 + | 3 | = 3 + 3 = 6 ; f ( – 3) = – 3 + | – 3| = – 3 + 3 = 0.
As 6 ≠ 0, so f(3) ≠ f ( – 3).
Note : If f(x) = f(– x) [i.e., f (3) = f (– 3)] then f(x) will be an even function of x.
Example 19 : Show that x 2 − 5x + 4 is not defined for 1 < x < 4
Solution:
x 2 − 5x + 4 = ( x − 1) ( x − 4)
Now for any value x > 1, but < 4 the expression becomes imaginary. So the expression is undefined for 1 < x < 4.
x
Example 20 : Find the domain of f ( x) = 2
x −9
Solution:
Here f(x) has a unique value except for x = 3, – 3.
3 3 −3 −3
For f (3) = = (undefined) and f (−3) = = (undefined)
9−9 0 9−9 0
∴ domain of the function f(x) is – ∞ < x < – 3 ; – 3 < x < 3 and 3 < x < ∞.
Example 21 : Given the function
f(x) = 5–2x –1, – 1 ≤ x < 0

182 The Institute of Cost Accountants of India


Calculus - Application in Business
x2 − 2
= ,0≤x<1
x−2
= 1≤ x < 3

Find f (– 1), f(0), f (1/2), f (2).


Solution:
f( – 1) = 5– 2 (–1) – 1 (since – 1 lies in the interval – 1 ≤ x < 0)
= 52 – 1 = 25 – 1 = 24.
x = 0 and x = 1 lie in the interval 0 ≤ x < 1,
2
1
()
1 −2 7
0−2
f (0) = = 1 , f 2 = 14 = .
6
0−2 −2
2
2.2 4 4
Now 2 lies in the third interval. So f (2) = 2 = = .
2 −1 4 −1 3
Example 22 : If f(x) = eax + b. Prove that eb­ f(x + y) = f (x). f(y)
Solution:
eb f(x + y) = eb.e­­a(x+y)+b = eb+ax+ay+b
= eax + b. eay + b = f(x). f(y). Hence proved.

Example 23 : If f(x) = x – a, q(x) = x + a then show that


{f(x)}2 – {q(x)}2 = – 2a {f(x) + q(x)}

Solution:
L.H.S. = (x – a)2 – (x + a)2 = x2 + a2 – 2ax – (x2 + a2 + 2ax) = – 4ax
R.H.S. = – 2a {x – a + x + a} = – 2a. 2x = – 4ax. So L.H.S = R.H.S (Proved)

LIMIT
Introduction:
Calculus is based, in general, on the idea of limit. At present this idea including its related concepts, continuity to
mention, will be discussed.
Some definitions:

(i) Meaning of “x tends to a”. When the difference |x–a| (i.e., numerical difference between the present value of
x and a) can be made less than any positive quantity, however small, we say x tends to a and is written as x→
a.

(ii) Meaning of “x tends to zero”. When the value of x goes on decreasing numerically and can be made
numerically less than any positive quantity, however small, we say x tends to zero and is written as x→ 0.

(iii) Meaning of “x tends to infinity”. When the value of x goes on increasing and can be made greater than any
positive quantity, however, large, we say x tends to infinity written as x → ∝.

The Institute of Cost Accountants of India 183


Fundamentals of Business Mathematics and Statistics

Neighbourhood or Proximity of a point


Let c be any real number, then any open interval around c is called the neighbourhood of c, e.g.,
] c – ∈, c + ∈[, ∈> 0
Is the neighbourhood of c.
Any neighbourhood from which the point c is excluded is called deleted neighbourhood of c.
Geometrically it means set of those points which are within an infinitely small distance ∈ from c on either side
except for the point c.
(iv) Limit of a function f (x). A number l is said to be the limit of f(x) as x→a written as lim f ( x) = l if
x→ a

a. The function is defined and single valued in the deleted neighbourhood of a.


b. For every positive number ∈, however small, there exists a positive number δ (usually depending on ∈),
such
|f (x) – 1| < ∈
Whenever 0<|x–a|<δ

From the above definition it is interesting to note lim f ( x) that may exist, even if the function f(x) is not defined at
x→ a
x = a. Sometimes both the things may happen, i.e.,
(i) The function is defined at x = a, and

(ii) lim f ( x) also exists.


x→ a

Right hand and left hand limits

The variable point x can approach ‘a’ either from the left or from the right. These respective approaches are
indicated by writing x → a − and x → a +
When

lim f ( x) = l1 and lim+ f ( x) = l2 then we can say lim f ( x) = l if and only if l1 = l2 = l


x→ a − x→ a x→ a

Methods of finding limit of a function f(x) as x tends to a finite quantity say ‘a’
There are three methods for finding limit of a function f(x) as x tends to a finite quantity say ‘a’:
(i) Method of factors
(ii) Method of substitution
(iii) Method of rationalization.
g (x)
In method I, if f(x) is of the form factorise g(x) and h(x), cancel the common factors and then put the value
h (x)
of x.
3
Example 24: Find the value of Lt x − 1 .
2 x→1 x −1

184 The Institute of Cost Accountants of India


Calculus - Application in Business

Answer:
x 3 − 1 (x − 1)(x 2 + x + 1)
Now, =
x2 − 1 (x − 1)(x + 1)
x2 + x + 1
= [As x → 1, x ≠ 1 i.e x –1 ≠ 0]
x +1
3 2
∴ Lt x 2 − 1 = Lt x + x + 1
x→1 x −1 x→1 x +1

= 1+ 1+ 1 = 3 .
1+ 1 2
Method II
The following steps are involved:
(i) Put x = a + h where h is very small but ≠ 0, i.e. x → a, h→0.
(ii) Simplify numerator and denominator and cancel common powers of h.
(iii) Put h = 0.
The result is the required limit.
n n
Example 25: Evaluate Lt x − a .
x→ a x−a
Answer:
Put x = a + h where h is very small, then
x n − an (a + h)n − an
=
x−a h
 h  n 
an 1+  − 1
 a
=  
h
 nh h2  
an 1+ + n (n − 1) 2 +  − 1
 a a  
=
h
 h h2 
an  n + n (n − 1) 2 + 
 a a 
=
h
n h 
= an  + n (n − 1) 2 + 
 a a 

x n − an n h 
∴ Lt = Lt an  a + n (n − 1) a2 + 
x→ a x−a h→ 0
 
n
= an ⋅ = nan−1
a

Method III. (Rationalisation)


This method is useful where radical signs are involved either in the numerator or denominator. The numerator or
denominator (as required) is rationalized and limit taken. The following example will make the method clear.

The Institute of Cost Accountants of India 185


Fundamentals of Business Mathematics and Statistics

1+ x − 1 − x
Example 26: Evaluate Lt
x→ 0 x
Answer:
Rationalising the numerator, we get

1+ x − 1− x 1+ x − 1− x 1+ x + 1− x
= ×
x x 1+ x + 1− x
2x
=
x  1+ x + 1− x 

2
= [As x → 0, x ≠ 0]
1+ x + 1 − x

∴ Lt 1+ x − 1 − x 2
= Lt =1
x→ 0 x x → 0
1+ x − 1 − x

Infinite Limits
g(x)
For finding the limit of f(x) = as a → ∝, we divide the numerator and denominator by highest power of x
h(x) 1 1
occurring in f(x) (numerator or denominator whichever has higher power of x) and then use , 2 , etc.→ 0 as
x x
x → ∝.
Example 27: Evaluate
(x + 1)(2 x + 3)
Lt
x→∞ (x + 2)(3 x + 4)
Answer:
(x + 1)(2 x + 3) 2 x2 + 5x + 3
Now = 2
(x + 2)(3 x + 4) 3 x + 10 x + 8
5 3
+ 2+
= x x2 [Dividing by x2]
10 8
3+ +
x x2
(x + 1)(2 x + 3) 22 + 5×0 + 3×0 1 1
∴ Lt = [As x → ∞, as well as 2 → 0 ]
x→∞ (x + 2)(3 x + 4) 3 +10×0 + 8×0 x x
2
=
3
Alternative approach:
(x + 1)(2 x + 3)
Lt
x→∞ (x + 2)(3 x + 4)

 1  3
1+ x   2 + x  1⋅ 2 2
= Lt = =
x→∞  2  4  1⋅ 3 3
1+ x   3 + x 

186 The Institute of Cost Accountants of India


Calculus - Application in Business

SOLVED EXAMPLES

Example 28: x2 − 4
lim = 4.
x→2 x−2

For if x = 2 + h, whether h be positive or negative,

x 2 − 4 ( x − 2 ) ( x + 2 ) h ( 4 + h)
= = = 4+h
x−2 ( x − 2) h

2
and by taking h numerically small, the difference of x − 4 and 4 can be made as small as we like. It may be
x−2
noted here that however small h may be, as h ≠ 0, one can cancel the factor (x – 2) i.e., h between numerator and
2 2
denominator here. Hence lim x − 4 = 4. But for x = 2, the function x − 4 is undefined as we cannot cancel the
x→2 x−4 x−2
factor x – 2, which is equal to zero.
2
Now writing f(x) = x − 4 , limf(x) = 4 whereas f(2) does not exist.
x−2 x→ 2

Example 29 : For f (x) = | x |, find, xlim


→0
f(x).

Answer:
 x, for x ≥ 0
f(x) = | x | means f(x) = 
 − x, for x < 0

lim f(x) = lim |x| = lim x = 0 [x → 0+ means x is approaching 0 from right i.e x > 0 now. Also | x | = x for x > 0]
x →0+ x →0+ x →0

lim f(x) = lim |x| = lim(− x) = 0. [x → 0 – means x is approaching 0 from left i.e x < 0 now. Also | x | = – x for
x →0 − x →0 − x →0

x <  0]

So lim+ f(x) = lim– f(x) = 0


x →0 x →0

∴ limf(x) = 0.
x→0

Example 30: Find analytically lim


x→3
x − 3, if it exists.

Answer:

lim x − 3 = 0, but lim x − 3 does not exist. Because when x → 3 – then x is approaching 3 from left which
x→3 + x→3 −

means x is definitely less than 3. Thus (x – 3) is negative and square root of negative quantity is never real but
imaginary.
∴ the limit does not exist.

Alternative approach:

Put x = 3 + h, as x → 3, h → 0

The Institute of Cost Accountants of India 187


Fundamentals of Business Mathematics and Statistics

lim x − 3 = lim 3 + h − 3 = lim h


x→3 h→0 h→0

lim h = 0, but = lim h does not exist (as the h near to but less than zero corresponds no real value of h ).
h→0 + h→0 −

∴ lim x − 3 does not exist.


x→3

Note : At x = 3, f (x) = x − 3 = 3 − 3 = 0 ∴ f(3) exists.

Example 31 : Do the following limits exist? If so find the values


 ( x − 1) 
2
1 1
(i) lim
x →−2 x + 2
(ii) lim
x →0 x x →1
( 2
)
(iii) lim  x − 1 +
x −1 

 
Answer:
1 1 1
(i) lim = lim = lim
x →−2 x+2 h → 0 −2 + h + 2 h → 0 h
1 1
Now lim = + ve; lim = −ve. As the two limits are not same, so the limit does not exist.
x→0+ h h→ 0 − h

1 1
(ii) xlim = +∞, lim = − ∞. The limit doesn’t exist as the two values are unequal.
→0+ x h→ 0 − x

 ( x − 1) ( x − 1)  = lim
(
(iii) lim  x 2 − 1 +
x →1

) ( x − 1)  x →1
{( x − 1) + ( x − 1)} , [As x → 1, x ≠ 1 i.e x –1 ≠ 0]
2

= lim
x →1
( )
x 2 + x − 2 = (1+ 1− 2 ) = 0.

On putting the limiting value of x, the value of the function exists and its value is 0.

Distinction between lim f ( x ) and f (a)


x →a

By lim f ( x ) we mean the value of f(x) when x has any arbitary value very near ‘a’ but not ‘a’. The quantity f (a)
x→a

is the value of f(x), when x is exactly equal to ‘a’.

Note : The following cases may arise :

(i) f (a) does not exist, but lim f ( x ) exists.


x→a

Such situation is explained in example 28 above.

(ii) f (a) exists, but lim f ( x ) does not exist.


x→a

Such situation is explained in example 30 above.

(iii) f (a) and both exist, lim f ( x ) both exist but unequal.
x→a

188 The Institute of Cost Accountants of India


Calculus - Application in Business

Let f (x) = 0, for x ≠ 0


= 1, for x = 0
lim f ( x ) = 0 = lim f ( x) and f(0) = 1, unequal values.
x → 0+ x → 0−

(iv) f (a) and lim f ( x) both exist and equal.


x→a

Such situation is explained in example 29 above.

(v) neither f (a) nor, lim f ( x ) exists.


x→a

Fundamental Theorem on Limits:

If lim f ( x ) = l and lim φ ( x) = m, where l and m are finite quantities then


x→a x→a

1. lim  f ( x ) ± φ ( x ) = l ± m
x→a 

2. lim  f ( x). φ ( x ) = lm.


x→a 

f ( x) l
3. lim = ,m≠0
x→ a φ ( x) m
4. If lim φ ( x) = b and lim f (u ) = f (b) then
x→ a x →b

x →b
{
lim f {φ ( x)} = f lim φ ( x) = f (b).
x →b
}
lim
5. x→ a k = k, where k = Constant

6. lim k . f ( x) = k lim f ( x), where k = Constant


x→ a x→ a

x 2 + 3x − 1
Example 32: Evaluate, lim .
x →1 2x + 4
Answer:
As the limit of the denominator ≠ 0, we get

x →1
(
lim x 2 + 3 x − 1 ) lim x 2 + lim 3 x − lim1
x →1 x →1 x →1
Required limit = = (by theorem 1)
lim ( 2 x + 4) lim 2 x + lim 4
x →1 x →1 x →1

1 + 3.1 − 1 3 1
= = =
2.1 + 4 6 2
 
1
We have not applied the definition to save labour. If we substitute x = 1, we get the value of the function =
(equal to the limit the value as x →1). Practically this may not happen always, as shown below. 2

The Institute of Cost Accountants of India 189


Fundamentals of Business Mathematics and Statistics

x 2 − 3x + 2
Example 33 : Evaluate lim .
x →1 x2 − 4x + 3
( x − 1) ( x − 2) = lim x − 2
Required limit = lim [As x → 1, x ≠ 1 i.e x – 1 ≠ 0]
x →1 ( x − 1) ( x − 3) x →1 x − 3
1− 2 1
= = .
1− 3 2

lim
( x − 1) ( x − 2) = lim x − 2 = 1 − 2 = 1 .
x →1 ( x − 1) ( x − 3) x →1 x − 3 1 − 3 2
0
If we put x = 1, the function becomes which is undefined. Further limit of the denominator is zero, we cannot
0
apply theorem 2, hence cancelling the common factor (x – 1) which is ≠ 0 as x →1, we obtain the above result.

Examples related to variable tends of infinity:

Example 34: Find the limit of lim


x →∞
2 x2 − 5x + 2 ( )
Answer:
 5 2
Now 2x2 – 5x + 2 = x2  2 − + 2 
 x x 

 5 2  5 2
x →∞
( x →∞
)
∴ lim 2 x 2 − 5 x + 2 = lim x 2  2 − + 2  = lim x 2 × lim
 x x  x→∞ x →∞
 2 − + 2 
x x
5 2 1 1
Now lim x 2 = ∞ ; lim 2 = 2 ; lim = 0; lim 2 = 0 [As x → ∝, & 2 = 0]
x →∞ x →∞ x →∞ x x →∞ x x x
(
∴ lim 2 x 2 − 5 x + 2 = ∞ × 2 = ∞.
x →∞
)
4 x5 + 2 x3 − 5
Example 35: Find lim
x →∞ 7 x8 + x 4 + 2
Answer:
Of all the terms in numerator and denominator the highest power of x is 8. We now divide both the numerator and

the denominator by x8 to avoid the undefined form . So we get

4 2 5
+ − 3 5 8
x 3 x 5 x8 = lim 4u + 2u − 5u 1
Given limit = lim 1 2 4 8 (put = u , so as x → ∞ , u → 0)
x →∞
7+ 4 + 8
u→0 7 + u + 2u x
x x

(
lim 4u 3 + 2u 5 − 5u 8
u→0
)
= 4 8
lim (7 + u + 2u )
u →0

4 lim u 3 + 2 lim u 5 − 5lim u 8 0+0−0 0


u→0 u→0 u →0
= = = =0
7 + lim u 4 + 2 lim u 8 7+0+0 7
u→0 u→0

190 The Institute of Cost Accountants of India


Calculus - Application in Business

5 − 2x2
Example 36: Find lim .
x →∞ 3x + 5 x 2
Answer:
5
−2
x2 5u 2 − 2  1 
Given limit = lim
x →∞ 3
= lim
u → 0 3u + 5  Let x = u , as x → ∞, u → 0
+5
x
5 lim u 2 − 2 0−2 2
u→0
= = =− .
3 lim u + 5 0+5 5
u→0

Example related to rationalisation:

x+h − x
Example 37: Find the value of : lim
h →0 h

lim
x+h − x
= lim
( x+h − x )( x+h + x ) = lim x+h−x
h →0 h h→0
h ( x+h + x ) h →0
h ( x+h + x )
h 1 1 1
= lim = lim , [As h → 0, h ≠ 0) = = .
h →0
h ( x+h + x ) h→0 x+h + x x+0+ x 2 x

Examples related to left hand and right hand limits:

Example 38: A function is defined as


f(x) = x2, for x > 1
= 4.1, for x=1
= 2x, for x < 1.
Does lim f ( x) exist?
x →1

Answer:
We have to find R.H. limit and L.H. limit i.e lim f ( x) and lim f ( x).
x →1+ x →1−

2 2
Now lim f ( x) = lim x = 1 = 1, [As f(x) = x for x > 1] 2
x →1+ x →1 +

Again lim f ( x ) = lim 2 x = 2 × 1 = 2 , [As f(x) = 2x for x < 1]


x→1+ x→1+

Since the values of R.H. L and L.H.L are not equal, so lim f ( x ) doesn’t exist.
x →1

4 x+ | x | | x−2|
Example 39: Evaluate (i) lim (ii) lim
x→0 3x+ | x | x→2 x − 2

The Institute of Cost Accountants of India 191


Fundamentals of Business Mathematics and Statistics

Answer:
4x + x 5x 5
(i) R.H. Limit = xlim = lim = , [As x → 0, So x ≠ 0 and also for x > 0, | x | = x]
→0+ 3x + x x → 0 4x 4
4x − x 3x 3
L.H. Limit = lim = lim = [As x → 0, So x ≠ 0, and for x < 0, | x | = – x]
x → 0 − 3x − x x→0− 2x 2
∴ the given limit doesn’t exist, as the values of R.H.L and L.H.L are unequal.

x−2 x−2
(ii) R.H.L = lim = lim = 1. [As x → 2, x ≠ 2 i.e (x–2) ≠ 0. Also for | x – 2 | > 0 i.e., for x > 2,
x →2 + x − 2 x →2 ( x − 2 )
|x–2|=x–2

x−2 − ( x − 2)
L.H.L = xlim = lim = −1 for | x – 2 | < 0 i.e., for x < 2, | x – 2 | = – (x – 2)
→2− x − 2 x → 2 − ( x − 2)
As the values of R.H.L and L.H.L are unequal, so the given limit does not exist.

Some Useful Limits :


1
(A) lim (1 + x)
1/ x
= e (B) lim log e (1 + x ) = 1
x→0 x
x→1
ex − 1 ax −1
(C) lim = 1 (D) lim = log e a
x→0 x x→0 x
(E) lim
xn − an
an nn −−1 1 (F)
= na lim
(1 + x ) − 1 = n.
n

x→a x−a x → 0 x

e3 x − 1
Example 40 : Evaluate lim
x→0 x
Answer:
Put 3x = u, As x → 0 u → 0.

e3 x − 1 eu − 1 eu − 1
lim = lim = 3 × lim . = 3 × 1 (by C ) = 3.
x→0 x u→0 u / 3 u →0 u

log (1 + 6 x )
Example 41: Evaluate lim = 6.
x→0 x
Answer:
Put 6x = u, As x → 0, u → 0
log (1 + u ) log (1 + u )
Given limit = lim = 6 × lim = 6 × 1 (by B) = 6.
u→0 u/6 u → 0 u

a x − bx
Example 42 : lim
x→0 x

192 The Institute of Cost Accountants of India


Calculus - Application in Business

Answer:
a x −1 − bx + 1 ax −1 bx −1
Given limit = lim = lim − lim
x→0 x x→0 x x →0 x
a
= log a – log b [by (D) above] = log .
b

e ax − ebx
Example 43: lim
x→0 x
Answer:

Given limit = lim


(e ax
) (
− 1 − ebx − 1 ) = lim e ax
−1
− lim
ebx − 1
x→0 x x→0 x x→0 x
eu − 1 ev − 1
= lim − lim [Let ax = u, As x → 0 u → 0 also. Similarly for bx = v, As x → 0, v → 0 also]
u→0 u / a v→0 v / b

eu − 1 eν − 1
= a lim − b lim
u→0 u ν →0 ν
= a. 1 – b.1 = a – b.

CONTINUITY
Introduction :
A function is said to be continuous if its graph is a continuous curve without any break. If, however, there is any
break in the graph, then function is not continuous at that point.
If for a value of k, the limit of f(x) does not exist i.e., if on the curve of f(x) a point is absent, the graph will be
discontinuous i.e., not continuous.
A function f(x) is said to be continuous at x = a, when lim f ( x) if exists is finite and is equal to f(a)
x→ a
i.e., lim f ( x) = lim f ( x ) = f ( a )
x→ a+ x→ a −

i.e., f (a + 0) = f (a – 0) = f (a) briefly or lim f (a + h) = f (a )


x→0
Thus we are to find following three values :
(i) lim f(x), (ii) xlim
→ a−
f(x) (iii) f(a)
x → a+

If however all these values are equal, then f (x) is continuous at x = a, otherwise it is discontinuous.

Example 44 : Show that f(x) = 3x2 – x + 2 is continuous at x = 1.

Answer:

Now,
x →1+ x →1+
( ) h→0
{ 2
}
lim f(x) = lim 3x 2 − x + 2 = lim 3 (1+ h) − (1+ h) + 2 [Putting x = 1 + h as x → 1, h → 0]

(
= lim 3h2 + 5h + 4 = 3.0 + 5.0 + 4 = 4
h→ 0
)
( ) { }
and lim f ( x ) = lim 3x 2 − x + 2 = lim 3 (1− h) − (1− h) + 2 [Putting x = 1 – h, h → 0 as x → 1]
h →1− x →1− h→ 0
2

= hlim
→0
(
3h2 − 5h + 4 = 3.0 − 5.0 + 4 = 4 )
The Institute of Cost Accountants of India 193
Fundamentals of Business Mathematics and Statistics

Again f (1) = 3.12 – 1 + 2 = 4


Thus we find that all the values are equal.
∴ f (x) is continuous at x = 1 ;

ε - δ Definition :
Again corresponding to definition of limit, we may define the continuity of a function as follows :
The functions f(x) is continuous at x = a, if f (a) exists and for any pre-assigned positive quantity ε, however
small we can determine a positive quantity δ, such that |  f (x) – f (a) | < ε, for all values of x satisfying
|x – a | < δ.

Some Properties :
1. The sum or difference of two continuous functions is a continuous function

x →a
{ }
i.e., lim f ( x ) ± φ ( x ) = lim f ( x ) ± lim φ ( x ).
x →a x →a

2. Product of two continuous functions is a continuous function.


3. Ratio of two continuous functions is a continuous function, provided the denominator is not zero.

Continuity in an Interval, at the End Points:


A function is said to be continuous over the interval (open or closed) including the end points if it is continuous at
every point of the same interval.
Let c be any point in the interval (a, b) and if lim f ( x ) = lim f ( x ) = f (c) , then f(x) is continuous in the interval
x → c+ x →c−
(a, b)

A function f (x) is said to be continuous at the left end ‘a’ of an interval a ≤ x ≤ b if lim f (a + h) = f (a ) , and at the
x→0
right end ‘b’ if lim f ( x) = f (b) .
x →b −

Discontinuity at a Point: If at any point x = a in its domain, at least one of values and f (a) be different from the
others, then f (a) fails to be continuous at that point, i.e., at x = a.

Example 45: Discuss the continuity of f (x) at x = 4, where


f (x) = 2x + 1, x ≠ 4
= 8, x = 4.
Answer:

lim f ( x) = lim (2 x + 1) = 2.4 + 1 = 9 [x → 4+ means x is approachig 4 from right i.e x > 4. Also f (x) = 2x + 1
Here 
x→ 4 − x→ 4
for x ≠ 4]

 lim f ( x) = lim (2 x + 1) = 2.4 + 1 = 9 [x → 4 means x is approachig 4 from left i.e x < 4. Also f (x) = 2x + 1

x→ 4 − x→ 4
for x ≠ 4]

f (4) = 8, which is different from the previous two values.


∴ f (x) is not continuous at x = 4.

194 The Institute of Cost Accountants of India


Calculus - Application in Business

SOLVED EXAMPLES
Example 46: Show that f (x) = 2x + 3 is continuous at x = 1.
Answer:

lim f ( x) = lim (2 x + 3) = 2.1 + 3 = 5.


x→1+ x→1

lim f ( x) = lim (2 x + 3) = 2.1 + 3 = 5. and at x = 1, f (1) = 2.1 + 3 = 5


x→1− x→1

∴ lim f ( x ) = lim f ( x ) = f (1) = 5 ; ∴ f (x) is continuous at x = 1.


x →1+ x →1−

Example 47: Discuss the continuity of f (x) = | x | at x = 0.


Answer:
f (x) = | x | means f (x) = x for x > 0
= 0 for x=0
= – x for x < 0.

lim f ( x) = lim x = 0 [x → 0 means x is approaching 0 from right i.e x > 0 now. Also f (x) = x for x > 0]
+

x→0 + x→0

lim f ( x) = lim ( − x) = 0 [x → 0 – means x is approaching 0 from left i.e x < 0 now. Also f (x) = – x for x < 0]
x→0 − x→0

f (0) = f (x) at x = 0 = 0

∴ lim f ( x ) = lim f ( x ) = f ( 0 ) = 0 ; ∴ f (x) is continous at x = 0.


x→0+ x→0 −

1
Example 48: Show that f ( x) = is discontinuous at x = 1.
x −1
Answer:
1 1
f (1) = = , undefined.
1− 1 0
1 1
Now lim f ( x ) = lim = lim = + ∞ (Putting x = 1 + h, As x → 1, h → 0)
x →1+ h→ 0 1+ h − 1 h → 0 h

1 1
And lim f ( x ) = lim = lim = −∞ (Putting x = 1 – h, As x → 1, h → 0)
x →1− h→ 0 1− h − 1 h → 0 −h

We find right hand limit is not equal to left hand limit.


∴ at x = 1, f (x) is discontinuous.

Example 49: Discuss the continuity at x = 2 where


f(x) = 4x + 8, x ≠ 2
= 12 , x=2

The Institute of Cost Accountants of India 195


Fundamentals of Business Mathematics and Statistics

Answer:
Now lim f ( x ) = lim 4 ( 2 + h) + 8 = 16 (Putting x = 2 + h As x  2, h  0)
x→2+ h→ 0

And lim f ( x ) = lim 4 ( 2 − h) + 8 = 16. [Putting x = 2 – h, h  0, As x  2]


x→2− h→ 0

Again at x = 2, f(x) = f(2) = 12.


Three values are not equal. Hence f(x) is discontinuous at x = 2.

OBJECTIVE QUESTIONS :
1. If f(x) = (x – 1) (x – 2) (x – 3), find f(0) [Ans. – 6]
2. If f(x) = x + | x |, find f(– 2) [Ans. 0]
x 2 − 6x + 8
3. If f ( x ) = find f(0) [Ans. 2
3]
x 2 + 8x + 12
x2
4. Given f(x) = x, F ( x ) = is F(x) = f(x) always? [Ans. equal for x ≠ 0]
x,
5. If f(x) = 2 + x, x < 3 [Ans. 4]
= 7 – x, x ≥ 3, find f(3)
6. If f(x) = (x – 2) (x – 3) (x + 4) find f(3) [Ans. 0]
7. Given f ( x ) = x , for what value of x, f(x) is unreal? [Ans. Any negative value]
x−2
8. Find the range of the function f ( x ) = , x≠2 [Ans. – 1]
2−x
9. If f(x) = x + 2x3, find – f(– x) [Ans. x + 2x3]
x
10. f ( x ) = , x ≠ 0 and is a real number, find f (c) − f ( −c) . [Ans. 2]
x

11. If f(x) = e3x + 4, find f(1) . f(2) . f(5) [Ans. e36]


1 1− x
find f ( x ) =
1
12. If x = , [Ans. ]
3 1+ x 2

13. If f(x) = x + | x | are f(2) and f(– 2) equal? [Ans. no]

x2 + 1 x2 − 1 x−2
14. Evaluate : (i) lim (ii) lim (iii) lim [Ans. (i) 1, (ii) 2, (iii) does not exist]
x →1 x +1 x → 1 x −1 x → 2 x−2

x 2 + 5x − 6 ex − 1
15. Evaluate : (i) lim (ii) lim [Ans. (i) 7 (ii) 1]
x →1 x −1 x→0 x

x 2 + 4x + 3 −2
16. Evaluate : lim [Ans. ]
x → −1 x 2 − 7x − 8 9

196 The Institute of Cost Accountants of India


Calculus - Application in Business

1− 1− x 1
17. Find the value of lim [Ans. ]
x→0 x 2

f ( x + h) − f ( x )
18. If f(x) = x2, evaluate : lim [Ans. 2x]
h→ 0 h

19. Is the function f(x) = | x | continuous at x = 0 [Ans. Yes]


2
6 − 5x 1
20. Evaluate : lim . [Ans. − ]
x →∞ 4x + 15x 2 3
21. A functions is defined as follows :
f(x) = 2x – 1, x < 3
=k , x=3
= 8 – x, x > 3.
For what value of k, f(x) is continuous at x = 3? [Ans. 5]
22. A function is defined as follows :
f(x) = 1, x>0
= 0, x=0
= –1 x < 0. Is the function is continuous at x = 0 [Ans. no]

DIFFERENTIATION
Introduction :
The idea of limit as discussed previously will now be extended at present in differentiating a function f(x) with
respect to x (the independent variable). For this let us know at first what the term ‘increment’ means.

Increment : By increment of a variable we mean the difference of initial value from the final value.
i.e., Increment = final value – initial value.
Let x change its value from 1 to 4, increment of x = 4 – 1 = 3.
Again if x changes from 1 to – 2, increment = – 2 – 1 = – 3.
(i.e., increment may be positive or negative).

Symbols : Increment of x will be denoted by h or, δx (delta x) or ∆x (delta x) and that of y will be represented by
k or, δy or, ∆y.
If in y = f(x), the independent variable x changes to x + δx, then increment of x = x + δx – x = δx (≠ 0).
So y = f(x) changes to y = f(x + δx).
∴ increment of y = f(x + δx) – f(x) [as y = f(x)]

δy f ( x + δx ) − f ( x ) f ( x + h) − f ( x )
Now the increment ratio = = [assuming δx = h]
δx δx h

The Institute of Cost Accountants of India 197


Fundamentals of Business Mathematics and Statistics

δy
If the ratio tends to a limit, as δx → 0 from either side, then this limit is known as the differential coefficient or
δx
derivative of y [ = f(x)] with respect to x. The operation of finding differential coefficient (or derivetive) is called
Differentiation.
Example : If y = 2x2, then y + dy = 2 (x + dx)2, dy = 2 (x + dx)2 – 2x2

δy 2 ( x + δx ) − 2x
2 2

∴ =
δx δx

1 1 1
Again for y = , δy = − 5
( x + δx ) x
5 5
x

f ( x + δx ) − f ( x )
or, lim f ( x + h) − f ( x )
δy
Definition : A function y = f(x) is said to be derivable at x if lim or, lim
δx → 0 δx δx → 0 δx h→ 0 h
exists and equal to l. Now this limit l that exists is known as derivative (or differential co-efficient) of y [or f(x)]
with respect to x.
Symbols : Derivative of y [ = f(x)] w.r.t.x (with respect to x) is denoted by
dy d
or, f ′ (x), or, f ( x ) or, Dy or, D [f(x)] or, y1
dx dx

dy δy f ( x + δx ) − f ( x ) f(x + h) − f ( x )
Now = lim = lim = lim , provided this limit exists.
dx δ x → 0 δx δ x → 0 δx h →0 h

dy d d
Note : does not mean the product of with y. The notation stands as a symbol to denote the operation of
dx dy dx dx
differentiation only. Read as ‘dee y by dee x’.
dx
SUMMARY :
dy
The whole process for calculating f′ (x) or
may be summed up in the following stages :
dx
1. Let the independent variable x has an increment h and then find the new value of the function
f (x + h).
2. Find f(x + h) – f(x).
f(x + h) − f ( x )
3. Divide the above value by h i.e., find .
h
f ( x + h) − f(x)
4. Calculate lim = f ′ (x)
h→ 0 h

SOME USEFUL DERIVATIVES :


d n d 1 n
1. x = nx n−1 2. . n = − n+ 1 .
dx dx x x

198 The Institute of Cost Accountants of India


Calculus - Application in Business

d x d x
3. e = e x . [e = constant = 2.718 (Approx)] 4. a = ax loge a.
dx dx
d 1 d 1
5.
dx
loge x = .
x
6.
dx
(loga x ) = x loga e. [a = Constant other than ‘e’]
dc
7. = 0 (c = constant)
dx
d du dv
8. (u ± v) = ± [u and v are functions of x]
dx dx dx

d dv du
9. (uv) = u +v [u and v are functions of x]
dx dx dx
This formula is knon as Product formula because the function to be differentiated is expressed as product of two
functions
du dv
d  u  v dx − u dx
10. = [u and v are functions of x]
dx  v  v2
This formula is known as Division formula because the function to be differentiated is expressed as quotient of
two functions.

dy dy du
11. = ⋅
dx du dx

d 4 d
Example 50 :
dx
( )
x = 4x 3 ;
dx
(x) = 1.x1−1 = 1x0 = 1.1 = 1.

d  1 d −1 −1
  = (x ) = −1.x −1−1 = −1x −2 = 2
dx  x  dx x

d
( x ) = dxd ( x ) = 21 .x 1
1
−1/2
. 2
= ;
dx 2 x

d  1 
.  =
dx  x  dx
d −1
( )
1
2
−1 d
. x 2 = − .x −3/2 = 3/2 ;
2x dx dx 2
( )
x x = d . x 3/2 = 3 x 12 = 3 x ;
2
( )
d 3 d 3 d 2
Examples 51: (x + x 2 ) = (x ) + (x ) = 3x 2 + 2x.
dx dx dx
d 2 x d x d 2
Example 52 : (x .e ) = x 2 (e ) + ex x = x 2ex + ex .2x = x 2ex + 2xex .
dx dx dx
d d(x 4 )
Example 53: (2x 4 ) = 2 = 2.4x 3 = 8x 3 .
dx dx
x2 dy
Example 54 : If y = find
x +1 dx

The Institute of Cost Accountants of India 199


Fundamentals of Business Mathematics and Statistics

Answer:
u du d 2
Let y =
v
where u = x2, =
dx dx
( )
x = 2x 2 −1 = 2x

dv d d d
And v = ( x + 1) , = ( x + 1) = x+ .1 = 1+ 0 = 1
dx dx dx dx
du dv
v − u.
dx = ( x + 1).2x − x .1 = 2x + 2x − x = x + 2x
2 2 2 2
dy d  u dx
Now = =
dx dx  v  v2 ( x + 1)
2
( x + 1)
2
( x + 1)
2

dy
Example 55: Find of the following functions :
dx
(i) x4 + 4x, (ii) 3x5 – 5x3 + 110, (iii) – 2 + (4/5) x5 – (7/8) x8.
Answer:
(i) Let y = x4 + 4x.
dy d 4 d 4 d d
=
dx dx
(
x + 4x =
dx
)
(x ) +
dx
(4x ) = 4.x 4−1 + 4 dx . x = 4x 3 + 4.

(ii) Let y = 3x5 – 5x3 + 110
dy d d d d

=
dx dx
(
3x 5 − 5x 3 + 110 =
dx
3x 5 −
dx
)
5x 3 + ( )
dx
.110 ( )
d d
= 3 ( x 5 ) − .5 ( x 3 ) + 0 (as 110 is a constant number)
dx dx
= 3.5x5 – 1 – 5.3x3 – 1 = 15x4 + 15x2.
4 5 7 8
(iii) Let y = −2 + x − x
5 8
dy d  4 7  d d 4 d 7
=
dx dx 
−2 + x 5 − x 8  =
5 8  dx
( −2) + dx  5 x 5  − dx  8 x 8 

4 d 5 7 d 8
= 0+ (x ) − (x ), (as – 2 is a constant number)
5 dx 8 dx
4 7
= .5x 5−1 − .8x 8 −1 = 4x 4 − 7x 7 .
5 8

1 2 ds
Example 56: If s = ut + ft , find when t = 2.
2 dt
Answer:

ds d  1  d d 1 dt 1 d
=
dt dt 
ut + ft2  =
2  dt
(ut) + dt  2 ft2  = u dt + 2 f dt (t2 )

200 The Institute of Cost Accountants of India


Calculus - Application in Business

1
(here u, f, are constants & t is a variable, since we are to differentiate w.r.t. t)
2
1 1
= u.1+ f.2t2 −1 = u + .2ft = u + ft
2 2
ds
For t = 2, = u + 2f.
dt

EXAMPLES RELATED TO PRODUCT FORMULA:

Example 57: Differentiate (x + 1) (2x3 – 21) with respect to x.


Answer:
Let y = (x + 1) (2x3 – 21) = u. v where u = x + 1, v = 2x3 – 21

du d d(x) d
=
dx dx
( x + 1) = dx + dx (1) = 1+ 0 = 1

dv d d d
=
dx dx
(
2x 3 − 21 = 2
dx
)
.(x 3 ) −
dx
(21) = 2.3x 3−1 − 0 = 6x 2 .
dy d dν du
Now =
dx dx
(uv) = u
dx

dx
(
= ( x + 1). 6x 2 + 2x 3 − 21 .1 )
= 6x3 + 6x2 + 2x3 – 21 = 8x3 + 6x2 – 21.
dy
Example 58: y = x (x2 – 1) (x3 + 2), find .
dx
Answer:
Let y = uvw, where u = x, v = x2 – 1, w = x3 + 2
du dv dw
= 1, = 2x and = 3x 2
dx dx dx

dy d du dv dw
=
dx dx
(uvw ) = vw
dx
+ uw
dx
+ uv
dx

= (x2 – 1) (x3 + 2).1 + x (x3 + 2). 2x + x (x2 – 1). 3x2

= 6x5 – 4x3 + 6x2 (on simplification).

dy
Example 59: If y = 10x x10 , find .
dx
Answer:
Let y = uv where u = 10x and v = x10
du d
Now = (10 x ) = (10 x )loge 10 ;
dx dx

The Institute of Cost Accountants of India 201


Fundamentals of Business Mathematics and Statistics

dv d
Again = . (x10 ) = 10. x10 −1 = 10x 9 .
dx dx
dy d dv du
∴ =
dx dx
(u.v.) = u dx + v dx = 10x 10x9 + x10. 10x loge 10 = 10x (10x9 + x10 loge 10)

EXAMPLE RELATED TO DIVISION FORMULA:

x −1 dy
Example 60: If y = , find
x +1 dx
Answer:
u du d d(x) d
Let y = , where u = x – 1, =
dx dx
( x − 1) = −
dx dx
.(1) = 1− 0 = 1
v

dv d d(x) d
v = x + 1, =
dx dx
( x + 1) = +
dx dx
(1) = 1+ 0 = 1

du dv
v −u
∴ dy
= dx dx = ( x + 1).1− ( x − 1).1 = x + 1− x + 1 = 2
dx ( x + 1) ( x + 1) ( x + 1)
2 2 2 2
v

( x + 1) (2x 2
) , find dy .
−1
Example 61: If y = 2
x +1 dx
Answer:

u
Let y = , where u = (x + 1) (2x2 – 1) = 2x3 + 2x2 – x – 1.
v
du
and = 6x 2 + 4x − 1,
dx
dv
v = x2 + 1 ; = 2x
dx

du dv
dy
=
v
dx
−u 2
( 2
)( 2
) 4
( )
dx = x + 1 6x + 4x − 1 − ( x + 1) 2x − 1 . 2x = 2x + 3x + 2x − 1
3

dx v2 ( ) ( )
2 2
x2 + 1 x2 + 1

DERIVATIVE OF FUNCTION OF A FUNCTION


A variable y may be a function of a second variable z, which again may be a function of a third variable x.
i.e., y = z2 + 3, and z = 2x + 1
Here y is a function of z and z again a function of x. Ultimately y is seen to depend on x, so y is called the function
of another function.

202 The Institute of Cost Accountants of India


Calculus - Application in Business

Symbolically, if y = f (z), z = φ (x) then y = f {φ (x)}


dy dy dz
Theorem. If y = f(z) and z = φ (x) then = . (proof is not shown at present)
dx dz dx
du du dv dw
Corr. If u = f(v), v = φ (w), w = ψ (x) then = . .
dx dv dw dx
dy
Example 62 : To find for y = 2z2 + 1, z = 4x – 2
dx
Answer:
dy dz
Now = 4z and =4
dz dx
dy dy dz
= . = 4z. 4 = 16z = 16 (4x − 2) = 64x − 32.
dx dz dx

dy
Rule 1. If y = ax + b, to find . Let y = z, and z = ax + b.
dx
So y = f (z) and z = f (x)

∴ dy dy dz
= . = 1. (a. 1 + 0) = a
dx dz dx

dy
Rule. 2. If y = (ax + b)n, to find Let y = zn and z = ax + b
dx
dy dz
Now = n.z n−1 and =a
dz dx
dy dy dz
∴ = . = nz n−1 a = na (ax + b)n – 1
dx dz dx

dy
Example 63 : If y = (2x + 5)4 find
dx
Answer:

Let y = z4, where z = 2x + 5.

dy dz
Now = 4z 3 , = 2
dz dx
dy dy dz
= 4z 3 . 2 = 4.2 ( 2x + 5) = 8 ( 2x + 5)
3 3
Now = .
dx dz dx

dy dy dy du
Rule 3. If y = log u (u is a function of x), then to find ; = .
dx dx du dx

The Institute of Cost Accountants of India 203


Fundamentals of Business Mathematics and Statistics

dy
Example 64 : y = log (4x), find
dx
Answer:
du
Let y = log u, where u = 4x, =4
dx
dy dy du d 1 1 1
= = (log u). 4 = ⋅ 4 = 4. = .
dx du dx du u 4x x

dy
( )
Example 65 : y = log 1+ x , find
dx
Answer:

Let y = log u, where u = 1+ x

du 1 1 dy dy du 1 1 1
= 0+ = ; = . = . =
dx 2 x 2 x dx du dx u 2 x 1+ x . 2 x ( )
dy
Example 66: If y = x 2 + 7, find .
dx
2 dz
Answer: Let y = z , where z = x + 7, = 2x.
dx
dy dy dz d 1 x x
= . = . z. 2x = . 2x = =
dx dz dx dz 2 z z x +72

2x + 1 dy
Example 67: Given y = , find .
x+2 dx
Answer:
d d
2x + 1 du ( x + 2) dx (2x + 1) − (2x + 1) dx ( x + 2)
Let y = u, where u = , =
x + 2 dx ( x + 2)
2

du ( x + 2 ).2 − ( 2x + 1).1 3
= =
dx ( x + 2) ( x + 2)
2 2


dy 1 1 x+2
= = .
du 2 u 2 2x + 1

dy dy du 1 x+2 3 3
= = . =
dx du dx 2 2x + 1 ( x + 2 )2
2 2x + 1. ( x + 2 )
3/2

dy
Example 68: If y = log log log x2, find
dx

204 The Institute of Cost Accountants of India


Calculus - Application in Business

Answer:
Let y = log u where u = log v and v = log x2 = 2 logx.
dy 1 1 1
= = = .
du u logv log log x 2

du 1 1 1
= = = .
dv v log x 2 2 log x

dv 2
=
dx x
dy dy du dv 1 1 2 1
= . . = . . =
dx du dv dx log log x 2 log x x x log x log log x 2
2

Example 69: Differentiate x5 w.r.t.x2


Answer:
d dy
Let y = x5, z = x2. So the requirement of the question is ( x5 ) =
d ( x2 ) dz
dy dz d 2 dx 1
= 5x 4 , = (x ) = 2x, so that = .
dx dx dx dz 2x

dy dy dx 1 5
Now = . = 5x 4 . = x3.
dz dx dz 2x 2

(
Example 70: y = loge x + x 2 + a2 , find ) dy
dx
.

Answer:
Let y = log u where u = x + x 2 + a2

dy 1 du d d 2 du 1
( )
1/2
= and = (x) + x + a2 or, = 1+ .2x
du u dx dx dx dx 2 x 2 + a2

dy dy du 1  x  1 x + x 2 + a2 1
Now = = . 1+ = . = .
dx du dx u  
x 2 + a2  x + x 2 + a2 x 2 + a2 x + a2
2

DERIVATIVE OF IMPLICIT FUNCTION


If f(x, y) = 0 defines y as a derivable function of x, then differentiate each term w.r.t.x. The idea will be clear from
the given example.

dy
Example 71: Find , if 3x4 – x2y + 2y3 = 0
dx

The Institute of Cost Accountants of India 205


Fundamentals of Business Mathematics and Statistics

Answer:
 2 dy  dy
Differentiating each term of the functions w.r.t.x we get, 3.4x3 –  x + 2xy + 6y 2 =0
 dx  dx
dy dy
or, 12x 3 − x 2 − 2xy + 6y 2 =0
dx dx
dy 2xy − 12x 3
or, (6y 2
− x2 ) dy
dx
= 2xy − 12x 3
or,
dx
=
6y 2 − x 2
.

DERIVATIVE OF PARAMETRIC FUNCTION


When each of the variables x and y can be expressed in terms of a third variable (say t) then the function y = f(x)
is known as parametric function.
dy dy dx dy dy dt dy / dt dx
For example. x = f1 (t), y = f2 (t). Now to find we are to find and so that = . = , ≠ 0.
dx dt dt dx dt dx dx / dt dt
dy
Example 72: Find when x = 4t, y = 2t2
dx
Answer:
dx dy dy dy / dt 4t
= 4, = 4t, = = =t
dt dt dx dx / dt 4

dy 3 at 3at2
Example 73: Find , when x = , y =
dx 1+ t 3 1+ t 3
Answer:
dx
=
( )
1+ t3 .3a − 3at 3t2
=
3a 1− 2t3 ( ) ( )
dt ( ) ( )
2 2
1+ t3 1+ t3

3
( 2
)
dy 1+ t 6at − 3at 3t
=
2

=
3at 2 − t3 ( ) ( )
dt ( ) ( )
2 2
1+ t 3 1+ t 3

3 2
(
dy dy / dt 1+ t 6at − 3at 3t
= =
2

+
)
3a 1− 2t3
=
( )
3at 2 − t3
=
t 2 − t3 ( ) ( ) ( )

dx dx / dt 1+ t3
2
(
1+ t3
2
)
3a 1+ 2t3 1− 2t3 ( ) ( )

SECOND ORDER DERIVATIVE

Introduction :
We have seen that the first order derivative of a function of x, say f(x), may also be a function of x. This new
function of x also may have a derivative w.r.t.x which is known as second order derivative of f(x) i.e. second order
derivative is the derivative of first order derivative.
Similarly the derivative of the second order derivative is known as third order derivative and so on up to nth order.

206 The Institute of Cost Accountants of India


Calculus - Application in Business

Symbols :
dy
For the function y = f(x), the first order derivative w.r.t.x is denoted by or f′ (x) or y1 as discussed before.
dx
d2 y d2 y
Now the second order derivative of y = f(x) is expressed as 2
or f ′′ (x) or y2. The notation is read as “dee
two y by dee x squared”. dx dx 2

d2 y
Example 74: If y = x4, find .
dx 2
Answer:

dy d2 y d  dy  d
y = x4 , = 4x 3 , again 2 = = (4x 3 ) = 4.3x 3 −1 = 12x 2 .
dx dx dx  dx  dx

d3 y d3 y d  d2 y  d d
Note : To find 3
dx dx
; 3
= =
dx  dx 2  dx
(
12x 2 = 12. (x 2 )
dx
)
= 12.2x2 – 1 = 24x i.e., third order derivative is 24x.

d2 y log x
Example 75 : Find 2 if
y= .
dx x
Answer:
1
dy x. − log x.1 1− log x
= y1 = x = ;
dx x2 x2
 1
2 x 2  −  − (1− log x ). 2x
dy d  dy  d  1− log x   x
= = =
dx 2 dx  dx  dx  x 2  x4

− x − 2x (1− log x ) −1− 2 (1− log x ) 2 log x − 3 .


= = =
x4 x3 x3
Example 76 :
d2 y
If y = x2 ex, Prove = (x2 + 4x + 2)ex
dx 2
Answer:

y = x2 ex

Differentiating both sides w.r.t x

dy
= x2.ex + ex.2x
dx
= (x2 + 2x)ex

Again differentiating both sides w.r.t x

The Institute of Cost Accountants of India 207


Fundamentals of Business Mathematics and Statistics

d2 y
= (x2 + 2x) ex + ex (2x + 2)
dx 2
= (x2 + 2x + 2x + 2)ex

= (x2 + 4x +2)ex

FOR IMPLICIT FUNCTION AND PARAMETRIC FORMS :

x 2 y2 d2 y
Example 77 : For 2
+ 2 = 1, find
a b dx 2
Answer:

2x 2y.y1 2x 2y b2 x
Diff. Both sides w.r.t. x we get + = 0 or 2
+ 2
.y1 = 0 or y1 = − .
a2 b2 a b a2 y

 b2 x 
y + x  a2 . y 
d2 y  b2  y.1− x.y1  b2 
= − . = − 2 [putting the value of y1]
dx 2  a2  y2  a  y2


2 2
(
 b2  a y + b x
2 2
)
 b2  a2b2
= − 2.
 a  a2 y 3
=  − a2  . a2 y 3 [from the given equation we have a 2 y 2 + b 2 x 2 = a 2b 2 ]

b4
= −
y3

Example 78:
d2 y
If y = t2 + t3, x = t – t4, find .
dx 2
Answer:
dy dx
= 2t + 3t2 , = 1− 4t3
dt dt

dy dy dt dy dx 2t + 3t2
= . = =
dx dt dx dt dt 1− 4t3

d2 y d  dy  d  dy  dt d  dy  dx d  2t + 3t2 
dx 2
= = . =
dx  dx  dt  dx  dx dt  dx 
    =
dt dt  1− 4t3 
(1− 4t )
3

=
(1− 4t ) (2 + 6t) − (2t + 3t ) ( −12t ) = 12t + 16t + 6t + 2
3 2 2 4 3

(1− 4t ) . (1− 4t ) (1− 4t )


2 2
3 2 3

208 The Institute of Cost Accountants of India


Revenue and Cost Function 3.2
Revenue
The term ‘revenue’ refers to the receipts obtained by a firm form the sale of certain quantities of a commodity at
various prices. The revenue concept relates to total revenue, average revenue and marginal revenue.
� Total Revenue: Total revenue refers to the total amount of money that the firm receives from the sale of its
products. Thus, total revenue is equal to the quantity sold multiplied by the selling price of the commodity.
Algebraically total revenue can be written as:
TR = PQ

where, TR = Total Revenue; P = Price per unit; Q = Quantity sold


� Average Revenue: Average Revenue can be obtained by dividing the total revenue by the number of units
sold. Thus, Average Revenue can be written as:
Total Revenue
Average Revenue =
Total output sold

TR PQ
Or, AR= = =P
Q Q
Thus, average revenue is the price of the commodity.
� Marginal Revenue: Marginal Revenue is the net revenue earned by selling one additional unit of the product
produced. In other words, marginal revenue is the addition made to the total revenue by selling one additional
unit of the commodity. Putting it in algebraic expression, marginal revenue is the addition to total revenue by
selling (n + 1) units of a product instead of n units. Therefore,
Marginal Revenue = difference in total revenue in increasing sales from (n + 1) units to n Units

MR = TRn +1 − TRn

In other words Marginal Revenue is the rate of change of Total Revenue with respect to the Quantity sold.
If TR stands for Total Revenue and Q stands for output, the marginal revenue (MR) can be expressed as:
d (TR)
MR =
dQ
d (TR)
Here, indicates derivative of TR with respect to Q which is also the slope of the total revenue curve. Thus,
dQ
if total revenue is given to us, we can find out marginal revenue at various levels of output by finding the slopes at
the corresponding points on the total revenue curve.

The Institute of Cost Accountants of India 209


Fundamentals of Business Mathematics and Statistics

Examples:
1. A schedule on Total Revenue and Amount of output sold is given. Find out Average Revenue and Marginal
Revenue
Quantity of Output sold Total Revenue Average Revenue Marginal Revenue
(units) (TR)
0 0
1 29
2 54
3 72
4 80
5 80
6 72
Solution:

Quantity of Output Total Revenue Average Revenue Marginal Revenue


sold (units) (TR)  TR PQ 
 Q = Q = P  ( MR = TRn +1 − TRn )

0 0 0 -
1 29 29 29
2 54 27 25
3 72 24 18
4 80 20 8
5 80 16 0
6 72 12 -8

2. Consider the following market demand schedule:


Price per unit (`) 3 5 8 12 15 20
Quantity (unit) 25 21 18 15 10 4
Now determine the AR and TR schedules of the seller

Solution:

Quantity of Price Total Revenue Average Revenue (`)


Output sold (Q) per unit (TR=P.Q)  TR PQ 
(units) (P) (`) (`)  Q = Q = P 
25 3 75 3
21 5 105 5
18 8 144 8
15 12 180 12
10 15 150 15
4 20 80 20

210 The Institute of Cost Accountants of India


Calculus - Application in Business

Cost
The term ‘total cost’ is the sum of all costs incurred by a firm in producing a certain level of output. In the
traditional theory total cost is divided into two groups: total fixed cost and total variable cost.
TC = TFC + TVC
where, TC = Total Cost; TFC = Total Fixed Cost; TVC = Total Variable Cost
� Total Fixed cost: There are some cost which do not vary with the level of output produced. These remain fixed
even when output changes. These are fixed costs. For eg, rent of land, license fees etc. Suppose, that the land
on which a factory is situated has an annual rent of ` 12,000. It is obvious that the amount of rent will not have
any relationship with the quantity of output produced by the factory. Whether the output is 20 tons or 2,000
tons, the rent will remain fixed at ` 12,000. In fact, the rent would be the same, even if the factory is temporarily
closed down. For this reason, the rent on land is a fixed cost.
� Total Variable Cost: On the other hand, some costs change with the level of output produced. This type
of cost rises when output increase, and falls when output decrease. For eg, cost of raw materials, wages and
salaries of casual employees, transport cost etc. Suppose in order to produce a huge amount of output, we
require higher amount of raw materials. So, the cost of raw materials will increase and hence variable cost will
increase.
Average Cost: The average cost is obtained by dividing Total cost by the level of output. Thus,

Total Cost (TC )


Average Cost ( AC ) =
Output

Total Fixed Cost (TFC )


Similarly, Average Fixed Cost ( AFC ) =
Output

Total Variable Cost (TVC )


Average Variable Cost ( AVC ) =
Output
Marginal Cost: The marginal cost is defined as the change in total cost which results from a unit change in output.
In other words, marginal cost is the addition to the total cost when the quantity of output produced increases by one
unit. In symbols it can be written as:
MC = TCn +1 − TCn

Mathematically, the marginal cost is the first order derivative of the Total cost function. Denoting total cost by TC
and output by Q we have:
d (TC )
MC =
dQ
It is worth noting that marginal cost is independent of fixed cost. Since fixed cost do not change with the level of
output, there is no marginal fixed cost when output is increased.

Profit: The term ‘profit’ is defined as the difference between total revenue and total cost. Thus,
Profit = Total Revenue – Total Cost
In order to obtain the maximum amount of output we differentiate the profit function with respect to the output
produced and set the equation equal to zero (First order condition for maximization). Then, we find the second
order derivative of the function. If the second order derivative is less than zero then we can conclude that the profit
is maximized at that level of output. Thus,

The Institute of Cost Accountants of India 211


Fundamentals of Business Mathematics and Statistics

d ( Profit )
= 0 [First Order Condition for maximisation]
dQ
d 2 ( Profit )
< 0 [Profit is maximized at Qmax ]
dQ 2

d ( Profit ) d (TR − TC ) d (TR) d (TC )


So for mamimum Profit, =0⇒ = 0⇒ − = 0 ⇒ MR − MC = 0 ⇒ MR
MR==MC
MC
dQ dQ dQ dQ
Thus profit is maximum at the equilibrium point under perfect competition. In other words maximum profit is
obtained when Marginal Cost and Marginal Revenue are equal.
Break-even Point: Break-even point is the point at which total cost and total revenue are equal which means that
the firm does neither earn any profit nor any loss. The firm faces a no gain no loss situation. Thus, at the Break –
even Point :
Profit = 0
Or, Total Cost = Total Revenue
Examples:
3. For a firm the Total Revenue (TR) and Total Cost (TC) functions are given as TR = 20Q and TC = Q2 + 4Q +
20 where Q = Output. Find the profit maximizing output and maximum profit.

Solution:
Profit = TR-TC
= 20Q − Q 2 − 4Q − 20
= 16Q − Q 2 − 20
d (Profit )
= 16 − 2Q
dQ
∴ 16 − 2Q = 0 [Find Order Condition for maximization]
Or, Q = 8

d 2 ( Profit ) d  d ( Profit )  d
2
=
dQ  dQ  = dQ (6 − 2Q) = – 2 < 0
dQ  
So, the profit is maximized at Q = 8
The maximum profit = 16 × 8 – 82 – 20 = 44

4. In a firm, price of a product is ` 4 and TC = Q3 – 15Q2 + 31Q + 100 where Q = Output. Find the profit
maximizing output and maximum profit

Solution:
Total Revenue = Price × Quantity = 4Q
Profit = TR-TC
= 4Q − Q 3 + 15Q 2 − 31Q − 100
= −27Q − Q 3 + 15Q 2 − 100

212 The Institute of Cost Accountants of India


Calculus - Application in Business

d ( Profit )
= −27 − 3Q 2 + 30Q
dQ
∴ 3Q 2 − 30Q + 27 = 0 [First Order Condition for maximization]
2
Or, Q − 10Q + 9 = 0
Or, Q 2 − Q − 9Q + 9 = 0
Or, Q (Q − 1) − 9 (Q − 1) = 0
Or, (Q − 9) (Q − 1) = 0
Or, Q = 9,1

d 2 ( Profit ) d  d (Profit)  d
= [ −27 − 3Q 2 + 30Q] = – 6Q + 30
dQ 2 dQ  dQ  dQ
At Q = 1, Profit = 24 > 0
At Q = 9, Profit = – 24 < 0
So, the profit is maximized at Q = 9
The maximum profit at Q = 9 is (– 27) × 9 –93 + 15 × 92 – 100 = 143

5. A firm has the following functions of Total Revenue (TR) and Total Cost (TC) for Q output TR = 30 Q – Q2 and
TC = (Q3 –15Q2 +10Q +100). Find the profit maximizing output and maximum profit and equilibrium price.

Solution:

Profit = TR–TC
= 30Q − Q 2 − Q 3 + 15Q 2 − 10Q − 100

= 20Q − Q 3 + 14Q 2 − 100


d (Profit )
= 20 − 3Q 2 + 28Q
dQ
∴ 3Q 2 − 28Q − 20 = 0 [F.O.C for maximization]
2
Or, Q = 28 ± 28 + 4 × 3 × 20 [by using Sridhar Acharya Formula]
2×3

28 ± 784 + 240
Or, Q =
6
28 ± 32
Or, Q =
6
2
Or, Q = 10, −
3

d 2 ( Profit ) d  d ( Profit )  d
= = [20 − 3Q 2 + 28Q] = –  6Q + 28
dQ 2 dQ  dQ  dQ

The Institute of Cost Accountants of India 213


Fundamentals of Business Mathematics and Statistics

At Q = 10, Profit = – 32 < 0


2
At Q = − , Profit = 32 > 0
3
So, the profit is maximized at Q = 10
The maximum profit at Q = 10 is 20 × 10 – 103 + 14 × 102 – 100 = 500
TR
We know that P = AR, so AR = = 30 − Q
Q
As profit is maximized at Q = 10 then P = 30 – 10 = ` 20 is the equilibrium price

6. A firm has the following functions of Total Revenue (TR) and Total Cost (TC) for Q output TR = 26Q – 3Q2
and TC = 2Q2 – 4Q + 10. Find the profit maximizing output and maximum profit and equilibrium price and
Total Revenue at that level of output.
Solution:
Profit = TR – TC
= 26Q − 3Q 2 − 2Q 2 + 4Q − 10
= 30Q − 5Q 2 − 10
d ( Profit )
= 30 − 10Q
dQ
∴ 30 − 10Q = 0 [F.O.C for maximization]
Or, Q = 3

d 2 ( Profit ) d  d ( Profit )  d
= = [30 − 10Q] = – 10 < 0
dQ 2 dQ  dQ  dQ

So, the profit is maximized at Q = 3


The maximum profit at Q = 3 is 30 × 3 – 5 × 32 – 10 = 35
TR
We know that P = AR, so AR = = 26 − 3Q
Q
As profit is maximized at Q = 3 then P = 26 – 3×3 = 17 Money units. This is the equilibrium price
Total Revenue at this price = 26 × 3 – 3 × 32 = 51 Money units

214 The Institute of Cost Accountants of India


Optimization Techniques
(Basic Concepts) 3.3
Slope and Curvature

dy
For a differentiable function y = f (x) the derivative or f ′( x) is often called the first derivative or the first order
dx
derivative. Generally this first order derivative f ′( x) is a function of x and therefore f ′( x) is further differentiable

with respect to x which is called the second order derivative of the function y = f (x). This second order derivative
d2y
is denoted by f ′′( x) or . In the same manner if f ′′( x) be differentiable then we can obtain the third order
3
dx 2
d y
derivative f ′′′( x) or . Thus, proceeding in the similar way one can introduce the nth order derivative of y = f
dx 3
dny
(x) which will be denoted by .
dx n
The first and second order derivatives are much used in the graphical analysis. First derivative of a function at any
point measures the slope of a function at a particular point. If f ′( x) > 0 for all values of x in the domain of the
function then we can say that f ( x) rises continuously from left to right. But we know that the function which rises
continuously is called monotonically increasing function. Thus, the condition of a function to be monotonically
increasing is that the first derivative must be positive all through. Similarly, a function y = f (x) is said to be
monotonically decreasing is f ′( x) < 0 for all x. In short the condition of monotonicity for a function y = f (x) is
either f ′( x) > 0 or f ′( x) < 0 for all x.

Examples:
1. f (x) = 2x
f ′( x) = 2 > 0 This function is monotonically increasing

2. f (x) = – 6x
f ′( x) = − 6 < 0 . This function is monotonically decreasing
To know whether a function is increasing at a constant rate or at an increasing rate or at a decreasing rate, we
perform the second order derivative. The nature of the function (constant, increasing, decreasing) can be known
form the knowledge of the curvature of the curve. By curvature we mean the bending of a curve. To measure
this bending we have to measure the rate of change of the slope of the curve. If the rate of change of the slope of
the curve is positive i.e. f ′′( x) > 0 the curve is said to be convex curve. If f ′( x) > 0 and f ′′( x) > 0 the curve
increases at an increasing rate. On the other hand, if f ′( x) < 0 and f ′′( x) > 0 the curve decreases at an increasing
rate. Lastly, there may be some curve which increase or decreases at a constant rate. Linear curves are examples of
this case and for this f ′′( x) = 0 .

The Institute of Cost Accountants of India 215


Fundamentals of Business Mathematics and Statistics

Now, we can also identify whether a curve is convex or concave by just looking at the curve and by drawing a
tangent at any point on the curve. If the curve lies above the tangent it will be convex at that point, otherwise if the
curve lies entirely below the tangent it will be concave at that point.
Y Y

0 X 0 X
Fig. 1 Fig. 2
Convex Concave
f ′( x) < 0 , f ′′( x) > 0 f ′( x) < 0 , f ′′( x) < 0
Y Y

0 X 0 X

Fig. 3 Fig. 4
f ′ ( x ) < 0 , f ′′ ( x ) = 0 f ′( x) > 0 , f ′′( x) = 0
Y Y

0 X 0 X

Fig. 5 Fig. 6
f ′ ( x ) > 0 , f ′′ ( x ) < 0 f ′( x) > 0 , f ′′( x) > 0

216 The Institute of Cost Accountants of India
Calculus - Application in Business

Maxima and Minima


In order to determine the relative extremum (relative maximum and minimum) of a function y = f ( x) , the first
derivative plays an important role.

A
Y
y = f (x)

0 x1 x2 X
Fig. 7
In the figure 7 given here the function y = f ( x) is attaining the maximum value at the point A, where x = x1 and
minimum value at the point B, where x = x2 . It appears that the slope of the curve is positive before it attains
maximum at the point x = x1 and is negatively sloped immediately after attaining the maximum value and at the
maximum point A the slope of the curve is zero as the tangent at A is horizontal. Since the slope of a function
dy dy
y = f ( x) is represented by its first derivative dx , so the necessary condition for maximization is = f ′( x) = 0 .
dx
Similarly it appears that the curve is sloping downward before attaining minimum value at B and immediately after
attaining the minimum it slopes upward indicating that the slope of the curve is zero at the minimum point B since
the tangent at B is horizontal. Since, the slope of the curves at both the extreme point are 0, first order derivative is
set at 0 is a necessary condition for both maximization and minimization.
For sufficient condition for maximum and minimum value of the function y = f ( x) , we have to see the slope of the
curve immediately after attaining the maximum or minimum. From the figure, it is seen that after attaining maximum
point at A the curve is sloping downward while it is sloping upwards after attaining minimum. Thus, a distinction
between maximum and minimum can be made by examining the direction of change of the first derivative. Given
the function y = f ( x) whose first derivative f ′( x) = 0 at point x = x1 will be a relative maximum if the derivative
f ′( x) changes its sign from positive to negative as x increases in the neighbourhood of x = x1 . Similarly, y = f ( x)
will attain a relative minimum if the derivative f ′( x) changes its sign from negative to positive as x increases in the
neighbourhood of x = x2 . Since the direction of change of first derivative can be stated in terms of the sign of the
second derivative f ′′( x) , the sufficient condition for maximization is f ′′( x) < 0 and for minimization is f ′′( x) > 0.
Examples:
2
1. If y = 20 x − 2 x then find the value of x for which y is optimum.
Answer:
dy
= 20 − 4 x .
dx
dy
As = 0 according to the first order condition,
dx
20 – 4x = 0 or, 4x = 20 or, x = 5
d2y
= − 4 < 0 So, the function is maximum at x = 5
dx 2

The Institute of Cost Accountants of India 217


Fundamentals of Business Mathematics and Statistics

2. If y = 2 x 2 − 16 x + 50, then find x where y is optimum.


Answer:

dy
= 4 x − 16 .
dx
dy
As = 0 according to the first order condition,
dx
4x – 16 = 0 or, 4x = 16 or, x = 4

d2y So, the function is minimum at x = 4


=4>0
dx 2
3. If y = x 3 − 9 x 2 + 15x + 20, then find the points where y has maximum and minimum values
Answer:
dy
= 3x 2 − 18x + 15 .
dx
As dy = 0 according to the first order condition,
dx
3x 2 − 18x + 15 = 0
Or, 3x 2 − 15x − 3x + 15 = 0
Or, 3x ( x − 5) − 3( x − 5) = 0
Or, (3x − 3) ( x − 5) = 0 or, x = 1,5
d2y
= 6 x − 18
dx 2
2
Putting x = 1, we get d y2 = −12 < 0
dx
d2y
Putting x = 5, we get = 12 > 0
dx 2
So, the function is maximum at x = 1 and minimum at x = 5
1 3 5 2
4. Find x where y = x − x + 4 x + 10 has optimum values.
3 2
Answer:
dy
= x 2 − 5x + 4 .
dx
As dy = 0 according to the first order condition,
dx
x 2 − 5x + 4 = 0
Or, x 2 − x − 4 x + 4 = 0
Or, x( x − 1) − 4 ( x − 1) = 0
Or, ( x − 4) ( x − 1) = 0 or, x = 1,4
d2y
= 2x − 5
dx 2
d2y
Putting x = 1, we get = −3 < 0
dx 2
2
d y
Putting x = 4, we get = 3> 0
dx 2
So, the function is maximum at x = 1 and minimum at x = 4

218 The Institute of Cost Accountants of India


Calculus - Application in Business

Exercise:
Theoretical Questions
 Multiple Choice Questions (MCQ)

1. lim x→3 ( x 3 + 1)
(a) 52
(b) 53
(c) 55
(d) 54
2
2. lim x→0 (4 x + 7 x + 5)
(a) 4
(b) 7
(c) 5
(d) 16

 x 2 − 16 
3. lim x→ 4 
 x − 4 
(a) 2
(b) 4
(c) 8
(d) 6
x dy
4. If y = xe then =?
dx
(a) xe x
x
(b) e ( x + 1)
x
(c) e ( x − 1)
(d) e x / x

x5 x 4 x3
5. f ( x) = + + − 7 x 2 + 18 ⋅ f ′ ( x) = ?
5 4 2
4 3 2
(a) x + x + 3x + 14 x
4 3 2
6 5 4
(b) x + x + x − 7 x 3 + 18x
6 5 4
2
(c) x 4 − x 3 − 3x + 14 x
2
2
(d) x 4 + x 3 + 3x − 14 x
2

The Institute of Cost Accountants of India 219


Fundamentals of Business Mathematics and Statistics

6. When y = 4 x then derivative of y is ——


(a) x (4 x −1)
4x
(b) 2 log 2
(c) 4 x 2 log 2
(d) None of these

ex
7. Find the differential coefficient of y =
ex + 1
ex
(a) x
(e + 1) 2
e2x
(b) x
(e + 1) 2
ex
(c) 2 x
(e + 1) 2
ex
(d) x
(e + 2) 2

8. y = (4 x − 3)3 + (5x − 2) 2 . Calculate y1


(a) 182 x 2 + 13x + 29
(b) 96 x 2 + 13x + 29
(c) 12 x 2 + 26 x + 29
(d) 192 x 2 + 26 x + 58

x2 y2
9. + = 1 is an Implicit function. The derivative of this function is ——
16 4
x
(a)
4y
x
(b) −
4y
x
(c)
2y
x
(d) −
2y

10. The result of differentiation of y = log 3x 2 + 13x + 10 is ——


6 x 2 + 13x
(a) 3x 2 + 13x + 10
6 x + 13
(b) 3x 3 + 13x 2 + 10 x

220 The Institute of Cost Accountants of India


Calculus - Application in Business

6 x + 13
(c)
3x 2 + 13x + 10

19
(d) 6 x + 13

11. A soft-drink manufacturer has a revenue function R = 7Q 2 − 19Q + 30 and the cost function is given by 9Q.
Find the number of cans produced by the firm, under perfect competition.
(a) 2
(b) 4
(c) 6
(d) 8

12. A tin manufacturer has a revenue function given by: R = 11Q 2 − 110Q + 70 and the cost function is given
by: C = 22Q. Find the number of tins to be produced by the manufacturer.
(a) 2
(b) 6
(c) 10
(d) 14

13. A demand function is given by: P = a – bQ and the cost function is given by C = Q2. Find the value of Q
for which profit will be maximum under perfect competition.
a
(a) (a + 1)
a
(b) 2 (b + 1) 2
a
(c) 2 (b + 1)
b
(d) 2 (a + 1)

14. The demand function is given by: P = 1400 – 25Q and the cost function is given by C = 10Q2. Find the value
of  Q at the equilibrium point.
(a) 10
(b) 20
(c) 30
(d) 40

The Institute of Cost Accountants of India 221


Fundamentals of Business Mathematics and Statistics

15. A revenue function is given by R = 3Q 2 − 8Q + 15 and the cost function is given by: C = 28Q. Find the value
of  Q for achieving highest profit.
(a) 3
(b) 9
(c) 6
(d) 12

16. A demand function is given by: P = 1500 – 3Q and the cost function is given by: C = 12Q2. Find the value
of Q at the equilibirum point.
(a) 50
(b) 100
(c) 150
(d) 125

17. A firm has a fixed production cost of ` 90 and a marginal variable production cost of ` 9. The price of the
product is `18. Find the cost function, revenue function, and the value of Q at the Break Even point.
(a) 2Q + 20; 9Q; 10
(b) 9Q + 90; 18Q; 10
(c) 4Q + 90; 36Q; 20
(d) Q + 10; 5Q; 50

18. A sugar industry has a fixed cost of `290 and a marginal cost of 50 paise. He sells sugar at the price of
`15/kg. Find the quantity of sugar sold for breaking even.
(a) 10
(b) 30
(c) 20
(d) 40

19. A cotton mill has a fixed cost of `1540 and a marginal cost of `33. He sells a shirt at a price of `110. Find
the minimum number of shirts sold so the mill incurs no loss.
(a) 23
(b) 20
(c) 26
(d) 29

222 The Institute of Cost Accountants of India


Calculus - Application in Business

2
20. Given: C ( x) = 900 + 30 x + 0.6 x , P = 90. Find the value of x at equilibrium point.
(a) 40
(b) 50
(c) 70
(d) 30

21. Given: C ( x) = 2 x 2 − 3x − 12 , P = 33. Find the value of x at equilibrium point.


(a) 9
(b) 12
(c) 15
(d) 18

22. Given: R( x) = 3x 2 + 24 x + 2 , MC (x) = 42 and the fixed cost is 90. Find the value of x at equilibrium point.
(a) 2
(b) 3
(c) 7
(d) 11

23. Given: R( x) = 6 x 2 − 11x − 35 , C ( x) : 5x 2 − 3x + 16 . Find the value of x for which profit is maximum.
(a) 2
(b) 4
(c) 6
(d) 12

24. Given: R( x) = 20 x 2 − 15x − 10 , C ( x) : x 2 + 99 x + 27 . Find the value of x for which profit is maximum.
(a) 3
(b) 15
(c) 25
(d) 10
25. Given: R( x) = 3x 2 + 4 x + 2 , MC ( x) = 16 and the fixed cost is 24. Find the profit maximising value of x
under perfect competition.
(a) 1
(b) 3
(c) 4
(d) 2

The Institute of Cost Accountants of India 223


Fundamentals of Business Mathematics and Statistics

26. A manufacturer has a monthly fixed cost of `1, 00,000 and a production cost of `50 per unit produced. The
product is sold at `75. Find the cost function and the number of products be sold by the manufacturerer to
have break even.
(a) 25x + 50,000; 2000
(b) 50x + 1,00,000; 4000
(c) 5x + 1,00,000; 3000
(d) 2.5x + 10,000; 5000

27. A cement industry has a yearly fixed cost of `96,000 and a monthly production cost of `13 per unit
produced. The product is sold at `39 per unit. Find the cost function
(a) 13x + 8000
(b) 13x + 96,000
(c) 39x + 96,000
(d) 39x + 8000

28. Find the monthly profit function if a firm’s yearly fixed cost is `60,000 and yearly production cost is `120
per piece. Each unit is sold at `15.
(a) π = 5x – 5000
(b) π (x) = 15x – 5000
(c) π (x) = 20x – 5000
(d) π (x) = 25x – 5000

29. Given: C (x) = 9x + 350 and P = 14. Find the condition of getting break-even point
(a) 5x – 350 = 0
(b) 7x – 350 = 0
(c) –14x – 350 = 0
(d) None of these

30. With reference to Q. 29 find the break-even quantity


(a) 50
(b) 70
(c) 110
(d) 100

224 The Institute of Cost Accountants of India


Calculus - Application in Business

From Q.31 to Q.39, find whether the function is maximum or minimum:

31. f ( x) = 6 x 2 + 11x − 35
(a) Maximum
(b) Minimum
(c) No Curvature
(d) None of the above

32. f ( x) = − 4 x 2 − 7 x − 35
(a) Maximum
(b) Minimum
(c) No Curvature
(d) None of the above

33. f ( x) = 20 x 2 − 15x − 10
(a) Maximum
(b) Minimum
(c) No Curvature
(d) None of the above

2
34. f ( x) = 3x − 4 x + 2
(a) Maximum
(b) Minimum
(c) No Curvature
(d) None of the above

35. f ( x) = − x + 6 x + 18
(a) Maximum
(b) Minimum
(c) No Curvature
(d) None of the above

The Institute of Cost Accountants of India 225


Fundamentals of Business Mathematics and Statistics

2
36. f ( x) = 9 x − 6 x + 1
(a) Maximum
(b) Minimum
(c) No Curvature
(d) None of the above

2
37. f ( x) = − x + 4 x − 2
(a) Maximum
(b) Minimum
(c) No Curvature
(d) None of the above

2
38. f ( x) = 3x + 2
(a) Maximum
(b) Minimum
(c) No curvature
(d) None of the above

2
39. f ( x) = x − 3x
(a) Maximum
(b) Minimum
(c) No curvature
(d) None of the above

Find the values of x for which the functions of Q. No. 40 to 50 have maximum and minimum points

40. f ( x) = ax 3 + bx 2 + cx + d ; a < 0 ; b < 0 ; c > 0 and a < b

−b + b 2 − 3ac −b − b 2 − 3ac
(a) Maximum, x = ; Minimum, x =
3a 3a

−b + b 2 − 3ac −b − b 2 − 3ac
(b) Minimum, x = ; Maximum, x =
3a 3a
(c) No Curvature
(d) None of the above

226 The Institute of Cost Accountants of India


Calculus - Application in Business

3
41. f ( x) = x − 9 x 2 + 81x + 70
3
(a) Maximum, x = 9; Minimum, x = 9
(b) Minimum, x = 3; Maximum, x = 5
(c) No curvature
(d) None of the above

2 3 9 2
42. f ( x) = 3 x + 2 x − 11x − 21

9
(a) Maximum, x = 2 ; Minimum, x = −
2
11
(b) Maximum, x = − ; Minimum, x = 1
2
3
(c) Maximum, x = − ; Minimum, x = –3
2
(d) No curvature

4 3
43. f ( x) = x − 5x 2 + 4 x − 9
3
3
(a) Maximum, x = 4 ; Minimum, x = −
2
1
(b) Maximum, x = ; Minimum, x = 2
2
(c) Maximum, x = 2 ; Minimum, x = –1
(d) No curvature

3 2
44. f ( x) = x − 2 x − 4 x
2
(a) Maximum, x = − ; Minimum, x = 2
3
3
(b) Maximum, x = ; Minimum, x = 1
4
1
(c) Maximum, x = 3 ; Minimum, x = −
3
(d) No curvature

x3 2
45. f ( x) = 3 − 4.5x − 8x + 2
(a) Maximum, x = 1 ; Minimum, x = 8
(b) Maximum, x = 4 ; Minimum, x = 2
(c) Maximum, x = 3 ; Minimum, x = – 5
(d) No curvature

The Institute of Cost Accountants of India 227


Fundamentals of Business Mathematics and Statistics

3
46. f ( x) = x − 3 x 2 + 2 x − 3
3 2
(a) Maximum, x = 3 ; Minimum, x = 5
(b) Maximum, x = 1 ; Minimum, x = 2
(c) Maximum, x = 6 ; Minimum, x = 1
(d) No curvature

2 3 3 2
47. f ( x) = 3 x − 2 x − 5x

(a) Maximum, x = – 1 ; Minimum, x = 5


2
(b) Maximum, x = 1 ; Minimum, x = 3
3
(c) Maximum, x = –1 ; Minimum, x = −
2
(d) No curvature

x3
+ 4 x 2 − 15x
48. f ( x) = −
3
(a) Maximum, x = 1 ; Minimum, x = 3
(b) Maximum, x = 5 ; Minimum, x = 3
(c) Maximum, x = –3 ; Minimum, x = – 5
(d) No curvature

x3 x 2
49. f ( x) =
− − 2x
3 2
(a) Maximum, x = – 1 ; Minimum, x = 2
(b) Maximum, x = 1 ; Minimum, x = – 2
(c) Maximum, x = – 3 ; Minimum, x = 5
(d) No curvature

2
50. f ( x) = x + 2 x 2 + 3x + 7
3
(a) Maximum, x = – 3 ; Minimum, x = – 1
(b) Maximum, x = 1 ; Minimum, x = 2
2
(c) Maximum, x = 1 ; Minimum, x = 4
(d) No curvature

228 The Institute of Cost Accountants of India


Calculus - Application in Business

Answer:
 Multiple Choice Questions (MCQ)

1 (c) Put x = h + 3 and then evaluate


2 (c) Put x = 0 and evaluate
3 (c) Use the formulae a 2 − b 2 = (a + b) (a − b) and then put x = 4
4 (c) By using the formulae
5 (d) By using the formulae
6 (c) Use logarithm on both sides
7 (a) By using the formulae
8 (d) By using the formulae
9 (b) By using the formulae
10 (c) By using the formulae
11 (a) dR dC
MR = and MC = , then
dQ dQ
Put MR = MC
12 (b) dR dC
MR and MC = , then
dQ dQ
Put MR = MC
13 (c) dR dC
MR = and MC = , then
dQ dQ
Put MR = MC
14 (b) dR dC
MR = and MC = , then
dQ dQ
Put MR = MC
15 (c) dR dC
MR = and MC = , then
dQ dQ
Put MR = MC
16 (a) dR dC
MR = and MC = , then
dQ dQ
Put MR = MC
17 (b) Profit Function = Revenue Function – Cost Function
Put profit function = 0
18 (c) Profit Function = Revenue Function – Cost Function
Put profit function = 0

The Institute of Cost Accountants of India 229


Fundamentals of Business Mathematics and Statistics

19 (b) Profit Function = Revenue Function – Cost Function


Put profit function = 0
20 (b) dR and dC then,
MR = MC =
dx dx
Put MR = MC
21 (a) dR and dC then,
MR = MC =
dx dx
Put MR = MC
22 (b) dR and dC then,
MR = MC =
dx dx
Put MR = MC
23 (b) dR and dC then,
MR = MC =
dx dx
Put MR = MC
24 (a) dR and dC then,
MR = MC =
dx dx
Put MR = MC
25 (d) dR and dC then,
MR = MC =
dx dx
Put MR = MC
26 (b) Profit Function = Revenue Function – Cost Function
Put profit function = 0
27 (a) Total Cost = Fixed Cost + Variable Cost (Production cost)
28 (a) First divide the production cost by 12 in order to find out the monthly
production cost.
Then, profit function = Revenue function – cost function
29 (a) Revenue Function = Price × Quantity. So, R ( x) = P × x
Then Profit Function = Revenue Function – Cost Function = 0
30 (b) Put Profit Function = 0
31 (b)
Find f ′′( x)
Then, if f ′′( x) > 0 → Minimum
if f ′′( x) < 0 → Maximum
32 (a)
Find f ′′( x)
Then, if f ′′( x) > 0 → Minimum
if f ′′( x) < 0 → Maximum

230 The Institute of Cost Accountants of India


Calculus - Application in Business

33 (b)
Find f ′′( x)

Then, if f ′′( x) > 0 → Minimum

if f ′′( x) < 0 → Maximum


34 (b)
Find f ′′( x)

Then, if f ′′( x) > 0 → Minimum

if f ′′( x) < 0 → Maximum


35 (a)
Find f ′′( x)

Then, if f ′′( x) > 0 → Minimum

if f ′′( x) < 0 → Maximum


36 (b)
Find f ′′( x)

Then, if f ′′( x) > 0 → Minimum

if f ′′( x) < 0 → Maximum


37 (a)
Find f ′′( x)

Then, if f ′′( x) > 0 → Minimum

if f ′′( x) < 0 → Maximum


38 (b)
Find f ′′( x)

Then, if f ′′( x) > 0 → Minimum

if f ′′( x) < 0 → Maximum


39 (b)
Find f ′′( x)

Then, if f ′′( x) > 0 → Minimum

if f ′′( x) < 0 → Maximum


40 (a)
Find f ′( x) and put f ′( x) = 0 to find out the roots.

Now find f ′′( x) and put the roots in f ′′( x) in order to find out the
value of f ′′( x)

Then, if f ′′( x) > 0 → Minimum

if f ′′( x) < 0 → Maximum

The Institute of Cost Accountants of India 231


Fundamentals of Business Mathematics and Statistics

41 (c) Find f ′( x) and put f ′( x) = 0 to find out the roots.


Now find f ′′( x) and put the roots in f ′′( x) in order to find out the
value of f ′′( x)
Then, if f ′′( x) > 0 → Minimum
if f ′′( x) < 0 → Maximum and if f ′′ ( x) = 0 → No curveture
42 (b)
Find f ′( x) and put f ′( x) = 0 to find out the roots.
Now find f ′′( x) and put the roots in f ′′( x) in order to find out the
value of f ′′( x)
Then, if f ′′( x) > 0 → Minimum
if f ′′( x) < 0 → Maximum
43 (b)
Find f ′( x) and put f ′( x) = 0 to find out the roots.
Now find f ′′( x) and put the roots in f ′′( x) in order to find out the
value of f ′′( x)
Then, if f ′′( x) > 0 → Minimum
if f ′′( x) < 0 → Maximum
44 (a)
Find f ′( x) and put f ′( x) = 0 to find out the roots.
Now find f ′′( x) and put the roots in f ′′( x) in order to find out the
value of f ′′( x)
Then, if f ′′( x) > 0 → Minimum
if f ′′( x) < 0 → Maximum
45 (a)
Find f ′( x) and put f ′( x) = 0 to find out the roots.
Now find f ′′( x) and put the roots in f ′′( x) in order to find out the
value of f ′′( x)
Then, if f ′′( x) > 0 → Minimum
if f ′′( x) < 0 → Maximum
46 (b)
Find f ′( x) and put f ′( x) = 0 to find out the roots.
Now find f ′′( x) and put the roots in f ′′( x) in order to find out the
value of f ′′( x)
Then, if f ′′( x) > 0 → Minimum
if f ′′( x) < 0 → Maximum

232 The Institute of Cost Accountants of India


Calculus - Application in Business

47 (a)
Find f ′( x) and put f ′( x) = 0 to find out the roots.
Now find f ′′( x) and put the roots in f ′′( x) in order to find out the
value of f ′′( x)
Then, if f ′′( x) > 0 → Minimum
if f ′′( x) < 0 → Maximum
48 (b)
Find f ′( x) and put f ′( x) = 0 to find out the roots.
Now find f ′′( x) and put the roots in f ′′( x) in order to find out the
value of f ′′( x)
Then, if f ′′( x) > 0 → Minimum
if f ′′( x) < 0 → Maximum
49 (a)
Find f ′( x) and put f ′( x) = 0 to find out the roots.
Now find f ′′( x) and put the roots in f ′′( x) in order to find out the
value of f ′′( x)
Then, if f ′′( x) > 0 → Minimum
if f ′′( x) < 0 → Maximum
50 (a)
Find f ′( x) and put f ′( x) = 0 to find out the roots.
Now find f ′′( x) and put the roots in f ′′( x) in order to find out the
value of f ′′( x)
Then, if f ′′( x) > 0 → Minimum
If f ′′( x) < 0 → Maximum

The Institute of Cost Accountants of India 233


Fundamentals of Business Mathematics and Statistics

234 The Institute of Cost Accountants of India


SECTION - B
FUNDAMENTALS OF BUSINESS STATISTICS
STATISTICAL REPRESENTATION OF DATA 4
This Module includes:

4.1 Diagrammatic Representation of Data


4.2 Frequency Distribution
4.3 Graphical representation of Frequency Distribution – Histogram, Frequency Polygon
Curve, Ogive, Pie-chart

The Institute of Cost Accountants of India 235


Fundamentals of Business Mathematics and Statistics

STATISTICAL REPRESENTATION OF DATA

Module Learning Objectives:


After studying this Module, the students will be able to understand –
 Meaning and definitions of Statistics
 Statistical Methods
 Representation of Information through Data
 Different analysis of Data

236 The Institute of Cost Accountants of India


Introduction 4

T
he word ‘Statistics’ has been derived from the Latin word ‘Status’ which means a political state. It has
also its root either to the Italian word ‘Statista’ or the German word ‘Statistik’ each one of which means a
political state. For several decades, the word ‘statistics’ was associated solely with the display of facts and
figures pertaining to the economic, demographic and political situations prevailing in a country,usually,
collected and brought out by the local governments.
Statistics is a tool in the hands of mankind to translate complex facts into simple and understandable statements
of facts.

Meaning and definition of Statistics:


Meaning of statistics: The word Statistics is used in two different senses - Plural and singular. In its plural form, it
refers to the numerical data collected in a systematic manner with some definite aim or object in view such as the
number of persons suffering from malaria in different colonies of Delhi or number of unemployed girls in different
states of India and so on. In Singular form, the word statistics means the science of statistics that deals with the
principles, devices or statistical methods of collecting, analyzing and interpreting numerical data.
Thus, ‘statistics’ when used in singular form refers to that branch of knowledge which implies Applied Mathematics.
The science of statistics is an old science and it has developed through ages. This science has been defined in
different ways by different authors and even the same author has defined it in different ways on different occasions.
It is impossible to enumerate all the definitions given to statistics both as “Numerical Data i.e., Plural Form: and
“Statistical Methods, i.e., Singular Form”. However, we have given below some selected definitions of both the
forms.
Definitions of “Statistics in Plural Form or Numerical Data”: Different authors have given different definitions
of statistics. Some of the definitions of statistics describing it quantitatively or in plural form are:
“Statistics are the classified facts representing the conditions of the people in a state especially those facts which
can be stated in number or in a table of numbers or in any tabular or classified arrangement.
This definition is narrow as it is confined only to the collection of data regarding the people in a state. But the
following definition given by Secrist is modern and convincing. It also brings out the major characteristics of
statistical data.
“By Statistics we mean the aggregate of facts affected to a marked extent by multiplicity of causes, numerically
expressed, enumerated or estimated according to reasonable standards of accuracy collected in a systematic manner
for a pre-determined purpose and placed in relation to each other”

The Institute of Cost Accountants of India 237


Fundamentals of Business Mathematics and Statistics

This definition makes it clear that statistics (in plural form or numerical data) should possess the following
characteristics.
I. Statistics means aggregate of facts
II. Statistics is affected by a large number of causes
III. Statistics is always numerically expressed
IV. Statistics should be enumerated or estimated according to reasonable standards of acuracy
V. Statistics should be collected in a systematic manner
VI. Statistics should be collected for a pre-determined purpose
VII. Statistics should be placed in relation to each other.

Statistics as Statistical methods or Statistics in Singular Sense:


We give below the definitions of statistics used in singular sense, i.e., statistics as statistical methods.
Statistical methods provide a set of tools which can be profitably used by different sciences in the manner they
deem fit. The term statistics in this context has been defined differently by different authors.
A few definitions are given below:
“Statistics may be called the science of counting”
This definition covers only one aspect, i.e., counting, but the other aspects such as classification, tabulation, etc.,
have been ignored. As such, the definition is inadequate and incomplete
“Statistics may be defined as the collection, presentation, analysis and interpretation of numerical data”
This definition given by Croxton and Cowden is simple, clear and concise.
According to this definition, there are four stages – collection of data, presentation of data, analysis of data, and
interpretation of data. However, one more stage may be added and that is the organization of data. Thus, there are
four stages:
1. Collection and Organization of data: There are various methods for collecting the data such as census,
sampling, primary and secondary data etc.
2. Presentation of data: The mass data collected should be presented in a suitable, concise form as the mass data
collected is difficult to understand and analyse
3. Analysis of data: The mass data collected should be presented in a suitable, concise form for further analysis.
Analysis includes condensation, summarisation conclusion, etc., by means of measures of central tendencies,
dispersion, skewness, kurtosis, correlation, regression, etc.
4. Interpretation of data: The last step is drawing conclusions from the data collected as the figures do not speak
for themselves.
Having briefly discussed some of the definitions of the term statistics and having seen their drawbacks we are
now in a position to give a simple and complete definition of the ‘Statistics’ in the following words:

238 The Institute of Cost Accountants of India


Statistical Representation of Data

Statistics (as used in the sense of data) are numerical statements of facts capable of analysis and interpretation and
the science of statistics is a study of the principles and methods used in the collection, presentation, analysis and
interpretation of numerical data in any sphere of enquiry.

Importance and Scope of Statistics:


I. Statistics and Economics: According to Prof. Alfred Marshall, “Statistics are the straws out of which I like
every other economist, have to make bricks.” The following are some of the fields of economics where statistics
is extensively used.
(a) Consumption: Statistical data of consumption enable us to find out the ways in which people in different
strata of society spend their incomes.
(b) Production: The statistics of production describe the total productivity in the country. This enables us to
compare ourselves with other countries of the world.
(c) Exchange: In the field of exchange, an economist studies markets, laws of prices which are determined by
the forces of demand and supply, cost of production, monopoly, competition, banking etc. A systematic
study of all these can be made only with the help of statistics
(d) Econometrics: With the help of econometrics, economics has become exact science. Econometrics is the
combination of economics, mathematics and statistics.
(e) Public Finance: Public finance studies the revenue and expenditure activities of a country. Budget, (a
statistical document), fiscal policy, deficit financing, etc., are the concepts of economics which are based
on statistics.
(f) Input-Output Analysis: The input-output analysis is based on statistical data which explain the
relationship between the input and the output. Sampling, Time series, Index numbers, Probability,
Correlation and Regression are some other concepts which are used in economic analysis.

II. Statistics and Commerce:


Statistical methods are widely applied in the solution of most of the business and trade activities such as production,
financial analysis, costing, manpower, planning, business, market research, distribution and forecasting etc. A
shrewd businessman always makes a proper and scientific analysis of the past records in order to predict the future
course of the business conditions. Index numbers help in predicting the future course of business and economic
events. Statistics or statistical methods help the business establishments in analysing the business activities such as:
(a) Organization of Business: Any businessman makes extensive use of statistical data to arrive at the conclusion
which guides him in establishing a new firm or business house
(b) Production: The production department of an organisation prepares the forecast regarding the production of
the commodity with the help of statistical tools.
(c) Scientific Management and Business Forecasting: Better and efficient control of a business can be achieved
by scientific management with the help of statistical data. “The success of businessman lies on the accuracy
of forecast made”. The successful businessman is one whose estimates most closely approach accuracy,” said
Prof. Boddington.
(d) Purchase: The price statistics of different markets help the businessman in arriving at the correct decision.
Raw material is purchased from those markets only where the prices are low.
III. Statistics and ‘Auditing and Accounting’: Statistics is widely used in accounting and auditing.
IV. Statistics and Economic Planning: According to Prof. Dickinson, “Economic Planning is making of major
decisions – what and how much is to be produced, and to whom it is to be allocated – by the conscious

The Institute of Cost Accountants of India 239


Fundamentals of Business Mathematics and Statistics

decisions of a determinate authority on the basis of a comprehensive survey of economy as a whole. “The
various documents accompanying preceding and following each of the eight Five Year Plans of India are a
standing testimony to the fact that statistics is an indispensable tool in economic planning.
V. Statistics and Astronomy: Statistics were first collected by astronomers for the study of the movement of
stars and planets. As there are a few things which are common between physical sciences, and statistical
methods, astronomers apply statistical methods to go deep in their study. Astronomers generally take a large
number of measurements and in most cases there is some difference between several observations. In order
to have the best possible measurement they have to make use of the technique of the law of errors in the form
of method of least squares.
VI. Statistics and Meteorology: Statistics is related to meteorology. To compare the present with the past or
to forecast for the future either temperature or humidity of air or barometrical pressures etc., it becomes
necessary to average these figures and thus to study their trends and fluctuations. All this cannot be done
without the use of statistical methods. Thus, the science of statics helps meteorology in a large number of
ways.
VII. Statistics and Biology: The development of biological theories has been found to be closely associated with
statistical methods. Professor Karl Pearson in his Grammar of Sciences has written, “The whole doctrine of
heredity rests on statistical basis”.
VIII. Statistics and Mathematics: Mathematics and Statistics have been closely in touch with each other ever
since the 17th Century when the theory of probability was found to have influence on various statistical
methods. Bowley was right when he said, “Acknowledge of Statistics is like knowledge of foreign language
or of algebra: it may prove of use at any time under any circumstances”.
Thus we observe that:
“Science without statistics bear no fruit, statistics without sciences have no Root”.
IX. Statistics and Research: Statistical techniques are indispensable in research work. Most of advancement in
knowledge has taken place because of experiments conducted with the help of statistical methods.
X. Statistics and natural sciences: Statistics finds an extensive application in physical sciences, especially in
engineering physics, chemistry, geology, mathematics, astronomy, medicine, botany, meteorology, zoology,
etc.
XI. Statistics and Education: There is an extensive application of statistics in Education. Statistics is necessary
for formulation of policies to start new courses, infrastructure required for new courses consideration of
facilities available for new courses etc.
XII. Statistics and Business: Statistics is an indispensable tool in all aspects of business. When a man enters
business he enters the profession of forecasting because success in business is always the result of precision
in forecasting and failure in business is very often due to wrong expectations which arise in turn due to faulty
reasoning and inaccurate analysis of various causes affecting a particular phenomenon. Boddington observes,
“The successful businessman is the one, whose estimate most closely approaches the accuracy”.

LIMITATIONS OF STATISTICS
Statistics and its techniques are widely used in every branch of knowledge. W.I. King rightly says:
“Science of statistics is the most useful servant, but only of great value to those who understand its proper use”.
The scope of statistics is very wide and it has great utility; but these are restricted by its limitations. Following are
the important limitations of statistics:

240 The Institute of Cost Accountants of India


Statistical Representation of Data

1. Statistics does not deal with individual item: King says, “Statistics from the very nature of the subject cannot
and never will be able to take into account individual cases”. Statistics proves inadequate, where one wants to
study individual cases. Thus, it fails to reveal the true position.
2. Statistics deals with quantitative data: According to Prof. Horace Secrist, “Some phenomenon cannot be
quantitatively measured; honesty, resourcefulness, integrity, goodwill, all important in industry as well as in
life, are generally not susceptible to direct statistical measurement”.
3. Statistical laws are true only on averages. According to W.I. King, “Statistics largely deals with averages
and these may be made up of individual items radically different from each other”.
Statistics are the means and not a solution to a problem.
4. Statistics does not reveal the entire story: According to Marshall, “Statistics are the straws, out of which, I,
like every other economist, have to make bricks”. Croxton says: “It must not be assumed that statistical method
is the only method or use in research; neither should this method be considered the best way to attack for every
problem”.
5. Statistics is liable to be misused: According to Bowley, “Statistics only furnishes a tool though imperfect,
which is dangerous in the hands of those who do not know its use and deficiencies”.
W.I. King states, “Statistics are like clay of which you can make a God or Devil as you please”. He remarks,
“Science of Statistics is the useful servant, but only of great value to those who understand its proper use”.
6. Statically data should be uniform and homogeneous

STATISTICAL TOOLS USED IN ECONOMIC ANALYSIS


The following are some of the important statistical techniques which are applied in economic analysis:
(a) Collection of data
(b) Tabulation
(c) Measures of Central Tendency
(d) Measures of Dispersion
(e) Time Series
(f) Probability
(g) Index Numbers
(h) Sampling and its uses
(i) Business Forecasting
(j) Tests of Significance and analysis of variance
(k) Statistical Quality Control

Collection of Data:
Data that is the information collected through censuses and surveys or in a routine manner or other sources is called
a raw data. The word data means information (its literary meaning is given as facts). The adjective raw attached to
data indicates that the information thus collected and recorded cannot be put to any use immediately and directly.

The Institute of Cost Accountants of India 241


Fundamentals of Business Mathematics and Statistics

It has to be converted into more suitable form or processed before it begins to make sense to be utilized gainfully.
A raw data is a statistical data in original form before any statistical techniques are used to redefine, process or
summarize it.
There are two types of statistical data:
(i) Primary data
(ii) Secondary data
1. Primary Data: It is the data collected by a particular person or organization for his own use from the primary
source
2. Secondary data: It is the data collected by some other person or organization for their own use but the
investigator also gets it for his use.

Methods of Collecting Primary Data:


The primary data can be collected by the following methods:
1. Direct personal observation: In this method, the investigator collects the data personally and, therefore, it
gives reliable and correct information.
2. Indirect oral investigation: In this method, a third person is contacted who is expected to know the necessary
details about the persons for whom the enquiry is meant.
3. Estimates from the local sources and correspondence. Here the investigator appoints agents and
correspondents to collect the data
4. Data through questionnaires. The data can be collected by preparing a questionnaire and getting it filed by
the persons concerned.
5. Investigations through enumerators. This method is generally employed by the Government for population
census, etc.

Methods of Collecting Secondary data:


The secondary data can be collected from the following sources:
1. Information collected through newspapers and periodicals.
2. Information obtained from the publications of trade associations.
3. Information obtained from the research papers published by University departments or research bureau or
UGC.
4. Information obtained from the official publications of the central and the state governments dealing with crop
statistics, industrial statistics, trade and transport statistics etc.
5. Information obtained from the official publications of the foreign governments for international organizations.
Classification of Data: The process of arranging things in groups or classes according to their common
characteristics and affinities is called the classification of data.
“Classification is the process of arranging data into sequences and groups according to their common characteristics
or separating them into different but related parts – Secrist.
Thus classification is the process of arranging the available data into various homogenous classes and sub-classes
according to some common characteristics or attributes or objectives of investigation.

242 The Institute of Cost Accountants of India


Statistical Representation of Data

Requisites of a Good Classification:


The main characteristics of a good classification are:
1. It should be exhaustive
2. It should be unambiguous
3. It should be mutually exclusive
4. It should be stable
5. It should be flexible
6. It should have suitability
7. It should be homogeneous
8. It should be a revealing classification
9. It should be reliable
10. It should be adequate

Advantages of classification of data:


(i) It condenses the data and ignores unnecessary details
(ii) It facilitates comparison of data
(iii) It helps in studying the relationships between several characteristics
(iv) It facilitates further statistical treatments

Types of Classification of Data:


There are four types of classification of data:
(i) Quantitative Classification
(ii) Temporal Classification
(iii) Spatial Classification and
(iv) Qualitative Classification
(i) Quantitative Classification: When the basis of classification is according to differences in quantity, the
classification is called quantitative
A quantitative classification refers to classification that is based on figures: In other words, it is a
classification which is based on such characteristics which are quantifiable such as height, weight, marks
obtained by students of a class.
(ii) Temporal Classification: When the basis of classification is according to differences in time, the classification
is called temporal or chronological classification.
(iii) Spatial or Geographical Classification: When the basis of classification is according to geographic location
or place, the classification is called spatial or geographical classification.
(iv) Qualitative Classification: When the basis of classification is according to characteristics or attributes like
social status etc. is called qualitative classification.

The Institute of Cost Accountants of India 243


Fundamentals of Business Mathematics and Statistics

Classification according to attributes is a method in which the data are divided on the basis of qualities.(i.e., married
or single; honest or dishonest; beautiful or ugly; on the basis of religion, viz., Hindu, Muslim, Sikh, Christian etc.,
known as attributes), which cannot be quantified.
Qualitative Classification is of two types:
(i) Simple Classification or Two-Fold Classification
(ii) Manifold Classification
1. Simple Classification or Two-fold Classification: If the data are classified only into two categories according
to the presence or absence of only one attribute, the classification is known as simple or two-fold classification
or Dichotomous For example, the population of India may be divided into males and females; literate and
illiterate etc.

POPULATION

FEMALE MALE

Moreover, if the classification is done according to a single attribute it is also known as one way classification.
2. Manifold Classification: It is a classification where more than one attributes are involved.

MODE OF PRESENTATION OF DATA


In this section we shall consider the following three modes of presentation of data
(a) Textual Presentation
(b) Tabular Presentation or Tabulation
(c) Diagrammatic presentation

Textual Presentation:
In this method presentation of data is done with the help of a paragraph or a number of paragraphs. The official
report of an enquiry commission is usually made by textual presentation. Following are the examples of textual
presentation.
Example1: In 1995, out of total 2,000 students in a college, 1,400 were for graduation and the rest for post-
graduation (P.G.). Out of 1,400 Graduation students 100 were girls. However, in all there were 600 girls in the
college. In 2000, number of graduation students increased to 1,700 out of which 250 were girls, but the number of
P.G. students fall to 500 of which only 50 were boys. In 2005, out of 800 girls 650 were for graduation, whereas
the total number of graduation students was 2,200. The number of boys and girls in P.G. classes were equal.
Merits and Demerits of Textual Presentation: The merit of this mode of presentation lies in its simplicity and
even a layman can present and understand the data by this method. The observations with exact magnitude can be
presented with the help of textual presentation. This type of presentation can be taken as the first step towards the
other methods of presentation.

244 The Institute of Cost Accountants of India


Statistical Representation of Data

Textual presentation, however, is not preferred by a statistician simply because it is dull, monotonous and
comparison between different observations is not possible in this method. For manifold classification, this method
cannot be recommended.
Tabular presentation or tabulation of data: Tabulation is a scientific process used in setting out the collected
data in an understandable form
Tabulation may be defined as logical and systematic arrangement of statistical data in rows and columns. It is
designed to simplify the presentation of data for the purposes of analysis and statistical inferences.
Secrist has defined tabulation in the following words:
“Tables are a means of recording in permanent form the analysis that is made through classification and by placing
in juxtaposition things that are similar and should be compared”.
The above definition clearly points out that tabulation is a process which gives classification of data in a systematic
form and is meant for the purpose of making comparative studies.
Professor Bowley refers to tabulation as:
“The intermediate process between the accumulation of data in whatsoever form they are obtained, and the final
reasoned account of the result shown by the statistics”
“Tabulation is the process of condensing classified data in the form of a table so that it may be more easily
understood and so that any comparison involved may be more readily made”.
Thus tabulation is one of the most important and ingenious devices of presenting the data in a condensed and
readily comprehensible form. It attempts to furnish the maximum information in the minimum possible space,
without sacrificing the quality and usefulness of the data.

Objectives of Tabulation:
The purpose of tabulation is to summarise lots of information in such a simple manner that it can be easily analysed
and interpreted.
The main objectives of the Tabulation are:
1. To simplify the complex data.
2. To clarify the objective of investigation
3. Economise space.
4. To facilitate comparison
5. To depict trend and pattern of data
6. To act as reference for future studies.
7. To facilitate statistical analysis.
8. To detect errors and omissions in the data
9. To clarify the characteristics of data.

Essential Parts of a Statistical Table:


A good statistical table should invariably have the following parts:
1. Table Number: A table should be numbered for identification, especially, when there are a large number of
tables in a study. The number may be put at the centre, above the title or at the bottom of the table.

The Institute of Cost Accountants of India 245


Fundamentals of Business Mathematics and Statistics

2. Title of the table: Every table should have a title. It should be clear, brief and self-explanatory. The title should
be set in bold type so as to give prominence.
3. Date: The date of preparation of a table should always be written on the table. It enables to recollect the
chronological order of the table prepared.
4. Stubs or Row Designations: Each row of the table must have a heading. The designations of the rows are
called stubs or stub items. Stubs clarify the figures in the rows. As far as possible, the same items should be
considered so that they can be included in a single row.
5. Captions or Column headings: A table has many columns. Sub-headings of the columns are called captions
or headings. They should be well-defined and brief.
6. Body of the table: It is the most vital part of the table. It contains the numerical information. It should be made
as comprehensive as possible. The actual data should be arranged in such a manner that any figure may be
readily located. Different categories of numerical variables should be set out in an ascending order, from left
to right in rows and in the same fashion in the columns, from top downwards.
7. Unit of Measurements: The unit of measurements should always be stated along with the title, if this is
uniform throughout. If different units have been adopted, then they should be stated along the stubs or captions.
8. Source Notes. A note at the bottom of the table should always be given to indicate the primary source as well
as the secondary source from where the data has been taken, particularly, when there is more than one source.
9. Foot Notes and References: It is always placed at the bottom of the table. It is a statement which contains
explanation of some specific items, which cannot be understood by the reader from the title, or captions and
stubs.
Different Parts of Table

TITLE
BOX HERD

N
C A P T IO
(1) (2) (3) (4) (5) (6) (7)
STUB

DY
BO

SOURCE
FOOT NOTE

Difference between Textual and Tabular Presentation: The tabulation method is usually preferred to textual
presentation as:
(i) It facilitates comparison between rows and columns
(ii) Complicated data can be represented using tabulation
(iii) Without tabulation, statistical analysis of data is not possible.
(iv) It is a must for diagrammatic representation.

246 The Institute of Cost Accountants of India


Diagrammatic Representation of
Data 4.1
The representation of statistical data through charts, diagrams and picture is another attractive and alternative
method. Unlike the first two methods of representation of data, diagrammatic representation can be used for both
the educated section and uneducated section of the society. Furthermore, any hidden trend presented in the given
data can be noticed only in this mode of representation.
However, compared to tabulation, this is less accurate. So if there is a priority for accuracy, we have to recommend
tabulation.
In this chapter we shall consider the following three types of diagrams:
I. Line diagram;
II. Bar diagram;
III. Pie chart.

LINE CHART
We take a rectangular axes. Along the abscissa, we take the independent variable (x or time) and along the ordinate
the dependent variable (y or production related to time). After plotting the points, they are joined by a scale, which
represents a line chart. The idea will be clear from the following example.
Example : Represent the following data by line chart.
The monthly production of motor cars in India during 2011-12

Jan Feb Mar April May June July Aug Sept Oct Nov Dce
70 90 80 120 100 120 110 125 130 150 100 120

Graph showing production of motor cars.

160
150
140
130
PRODUCTIONS

120
110
100
90
80
70
60

J F M A M J JL A S O N D

MONTHS

The Institute of Cost Accountants of India 247


Fundamentals of Business Mathematics and Statistics

BAR DIAGRAM
The simplest type of graph is the bar diagram. It is especially useful in comparing qualitative data or quantitative
data of discrete type. A bar diagram is a graph on which the data are represented in the form of bars. It consists of a
number of bars or rectangles which are of uniform width with equal space between them on the x-axis. The height
(for vertical bars) or length (for horizontal bars) of the bar is proportional to the value it represents. It should be
seen that the bars are neither too short nor too long. The scale should be clearly indicated and baseline be clearly
shown.
Bars may be drawn either horizontally or vertically. A good rule to use in determining the direction is that if
the legend describing the bar is to be written under the bars then vertical bars should be used; when it is not a
requirement, horizontal ones must be used. In this way, the legends can be read without turning the graph. The
descriptive legend should not be written at the ends of the bars or within the bars, since such writing may distort
the comparison. Usually the diagram will be more attractive if the bars are wider than the space between them.
The width of bars is not governed by any set rules. It is an arbitrary factor. Regarding the space between two bars,
it is conventional to have a space about one half of the width of a bar.
The data capable of representation through bar diagrams, may be in the form of row scores, or total scores, or
frequencies, or computed statistics and summarised figures like percentages and averages etc.
The bar diagram is generally used for comparison of quantitative data. It is also used in presenting data involving
time factor. When two or more sets of data over a certain period of time are to be compared a group bar diagram
is prepared by placing the related data side by side in the shape of bars. The bars may be vertical or horizontal in
a bar diagram. If the bars are placed horizontally, it is called a Horizontal Bar Diagram. When the bars are placed
vertically, it is called a Vertical Bar Diagram.

There are six types of Bar diagram:


(i) Simple Bar Diagram;
(ii) Multiple or Grouped Bar Diagram;
(iii) Subdivided or Component Bar Diagram;
(iv) Percentage Subdivided Bar Diagram;
(v) Deviation or Bilateral Bar Diagram;
(vi) Broken Bars.

Simple Bar Diagram:


It is used to compare two or more items related to a variable. In this case, the data are presented with the help of
bars. These bars are usually arranged according to relative magnitude of bars. The length of a bar is determined
by the value or the amount of the variable. A limitation of Simple Bar Diagram is that only one variable can be
represented on it.

Multiple or Grouped Bar Diagram:


A multiple or grouped bar diagram is used when a number of items are to be compared in respect of two, three or
more values. In this case, the numerical values of major categories are arranged in ascending or descending order
so that the categories can be readily distinguished. Different shades or colours are used for each category.

248 The Institute of Cost Accountants of India


Statistical Representation of Data

Sub-divided or Component Bar Diagram:


A component bar diagram is one which is formed by dividing a single bar into several component parts.
A single bar represents the aggregate value whereas the component parts represent the component values of the
aggregate value. It shows the relationship among the different parts and also between the different parts and the
main bar.
Percentage Sub-divided Bar Diagram:
It consists of one or more than one bars where each bar totals 100%. Its construction is similar to the subdivided
bar diagram with the only difference is that in the sub-divided bar diagram segments are used in absolute quantities
and in the percentage bar diagram the quantities are transformed into percentages.

PIE DIAGRAM OR ANGULAR DIAGRAM


A pie diagram is a circular graph which represents the total value with its components. The area of a circle represents
the total value and the different sectors of the circle represent the different parts. The circle is divided into sectors
by radii and the areas of the sectors are proportional to the angles at the centre. It is generally used for comparing
the relation between various components of a value and between components and the total value. In pie diagram,
the data are expressed as percentages.
Each component is expressed as percentage of the total value. A pie diagram is also known as angular diagram.
The name pie diagram is given to a circle diagram because in determining the circumference of a circle we have to
take into consideration a quantity known as ‘pie’ (written as π).

Method of Construction: The surface area of a circle is known to cover 2 radians or 360 degrees.
The data to be represented through a circle diagram may therefore be presented through 360 degrees or parts
or sections of a circle. The total frequencies or value is equated to 360° and then the angles corresponding to
component parts are calculated (or the component parts are expressed as percentages of the total and then multiplied
by 360/100 or 3.6). After determining these angles the required sectors in the circle are drawn. Different shades or
colours of designs or different types of hatchings are used to distinguish the various sectors of the circle.

Illustration 1.
120 students of a college were asked to opt for different work experiences. The details of these options are as under.

Areas of work experience No. of students


Photography 6
Clay modeling 30
Kitchen gardening 48
Doll making 12
Book binding 24

Represent the above data through a pie diagram.

The Institute of Cost Accountants of India 249


Fundamentals of Business Mathematics and Statistics

Solution:
The numerical data may be converted into the angle of the circles as given below:

Area of Work Experience No of Students Angle subtended at the centre of


the circle (degrees)
Photography 6 (6/120) × 360 = 18
Clay Modelling 30 (30/120) × 360 = 90
Kitchen Gardening 48 (48/120) × 360 = 144
Doll Making 12 (12/120) × 360 = 36
Book Binding 24 (24/120) × 360 = 72
Total 120 360

With the help of above computations the following Pie diagram is constructed.

Photography

Book
Binding
Clay
Modeling

Doll
Making

Kitchen
Gardening

250 The Institute of Cost Accountants of India


Frequency Distribution 4.2
TALLY BARS AND FREQUENCY
In order to make the data easily understandable, we tabulate the data in the form of tables or charts.
A table has three columns
(i) Variable
(ii) Tally marks
(iii) Frequency
(i) Variable: Any character which can vary from one individual to another is called a variable or a variate. For
example, age, income, height, intelligence, colour etc. are variates. Some variates are measurable and others are not
directly measurable. The examples of measurable variates are age, height, temperature, etc., whereas colour and
intelligence are the examples of those variates which cannot be measured numerically. Variables or observations
having numerics as possible values are called quantitative variables, whereas those with names of places, quality,
and things etc., as possible values are called qualitative variables or attributes.
Variables are of two types i) Continuous; ii) Discontinuous or Discrete. Quantities which can take all numerical
values within a certain interval are called continuous variables; But those variables which can take only a finite
number of values are called discrete variables; For example, number of students in a particular class, number of
sections in a school etc.
(ii) Tally: It is a method of keeping count in blocks of five.

For example: | = 1, || = 2, ||| =3, |||| = 4, |||| = 5

Tally Bars: These are the straight bars used in the Tally.
Each item falling in the class interval, a stroke (vertical Bar) is marked against it. This stroke (Vertical Bar)is called
the Tally Bar. Usually, after every four strokes (Tally Bar), in a class, the fifth item is marked by a horizontal or
slanted line across the Tally Bars already drawn. For example the frequency 5, 6, 7 is represented by ||||, |||| |, |||| ||
respectively.
The above method of presentation of data is known as ‘Frequency Distribution’. Marks represent the numerical
value of the number of students who secured a particular marks and is called Frequency of that particular value of
the variate.
In the first column of the table, we write all marks from lowest to highest. We now look at the first mark or value
in the given raw data and put a bar (vertical line) in the second column against it. We then, see the second mark or
value in the given raw data and put a bar against it in the second column.

The Institute of Cost Accountants of India 251


Fundamentals of Business Mathematics and Statistics

This process is repeated till all the observations in the given raw data are exhausted. The bars drawn in the second
column are known as tally marks and to facilitate we record tally marks in bunches of five, the fifth tally marks is
drawn across the first four.
For example ||||||| = 8. We finally count the number of tally marks corresponding to each observation and write in
the third column headed by frequency or number of students.
(iii) Frequency: The number of times an observation occurs in the given data is called the frequency of the
observation.
Frequency Distribution: A frequency distribution is the arrangement of the given data in the form of a table
showing frequency with which each variable occurs. In other words, Frequency distribution of a variable is the
ordered set {x, f}, where f is the frequency. It shows all scores in a set of data together with the frequency of each
score.

Types of frequency distributions:


Frequency distributions are of two types:
(i) Discrete Frequency Distribution
(ii) Grouped (or Continuous) Frequency Distribution

Discrete Frequency Distribution


The construction of discrete frequency distribution from the given raw data is done by the method of tally marks
as explained earlier.

Construction of Discrete Frequency Distribution Table


The frequency distribution table has three columns headed by
1. Variables (or classes)
2. Tally Mark or Bars
3. Frequency
The table is constructed by the following steps:
Step 1: Prepare three columns, viz., one for the variable (or classes), another for tally marks and the third for the
frequency corresponding to the variable (or class).
Step 2: Arrange the given data (or values) from the lowest to the highest in the first column under the heading
variable (or classes)
Step 3: Take the first observation in the raw data and put a bar (or vertical line|) in the second column under Tally
Marks against it. Then take a second observation and put a tally marks against it, continue this process till all the
observations of the given raw data are exhausted. For the sake of convenience, record the tally marks in bunches
of five, the fifth bar is placed diagonally crossing the other four (5 is represented by or ||||) leave some space
between each block of bars.
Step 4: Count the tally marks of column 2 and place this number opposite to the value of the variable in the third
column headed by Frequency.
Step 5: Give a suitable title to the frequency distribution table so that it exactly conveys the information contained
in the table.

252 The Institute of Cost Accountants of India


Statistical Representation of Data

SOME STATISTICAL TERMS


Raw Data or Data: A raw data is a statistical data in original form before any statistical technique is applied to
redefine process or summarize it.
Continuous Variable: A continuous variable is capable of assuming any value within a certain range or interval.
The height, weight, age and temperature of any person can be expressed not only in integral part but also in fractions
of any part. For example, the weight of a boy may be 44.0 kg or 44.6 kg or 44.65kg, similarly, his height may be
56 inches or 56.4 inches and age may be 10 years or 10.5 years. Thus, the height, weight, age or temperature etc.
are continuous variables.
Discrete Variable: A discrete variable can assume only integral values and can have exact measurement. In other
words, those variables which can take only a finite set of values are called discrete variables. For example: the
number of students in a particular class, or the number of sections in a school, etc. are the examples of discrete
variables. Discrete variables are also known as discontinuous variables.
Continuous Series: When the continuous variables are arranged in the form of a series, it is called continuous
series or exclusive series.
Discrete or Discontinuous Series: When the discrete variables are arranged in the form of a series, it is called a
discrete or discontinuous series.
Array: An array is an arrangement of data in order of magnitude either in descending or ascending order.
Descending Order: When data is arranged from the highest value to the lowest value, the array so formed is in
descending order.
Ascending order: When the data is arranged from the lowest value to the highest value, the array so formed is in
ascending order.

Illustration 2.
If the given data is 17, 7, 11, 5, 13, 9 then
Array in ascending order: 5, 7, 9, 11, 13, 17.
Array in descending order: 17,13,11,9,7, 5.
Range: It is the difference between the largest and the smallest number in the given data
The range of the data given in illustration above is 17 – 5 = 12.
Class, Class-Interval and Class limits. If the observations of a series are divided into groups and the groups are
bounded by limits, then each group is called a class. The end values of a class are called class limits. The smaller
value of the two limits is called the lower limit and the higher value of the same is called the upper limit of the class.
These two class limits are sometimes called the stated class limits.
Class Width: The difference between the lower limit (L) and the upper limit (U) of the class is known as class
Width (I).
Thus: I = U – L.
In other words, the range of a class is called its Class Width.

The Institute of Cost Accountants of India 253


Fundamentals of Business Mathematics and Statistics

Illustration 3.
The given data is

Marks obtained Tally Marks No of Students


1-10 |||| | 6
11- 20 ||| 3
21-30 |||| || 7
31-40 || 2
Total 18
In the above data the classes are: 1-10, 11-20, 21-30, and 31-40.
Class Interval: The range of the marks from 1 to 40 is grouped into four classes or groups viz: 1 – 10, 11 - 20, 21-
30, and 31-40. Each group is known as class interval. The interval between one class and its adjacent class being
9. [As 10-1 = 9, 20 – 11 = 9, 30 – 21 = 9, etc.]
Class Limits: In the first class 1 – 10, its lower limit is 1 and upper limit is 10. Similarly, 31 is the lower limit and40
is the upper limit of the class interval 31-40.
Actual Class Limit or Class Boundaries: In the illustration, there is a gap of 1 mark between the limits of any
two adjacent classes. This gap may be filled up by extending the two limits of each class by half of the value of
the gap. Thus
Lower class boundary = lower class limit – ½ of the gap
Upper class boundary = Upper class limit + ½ of the gap
The class boundary of the class 11 to 20 are
Lower class boundary = 11– ½ of 1 = 11 – 0.5 = 10.5
Upper class boundary = 20+ ½ of 1 = 20+ 0.5 = 20.5
In other words, the class boundaries are the limits up to which the two limits, (actual) of each class maybe extended
to fill up the gap that exists between the classes. The class boundaries of each class, so obtained are called the
Actual class limits or True class limits.
True lower class limit = Lower class limit - ½ of the gap
True upper class limit = Upper class limit + ½ of the gap
Note: In the case of exclusive series True class limits are the same as class limits

Illustration 4.

Class Interval Class Boundaries


11 – 20 10.5 – 20.5
21 – 30 20.5 – 30.5
31 – 40 30.5 – 40.5
Class-mark or Mid-point or Mid-value: The central value of the class interval is called the mid-point or mid-
value or class mark. It is the arithmetic mean of the lower class and upper class limit of the same class.

254 The Institute of Cost Accountants of India


Statistical Representation of Data

Mid-value of Class = (Lower class limit + Upper class limit)/2


Or,
Class mark = (True Upper class limit + True Lower class limit)/2
The class mark of the class 11-20 is (11+ 20)/2= 15.5
Class Magnitude: It is the difference between the upper class boundary and the lower class boundary of the class.
In the illustration the class magnitude of the class, 20.5 – 30.5 is (30.5 – 20.5) = 10.
Inclusive and Exclusive Series: In the above illustration, all the marks we considered were integers.
Hence, it was possible for us to choose classes 11 to 20, 21 to 30 etc. there is a gap of 1 between the upper limit of
a class and the lower limit of its succeeding class, which has not created any difficulty. But there can be situations
where the raw data is not in integers. For example, in the information regarding maximum temperature of the city
or time required to solve a statistical problem is recorded in the data, it may contain fractions as well. In such cases,
the consecutive classes have to be necessarily continuous. We have the following:
Inclusive Series: When the class-intervals are so fixed that the upper limit of the class is included in that class, it
is known as inclusive method of classification, e.g., 0-5, 6-10, 11-15, 16-20.
In the inclusive series, the upper limit and lower limit are included in that class interval. For example, in illustration,
the marks 11 and 20 are included in the class 11-20. It is a discontinuous series or inclusive series. In order to
make it a continuous one, some adjustment with the class limits is necessary. The class limits are extended to class
boundaries by the adjusting adjustment factor, which is equal to half of the difference between the upper limit of
the one class and lower limit of the next class. The series so obtained is continuous and is known as exclusive
series.
Exclusive or Continuous Series: In this series the upper limit of the class is the lower limit of the other class, the
common point of the two classes is included in the higher class. For example, 10-15, 15-20, 20-25… represent a
continuous series or the exclusive series. In this series, 15 is included in the class 15-20 and 20 is included in 20-30.
Here the class intervals overlap and the upper limit of each class is treated as less than that limit and lower limit
of each class actually represents exact value. Thus when the class-intervals are so fixed that the upper limit of one
class is the lower limit of the next class, it is known as Exclusive method of classification.
Relative frequency, Percentage frequency and Frequency Density of a class interval
Relative Frequency: Frequency of each class can also be expressed as a fraction of total frequency. These are
known as relative frequencies. In other words, a relative frequency is the class frequency expressed as a ratio of
the total frequency, i.e.
Relative frequency = Class frequency/Total frequency
Percentage Frequency: Percentage frequency of a class interval may be defined as the ratio of the class frequency
to the total frequency expressed as a percentage.
Percentage frequency = (Class frequency/ Total frequency) × 100

Frequency Density: This in defined as the ratio of Frequency of a class to its Width.
Frequency Density = Class Frequency / Class Width

The Institute of Cost Accountants of India 255


Graphical Representation of
Frequency Distribution 4.3
The graphs of frequency distribution are designed to present the characteristic features of a frequency data. They
facilitate comparative study of two or more frequency distributions regarding their shape and pattern.
The most commonly used graphs are:
1. Histogram
2. Frequency Polygon
3. Frequency Curve
4. Cumulative Frequency Curve or Ogive.

Illustration 5.
Draw the pie diagram from the following information
Vehicle Sales by type at a Car Dealers shop are given as:

Type Numbers Sold


Wagon R 72
S-Cross 54
Amaze 27
Punch 18
City 9
Total 180
Answer:
The detail computations are as follows:
Type Nos. Sold Angle subtended at the centre by the
Area (degrees)
Wagon R 72 144
S-Cross 54 108
Amaze 27 54
Punch 18 36
City 9 18
Total 180 360

256 The Institute of Cost Accountants of India


Statistical Representation of Data

Angle subtended at the centre of the circle by differnent Areas (in Degrees) are

Wagon R = 360 × 72 = 144


180
360
S-Cross = × 54 = 108
180

Amaze = 360 × 27 = 54
180

Punch = 360 × 18 = 36
180

City = 360 × 9 = 18
180

City
Punch

Wagon-R
Amaze

S-Cross

Illustration 6.
Prepare a statistical table from the following information of Daily earnings (in `) by Doctors:
88 23 27 28 86 96 94 93 86 99
82 24 24 55 88 99 55 86 82 36
96 39 26 54 87 100 56 84 83 46
102 48 27 26 29 100 59 83 84 48
104 46 30 29 40 101 60 89 46 49
106 33 36 30 40 103 70 90 49 50
104 36 37 40 40 106 72 94 50 60
24 39 49 46 66 107 76 96 46 67
26 78 50 44 43 46 79 99 36 68
29 67 56 99 93 48 80 102 32 51

The Institute of Cost Accountants of India 257


Fundamentals of Business Mathematics and Statistics

Answer:
Detailed calculations are as follows:
Class Internal of Earnings (`) Tally Marks No of Doctors Class Boundaries
20 – 29 |||| |||| ||| 13 19.5 – 29.5
30 – 39 |||| |||| | 11 29.5 – 39.5
40 – 49 |||| |||| |||| ||| 18 39.5 – 49.5
50 – 59 |||| |||| 10 49.5 – 59.5
60 – 69 |||| | 6 59.5 – 60.5
70 – 79 |||| 5 60.5 – 79.5
80 – 89 |||| |||| |||| 14 79.5 – 89.5
90 – 99 |||| |||| || 12 89.5 – 99.5
100 – 109 |||| |||| | 11 99.5 – 109.5
Total – 100 –
Illustration 7.
The national income of a country for the year 1999-2000 to 2001-2002 at current prices was 8650, 9010 and 9530
crores of rupees respectively and per capita income for these years was 250, 256 and 267 rupees. The corresponding
figures of national income and per capita income at 1999-2000 prices for the above years were 8650, 8820 and
8850 crores of rupees and 250, 252 and 248 respectively. Present the above data in a table.
Answer:
National Income and Per capita income of a country for the year 1999-2000 to 2001-2002
Year National Income Per Capita Income
At Current At 1999-2000 At Current At 1999-2000 Prices
Prices (` crores) Prices (` crores) Prices
1999-2000 8650 8650 200 250
2000-2001 9010 8820 256 251
2001-2002 9530 8850 267 248

Illustration 8.
Draw a Histogram of the frequency distribution given below:
Class Interval Frequency Class Interval Frequency
58 – 61 2 70 – 73 56
61 – 64 10 73 – 76 16
64 – 67 48 76 – 79 4
67 – 70 64
Answer:
In this case widths of the classes are of equal magnitude throughout. Width of each class interval is drawn on the X
axis i.e. base of each rectangle is proportional to the magnitude of the width of class interval. With width of each
class interval as base a rectangle is drawn with height proportional to the corresponding frequency of the class.

258 The Institute of Cost Accountants of India


Statistical Representation of Data

Histogram is as follows:

Histogram
70
60
50
Frequency

40
30
20
10
0
58 61 64 67 70 73 76 79
Value (Class Boundary)

Illustration 9.
Draw a Histogram of the frequency distribution given below:
Class Interval Frequency Class Interval Frequency
10-14 4 40 – 49 14
15 – 19 12 50 – 74 25
20 – 29 20 75 – 99 10
30 – 39 18
Answer:
In the given data it is to be noted that the widths of the class intervals are unequal and class boundaries are also not
provided. There is a gap of 1 between the limits of any two adjacent classes. Since rectangle in a histogram should
have area proportional to the class frequencies, the height of each rectangle should be made proportional to the
frequency density. Secondly we can eliminate the gap between the limits of two adjacent classes by bringing in the
class boundaries instead of class limits. Thus in this case the class boundaries are 9.5 – 14.5, 14.5 – 19.5 and so on.
Calculaltion of Frequency Density
Class Internal Showing Class Internal Showing Class Frequency
Frequency
Class Limits Class Boundaries Width* Density
(1) (2) (3) (4) (5) = (4) ÷(3)
10 - 14 9.5 - 14.5 5 4 0.8
15 - 19 14.5 - 19.5 5 12 2.4
20 - 29 19.5 - 29.5 10 20 2
30 - 39 29.5 - 39.5 10 18 1.8
40 - 49 39.5 - 49.5 10 14 1.4
50 - 74 49.5 - 74.5 25 25 1
75 - 99 74.5 - 99.5 25 10 0.4
* Class Width = Upper Class Boundary – Lower Class Boudary

The Institute of Cost Accountants of India 259


Fundamentals of Business Mathematics and Statistics

Histogram
3

2.5
Frequency Density

1.5

0.5

0
9.5 14.5 19.5 29.5 39.5 49.5 74.5 99.5
Values (Class Boundary)

The histogram above was thus drawn by plotting the Values (Class boundary) on X axis and frequency density on
Y axis

Illustration 10.
Draw histogram, frequency polygon and ogives (both “less-than” and “more-than” types) for the following
frequency distribution:

Wages (`) 50-59 60-69 70-79 80-89 90-99 100-109 110-119


No.of employees 8 10 16 14 10 5 2

Answer:
[Note: Here the class intervals are defined by class limits and so we have to find the class boundaries for drawing
the histogram. All the classes have the same width and therefore when drawing the histogram, heights of the
rectangles may be represented by the class frequencies. Also for drawing the ogives, we have to calculate both
‘less-than’ and ‘more-than’ cumulative frequencies.]

Calculations for Drawing Histogram

Class Class boundaries Frequency


50 – 59 49.5 – 59.6 8
60 – 69 59.5 – 69.5 10
70 – 79 69.5 – 79.5 16
80 – 89 79.5 – 89.5 14
90 – 99 89.5 – 99.5 10
100 – 109 99.5 – 109.5 5
110 – 119 109.5 – 119.5 2

260 The Institute of Cost Accountants of India


Statistical Representation of Data

Histogram and Frequency Polygon for Wage Distribution

D
16
n
E ly go
Po
cy
u en
F req
12
m
C F to gra
His
Frequency

B
8

4
H

0 A I
39.5 49.5 59.5 69.5 79.5 89.5 99.5 109.5 119.5 129.5

. Wages (`)

Frequency Polygon is drawn by joining the mid points of the top sides of the frequency bars. Here the points B, C,
D, E, F, G and H are the mid points of the frequency bars. Two end points of the Polygon are drawn by considering
mid points of two class widths drawn on the two sides of the Histogram. Here A is the mid point of class width
considered on the left side of the Histogram and I is that of the width considered on the right side of the Histogram.
Thus for the given data the Frequency Polygon is ABCDEFGHI.

Calculations for Drawing Ogives

Class Cumulative Frequency


Boundary
‘less-than’ ‘more-than’
49.5 0 65 = N
59.5 8 57
69.5 18 47
79.5 34 31
89.5 48 17
99.5 58 7
109.5 63 2
119.5 65 = N 0

The Institute of Cost Accountants of India 261


Fundamentals of Business Mathematics and Statistics

Ogives for Wage Distribution

70

60 “M ”
O
R HAN
E- -T
TH S
A ES
N
” “L
50
Cumulative Frequency

40

30

20

10
M
ED
IA
N

0
49.5 59.5 69.5 79.5 89.5 99.5 109.5 119.5

Wages (`)

262 The Institute of Cost Accountants of India


Statistical Representation of Data

Exercise:
Theoretical Questions
 Multiple Choice Questions (MCQ)

1. The word ‘Statistics’ has been derived from the Latin word ‘Status’ which means

(a) A numerical state

(b) A mathematical state

(c) A neutral state

(d) A political state

2. Which one of the following has synonymous words?

(a) Status, Staistik, Statista

(b) Staistik, Statista, Stats

(c) Statistic, Statistia, Stats

(d) Statistic, Statistia, Status

3. Statistics is a tool in the hands of mankind

(a) To translate complex facts into simple and understandable statements of facts

(b) To translate statistical information into mathematical statements of facts

(c) To translate business information into simple and understandable statements of facts

(d) To translate complex facts into presentable forms through tables and diagrams

4. The word statistics in plural form means

(a) The science of statistics that collect data in a systematic manner with some definite aim or object
towards further analysis in graphical way

(b) The numerical data collected in a systematic manner with some definite aim or object in view such as
the number of persons unemployed in a country.

(c) The numerical data collected in a systematic manner for analysing the state of affairs with the help of
mathematical applications

(d) The science of statistics that deals with the principles, devices or statistical methods of collecting,
analyzing and interpreting numerical data

The Institute of Cost Accountants of India 263


Fundamentals of Business Mathematics and Statistics

5. The word statistics in singular form means

(a) The science of statistics that collect data in a systematic manner with some definite aim or object
towards further analysis in graphical way

(b) The numerical data collected in a systematic manner with some definite aim or object in view such as
the number of persons unemployed in a country.

(c) The numerical data collected in a systematic manner for analysing the state of affairs with the help of
mathematical applications

(d) The science of statistics that deals with the principles, devices or statistical methods of collecting,
analyzing and interpreting numerical data

6. Which one of the following is not a characteristic of Plural form of Statistics?

(a) Statistics should be collected for a pre-determined purpose

(b) Statistics should be enumerated or estimated

(c) Statistics are always graphically expressed

(d) Statistics are affected by a large number of causes

7. Statistics in singular sense has

(a) 5 stages

(b) 6 stages

(c) 2 stages

(d) 4 stages

8. Measurement of skewness is

(a) Analysis of data

(b) Presentation of data

(c) Organisation of data

(d) Interpretation of data

9. Find the odd man out from the following

(a) Regression

(b) Kurtosis

264 The Institute of Cost Accountants of India


Statistical Representation of Data

(c) Sampling
(d) Central Tendency

10. Raw data is

(a) Information which can be interpreted to take decision

(b) Information which can’t be put to use directly

(c) Information which is not amenable to conversion

(d) Information which are useless

11. There are four person named A, B, C, & D. A is a sales person whereas B, C, D are students. A collected
sales figures for his region and B, C, D used these data in order to study sales pattern. Which one of the
following is correct?

(a) B uses secondary data

(b) A & B both are using primary data

(c) A, B, C, D all are using secondary data

(d) B, C, D are using primary data

12. Which one of the following is a method of collecting primary data?

(a) Information collected through newspapers and periodicals

(b) Information obtained from the publications of trade associations

(c) Information collected by Government through Census

(d) Information gathered from research paper published in research journal

13. Classification of data is

(a) The process of arranging things in groups or classes according to their common frequencies

(b) The process of arranging things in groups or classes according to their common characteristics and
affinities

(c) The process of arranging things in groups or classes according to their common differences and tally
marks

(d) The process of arranging things in groups or classes according to their common deviations from
respective mean

The Institute of Cost Accountants of India 265


Fundamentals of Business Mathematics and Statistics

14. In Statistics classification

(a) Separates data into different unrelated parts

(b) Separates data into different dispersed groups

(c) Separates data into different but related parts

(d) Separates data into different modal groups

15. Which one of the following is a characteristic of a good classification?

(a) Classification should be heterogeneous

(b) Classified groups must have overlapping data

(c) Classification should be stable

(d) Classification should be inclusive

16. When the basis of classification is according to differences in time it is called

(a) Temporal Classification

(b) Quantitative Classification

(c) Spatial Classification

(d) Qualitative Classification

17. Dichotomous Classification is

(a) When data is classified according to presence or absence of two attributes

(b) When data is classified into two groups containing all the attributes

(c) When data is classified according to presence of two attributes

(d) When data is classified into two groups according to presence or absence of one attribute

18. Tabulation Condenses classified data so that

(a) Data may be more easily understood

(b) Data may be easily presented textually

(c) Data may be more easily synchronized

(d) More comprehensive secondary result could be obtained

266 The Institute of Cost Accountants of India


Statistical Representation of Data

19. Tabulation is preferred to textual presentation because

(a) Tabulation never compares between rows and columns

(b) Tabulation is a must for diagrammatic representation

(c) Tabulation does not require complex statistical analysis

(d) Tabulation always get constructed with simple data

20. With respect to accuracy

(a) Diagrammatic presentation is preferable to Tabular presentation

(b) Textual presentation is preferable to diagrammatic presentation

(c) Tabular presentation is preferable to Diagrammatic presentation

(d) Textual presentation is preferable to Tabular presentation

 State True or False

1. The class boundaries are the limits up to which the two limits, (actual) of each class may be extended to
fill up the gap that exists between the classes;

2. In an exclusive series the class limits are extended to class boundaries by the adjusting adjustment factor;

3. In a Continuous Series lower limit of each class actually represents exact value;

4. Ratio of the class frequency to the total frequency expressed as a percentage is called percentage frequency

5. In Histogram class intervals are taken as heights and corresponding frequencies as breadth;

6. Frequency density = Class frequency/ Width of class interval;

7. A discrete variable can assume only integral values and therefore is not capable of exact measurement;

8. Variables or observations those with names of places, attributes, and things etc., as possible values are
called descriptive variables;

9. A raw data is a statistical data in original form before any statistical technique is applied to redefine
process or summarize it;

10. Any character which can vary from one individual to another is called a variable or a variate

The Institute of Cost Accountants of India 267


Fundamentals of Business Mathematics and Statistics

Answer:

 Multiple Choice Questions (MCQ)

1. 2. 3. 4. 5.
(d) (a) (a) (b) (d)
6. 7. 8. 9. 10.
(c) (d) (a) (c) (b)
11. 12. 13. 14. 15.
(a) (c) (b) (c) (c)
16. 17. 18. 19. 20.
(a) (d) (a) (b) (c)

 State True or False

1 2 3 4 5 6 7 8 9 10
T F T T F T F F T T

268 The Institute of Cost Accountants of India


MEASURES OF CENTRAL TENDENCY AND
DISPERSION
5
This Module includes

5.1 Mean, Median, Mode, Mean Deviation


5.2 Range, Quartiles and Quartile Deviation
5.3 Standard Deviation
5.4 Co-efficient of Variation
5.5 Karl Pearson and Bowley’s Co-efficient of Skewness

The Institute of Cost Accountants of India 269


Fundamentals of Business Mathematics and Statistics

MEASURES OF CENTRAL TENDENCY


AND DISPERSION
Module Learning Objectives:
After studying this Module, the students will be able to understand the –
 Concept of Averaging
 Concept of Arithmatic Mean – different methods of computations with grouped & ungrouped date
 Concept of Geometric Mean – different methods of computations with grouped & ungrouped data
 Concept of Harmonic Mean – different methods of computations with grouped & ungrouped data
 Relation between Arithmatic mean, Geometric mean and Harmonic mean

270 The Institute of Cost Accountants of India


Mean, Median, Mode, Mean
Deviation 5.1
In the previous chapters, data collection and presentation of data were discussed. Even after the data have been
classified and tabulated one often finds too much details for many uses that may be made of the information
available. We, therefore, frequently need further analysis of the tabulated data. One of the powerful tools of
analysis is to calculate a single average value that represents the entire mass of data. The word average is very
commonly used in day-to-day conversation. For example, we often talk of average work, average income, average
age of employees, etc. An ‘Average’ thus is a single value which is considered as the most representative or typical
value for a given set of data. Such a value is neither the smallest nor the largest value, but is a number whose value
is somewhere in the middle of the group. For this reason an average is frequently referred to as a measure of central
tendency or central value. Measures of central tendency show the tendency of some central value around which
data tends to cluster.

Objectives of Averaging:
There are two main objectives of the study of averages:
(i) To get one single value that describes the characteristic of the entire data. Measures of central value, can
be looked into as condensing the mass of data to one single value which, enable us to get an idea of the entire
data. Thus one value can represent thousands, lakhs and even millions of values. For example, it is impossible
to remember the individual incomes of millions of earning people of India and even if one could do it there
is hardly any use. But if the average income is obtained, we get one single value that represents the entire
population. Such a figure would throw light on the standard of living of an average Indian.
(ii) To facilitate comparison. By reducing the mass of data to one single figure, compare various sets of data.
Comparison can be made either at a point of time or over a period of time. For example, the figure of average
sales for December may be compared with the sales figures of previous months or with the sales figures of
another competitive firm.

Characteristics of a Good Average:


Since an average is a single value representing a group of values, it is desirable that such a value satisfies the
following properties:
(i) It should be easy to understand. Since statistical methods are designed to simplify complexity, it is desirable
that an average be such that it can be readily understood; otherwise, its use is bound to be very limited.
(ii) It should be simple to compute. Not only an average should be easy to understand but also it should be
simple to compute so that it can be used widely. However, though ease of computation is desirable, it should
not be sought at the expense of other advantages, i.e., if in the interest of greater accuracy, use of a more
difficult average is desirable one should prefer that.

The Institute of Cost Accountants of India 271


Fundamentals of Business Mathematics and Statistics

(iii) It should be based on all the observations. The average should depend upon each and every observation so
that if any of the observation is dropped average itself is altered.
(iv) It should be rigidly defined. An average should be properly defined so that it has one and only one
interpretation. It should preferably be defined by an algebraic formula so that if different people compute the
average from the same figures they all get the same answer.
(v) It should be capable of further algebraic treatment. We should prefer to have an average that could
be used for further statistical computations. For example, if we are given separately the figures of average
income and number of employees of two or more factories we should be able to compute the combined
average.
(vi) It should have sampling stability. We should prefer to get a value which has what the statisticians call
‘sampling stability’. This means that if we pick 10 different groups of college students, and compute the
average of each group, we should expect to get approximately the same values. It does not mean, however,
that there can be no difference in the value of different samples. There may be some difference but those
averages in which this difference, technically called sampling fluctuation, is less are considered better than
those in which this difference is more.
(vii) It should not be unduly affected by the presence of extreme values. If one or two very small or very large
observations unduly affect the average, i.e., either increase its value or reduce its value, the average cannot
be really considered as typical representatitive of the entire set of data. In other words, extremes may distort
the average and reduce its usefulness.
The following are the important measures of central tendency which are generally used in business:
A. Arithmetic mean.
B. Geometric mean,
C. Harmonic mean
D. Median
E. Mode

Measures of Central Tendency

Mathematical Positional
Measure Measure

AM GM HM Median Mode

I. ARITHMETIC MEAN: A.M is denoted by X . It is a mathematical measurement. It is calculated by different


methods as follows:
(a) Individual Series/Discrete data: -

(i) Direct Method: X = ΣX


N

272 The Institute of Cost Accountants of India


Measures of Central Tendency and Dispersion

(ii) Short Cut Method (or) Indirect Method: X = A + Σdx


N
(iii) Step Deviation method: X = A + Σ dx ′
×i
N
Where ΣX is Sum of terms, A is the assumed mean,
dx
dx is the deviation of items from assumed mean A i.e. dx = X – A, i is the common factor, dx′ = , N = No. of
i
observations.

(b) With grouped data:


ΣfX
(i) Direct Method: X =
Σf
Σfdx
(ii) Short Cut Method (or) Indirect Method: X = A + Σf
Σf dx′
(iii) Step Deviation method: X = A + ×i
Σf
Where A is the assumed mean,
dx is the deviation of items from assumed mean A
Σf = N , i is the common factor in dx, dx′ = dx
i

(c) Combined mean:


When there are two or more than two i.e. multiple series with n1,n2 , n3 ... .. nn items in respective series with respective
means X 1, X 2, X 3    X n , then the mean of all the series taken together i.e. combined mean is
n1 X 1 + n2 X 2 + ..... + nn X n
X=
n1 + n2 + ..... + nn
(d) Weighted mean:
For calculating simple arithmetic average, we suppose that all the values in the series have equal importance. But
in practice this may not be. Different values could have different importance and we assign different weights to
different values according to its importance. In such cases we measure weighted mean as per formula:
w1 X 1 + w2 X 2 +  + wn X n
X=
w1 + w2 +  + wn
Where w1, w2, w3, ... are the weights assigned to respective items of a series. Normally frequency values
corresponding to the different values of X are taken as the weights.
Illustration 1. Find Mean for the following figures:
30, 41, 47, 54, 23, 34, 37, 51, 53, 47
Answer:
Sl. No. 1 2 3 4 5 6 7 8 9 10 Total
x 30 41 47 54 23 34 37 51 53 47 417

ΣX 417
So mean = X = = = 41.7
N 10

The Institute of Cost Accountants of India 273


Fundamentals of Business Mathematics and Statistics

Illustration 2. Calculate A.M. from the following data:

Marks Obtained No of Students


4 6
8 12
12 18
16 15
20 9

Answer:
Calculations for A.M

Marks No of Students
Obtained (x) (f) fx
4 6 24
8 12 96
12 18 216
16 15 240
20 9 180
Total 60 756

So A.M = X = Σ fX =
756
= 12.6
Σf 60

Illustration 3. Use direct method to find X

Income (`) No of persons


10-20 4
20-30 7
30-40 16
40-50 20
50-60 15
60-70 8

Answer:

ΣfX
X=
Σf

274 The Institute of Cost Accountants of India


Measures of Central Tendency and Dispersion

Detail computations are as follows:


Calculations for X

Income (`) X = mid term of income class interval No of persons (  f  ) f X
10-20 15 4 60
20-30 25 7 175
30-40 35 16 560
40-50 45 20 900
50-60 55 15 825
60-70 65 8 520
Total – 70 3040

ΣfX 3040
X= = = ` 43.43
Σf 70

Illustration 4. X of 20 terms was found to be 35. But afterwards it was detected that two terms 42 and 34 were
misread as 46 and 39 respectively. Find correct X .

Answer:
X of 20 terms = 35
Incorrect Total of 20 terms = 35 × 20 = 700
Correct Total = 700 + 42 + 34 – 46 – 39 = 691
Correct X = 691/20 = 34.55

Illustration 5. The mean of wages in factory A of 100 workers is `720 per week. The mean wages of 30 female
workers in the factory is `650 per week. Find out average wage of male workers in the factory.

Answer:
N f + N m = 100 = N
Total wage of all workers
X = 720 = Mean wage of total workers =

So total wage of all workers = 720 × N = 720 × 100 = 72000

Total wage of female workers


X f = 650 = Mean wage of total female workers =
Nf
So total wage of female workers = 650 × Nf = 650 × 30 = 19500
So total wage of male workers = 72000 – 19500 = 52500
No of male workers in the factory = 100 – 30 = 70

52599
So Mean wage of total male workers = X m = = ` 750 per week
70

The Institute of Cost Accountants of India 275


Fundamentals of Business Mathematics and Statistics

Illustration 6. The following table gives the life time in hours of 400 tubes of a certain make. Find the mean life
time of the tubes.

Life time (hours) Number of tubes Life time (hours) Number of tubes
Less than 300 0 Less than 800 265
Less than 400 20 Less than 900 324
Less than 500 60 Less than 1000 374
Less than 600 116 Less than 1100 392
Less than 700 194 Less than 1200 400

Answer:
Σfdx′
As per Step Deviation method: X = A + Σ f × i. Here we have taken A = 749.5

Calculations for Mean

Life time Number Class Class Mid d= d’ = fd´


(in Hours) of Tubes Interval frequency value m-749.5 d/100
(f) (m)
Less than 300 0
Less than 400 20 300-399 20 349.5 -400 -4 -80
Less than 500 60 400-499 40 449.5 -300 -3 -120
Less than 600 116 500-599 56 549.5 -200 -2 -112
Less than 700 194 600-699 78 649.5 -100 -1 -78
Less than 800 265 700-799 71 749.5 0 0 0
Less than 900 324 800-899 59 849.5 100 1 59
Less than 1000 374 900-999 50 949.5 200 2 100
Less than 1100 392 1000-1099 18 1049.5 300 3 54
Less than 1200 400 1100-1199 8 1149.5 400 4 32
Total — — 400 — — — -145
Mean Life time of Tubes = 749.5 + (–)145/400 × 100 = 713.25 hours

Illustration 7. The average weight of the following distribution is 58.5 kg

Weight in Kg 50 55 60 X+12.5 70 Total


Number of men 1 4 2 2 1 10

Find the value of X.

276 The Institute of Cost Accountants of India


Measures of Central Tendency and Dispersion

Answer:
Calculations for Mean
Weight w (kg) Frequency (f) f w
50 1 50
55 4 220
60 2 120
X + 12.5 2 2X + 25
70 1 70
Total 10 2X + 485

(2 X + 485)
We have Mean = ∑ fw / ∑ f =
10
2 X + 485 58.5 × 10 − 485
By question we can write = 58.5 from where X is calculated as = 50 kg.
10 2

Illustration 8. A appeared in three test of full marks 20, 50 and 30 marks respectively. He obtained 75% marks in
the first and 60% marks in the second test. What should be his percentage of marks in the third test if his aggregate
is 60%?
Answer:
Here the mean is obtained by the formula

Average =
∑ ( Full Marks in test i × %age of marks in test i )
3
i =1
20 + 50 + 30
20 × 0.75 + 50 × 0.6 + 30 × x 15 + 30 + 30 x
Or, 0.6 = = [x = Percentage of marks in the 3rd test]
100 100
Or, 60 = 45 + 30x
Or, 15 = 30x
Or, x = 50%
Illustration 9. A certain number of salesmen were appointed in different territories and the following data were
compiled from their sales reports:

Sales (‘000 `) No of Salesmen


4-8 11
8-12 13
12-16 16
16-20 14
20-24 ?
24-28 9
28-32 17
32-36 6
36-40 4
If the average sale is believed to be `19, 920 find the missing information.

The Institute of Cost Accountants of India 277


Fundamentals of Business Mathematics and Statistics

Answer:
Let the missing frequency be y.
In this problem mean is obtained by the formula
Σfd
X = A+
Σf
Here d = deviation of respective class mark derived from A = 22 and the detail computations are as follows:

Calculations for Mean

Class Interval of Sales No. of Salesmen (f) Mid Value (x) d = x – 22 fd


(‘000`) (000 `)
4-8 11 6 -16 -176
8-12 13 10 -12 -156
12-16 16 14 -8 -128
16-20 14 18 -4 -56
20-24 y 22 0 0
24-28 9 26 4 36
28-32 17 30 8 136
32-36 6 34 12 72
36-40 4 38 16 64
Total 90 + y = Σf – – -208

Now by question we can write


Σfd
X = A+
Σf
Or, 19920 = 22000 + (−)208000 , as sales are in ` thousands
Σf

−208000
Or, 19920 – 22000 = Σf
−208000
Or, – 2080 = Σf
Or, Σ f = 100. Or, 90 + y = 100 Or, y = 10.
So the missing frequency is 10

Illustration 10. The numbers 3.2, 5.8, 7.9 and 4.5 have frequencies x, x + 2, x – 3 and x + 6 respectively. If the
arithmetic mean is 4.876 find the value of x.

Answer:
Σfn
Mean is obtained by the formula: n =
Σf
So detail computations are

278 The Institute of Cost Accountants of India


Measures of Central Tendency and Dispersion

Calculations for A.M

Number (n) frequency ( f ) fn


3.2 x 3.2x
5.8 x+2 5.8x + 11.6
7.9 x–3 7.9x – 23.7
4.5 x+6 4.5x + 27
Total 4x + 5 21.4x + 14.9

So we can write
21.4 x + 14.9
Mean = 4.876 = 4x + 5
So x = 5

Illustration 11. The sum of deviations of a certain number of observations (ungrouped data) measured from 4 is
72 and the sum of the deviations of the observations from 7 is –3. Find the number of observations and their mean.
Answer:
In this problem mean is obtained by the formula
Σd
X = A+
n
So by question we can write that
72 3
4+ =7−
n n
Or, 4n + 72 = 7n – 3
Or, 3n = 75
Or, n = 25
72
So mean = 4 + 25 = 6.88

Illustration 12. The marks of a student in written and oral tests in subjects A, B and C are as follows: The written
test marks are out of 70 and the oral test marks are out of 30. Find the weighted mean of the marks in written test
taking the percentage of marks in oral test as weights

Subjects Marks in Written test Marks in Oral test


A 43 15
B 32 12
C 29 18

The Institute of Cost Accountants of India 279


Fundamentals of Business Mathematics and Statistics

Answer:
Calculations for Weighted Mean

Oral test Weight of Oral test Written test marks (y) xy


marks marks out of 30 (x)
15 0.5 43 21.5
12 0.4 32 12.8
18 0.6 29 17.4
Total 1.5 — 51.7
Mean = Σxy/Σx = 51.7/1.5 = 34.47

Illustration 13: Calculate simple and weighted arithmetic averages from the following data:

Designation Monthly salary (`) Strength of the cadre


Class I officer 1500 10
Class II officer 800 20
Subordinate Staff 500 70
Clerical Staff 250 100
Lower Staff 100 150

Answer:
Detail computations are:
Calculations for Simple and Weighted A.M

Designation Monthly Strength of the w×x


Salary (x) Cadre (w)
Class I Offiicer 1500 10 15000
Class II Officer 800 20 16000
Subordinate Staff 500 70 35000
Clerical Staff 250 100 25000
Lower Staff 100 150 15000
Total 3150 350 106000

Σ x 3150
So simple arithmetic average = = = ` 630
n 5
Σ wx 106000
So weighted arithmetic average = = = ` 302.86
Σw 350
Properties of Arithmetic Mean: The important properties of arithmetic mean are given below:
(i) The sum of the deviations of the terms from the Actual mean is always zero.
(ii) The sum of the squared deviations of the items from arithmetic mean is minimum i.e. less than the sum of the
squared deviations of the items from any other value.

280 The Institute of Cost Accountants of India


Measures of Central Tendency and Dispersion

(iii) If we have arithmetic mean and the number of items of two or more than two groups, we can calculate the
combined average of groups.
(iv) If the terms of a series are increased, decreased, multiplied or divided by some constant, the mean will also be
increased, decreased, multiplied or divided by the same constant.
(v) The standard error of the arithmetic mean is less than that of any other measure of central tendency.

Merits of A.M:
� Different interpretation by different persons not possible as AM is rigidly defined
� Easy to understand
� It takes all values in consideration
� It lends itself easily to further mathematical treatment

Demerits of A.M:
� It cannot be determined by inspection nor can it be located graphically
� It is affected very much by extreme values. Average of 1, 2 & 3 is 2 but average of 1, 2, 33 is extremely
effected by presence of 33 and the result is 12
� It cannot be calculated if the distribution has open ended classes

II. GEOMETRIC MEAN (g): The geometric mean is obtained by multiplying the values of the items together and
then taking it to its root corresponding to the number of items. It is denoted by ‘g’. i.e., g = n X 1 X 2 X 3  X n

(a) Individual Series: g = Antilog  ∑ i =1


 n log x 
i

 n 

Where g is G.M, xi is items, n is No. of terms

 ∑ n f log x 
i i
(b) Discrete Series: g = Antilog  i =1 n 
 ∑ fi 
i =1

 ∑ f log m 
(c) Grouped Series or Continuous Series: g = Antilog   , where m = Mid value of the class Interval
 ∑f 
Σ w log x 
(d) Weighted Geometric mean:Anti Log 
Antilog
 Σw 

 n log g1 + n2 log g 2 
(e) Combined Geometric Mean: Antilog  1 
 n1 + n2
� The Geometric mean is relative value and is dependent on all items
� The geometric mean is never larger than the arithmetic mean. It is rare that it may be equal to the arithmetic
mean.
� The Geometric mean of the products of corresponding items is two series is equal to product of their geometric
means.

The Institute of Cost Accountants of India 281


Fundamentals of Business Mathematics and Statistics

Illustration 14: Calculate the GM of the following series of monthly income of a batch of families: 180, 250, 490,
120, 1400, 7000, 1050, 150, 360, 100, 80, 200, 500, 240.
Answer:
 n log x 
∑ i
G.M = g = Antilog  i =1
n 
 

Detail computations are


Calculations for G.M.

Income (x) Log x


180 2.255273
250 2.39794
490 2.690196
120 2.079181
1400 3.146128
7000 3.845098
1050 3.021189
150 2.176091
360 2.556303
100 2,000000
80 1.90309
200 2.30103
500 2.69897
240 2.380211
Total 35.4507

( )
 ∑ n log xi  35.4507
Log  i So, g = Antilog
Antilog = 340.6
 n  14

Illustration 15: Calculate the average income per head from the data given below using GM:

Class of People Number of Families Annual Income/head (`’00)


Landlords 2 5000
Cultivators 100 400
Landless Labourers 50 200
Money lenders 4 3750
School teachers 6 3000
Shop-Keepers 8 750
Carpenters 6 600
Weavers 10 300

282 The Institute of Cost Accountants of India


Measures of Central Tendency and Dispersion

Answer:
 n 
 ∑ fi log xi 
For Discrete Series: G.M = g = Antilog  i =1 
 ∑ fi 
 
Detail computations are as follows:
Calculations for G.M.

Class of people No. of Families ( f ) Income (x) log x f log x


Landlords 2 5000 3.69897 7.39794
Cultivators 100 400 2.60206 260.206
Landless Labourers 50 200 2.30103 115.0515
Money Lenders 4 3750 3.574031 14.29613
School Teaches 6 3000 3.477121 20.86273
Shop Keepers 8 750 2.875061 23.00049
Carpenters 6 600 2.778151 16.66891
Weavers 10 300 2.477121 24.77121
Total 186 – – 482.2549

 482.2549 
g = Antilog  = 391.54 (`’00)
 186 

Illustration 16: The weighted geometric mean of four numbers 8, 25, 19 and 28 is 22.15. If the weights of the first
three numbers are 3, 5, 7 respectively, find the weight of the fourth number.
Answer:

Weighted Geometric mean: Antilog  Σ


 w log X 
= 22.15
 Σ w 

 Σ w log X 
So  Σ w  = Log ( 22.15) = 1.3454

Detail computations are as follows:


Calculations of Weighted G.M.
X w log X w log X
8 3 0.90309 2.71
25 5 1.39794 6.99
19 7 1.278754 8.95
28 w 1.447158 1.45w
Total 15 + w 18.65+1.45w

18.65 + 1.45w
So = 1.3454 So w = 15 (approx)
15 + w

The Institute of Cost Accountants of India 283


Fundamentals of Business Mathematics and Statistics

Merits of G.M:
� Different interpretation by different persons not possible as GM is rigidly defined
� It takes all value in consideration
� Unlike A.M, it is not affected much by the presence of extremely small or large values
� It lends itself easily to further mathematical treatment

Demerits of G.M:
� It is difficult to understand
� It is difficult to calculate particularly when the items are very large or when there is graphical data
� It becomes imaginary if any of the observations is negative and GM is zero if any of the observation is zero

III. HARMONIC MEAN:


Harmonic mean is the reciprocal of the arithmetic mean
n
(a) For ungrouped data: H .M = n
, where xi = Values of the items and n = No. of terms
1
∑ x
i =1 i

(b) For data with frequency: H .M =


Σf i
, where xi = Values of the items (for ungrouped data) or mid value
n
fi
∑ x
i =1 i
of the class (for grouped data)

(c) Weighted H .M = ∑ w , where w = Weights and x = Values of the items


w
∑x
Illustration 17. Find the H.M of 3, 6, 12, 24 whose weights are 6, 2, 4, 8 respectively.

Answer:
∑w 6+ 2+ 4+8 20 20 2
H .M = = = = =6
w 6 2 4 8 1 1 1 2 +1 3

∑ x 3 + 6 + 12 + 24 2 + 3 + 3 + 3

Illustration 18. The marks secured by some students of a class are given below. Calculate the harmonic mean.

Marks 20 21 22 23 24 25
No of students 4 2 7 1 3 1

284 The Institute of Cost Accountants of India


Measures of Central Tendency and Dispersion

Answer:

Calculations for H.M

Marks (X) No of Students (f) Reciprocal of X f × Reciprocal of X


20 4 0.05000 0.20000
21 2 0.04762 0.09524
22 7 0.04545 0.31815
23 1 0.04348 0.04348
24 3 0.04167 0.12501
25 1 0.04000 0.04000
Total 18 – 0.82188

∑f =
18
H.M = f 0.82188 = 21.90
∑x
Merits of H.M:
� Different interpretation by different persons not possible as HM is rigidly defined
� It takes all value in consideration
� It is most suitable average when it is desired to give greater weight to smaller observations and less weight to
the larger ones
� It lends itself easily to further mathematical treatment

Demerits of H.M:
� It is difficult to understand
� It is difficult to compute
� It is only a summary figure and may not be the actual item in the series

Relationship among the different Averages:


(a) In any distribution where the original items differ in size, then either the values of A.M > G.M > H.M (or)
H.M < G.M < A.M. In case all items are identical then A.M. = G.M = H.M

X +Y 2 XY
(b) Let X, Y be two numbers then: A.M = , G.M = X ⋅ Y , H .M =
2 X +Y

IV. MEDIAN AND OTHER POSITIONAL MEASURES: Median is denoted by Md. It is a positional
measurement.Median divides a series into two equal parts. i.e., the middle most item is called median. It is also
indicated as Md .
(a) Individual Series: First of all the terms are to be arranged in ascending (or) descending order.
(1) When number of terms is odd then

The Institute of Cost Accountants of India 285


Fundamentals of Business Mathematics and Statistics

n +1
Median (M d ) = th item
2
Where Md is Median, n is No. of terms in the given series.
(2) When number of terms is even, then
n n
() ( )
Median (M d ) = Average of 2 & 2 + 1 th terms.
 N + 1
(b) Discrete Series with frequency: Median (Md) is the  th term. Here N = Σf = Total frequency
 2 
(c) Grouped data (Less than type i.e. When cumulative frequency is calculated from above)
N
−C
M d = l1 + 2 ×h
f

Where l1 = Lower boundary of the median class


N=Σ f
C′′ = cumulative frequency just preceding the median class.
f = simple frequency of the median class
h = width of the median class
(d) Grouped data (Greater than type i.e. When cumulative frequency is calculated from below)
N
− C ′′
M d = l2 − 2 ×h
f

Where l2 = Upper boundary of the median class

N=Σ f
C′′ = cumulative frequency just succeeding the median class.

f = simple frequency of the median class

h = width of the median class

Illustration 19. Find Median from following data:


17, 19, 21, 13, 16, 18, 24, 22, 20

Answer:
Arranging the data in ascending order we get: 13, 16, 17, 18, 19, 20, 21, 22, 24
No of observations in the data n = 9

So Median M d = n + 1 = 9 + 1 = 5 th item i.e. 19


2 2

286 The Institute of Cost Accountants of India


Measures of Central Tendency and Dispersion

Illustration 20. Find Median from following data:


17, 19, 21, 13, 16, 18, 24, 22, 20, 26

Answer:
Arranging the data in ascending order we get: 13, 16, 17, 18, 19, 20, 21, 22, 24, 26
No of observation in the data n = 10
 n n  19 + 20
So Median Md = Average of   &  + 1 th items i.e 5th & 6th items = = 19.5
 2 2  2

Illustration 21. Compute Median for the following data:

X 10 20 30 40 50 60 70
f 4 7 21 34 25 12 3

Answer:
Calculations for Median

X f C.f.
10 4 4
20 7 11
30 21 32
Median 40 34 66 53.5

50 25 91
60 12 103
70 3 106 = n

n +1 106 + 1
Median = M d = th term = = 53.5th term = 40
2 2

[From the above table all the values between 33rd and 66th items are 40. Thus 53.5th term should be 40]

Illustration 22. Calculate Median from the following data.

Class 4-8 8-20 20-28 28-40 40-60 60-72


f 7 12 42 56 39 22

Answer:
Here N/2 = 178/2 = 89 i.e 89th value is the Median. Hence it is a value in the class 28 – 40.

The Institute of Cost Accountants of India 287


Fundamentals of Business Mathematics and Statistics

Calculations for Median

Class Frequency ( f ) C.f (less than type)


4-8 7 7
8-20 12 19
20-28 42 61
28-40 56 117 Median Class
40-60 39 156
60-72 22 178 = N
So
N
−C
M d = l1 + 2 ×h
f
Where l1 = Lower boundary of the median class = 28
N = Σ f = 178
C = cumulative frequency just preceding the median class = 61
f = simple frequency of the median class = 56
h = width of the median class = 40 – 28 = 12
N 178
−C − 61
M d = l1 + 2 × h = 28 + 2 × 12 = 34
f 56

OR,
N
− C ′′
M d = l2 − 2 ×h
f

Where l2 = Upper boundary of the median class = 40


N = Σ f = 178
C′′ = cumulative frequency just succeeding the median class = 61
f = simple frequency of the median class = 56
h = width of the median class = 12

Class Frequency C.f (More than type)


4-8 7 178 = N
8-20 12 171
20-28 42 159
28-40 56 117 Median Class
40-60 39 61
60-72 22 22

288 The Institute of Cost Accountants of India


Measures of Central Tendency and Dispersion

So
N 178
− C ′′ − 61
M d = l2 − 2 × h = 40 − 2 × 12 = 34
f 56

Graphical method for locating median:


� This can be determined by drawing Ogive i.e. cumulative frequency curves.
� Cumulative frequency curves are of two types—More than c.f curve & less than c.f. curve
� These curves are drawn against class boundaries.
� The intersection of these two curves indicate the Median value on the X axis

For example let us consider the following data

Marks 0-10 10-20 20-30 30-40 40-50 50-60 60-70 70-80 80-90 90-100
No of Students 5 4 8 12 16 25 10 8 5 2

To draw the two ogives we first find less than type c.f and more than type c.f against respective class boundary and
the detail computations are

Class Frequency Class Boundary Cf (More thantype) C f (Less than type)
0-10 5 10 95 5
10-20 4 20 90 9
20-30 8 30 86 17
30-40 12 40 78 29
40-50 16 50 66 45
50-60 25 60 50 70
60-70 10 70 25 80
70-80 8 80 15 88
80-90 5 90 7 93
90-100 2 100 2 95

The Institute of Cost Accountants of India 289


Fundamentals of Business Mathematics and Statistics

And the ogives will be as follows: (Class boundaries on x axis and c.f. or y axis)
Cumulative Frequency (Cf)

Ogive Less than type


Ogive More than type

Median
Value (Class boundary)

Median is obtained by drawing an ordinate at the point of intersection of the two Ogives, as shown. The ‘Value’
corresponding to this ordinate is the Median.
n +1
Median divides the distribution into two equal parts. So we take for finding out median of ungrouped data
2
Similarly for ungrouped data we can find out –

(A) Quartile: Divides the distribution into 4 equal parts:


n +1
1st quartile = Q1 = th term
4
n +1
2nd quartile = Q2 = 2 × th term
4
n +1
3rd quartile = Q3 = 3 × 4 th term

(B) Decile: Divides the distribution into 10 equal parts:


n +1
1st decile = D1 = 10 th term
n +1
2nd decile = D2 = 2 × th term
10
……….
n +1
5th decile = D5 = 5 × th term
10
……….
n +1
9th decile = D9 = 9 × th term
10

290 The Institute of Cost Accountants of India


Measures of Central Tendency and Dispersion

(C) Percentile: Divides the distribution into 100 equal parts:


n +1
1st Percentile = P1 = th term
100
n +1
2nd Percentile = P2 = 2 × th term
100
………..
n +1
50st Percentile = P50 = 50 × th term
100
……….
n +1
99th Percentile = P99 = 99 × th term
100

From above it is clear


n +1 n +1
2nd quartile = Q2 = 2 × th term = 5th decile = D5 = 5 × th term = 50th decile = P50
4 10
n +1 n +1
= 50 × th term = 2 th term = median
100
Similarly for grouped data we can write

N
−C
M d = l1 + 2 ×h
f
N
−C
1st quartile = Q = l + 4 ×h
1 1
f

N
2× −C
2nd quartile = Q = l + 4 ×h
2 1
f

N
3× −C
3rd quartile = Q = l + 4 ×h
3 1
f
Similarly Deciles and Percentiles can be calculated.

N=Σ f

C = C.f just preceding to respective quartile class,

l1 = real lower limit of respective quartile class,

f = simple frequency of respective quartile class,

h = width of quartile class

Similarly Deciles and Percentiles can be calculated.

The Institute of Cost Accountants of India 291


Fundamentals of Business Mathematics and Statistics

Merits of Median:
� It is not at all affected by the items on the extremes
� It can be computed for the distributions which have open ended classes
� The value of the median can be located graphically
� The median is centrally located. The absolute sum of the deviations of the individual values from the median
is always the minimum

Demerits of Median:
� In case of an even number of observations ,median cannot be determined exactly
� It does not lend itself to algebraic treatment in a satisfactory manner
� It is unsuitable if it is desired to give greater importance to large or small values

Illustration 23. From the following grouped data find Median, 2nd Quartile, 5th Decile and 50 th percentile

Weekly 30-32 32-34 34-36 36-38 38-40 40-42 42-44 44-46 46-48 48-50
Wages (`)
Frequency 3 8 24 31 50 61 38 21 12 2

Answer:
Detail less than type cummulation is as follows:
Wages Frequency c.f (less than type)
30-32 3 3
32-34 8 11
34-36 24 35
36-38 31 66
38-40 50 116
40-42 61 177 Median Class
42-44 38 215
44-46 21 236
46-48 12 248
48-50 2 250 = N

N 250
Since = = 125., the median is 125th value. Now class 38-40 contains only upto 116th value. Any value
2 2
having rank greater than 116 but less than or inclusive 177 falls in the class 40-42. So median class is marked
against 40-42
N
−C 125 − 116
Median = M d = l1 + 2 × h = 40 + × 2 = 40.295
f 61

N 250
2× −C 2× − 116
2nd quartile = Q = l + 4 × h = 40 + 4 × 2 = 40.295
2 1
f 61

292 The Institute of Cost Accountants of India


Measures of Central Tendency and Dispersion

N 250
5× −C 5× − 116
5th decile = D5 = l1 + 10 × h = 40 + 10 × 2 = 40.295
f 61
N 250
50 × −C 50 × − 116
50th percentile = P50 = l1 + 100 × h = 40 + 100 × 2 = 40.295
f 61
[From the above computations we can conclude, Median = 2nd Quartile = 5th Decile = 50th Percentile]

Illustration 24. Refer to illustration 22. Compute 6th Decile and 88th Percentile

Answer:
N 250
6× −C 6× − 116
6th decile = D6 = l1 + 10 × h = 40 + 10 × 2 = 41.115
f 61

250 250
[6× = 6× = 150th value is the 6th Decile. Thus the class 40 – 42 is the 6th Decile class]
10 10
N 250
88 × −C 88 × − 215
88th percentile = P88 = l1 + 100 × h = 44 + 100 × 2 = 44.476
f 21
N 250
[ 88 × = 88 × = 220th value is the 88th Percentile which belongs to the class 44 – 46]
100 100

Calculations for 6th Decile and 88th Percentile

Wages Frequency c.f


30-32 3 3
32-34 8 11
34-36 24 35
36-38 31 66
38-40 50 116
40-42 61 177 ← 6th Decile Class
42-44 38 215
44-46 21 236 ← 88th Percentile Class
46-48 12 248
48-50 2 250

V. MODE: It is denoted by ‘M0’. Mode may be defined as the value that occurs most frequently in a statistical
distribution.
(i) Individual Series and Discrete Series: The terms are arranged in any order, Ascending or Descending. If
each term of the series is occurring once, then there is no mode, otherwise the value or values which occurs
maximum times is/are known as Mode/Modes.

The Institute of Cost Accountants of India 293


Fundamentals of Business Mathematics and Statistics

(ii) Grouped data:


f 0 − f1
M O = l1 + ×h
2 f 0 − f1 − f 2

Where l1 = Lower boundary of the modal class
f0 = Simple frequency of the modal class
f1 = Simple frequency of the class just preceding the modal class
f2 = Simple frequency of the class just succeeding the modal class
h = width of the modal class

Illustration 25. Find Mode from the following data:


12, 14, 16, 18, 26, 16, 20, 16, 11, 12, 16, 16, 20, 24

Answer:
In the above data
12 appeared twice;
16 appeared 5 times;
20 appeared twice;
Other terms only once
So mode is 16 as it appeared maximum times

Illustration 26. Find Mode from the following data:

Class 0-10 10-20 20-30 30-40 40-50 50-60 60-70


f 4 13 21 44 33 22 7

Answer:
Table showing Frequency Distribution

Class Frequency (f)


0-10 4
10-20 13
20-30 21 = f1
Modal Class → 30-40 44 = f0
40-50 33 = f2
50-60 22
60-70 7

294 The Institute of Cost Accountants of India


Measures of Central Tendency and Dispersion

So
f 0 − f1 44 − 21
Mode = M O = l1 + 2 f − f − f × h = 30 + 2 × 44 − 21 − 33 × 10 = 36.765
0 1 2

Mean, Median and Mode – Their Relation:


Mean – Mode = 3 (Mean – Median)
This formula was expressed by Karl Pearson. It is called Bi-mode.

Illustration 27. Compute the arithmetic average if the changes in the prices of securities in a certain stock exchange
are as follows:

Class Frequency Class Frequency


10% & over 54 -2% to 0% 149
8% to 10% 30 -4% to -2% 100
6% to 8% 50 -6% to -4% 74
4% to 6% 71 -8% to -6% 31
2% to 4% 119 -10% and under 51
0% to 2% 346 Total frequency 1075

Answer:
Classes are open ended type. So for open ended class AM can be found out by the relation
Mean – Mode = 3 (Mean – Median)
N 1075
Here = = 537.5. So Median is the 537.5th value which belongs to the class 0 to 2.
2 2
Median is calculated from the formula
N
−C
M d = l1 + 2 ×h
f

537.5 − 405
Md = 0 + × 2 = 0.7659
346

As the class 0 - 2 has maximum frequency 346, it is the modal class.


Mode is calculated from the formula

f 0 − f1
Mo = l1 + f − f + f − f × h
( 0 2 ) ( 0 1)
346 − 149
Mo = 0 + × 2 = 0.9292
(346 − 119) + (346 − 149)

The Institute of Cost Accountants of India 295


Fundamentals of Business Mathematics and Statistics

So Mean – 0.9292 = 3 × (Mean – 0.7659) Or, 2 Mean = 1.3685. So Mean = 0.684 Details computation for less than
cumulative frequencies are as follows:
Calculations for Median and Mode

Class Interval for changes in Frequency Cumulative frequency


the price of the securities (less than)
below (-10) 51 51
8 to 6 31 82
6 to 4 74 156
4 to 2 100 256
2 to 0 149 = f1 405
N
0 to 2 346 = f0 751 ← = 537.5
2
2 to 4 119 = f2 870
4 to 6 71 941
6 to 8 50 991
8 to 10 30 1021
10 to 12 54 1075 = N
Total 1075

Uses of various Averages: The use or application of a particular average depends upon the purpose of the
investigation. Some of the cases of different averages are as follows:
(a) Arithmetic Mean: Arithmetic mean is considered to be an ideal average. It is frequently used in all the
aspects of life. It possesses many mathematical properties and due to this it is of immense utility in further
statistical analysis. In economic analysis arithmetic mean is used extensively to calculate average production,
average wage, average cost, per capita income, exports, imports, consumption, prices etc. When different
items of a series have different relative importance, then weighted arithmetic mean is used.
(b) Geometric Mean: Use of Geometric mean is important in a series having items of wide dispersion. It is
used in the construction of Index Number. The averages of proportions, percentages and compound rates are
computed by geometric mean. The growth of population is measured in it as population increases in geometric
progression.
(c) Harmonic Mean: Harmonic mean is applied in the problems where small items must get more relative
importance than the large ones. It is useful in cases where time, speed, values given in quantities, rate and
prices are involved. But in practice, it has little applicability.
(d) Median and Partition values: Median and partition values are positional measures of central tendency. These
are mainly used in the qualitative cases like honesty, intelligence, ability etc. In the distributions which are
positively skewed, median is a more suitable average. These are also suitable for the problems of distribution
of income, wealth, investment etc.
(e) Mode: Mode is also positional average. Its applicability to daily problems is increasing. Mode is used to
calculate the ‘modal size of a collar’, modal size of shoes,’ or ‘modal size of ready-made garments’ etc. It is
also used in the sciences of Biology, Meteorology, Business and Industry.

296 The Institute of Cost Accountants of India


Range, Quartiles and Quartile
Deviation 5.2
Dispersion:
Follow the diagrammatic representation of two frequency distribution A and B. Both has same central tendency
(mean) located as shown at A and B. But a close observation of the diagram indicates that individual values fall
away from the central value (A and B) in different way. The drawn curves have different variability.

9
A
8
7
B
6
No of Items

5
4
3
2
1
0
0 2 4 6 8 10
Value of the Items

The measure of this variability of individual items with respect to central tendency is called DISPERSION.
Why knowledge on Dispersion is Important?
� It gives additional information on the data and enables us to judge the reliability of our measure of central
tendency. If individual items are widely spread with respect to the measure of central tendency measured
(mean/median/mode) then the central tendency is considered as less representative of the distribution as a
whole.
� There are problems peculiar to wide spread data items. So dispersion facilitates us to locate those data before
tackling those peculiar problems.
� On many occasions we require comparison of various samples of distribution in order to eliminate widely
fluctuated data with an ultimate objective to mitigate risks. More fluctuation means more risks. We need help
of dispersion on those occasions.

The Institute of Cost Accountants of India 297


Fundamentals of Business Mathematics and Statistics

Illustration 28. A firm using two different methods to ship orders to its customers found the following distributions
of delivery time for the two methods, based on past records. From available evidence, which shipment method you
recommend?

B
Frequency

Delivery Time

Answer:
A is more preferable as B has wider fluctuation

Illustration 29. For which one of the following distributions is the mean more representative of the data as a
whole? Why?

X
Frequency

Value of the variable


Answer:
X, because the values tend to cluster more around the mean.

298 The Institute of Cost Accountants of India


Measures of Central Tendency and Dispersion

So DISPERSION is the degree of the scatter or variation of the variables about a central value.

MEASURES of DISPERSION

Algebraic Graphic
(Absolute and Relative) (Lorenz Curve)

Standard
Range & Quartile Deviation Mean Deviation
Deviation &
Coefficient of & Coefficient of & Coefficient of
Coefficient of
Range Quartile Deviation Mean Deviation
Variation

Dispersion can be measured by (1) Distance Measures (2) Average Deviation Measures

Distance measure:

Range:

Range is the difference between the highest and lowest observed values.

Range = Value of highest observation – Value of lowest observation

The Institute of Cost Accountants of India 299


Fundamentals of Business Mathematics and Statistics

Illustration 30. Marks obtained by 12 students in the recently concluded examination is as follows:

Student Marks
1 52
2 13
3 35
4 46
5 89
6 10
7 29
8 95
9 66
10 72
11 48
12 71

From the above table it can be noted that Highest observed value is 95 whereas Lowest observed value is 10.
So Range of marks = 95 – 10 = 85

Quasi range: it refers to the difference between the values leaving the extreme values. If we leave the two extreme
values it will be
Quasi Range = 2nd highest observation – 2nd lowest observation = 89 – 13 =76 (in above case)

� Range is easy to understand but its usefulness as a measure of dispersion is limited;

� It considers only highest and lowest values and ignores all others. So it is highly influenced by extreme values;

� Since it measures only two values, the range is likely to change drastically from one sample to the next in a
given population;

� Open ended distribution has no range (Open ended example: Data < X or Data > Y, where X and Y are any
values ) ;

Coefficient of Range:
highest value − lowest value 95 − 10
Coefficient of range = = = 0.810
highest value + lowest value 95 + 10

Interquartile Range and Quartile Deviation:

Interquartile Range = Q3 – Q1, where Q3 is the third Quartile & Q1 is the first quartile

300 The Institute of Cost Accountants of India


Measures of Central Tendency and Dispersion

Lowest Observation Highest Observation

1/4 of 1/4 of
data data

2nd Quartile Q2

1st Quartile Q1 3rd Quartile Q3

The above figure shows the concept of the interquartile range graphically.
Q3 − Q1
One half of the interquartile range is a measure called the Quartile deviation= 2 . It is also called Semi Inter
Quartile Range.
The quartile deviation, then measures the average range of one-fourth of the data.
For our sample example let us calculate interquartile range and quartile deviation:

We begin by dividing the items into 4 equal parts as we have done in the following table:

FIRST FOURTH SECOND FOURTH THIRD FOURTH LAST FOURTH


10 35 52 72
13 46 66 89
29 Q1 48 71 Q3 95
Interquartile range = Q3 − Q1 = 71 − 29 = 42

Q3 − Q1 42
Interquartile deviation =
Quartile deviation = = 21
2 2
Like the range the interquartile range and the quartile deviation are based on only two values from the data set.
Although they are more complicated to calculate than the range, they avoid extreme values by using only the
middle half of the data. So effects of extreme data are avoided here.

Coefficient of Quartile Deviation:


Q3 − Q1
Coefficient of quartie deviation =
Q3 + Q1
Merits:
• Easy to calculate and simple to understand
• It can be easily computed from distribution with open ended classes
• It is useful when it is desired to know variability in the central half of data.

The Institute of Cost Accountants of India 301


Fundamentals of Business Mathematics and Statistics

Demerits:
• It ignores completely 50% of the data
• It does not lend itself to further mathematical treatment
• It is in fact not a measure of dispersion as it really does not show scatter around the average

Illustration 31. For the following data, compute the a) Interquartile range; b) Quartile deviation;

97 72 87 57 39 81 70 84 93 79
84 81 65 97 75 72 84 46 94 77

Answer:
First arrange the data in ascending order and divide the data in four equal parts as given in the following table:

First Fourth Second Fouth Third Fourth Last Fourth


39 72 81 87
46 72 81 93
57 75 84 94
65 77 84 97
70 79 84 = Q3 97

(a) Interquartile range = Q3 − Q1 = 84 − 70 = 14

Q3 − Q1 14
(b) Quartile deviation =
Interquartile = =7
2 2
Average Deviation Measures:
Under these two important measures are Variance and Standard Deviation
For a clear understanding on these two measures let us first discuss Average Absolute Deviation.

Average Absolute Deviation or Mean Deviation:


To compute this, steps are—
• Find out the mean of the given samples;
• Determine the absolute value of the difference between each item in the data set and the mean;
• Add all these differences together and divide by the total number of items in the sample;

For ungrouped data:


Average Absolute Deviation = ∑ ( x − µ) , for a population.
N

Average Absolute Deviation = ∑ (x − x) , for a sample.


n

302 The Institute of Cost Accountants of India


Measures of Central Tendency and Dispersion

Where:
x = the item or observation;
μ = the population mean;
N = number of items in the population;
x = sample mean;
n = no of items in the sample;
These are the average absolute deviation from Mean. Actually we can measure the average absolute deviation from
Mean, Median or Mode. i.e.

Average Absolute Deviation =


∑ ( x − A)
n
Where A can be Mean = X , Median = M d or Mode = M o

Illustration 32. Compute the average absolute deviation from the following data table which represents payments
received by 12 officers in a company on account of local conveyance during a period.

Payment in ` 863 903 957 1041 1138 1204 1354 1624 1698 1745 1802 1883

Answer:

x= Σx
n
863 + 903 + 957 + 1041 + 1138 + 1204 + 1354 + 1624 + 1698 + 1745 + 1802 + 1883
= 12
16212
= = 1351
12
Calculations for Mean Absolute Deviation
Observation Deviation from Mean Absolute deviation from Mean
(x - mean) = (x – x ) = x – 1351 |(x-Mean)| = | x- x |
863 -488 488
903 -448 448
957 -394 394
1041 -310 310
1138 -213 213
1204 -147 147
1354 3 3
1624 273 273
1698 347 347
1745 394 394
1802 451 451
1883 532 532
∑ |(x – x ) = 4000

The Institute of Cost Accountants of India 303


Fundamentals of Business Mathematics and Statistics

Average Absolute Deviation about Mean = Σ ( x − x ) = 4000 ==333.3


` 333.3
n 12
This average absolute deviation is a better measure of dispersion than the range (we have already discussed)
because it takes every observation into account. It gives weightage to each observation equally and indicates how
far on average each observation lies from the mean.
For Grouped data:

Average Absolute Deviation =


Σ f ( x − A)
Σf
Where A can be Mean = X , Median = Md or Mode = M0 & f = frequency

Merits:
• Mean deviation is easy to understand.
• It is based on all the items of the series.
• It is less affected by extreme observations when compared with Standard deviation.

Demerits:
• It is the arithmetic mean of the absolute values of the deviation. It ignores the positive and negative signs of
deviations. This weakness creates the demand for a more reliable measure of dispersion.
• It is not amenable for further algebraic treatment.

304 The Institute of Cost Accountants of India


Standard Deviation 5.3
Population Variance:
If we shift our calculation from samples to population we will get population variance symbolized by σ 2 (sigma
squared).
The population variance is similar to an average absolute deviation computed for an entire population. It is the
Mean Square deviation about Mean.

Σ ( x − µ) Σx
2 2
σ2 = = − µ2
N N
Where:
σ2 = population variance;
x = the item or observation;
μ = population mean;
N = total number of items in the population;
σ2 has the unit which is square of the units of the data. These types of units are not clearly interpreted. To avoid this
problem instead of variance we have another measure called Standard Deviation.

Population Standard Deviation:


Population standard deviation is simply the square root of the population variance.
The standard deviation is the square root of the average of the squared distances of the observations from the mean.

= σ2 = Σ ( x − µ) = Σx − µ2
N N

Where the symbols have same meaning as described in case of Population Variance.

Illustration 33. The following table shows the results of purity test on some compound. Find Variance and S.d.

Observed percent of impurity


0.04 0.14 0.17 0.19 0.22
0.06 0.14 0.17 0.21 0.24
0.12 0.15 0.18 0.21 0.25

The Institute of Cost Accountants of India 305


Fundamentals of Business Mathematics and Statistics

Answer:
Detailed computations are as follows:
Calculations for Variance and S.d
Sr Observation Deviation from mean ( x – µ)2 x­2
No (x) ( x – µ) = x – 0.166
1 0.04 -0.126 0.016 0.0016
2 0.06 -0.106 0.011 0.0036
3 0.12 -0.046 0.002 0.0144
4 0.14 -0.026 0.001 0.0196
5 0.14 -0.026 0.001 0.0196
6 0.15 -0.016 0.000 0.0225
7 0.17 0.004 0.000 0.0289
8 0.17 0.004 0.000 0.0289
9 0.18 0.014 0.000 0.0324
10 0.19 0.024 0.001 0.0361
11 0.21 0.044 0.002 0.0441
12 0.21 0.044 0.002 0.0441
13 0.22 0.054 0.003 0.0484
14 0.24 0.074 0.005 0.0576
15 0.25 0.084 0.007 0.0625
Total 2.49 – 0.051 0.4643

∑ x 2.49
Mean (µ ) = = = 0.166
N 15
∑ ( x − µ ) 2 0.051
Variance (σ 2 ) = = = 0.0034 percent squared
N 15
OR
∑ x2 0.4643
Variance (σ 2 ) = − µ2 = − (0.166) 2 = 0.0034 percent squared
N 15

S.d ( σ) = σ2 = 0.0034 = 0.058%

Standard deviation enables us to determine, where the values of a frequency distribution are located in relation to
the mean. We can measure the percentage of items that fall within specific ranges under a systematic bell shaped
curve like the one below from where we can say that:
1. About 68 % of the values in the population will fall within plus and minus 1 standard deviation from the mean.
2. About 95 % of the values in the population will fall within plus and minus 2 standard deviation from the mean.
3. About 99 % of the values in the population will fall within plus and minus 3 standard deviation from the mean.

306 The Institute of Cost Accountants of India


Measures of Central Tendency and Dispersion

The standard deviation is also useful in describing how far individual items in a distribution disperse from the
mean of the distribution. A measure called Standard Score gives us the number of standard deviations a particular
observation lies below or above the mean.
x−µ
Standard score computed from population data as = σ
Suppose we observe a compound that is 0.108 percent impure. Since in the above Illustration 6 population has
a mean of 0.166 and a standard deviation of 0.058, an observation of 0.108 would have a standard score of -1 as
follows:
x − µ 0.108 − 0.166
Standard score = = = −1
σ 0.058
For Grouped data variance and standard deviation are formulated as follows:

∑ f ( x − µ)
2
∑ fx 2
σ2 = = − µ2
∑f ∑f

∑ f ( x − µ)
2
∑ fx 2
σ = σ2 = = − µ2
∑f ∑f

Where:
σ2 = population variance;
σ = population standard deviation;
x = Class mark of each class;
μ = population mean;
f = frequency of each class
Illustration 34. From the following frequency distribution. Calculate variance and S.d

Annual Salary (` ’000) No of people


700-799 4
800-899 7
900-999 8
1000-1099 10
1100-1199 12
1200-1299 17
1300-1399 13
1400-1499 10
1500-1599 9
1600-1699 7
1700-1799 2
1800-1899 1

The Institute of Cost Accountants of India 307


Fundamentals of Business Mathematics and Statistics

With such type of grouped data we can use the following formulae to calculate the variance and standard deviation:

Σf ( x − µ )
2
Σfx 2
σ2 = = − µ2
Σf Σf

Σf ( x − µ )
2
Σfx 2
σ = σ2 = = − µ2
Σf Σf
Where symbols have their usual meaning mentioned before

Answer:
Calculations for Mean, Variance and S.d
Class interval of Class Frequency ( f ) f x x – m = x – 1249.5 (x – m)2 f (x – m)2
Annual Salary Mark (x)
700-799 749.5 4 2998 -500 250000 1000000
800-899 849.5 7 5946.5 -400 160000 1120000
900-999 949.5 8 7596 -300 90000 720000
1000-1099 1049.5 10 10495 -200 40000 400000
1100-1199 1149.5 12 13794 -100 10000 120000
1200-1299 1249.5 17 21241.5 0 0 0
1300-1399 1349.5 13 17543.5 100 10000 130000
1400-1499 1449.5 10 14495 200 40000 400000
1500-1599 1549.5 9 13945.5 300 90000 810000
1600-1699 1649.5 7 11546.5 400 160000 1120000
1700-1799 1749.5 2 3499 500 250000 500000
1800-1899 1849.5 1 1849.5 600 360000 360000
Total – 100 124950 – – 6680000

Σ ( fx ) 124950
µ= = = 1249.5
Σf 100

Σf ( x − µ )
2
6680000
σ2 = = = 66800 Sq. ` (’000)
Σf 100

σ = σ 2 = 66800 = ` 258.5 (’000)

Standard deviation cannot be computed from open ended distributions. It considers every observation but extreme
values in the data set distort the value of the standard deviation, although to a lesser extent than do the range.

Illustration 35. Calculate the population variance for the following set of grouped data:

Class 0-199 200-399 400-599 600-799 800-999


Frequency 8 13 20 12 7

308 The Institute of Cost Accountants of India


Measures of Central Tendency and Dispersion

Answer:
Σf ( x − µ )
2
Σfx 2
We know that population variance is computed as σ 2 = = − µ2
N N

Calculations for Variance


Class Class Frequency fx x–m (x – m)2 f (x – m)2
Mark (x) (f) = x – 489.5
0-199 99.5 8 796 -390 152100 1216800
200-399 299.5 13 3893.5 -190 36100 469300
400-599 499.5 20 9990 10 100 2000
600-799 699.5 12 8394 210 44100 529200
800-999 899.5 7 6296.5 410 168100 1176700
Total – 60 29370 – – 3394000

∑ fx 29370
Mean = µ = = = 489.5
∑f 60
∑ f ( x − µ ) 2 3394000
Variance = σ 2 = = = 56566.67
∑f 60

Generally we calculate variance and standard deviation for ungrouped & grouped data through various formulae
as given below.
1. Ungrouped data: (a) Direct Method; (b) Shortcut Method; (c) Step deviation method
Direct method:

Σ(x − x)
2
(a) σ =
n

Σx −Σ 
2 2
 x
(b) σ = n  n 

Shortcut Method

Σdx −  Σ  , where dx = x – A, A is the assumed mean


2 2
 dx 
(a) σ =
n  n 
Step Deviation Method

Σdx −Σ  ×h
'2 2
 dx′ 
(b) σ =
n  n 
where dx′ = x − A , A is the assumed mean, h is the common factor
h

The Institute of Cost Accountants of India 309


Fundamentals of Business Mathematics and Statistics

2. Grouped data: (a) Direct Method; (b) Shortcut Method; (c) Step deviation method
Direct method:

∑ f (x − x)
2
(a) σ =
∑f

Σ fx −Σ 
2 2
 fx 
(b) σ =
∑f  ∑f 

Shortcut Method

Σ fdx −Σ
2 2
 fdx 
(a) σ = , where dx = x – A, A is the assumed mean
∑f  ∑ f 

Step Deviation Method

Σ fdx '2  Σ fdx′ 


2

(b) σ = − ×h
∑f  ∑ f 
x− A
Where dx′ = , A is the assumed mean, h is the common factor (Generally it is the Common Width of the
h
class intervals).

3. Standard Deviation of combined Series:


If two groups contain n1 and n2 observations with mean X 1 & X 2 and standard deviations σ1 & σ 2 respectively, the
S.D (σ12) of the combined group is given by

n1 (σ12 + d12 ) + n2 (σ 2 2 + d 2 2 )
σ12 =
n1 + n2
Where
d1 = X 12 − X 1

d 2 = X 12 − X 2
n1 ⋅ X 1 + n2 ⋅ X 2
X 12 = Combined mean =
n1 + n2
Illustration 36. Calculate the standard deviation of the following distribution:
Age No of people
20-25 170
25-30 110
30-35 80
35-40 45
40-45 40
45-50 35

310 The Institute of Cost Accountants of India


Measures of Central Tendency and Dispersion

Answer:
Let the assumed mean is A = 32.5
2
∑ fdx ∑ fdx 2
We have, Standard Deviation = σ = −( ) acording to Shortcut Method.
∑f ∑f
The following table shows how to apply the above formula to find the standard deviation from the given grouped
data.

Calculations for S.d


Class Class Frequency ( f ) d=x–A fd f d 2
Mark (x) = x – 32.5
20-25 22.5 170 -10 -1700 17000
25-30 27.5 110 -5 -550 2750
30-35 32.5 80 0 0 0
35-40 37.5 45 5 225 1125
40-45 42.5 40 10 400 4000
45-50 47.5 35 15 525 7875
Total – 480 – -1100 32750

32750 −1100 2
σ= −( ) = 68.23 − 5.25 = 7.936
480 480

Another method of calculating standard deviation is called Step deviation method by which

2 2
Σ fd ′  Σ fd ′ 
σ= − × h
N  N 
x− A
Where d ′ = h ,
h is a common factor, A is the assumed average and x is the mid value of the class interval

Illustration 37. Calculate the mean and standard deviation from the following data:
Value Frequency
90-99 2
80-89 12
70-79 22
60-69 20
50-59 14
40-49 4
30-39 1

The Institute of Cost Accountants of India 311


Fundamentals of Business Mathematics and Statistics

Answer:
Calculations for Mean and S.d

Class Interval Class Interval with Class Mid value x − 64.5 f


with Class Limits Boundaries (x) d′ = fd ′ f d′ 2
10

90-99 89.5 - 99.5 94.5 3 2 6 18


80-89 79.5 - 89.5 84.5 2 12 24 48
70-79 69.5 - 79.5 74.5 1 22 22 22
60-69 59.5 - 69.5 64.5 0 20 0 0
50-59 49.5 - 59.5 54.5 -1 14 -14 14
40-49 39.5 - 49.5 44.5 -2 4 -8 16
30-39 29.5 - 39.5 34.5 -3 1 -3 9
Total – – – 75 27 127

In the above computation


A = 64.5 , h = 10

2 2
∑ fd ′  ∑ fd ′  127 27 2
σ= − × h= − ( ) × 10 = 1.6933 − 0.1296 × 10 = 12.505
∑f  ∑f  75 75

∑ fd ′ 27
X = A+ × h = 64.5 + × 10 = 68.1
∑f 75

Illustration 38. Calculate standard deviation from the following set of observations

8 9 15 23 5 11 19 8 10 12

Answer:
These are ungrouped data. For these sort of data standard deviation is computed by the following formula –
Direct method:

(X − X )
2

σ= , where X is the arithmetic mean, n = number of items………….(A)


n

Or, σ = ΣX 2 ΣX 2 ………………………. (B)


−( )
n n

312 The Institute of Cost Accountants of India


Measures of Central Tendency and Dispersion

The detail computation as per formula (A) and (B) are given in the following tables

Calculations for S.d using Equation (A)

Sl. No. Values (X) Deviation Deviation Squared


x = X − X = X =12 x2
1 8 -4 16
2 9 -3 9
3 15 3 9
4 23 11 121
5 5 -7 49
6 11 -1 1
7 19 7 49
8 8 -4 16
9 10 -2 4
10 12 0 0
Total 120 – 274

X = Σ X/n = 12
274
So σ = 10
= 5.23.

Alternative Method-1

Calculations for S.d using Equation (B)

Sl. No. Values Square of


(X) Values (X2)
1 8 64
2 9 81
3 15 225
4 23 529
5 5 25
6 11 121
7 19 361
8 8 64
9 10 100
10 12 144
Total 120 1714

( )
2
ΣX 2 ΣX 2 1714 120
So σ = n
−(
n
) =
10

10
= 5.23

The Institute of Cost Accountants of India 313


Fundamentals of Business Mathematics and Statistics

Alternative Method-2
The s.d of above ungrouped data can also be found by short cut method by the following formula:

2
Σd Σd
σ= − ( ) 2 where d = X – A and A is assumed mean = 10, in this case.
n n

The detailed computation is given in the following table


Calculation for s.d
Values Deviation
(X) d = X – A = X – 10 d 2
8 -2 4
9 -1 1
15 5 25
23 13 169
5 -5 25
11 1 1
19 9 81
8 -2 4
10 0 0
12 2 4
Total 20 314

2
Σd Σd 314 20 2
So σ = − ( )2 = − ( ) = 5.23
n n 10 10

Illustration 39. The analysis of the results of a budget survey of 150 families gave an average monthly expenditure
of `120 on food items with a standard deviation of `15. After the analysis was completed it was noted that figure
recorded for one household was wrongly taken as `15 instead of `105. Determine the correct value of the average
expenditure and its standard deviation.
Answer: We are given n = 150, X = 120 and σ = 15
Wrong value used =15 and correct value =105
ΣX ΣX
Now X = or 120 = 150
n
So that Σ = 120 × 150 = 18000
Corrected ΣX = 18000 − 15 + 105 = 18090

( )
2
ΣX 2 ΣX
Moreover σ 2 = −
n n
2
ΣX
Or, (15) = − (120)
2 2
150

314 The Institute of Cost Accountants of India


Measures of Central Tendency and Dispersion

So ΣX 2 = 2193750
Therefore Corrected ΣX 2 = 2193750 − 152 + 1052 = 2204550
18090
So Corrected Mean = 150 = 120.6
` 120.6

( )
2
ΣX 2 ΣX 2204550
− (120.6) = 152.64
2 2
Corrected Variance. = σ = n − n =
150
So Corrected S.d = σ = 152.64 = 12.35
` 12.35

Illustration 40. For a group containing 100 observations, the arithmetic mean and standard deviations are 8 and
10.5. For 50 observations selected from these 100 observations, the mean and the standard deviation are 10 and
2 respectively. Find the arithmetic mean and standard deviation of the other half.
Answer:
Suppose the 100 observations have been split up into two groups – Group I containing 50 observations with mean
X 1 = 10 and standard deviation σ1 = 2 and Group II containing the remaining 50 observations. The combined
group of 100 observations has mean X 12 = 8 and s.d. = σ1 = 10.5 . It is required to find the mean X 2 and s.d. (σ 2 )
of group II.
The values are shown in the following table:
Characteristics Groups Combined Group
Group I Group II
No of observations n1 = 50 n2 = 50 100
Mean X 1 = 10 X2 = ? X 12 = 8
S.D. σ1 = 2 σ2 = ? σ12 = 10.5

n1 X 1 + n2 X 2 50 × 10 + 50 × X 2
The mean of combined group = X 12 = n1 + n2
or,
50 + 50
= 8 (Given)
From above we have X 2 = 6
Again the standard deviation of the combined group is

σ12 =
(n σ
1 1
2
+ n2σ 2 2 ) + ( n1d12 + n2d 2 2 )
n1 + n2

Where d1 = X 1 − X 12 & d 2 = X 2 − X 12
d1 = 10 − 8 = 2
d 2 = 6 − 8 = −2
We can write from the s.d. formula of the combined group above —
(n1 + n2 ) σ12 2 = ( n1 σ12 + n2 σ 2 2 ) + ( n1 d12 + n2 d 2 2 )

Or, (50 + 50)10.5 = (50 × 22 + 50 × σ 2 2 ) + [50 × 22 + 50 × (−2) 2 ]


From here we have σ 2 = 3

The Institute of Cost Accountants of India 315


Fundamentals of Business Mathematics and Statistics

Illustration 41. Find Range and Coefficient of Range for the following data

X 5 10 15 20 25 30 35 40
f 4 7 21 47 53 24 12 6

Answer:
Going through the values of X we get smallest value S = 5 and largest value L = 40
Range = L – S = 40 – 5 = 35. It is an absolute measure.
L − S 40 − 5
Coefficient of Range = = = 0.77 . It is a relative measure.
L + S 40 + 5
Illustration 42. Calculate Range and Coefficient of Range for following data

X 20-30 30-40 40-50 50-60 60-70 70-80


f 4 9 16 21 13 6

Answer:
Here L = 80 and S = 20
\ Range = L – S = 80 – 20 = 60
L − S 80 − 20
And Coefficient of Range = L + S = 80 + 20 = 0.6

Illustration 43. Compute Inter quartile Range, Coefficient of Quartile Deviation and Percentile Range for the
following data.

X 4 8 12 16 20 24 28 32
f 4 9 17 40 53 37 24 16

Answer:
Computation details are

X f C.f.
4 4 4
8 9 13
20.1
P10 12 17 30
50.25
Q1 16 40 70
20 53 123
150.75
Q3 24 37 160
180.9
P90 28 24 184
32 16 200

316 The Institute of Cost Accountants of India


Measures of Central Tendency and Dispersion

Ranks of Q1 = (200 +1)/4 = 50.25 i.e 50.25th term is Q1. From the above Table we find that 31st to 70th all the
values are 16. Thus 50.25th value should be 16. Hence Q1 = 16.

Ranks of Q3 = 3(200+1)/4 = 150.75 i.e 150.75th term is Q3. Using the similar concept as above Q3 = 24.

Inter Quartile Range = Q3 – Q1 = 24 – 16 = 8

Q3 − Q1 24 − 16 8
Coefficient of Q.D = Q + Q = 24 + 16 = 40 = 0.2
3 1

To Compute Percentile Range:

Rank of P90 = 90 × (200+1)/100 = 180.9 i.e 180.9th term is P90

So P90 = 28

Rank of P10 = 10 × (200+1)/100 = 20.1 i.e 20.1th term is P10

So P10 = 12

Percentile Range = P90 – P10 = 28 – 12 = 16

Illustration 44. Compute M.D. and Coefficient of M.D. from mean and median for the following series.

X 3 7 12 14 15 18 22

Answer:
Detail computations of mean and median (14 as found from data) are:
Calculations for M.D from Mean & Median

Sl. No.  X |X-Mean| |X-Median|


 1 3 10 11
 2 7 6 7
 3 12 1 2
 4 14 1 0
 5 15 2 1
 6 18 5 4
 7 22 9 8
Total 91 34 33

Mean = ΣX/n = 91/7 = 13


∑ ( X − X ) 34
Mean Deviation from mean = = = 4.86
n 7

The Institute of Cost Accountants of India 317


Fundamentals of Business Mathematics and Statistics

∑ ( X − M d ) 33
Mean Deviation from median = = = 4.71
n 7

Mean Deviation from Mean 4.86


Coefficient of Mean Deviation from Mean = = = 0.37
Mean 13
Mean Deviation from median 4.71
Coefficient of Mean Deviation from Median = = = 0.34
Median 14

Illustration 45. Compute M.D. from Mean and Median for the given series:

X 5 10 15 20 25 30
f 3 4 8 12 7 2

Answer:
Detail Computation of M. D from mean and median are:
Calculations for M.D about Mean and Median

X f C.f. Xf |X-Mean| f |X-Mean| |X-Median| f |X-Median|


5 3 3 15 13.06 39.18 15 45
10 4 7 40 8.06 32.24 10 40
15 8 15 120 3.06 24.48 5 40
20 12 27 240 1.94 23.28 0 0
25 7 34 175 6.94 48.58 5 35
30 2 36 60 11.94 23.88 10 20
Total 36 – 650 – 191.64 – 180.000

Mean =
Σ Xf =
650
= 18.06
Σf 36

Mean Deviation from mean =


Σ f (X − X ) =
191.64
= 5.323
Σf 36

Σ f = 36 Thus Median class is 36/2 =18 th


c.f class. So Median is 20

∑ f ( x − M d ) 180
Mean Deviation from median = = =5
∑f 36

318 The Institute of Cost Accountants of India


Co-efficient of Variation 5.4
Coefficient of Variation (Coefficient of variability):
The relative measure of dispersion based upon standard deviation is called Coefficient of Variation. The formula is
Standard Deviation
Coefficient of variation (C.V) = × 100
Mean
� Coefficient of variation being a pure number is independent of the units of measurement and thus is suitable
for comparing the variability of two or more series of data.
� Higher the value of CV more will be the variability of the data series under consideration.

Illustration 46. From some financial statistics, it is found that the monthly average electricity charges were `2400
and S.D. `120. The monthly average direct wages were `42000 and S.D. `1200. Which one of electricity charges
and direct wages is more variable?

Answer:
Standard Deviation 120
Coefficient of variation for Electricity charges = × 100 = × 100 = 5%
Mean 2400

Standard Deviation 1200


Coefficient of variation for Direct wages = × 100 = × 100 = 2.9%
Mean 42000
So electricity charges is more variable

Properties of Standard Deviations:


1. S.D. is independent of change of origin.
For example S.D. of 3, 4, 10, 19 will be same as that of 5, 6, 12, 21. Here 2 has been added to each term of the
1st Series to get the 2nd Series, but S.D. remains same.
2. S.D. is not independent of change of scale.
For example S.D. of 2, 4, 7, 9 is half of the S.D of 4, 8, 14, 18 as terms are half of the latter series.
2
3. Sum of squares taken from the Arithmetic Mean is always minimum i.e. Σ( X − X ) is always least.

1 2
12 (
4. S.D. of first n natural number is n − 1)

5. For a given set of observation Standard Deviation ≥ Mean Deviation

The Institute of Cost Accountants of India 319


Fundamentals of Business Mathematics and Statistics

Relation between various measures of Dispersion:


Q.D = 2/3 (S.D) and M.D = 4/5 (S.D)
In case of QD, MD and SD, the relation is
(1) X ± QD covers 50% of the total terms.
(2) X ± MD covers 57.51% of the total terms.
(3) X ± SD covers 68.27% of the total terms.
(4) X ± 2SD covers 95.45% of the total terms.
(5) X ± 3SD covers 99.73% of the total terms.
Conclusion: If we discuss about the various measures of dispersion we come to Range first. But Range is not a
stable measure and has many drawbacks such as fluctuations of sampling. Thus can’t be taken as a good measure
of dispersion.
Quartile Deviation: It is certainly better than Range. But here all the terms are not taken into account. It also suffers
from sampling instability. It can be effectively used when class intervals are open; and to calculate M.D and S.D.
these limits have to be assumed.
Mean Deviation: It is not capable of further Algebraic Treatment, although it takes into account all the terms but
still, if the extreme values are big it will distort the result. Moreover, it ignores plus and minus signs; therefore can’t
be called a good measure of dispersion.
Standard Deviation: It is the most important measure of dispersion and is widely used. In this, the drawback of
ignoring the algebraic sign as in Mean Deviation is overcome by taking the square of deviations thereby making
all the deviations positive.
Formula Recapitulation:
Important formulae on dispersion are:

� Range = L – S, Where L is the largest term, S is the smallest term;


L−S
� Coefficient of Range = ;
L+S
� Interquartile range = Q3 – Q1 where Q1 is lower quartile and Q3 is the upper quartile;
Q3 − Q1
� Semi Interquartile range or quartile deviation = 2 ;
Q −Q
� Coefficient of quartile deviation = Q3 + Q1 ;
3 1

� Percentile Range = P90 – P10;

� Decile Range = D9 – D1;


Σf D
� Mean deviation or average deviation MD = N
, where D = X − A , N = Σf , A = Mean, median or mode;
MD
� Coefficient of mean deviation = A
(X − X )
2

� Standard deviation for discrete data = σ = N , where X is the mean and N total no of items;

320 The Institute of Cost Accountants of India


Measures of Central Tendency and Dispersion

( )
2
Σdx 2 Σdx
� Standard deviation for discrete data from assumed mean = σ = − where dx = X − A, A is
N N
assumed mean = any number

2
Σfdx 2  Σfdx 

� Standard deviation for grouped data = σ = N  N  where dx = X – A, A is assumed mean = any
number or actual mean and N = Σf

� Standard deviation of combined group – Group I and Group II

σ12=
(n σ
1 1
2
+ n2σ 2 2 ) + ( n1d12 + n2d 2 2 )
n1 + n2

Where
σ1 = S.d of Group I, σ2 = S. d of Group II, n1 = no of items in Group I, n2 = no of items in Group II, d1 & d2 are
deviations of group means from combined group mean

S.D
� Coefficient of Variation = Mean × 100

The Institute of Cost Accountants of India 321


Karl Pearson and Bowley’s
Co-efficient of Skewness 5.5
Skewness:
Skewness refers to the asymmetry or lack of symmetry in the shape of a frequency distribution curve.

NOT SKEWED or SYMMETRICAL


Frequency

Value

Figure-1: Showing typical Frequency Distribution curves with different types of skewness

So skewness is the attribute of a frequency distribution that extends further on one side of the class with the highest
frequency that on the other.
Let us take the following frequency distribution (Table 1) of three groups –Group I, Group II and Group III
For each group Σf = 250 and size pattern same as shown.
N
Σxf −C
For each group we find Average as X = , Median as M d = l1 + 2 × h and Mode as
Σf f
f1 − f 0
M o = l1 + ×h
2 × f1 − f 0 − f 2

322 The Institute of Cost Accountants of India


Measures of Central Tendency and Dispersion

Where
x = mid values of different classes, f = frequency, N = Σf, l1 = lower boundary of Median or Modal class,
C = cumulative frequency of class just preceeding median class, f in median formula = frequency of median class,
f1 = frequency of modal class, f0 = frequency of the class just preceeding modal class,  f2 = frequency of the class
just succeeding modal class, h = width of median and modal class.

Group I Group II Group III


Size
Mid value Frequency-I Frequency-II Frequency-III
0–5 2.5 10 10 10
5 – 10 7.5 90 30 20
10 – 15 12.5 50 50 30
15 – 20 17.5 40 70 40
20 – 25 22.5 30 50 50
25 – 30 27.5 20 30 90
30 – 35 32.5 10 10 10
Total – 250 250 250

Table -I
From the above data and formula given for Mean Median and Mode we have the following result, for different
groups.

Group I Group II Group III


Mean 14.3 17.5 20.7
Median 12.5 17.5 26.25
Mode 8.33 17.5 26.67

From the above table we can say

For group I: Mean > Median > Mode

For group II: Mean = Median = Mode

For group III: Mean < Median < Mode

The Institute of Cost Accountants of India 323


Fundamentals of Business Mathematics and Statistics

For these three groups if we plot the data in a graph paper then we will observe the following:
For Group I
• Refer figure 2— a positively skewed curve— a positively skewed curve occurs when a frequency distribution
has Mean > Median > Mode
Frequency

Mode Median Mean

Value

Figure-2: Showing a typical Positivity Skewed Frequency distribution curve


For Group II
• Refer figure 3— a symmetrical curve— a symmetrical curve occurs when a frequency distribution has Mean
= Median = Mode
Frequency

Mean = Median = Mode

Value
Figure-3: Showing a typical Symmetrical Frequency distributation curve

324 The Institute of Cost Accountants of India


Measures of Central Tendency and Dispersion

For Group III


• Refer figure 4— a negatively skewed curve— a negatively skewed curve occurs when a frequency distribution
has Mean < Median < Mode
Frequency

Mean Median Mode

Value

Figure-4: Showing a typical Negatively Skewed Frequency distribution curve

From the above discussion it can be concluded that


In case of Symmetrical Series, the mean, median and mode coincide. i.e., Mean = Median = Mode
In case of Positive Skewed: If the tail is towards right, then it is called positive skewness. It means
Mode < Median < Mean
In case of Negative Skewed: When the tail of a distribution is towards left, then the skewness is negative i.e.
Mode > Median > Mean
It should be noted that in both ‘+’ve and ‘-‘ve skewed distributions that median lies in between the mode and the
mean.
Other characteristics of Positively Skewed (Group –I), Symmetrical (Group-II) and Negatively Skewed (Group-
III) distributions are
For Group –I : Q3 − M d > M d − Q1
For Group –II : Q3 − M d = M d − Q1
For Group –III : Q3 − M d < M d − Q1
Where Md is the median Q3 is the third quartile, Q1 is the first quartile
There are three Absolute measures of skewness:
Absolute skewness (SK) = Mean – Mode
Absolute skewness (SK) = Mean – Median
Absolute skewness (SK) = Median – Mode

The Institute of Cost Accountants of India 325


Fundamentals of Business Mathematics and Statistics

There are four types of relative measures of skewness namely:


(1) The Karl Pearson’s Coefficient of Skewness;
(2) The Bowley’s Coefficient of Skewness;
(3) The Kelly’s Coefficient of Skewness;
(4) Measurement of Skewness based on Moments;
The Karl Pearson’s Coefficient of Skewness:
Mean - Mode
Sk =
Standard Deviation
If in a particular frequency distribution, it is difficult to determine precisely the mode, or the mode is ill defined
then:
3 (Mean – Median)
Sk =
Standard Deviation
Skewness obtained by both of these formulae may vary between ± 3. But mostly it varies in the range ± 1.
For our Groups the Skewness values as per this formula are given in the following table:

Group I Group II Group III

0.77184 0 -0.77184

Skewness value of Group I is positive and that of Group III is negative, as observed from graph. Also Grpah II
being symmetrical shows zero skewness.

Bowley’s Coefficient of Skewness:


Q3 + Q1 − 2M d
Sk =
Q3 − Q1

where Q3 & Q1 are third and first quartile and Md is the Median of the distribution
This is also called Quartile measure of skewness and it varies between +1 and – 1.
For our Groups the Skewness values as per this formula are given in the following table:

Group I Group II Group III


Q3 19.69 22.75 27.08
Md 12.50 17.50 22.50
Q1 7.92 12.25 15.31
Sk 0.22 0.00 -0.22
While obtaining the above results the following formula are used for determining standard deviation, third quartile
and first quartile:

Σfx 2  Σfx 
2 Group I Group II Group III
σ= − =
Σf  Σf  7.73 7.21 7.73

326 The Institute of Cost Accountants of India


Measures of Central Tendency and Dispersion

3N N
−C −C
Q3 = l1 + 4 × h & Q1 = l1 + 4 ×h
f f

Where l = lower boundary of the respective quartile class


C = cumulative frequency of the class just preceeding the respective quartile class
f = simple frequency of the respective quartile class
h = width of the respective quartile class
In practice Karl Pearson’s Coefficient of Skewness is usually used.

The Kelly’s Coefficient of Skewness:


P90 + P10 − 2P50
Sk =
P90 − P10

OR
D9 + D1 − 2D5
Sk =
D9 − D1

Kelly’s Coefficient of Skewness is also called percentile skewness or decile skewness. It is seldom used in practice.
It’s value lies between ±1.

Illustration 47. For a group of 10 items, ΣX = 452, ΣX2 = 24270, and Mode = 43.7; Find the Pearson’s coefficient
of Skewness

Answer:
ΣX 452
Mean ( X ) = n = 10 = 45.2

( )=
2
ΣX 2 ΣX 24270
S.D. (σ) = − − (45.2) 2 = 19.59
n n 10

Pearson’s coefficient of skewness:


Mean - Mode 45.2 − 43.7
Sk = = = 0.08
Standard Deviation 19.59
Illustration 48.
Consider the following distribution and comment on the correctness of the statements (i) and (ii) with reasons.

Distribution A Distribution B
Mean 100 90
Median 90 80
Standard Deviation 10 10

(i) Distribution A has the same degree of the variation as distribution B


(ii) Both distributions have the same degree of skewness.

The Institute of Cost Accountants of India 327


Fundamentals of Business Mathematics and Statistics

Answer:
σA 10
(i) Coefficient of variation (for distribution A) = X × 100 = 100 × 100 = 10%
A

σB 10
Coefficient of variation (for distribution B) = × 100 = × 100 = 11.11%
XB 90
Since C.V of (B) > C.V. of (A), the distribution B is more variable than the distribution A. Thus the given
statement that the distribution A has the same degree of variation as distribution B is false.
(ii) Karl Pearson’s coefficient of skewness for distribution A and B is given by
3( Mean − Median ) 3(100 − 90)
Sk (A) = = =3
σ 10
3( Mean − Median ) 3(90 − 80)
Sk (B) = = =3
σ 10
Thus the given statement that both distributions have same degree of skewness is true.

Illustration 49. You are given below the following details relating to wages in respect of two factories from which
it is concluded that the skewness and variability are the same in both the factories:

Factory A Factory B
Mean `50 `45
Mode `45 `50
Variance Sq. ` 100 Sq. ` 100
Point out the mistake, if any, in the above statement
Answer:

Standard deviation = σ = variance = 100 = 10

σA 10
Coefficient of variation (for distribution A) = × 100 = × 100 = 20%
XA 50

σB 10
Coefficient of variantion (for distribution B) = × 100 = × 100 = 22.22%
XB 45

Since C.V of (B) > C.V. of (A), the distribution B is more variable than the distribution A. Thus the given statement
that variability of wages in factories A & B are same is false.
Karl Pearson’s coefficient of skewness for factories A and B is given by

( Mean − Mode) = (50 − 45) = 0.5


Sk (A) = σ 10
( Mean − Mode) = ( 45 − 50) = −0.5
Sk (B) = σ 10
Thus the given statement that both the factories have same degree of skewness in respect of wages is false.

328 The Institute of Cost Accountants of India


Measures of Central Tendency and Dispersion

Illustration 50. In a moderately skewed frequency distribution the mean is `30 and the median is `20. If the co-
efficient of variation is 70%, find the Karl Pearson’s coefficient of skewness.

Answer:
σ σ
Coefficient of variation for the distribution = × 100 = × 100 = 70%
X 30
So standard deviation = σ = 0.7 × 30 = 21

3( Mean − Median ) 3(30 − 20)


Sk = = = 1.428
σ 21

Illustration 51. Given X = 20 , median = 22 and coefficient of skewness = – 0.16. Find out Standard deviation.

Answer:
3( Mean − Median )
Karl Pearson’s coefficient of skewness = Sk = = −0.16
σ
3( 20 − 22)
Or, = −0.16
σ
3( 20 − 22) −6
Or, σ = −0.16
=
−0.16
= 37.5

Illustration 52. The following facts are gathered before and after an industrial dispute:

Description of items Before Dispute After Dispute


No of workers employed 515 509
Mean Wages ` 49.5 ` 52.75
Median Wages ` 52.80 ` 50.00
Variance of wages Sq. `121.00 Sq. ` 144.00

Compare the position before and after the dispute in respect of (a) total wages; (b) median wages; (c) modal wages;
(d) standard deviation; (e) coefficient of variation; (f) coefficient of skewness.

Answer:
(a)

Total Wages Mean Wages × No of Before dispute After dispute


in ` workers employed 49.5 × 515 = 25492.5 52.75 × 509 = 26849.75

(b) The median wages after the dispute has come down from `52.80 to `50.00. This implies that before the
dispute upper 50% of the workers were getting wages above `52.80 whereas after the dispute they get wages
only above `50.

(c) We know that Mean – Mode = 3 (Mean – Median)

The Institute of Cost Accountants of India 329


Fundamentals of Business Mathematics and Statistics

Before Dispute: 49.5 – Mode = 3 (49.5 – 52.8)


Or, Mode = 49.5 – 3 (49.5 – 52.8) = 49.5 + 9.9 = ` 59.40
After Dispute: 52.75 – Mode = 3(52.75 – 50.00)
Or, Mode = 52.75 – 3 (52.75 – 50.00) = 52.75 – 8.25 = ` 44.50
It means that there is a concentration of wages around a much smaller value (44.5) after the dispute
(d)
Standard Deviation Before dispute After dispute
`11
121 = 11 `12
144 = 12
It means that variability of individual wages increased after the dispute.
(e)

Before dispute After dispute


Coefficient of Standard Deviation
Variation × 100 11 × 100 12 × 100
Mean = 22.22% = 22.75%
49.50 52.75

It means that after dispute more variability of wages are there as CV (after) > CV (before)

Mean − Mode
(f) Coefficient of Skewness: Sk =
Standard Deviation

Before dispute: Sk = 49.50 − 59.4 = −0.9


11
52.75 − 44.5
After dispute: Sk = = 0.6875
12
Thus the negatively skewed distributions (before the dispute) has become a positively skewed distribution (after
the dispute) which means that the curve of distribution of wages after the dispute has a fairly longer tail towards the
right. Hence we conclude that the number of workers getting higher wages has decreased after the dispute.

330 The Institute of Cost Accountants of India


Measures of Central Tendency and Dispersion

EXERCISE PROBLEMS

1. Calculate X for the following data by direct, short cut and step deviation methods.

70 65 55 75 80 85 65 70 95

Answer:
(i) Direct Method: X = Σx
n
Σdx
(ii) Short Cut Method (or) Indirect Method: X = A +
n
Σdx′
(iii) Step Deviation method: X = A + ×i
n
X dx = X – 80 dx´ = dx/5
70 –10 –2
65 – 15 –3
55 – 25 –5
75 –5 –1
80 0 0
85 5 1
65 –15 –3
70 – 10 –2
95 15 3
Total 660 –60 –12

Σx 660
Direct Method: X = = = 73.33
n 9
Short Cut Method (or) Indirect Method: X = A + Σ = 80 − 60 = 73.33
dx
n 9
Step Deviation method: X = A + Σ dx ′ 12
× i = 80 − × 5 = 73.33
n 9

2. Calculate X for the following series by Direct, Short-cut and step-deviation Methods.

X 5 10 15 20 25 30 35 40
f 6 17 28 34 18 11 9 7

Answer:
ΣfX
(i) Direct Method: X =
Σf
Σfdx
(ii) Short Cut Method (or) Indirect Method: X = A +
Σf
(iii) Step Deviation method: X = A + Σ
fdx′
×i
Σf
The Institute of Cost Accountants of India 331
Fundamentals of Business Mathematics and Statistics

X f Xf dx =X-20 fdx dx´ = dx/5 fdx´


5 6 30 -15 - 90 -3 -18
10 17 170 -10 -170 -2 -34
15 28 420 -5 -140 -1 -28
20 34 680 0 0 0 0
25 18 450 5 90 1 18
30 11 330 10 110 2 22
35 9 315 15 135 3 27
40 7 280 20 140 4 28
Total 130 2675 20 75 4 15

Direct Method: X =
Σ fX =
2675
= 20.58
Σf 130
Σfdx 75
Short Cut Method (or) Indirect Method: X = A + = 20 + = 20.58
Σf 130
Σfdx′ 15
Step Deviation method: X = A + × i = 20 + × 5 = 20.58
Σf 130

3. A class has 50 students with average weight of 45 kgs. Out of these there are 30 girls with average weight of
42.5 kgs. Find average wt. of boys.
Answer:
X 1 × n1 + X 2 × n2 + …….. + X n × nn
X=
n1 + n2 + ………+ nn
X = combined mean of boys & girls = 45 & n = Total of boys & girls = 50
X 1 = Mean of girls = 42.5 & n1 = Total of girls = 30
Total of boys = n2 = n – n1 = 50 – 30 = 20
Total weight of boys and girls =X × n = 45 × 50 = 2250
Total weight of girls = X 1 × n1= 42.5 × 30 = 1275
Total weight of boys = Total weight of boys & girls-Total weight of girls = 2250 – 1275 = 975
975
So Average weight of boys = 20 = 48.75

4. A class of 40 students has an average of 56 marks in Math exam. But later on it was found thatterms 48, 54
and 67 were misread as 68, 45 and 87. Find correct mean.
Answer: Total marks of 40 students = 40 × 56 = 2240
Correct total = 2240 – 68 – 45 – 87 + 48 + 54 + 67 = 2209
2209
So Correct mean = = 55.225
40
5. X for 20 items was 36, but two terms were taken as 47 and 56 instead of 67 and 65. Find correct mean.

332 The Institute of Cost Accountants of India


Measures of Central Tendency and Dispersion

Answer:
Total of 20 students = 20 × 36 = 720
Correct total = 720 – 47 – 56 + 67 + 65 = 749
749
So Correct mean = 37.45
20

6. Average of 10 terms is 6, Find new average if each term is (i) multiplied by 2 (ii) divided by 5 (iii) 3 is added
to each term (iv) 4 is subtracted from each term.
Hints:
Total of 10 terms = 10 × 6 = 60
if each term is multiplied by 2, new total will be = 60 × 2 = 120
60
if each term is divided by 5, new toal will be = 5 = 12
if 3 is added to each term, new toal will be = 60 + 10 × 3 = 90
if 4 is subtracted from each term, new toal will be = 60 – 10 × 4 = 20
120 12 90 20
So (i) = 12 ; (ii)= = 1.2 ; (iii)= = 9 ; (iv)= =2;
10 10 10 10

7. Find X for the following series. (Inclusive Series)

Class interval 3-5 6-8 9-11 12-14 15-17 18-20


f 3 7 16 34 17 3
Answer:

Direct Method: X = Σfx , here x we take as the mid value of each class
Σfa

Class X f Xf
3-5 4 3 12
6-8 7 7 49
9-11 10 16 160
12-14 13 34 442
15-17 16 17 272
18-20 19 3 57
Total 80 992
Σfx 992
X= = = 12.4
Σf 80

The Institute of Cost Accountants of India 333


Fundamentals of Business Mathematics and Statistics

8. Find median

Class Less than 50 50-75 75-100 100-125 125-150 More than 150
Interval
f 21 47 67 89 55 21
Answer:
N
−C
M d = l1 + 2 ×h
f

Where l1 = Real lower limit of the median class


N = Σf
C = Cumulative frequency just preceding the median class
f = simple frequency of the median class
h = width of the median class

Class f c.f
less than 50 21 21
50-75 47 68
75-100 67 135
Median Class
100-125 89 224
125-150 55 279
More than 150 21 300
Total 300
So
N 300
−C − 135
M d = l1 + 2 × h = 100 + 2 × 25 = 104.21
f 89

9. Mean weight of students in a class is 48 kg. If mean weight of girl students is 40 kg. And that of boysis 60
kg. Find (a) % age of boys and girls (b) If there are total of 75 students, find number of each.
Answer:
let there are x no of boy students. So there are 75 – x no of girls
x × 60 + ( 75 − x ) × 40
So by question we can write: 48 =
75
Or, 75 × 48 = 60x + 75 × 40 – 40x
Or, 3600 = 20x + 3000
600
Or, x = = 30
20
So no of girls students = 75 – 30 = 45

334 The Institute of Cost Accountants of India


Measures of Central Tendency and Dispersion

30 45
% age of boys and girls = × 100 = 40% & × 100 = 60%
75 75

10. Mean of a series with 50 terms is 80; but afterwards it was noted that three terms 63, 47 and 88were misread
as 36,74 and 63. Find correct Mean.
Answer:
Total of 50 terms = 50 × 80 = 4000
Correct total = 4000 – 36 – 74 – 63 + 63 + 47 + 88 = 4025
4025
So Correct mean = = 80.5
50

11. Median of a series is 80, but four terms 38, 66, 93 and 96 were misread as 83, 88, 39 and 69 find correct
Median.

12. Calculate M, Q1, Q3 for the following data

21 13 17 11 19 9 16 23 14

Answer:
Arranging the data in ascending order we get

9 11 13 14 16 17 19 21 23

Median = Md = M =
( n + 1) th term 9 + 1 = 5th term = 16
2 2

Q1 =
( n + 1) th term = 9 + 1 = 2.5 th term = (11 + 13)/2 = 12
4 4
3( n + 1) 3(9 + 1)
Q3 = th term = = 7.5th term = (19 + 21) / 2 = 20
4 4

13. Calculate M, Q1, Q3, D6 and P91for following data

X: 8 12 20 25 30 40
F: 9 16 28 46 20 10

14. Calculate Median Q1, Q3, D4 and P86 for following data

X: 0-4 4-8 8-12 12-16 16-20 20-24 24-28 28-32


F: 4 17 36 90 123 110 66 14

The Institute of Cost Accountants of India 335


Fundamentals of Business Mathematics and Statistics

Answer:

Class f c.f
0-4 4 4
4-8 17 21
8-12 36 57
12-16 90 147 Q1 class
16-20 123 270 Median Calss,D4 class
20-24 110 380 Q3 class
24-28 66 446 P84 class
28-32 14 460
So
N 460
−C − 147
M d = l1 + 2 × h = 16 + 2 × 4 = 18.699
f 123
N 460
−C − 57
Q1 = l1 + 4 × h = 12 + 4 × 4 = 14.58
f 90

3N 3 × 460
−C − 270
Q2 = l1 + 4 × h = 20 + 4 × 4 = 22.72
f 110

4N 4 × 460
−C − 147
D4 = l1 + 10 × h = 16 + 10 × 4 = 17.203
f 123

84 N 84 × 460
−C − 380
P84 = l1 + 100 × h = 24 + 100 × 4 = 24.388
f 66

15. Find M, Q1, Q3 for the following figures

X: 4-7 8-11 12-15 16-19 20-23 24-27 28-31


F: 7 11 23 47 36 29 17

16. Calculate D1, M1, Q3 and P95 for the following data (Unequal Intervals)

Class Interval: 0-20 20-30 30-80 80-120 120-180 180-200


Frequency: 7 19 38 78 45 13

17. Find the value of missing frequencies if Q1 = 36 and N = 840

X: 20-25 25-30 30-35 35-40 40-45 45-50 50-55 55-60


F: 40 50 ? 150 180 130 ? 60

336 The Institute of Cost Accountants of India


Measures of Central Tendency and Dispersion

Hints:
Let the missing figures are f1 and f2

Class f c.f
20-25 40 40
25-30 50 90
30-35 f1 90 + f1 120
35-40 150 240 + f1 Q1 class
40-45 180 420 + f1
45-50 130 550 + f1
50-55 f2 550 + f1 + f2
55-60 60 610 + f1 + f2

So by question we can write:


610 + f1 + f2 = 840, Or, f1 + f2 = 230
N 840
Rank of Q1 = = = 210 i.e 210th rank in Q1.
4 4
N 840
−C − 90 − f1 120 − f1
Q1 = 36 = l1 + 4 × h = 35 + 4 × 5 = 35 + So f1 = 90
f 150 30
So from, f1 + f2 = 230, f2 = 140

18. Find out median and quartiles for the following data:

Marks 0-10 10-20 20-30 30-40 40-50 50-60 60-70 70-80 80-90 90-100
No.of Stu 11 18 25 28 30 33 22 15 12 10

19. Calculate P22 and D8

X: 20-29 30-39 40-49 50-59 60-69 70-79


F: 2 19 20 21 15 13

20. In the batch of 15 students, 5 students failed. The marks of 10 students who passed were 9, 6, 7, 8, 8, 9, 6, 5,
4, 7 what were the median marks of all 15 students?

The Institute of Cost Accountants of India 337


Fundamentals of Business Mathematics and Statistics

Exercise:
Theoretical Questions
 Multiple Choice Questions (MCQ)

1. Which one of the following is a Positional Average?


(a) Geometric Mean;
(b) Harmonic Mean;
(c) Mode;
(d) Progressive Average;

2. Because of heavy rain on Sunday average rainfall of a city for the week increased to 0.6 inch from the
average rainfall 0.3 inch measured from Monday to Saturday. The rainfall on Sunday was-
(a) 2.4 inch;
(b) 0.3 inch;
(3) 2.1 inch;
(d) 1.5 inch

3. The mean of the frequency distribution ( ( x1, f1 ) , ( x2, f 2 ) , ( x3, f 3 )…………( xn , f n ) ) is

Σx
(a) ;
Σf
Σx
(b) ;
n
Σfx
(c) ;
n
Σfx
(d) ;
Σf


20
4. x =1
x = 54120 ; While computing this, it was observed that two entries were wrongly entered as 850 and
320 instead of 580 and 230. Correct value of x is
(a) 2688;
(b) 2746.5;
(c) 2720;
(d) 2662;

338 The Institute of Cost Accountants of India


Measures of Central Tendency and Dispersion

5. Σ( X − X ) is always equal to
(a) 1;
(b) -1;
(c) 0;
(d) ∞ ;
6. The sum of the squares of deviations of a set of observations is the minimum when deviations are taken
from the
(a) Geometric Mean;
(b) Harmonic Mean;
(c) Arithmetic Mean;
(d) Mode;

7. If each of the values of a variable x with mean x , is multiplied by K then the new mean of the variable is
x
(a) ;
k
(b) k x ;
(c) x ;
(d) kx ;

8. Assumed mean is 35, Σ f d = − 4 2 5 & Σ f = 6 3 . x is


(a) 20;
(b) 25.87;
(c) 28.25;
(d) 19.34;

9. If A = 500, i = 1000 & Σfd ′ = 232 & Σf = 120 x is


(a) 2433.33;
(b) 527.84;
(c) 501.93;
(d) 1017.24;

10. The mean daily salary paid to all employees in a certain company was `600. The mean daily salaries paid
to the male and female employees were `620 and `520 respectively. Male to female employees ratio in the
company is
(a) 3:2;
(b) 4:5;

The Institute of Cost Accountants of India 339


Fundamentals of Business Mathematics and Statistics

(c) 5:7;
(d) 4:1;

11. The pass result of 50 students who took up a class test is given below:

Marks 4 5 6 7 8 9
No of Students 8 10 9 6 4 3
If the average marks for all the fifty students was 5.16, the average marks of the students who failed is
(a) 0.42;
(b) 3.06;
(c) 4.74;
(d) 2.1;

12. For a certain frequency table which has only been partly reproduced below for which the mean is 1.46

No of accidents 0 1 2 3 4 5
Frequency 46 f1 f2 25 10 5

If Σf = 200 Unknown frequencies are


(a) f1 = 0, 2
= 114;
(b) f1 = 114, f2 = 0;
(c) f1 = 76, f2 = 38;
(d) f1 = 57, f2 = 57;

13. The sum of the deviations of a certain number of observations measured from 4 is 72 and the sum of the
deviations of the observations from 7 is -3. Mean of the observations is
(a) 6.88;
(b) 25;
(c) 3.63;
(d) Cannot be ascertained with given data;

14. The mean of a certain number of items is 42. If one more item 64 is added to the data, the mean becomes
44. The no of items in the original data is
(a) 20;
(b) 10;
(c) 43;
(d) 440;

340 The Institute of Cost Accountants of India


Measures of Central Tendency and Dispersion

15. The weighted average from the following observation is `46.23.

Price per tonne (`) 45.60 50.70 ?


Tonnes Purchased 135 40 25
Simple average of the observation is
(a) `46.23;
(b) `46.26;
(c) `66.63;
(d) `46.24;

16. In a certain factory a unit of work is completed by A in 4 minutes, by B in 5 minutes, by C in 6 minutes,


by D in 10 minutes, and by E in 12 minutes. Average number of units of work completed per minute is
(a) 25/4;
(b) 5/48;
(c) 4/25;
(d) 25/48;

17. A person walks 8 km at 4km an hour, 6km at 3km an hour and 4km at 2km an hour. Average speed per
hour is
(a) 0.33;
(b) 2;
(c) 3;
(d) 0.5;

18. It is the most suitable average when it is desired to give greater weight to smaller observations and less
weight to larger ones. It is
(a) AM;
(b) HM;
(c) GM;
(d) Median;

19. Which one of the following is not a feature of Arithmetic Mean (AM)?
(a) AM is affected very much by extreme values;
(b) AM is widely used in the study of qualitative phenomenon;

The Institute of Cost Accountants of India 341


Fundamentals of Business Mathematics and Statistics

(c) AM provides a good basis for comparison;


(d) AM is rigidly defined so different interpretation by different people are not possible;

20. Which one of the following is a feature of Harmonic Mean (HM)?


(a) GM is affected much by the presence of externally small or large observations;
(b) GM gives the actual value of the series;
(c) GM is useful when a given phenomenon has a limit for lower value;
(d) GM is imaginary if any of the observations is zero;

 State True or False

1. The arithmetic mean is always the best measure of central tendency


2. The sum of individual observations from mean is zero
3. In a moderately skewed distribution A.M. < G.M. < H.M
4. The addition of a constant value to each of the values of a series increases the average by thesame value
of the constant.
5. Average alone is enough to throw light on the main characteristics of a statistical series.
6. Median is a mathematical average
7. The value of median and mode can be determined graphically
8. Combined median can be calculated as in case of arithmetic mean
9. Percentile divides the series in ten equal parts
10. Range is the best measure of Dispersion.
11. Mode is the value that has maximum frequency
12. Mode can be located graphically
13. In a positively skewed distribution mode > mean
14. Mode is a mathematical average
15. A distribution with more than two modes is called multimodal
16. Geometric mean is a positional measure of central tendency
17. Geometric mean is more suitable for dealing with problems of rates and speed
18. Geometric mean can be computed in case of open end series
19. Combined G.M. of two or more series can be calculated
20. The geometric mean is the nth root of the product of n items in a given distribution.

342 The Institute of Cost Accountants of India


Measures of Central Tendency and Dispersion

 Short Answer Type Questions

1. What do you mean by “Measures of Central Tendency”?


2. What are different types of averages?
3. If some AMs are given of different series, how can we find combined X for all of those? Also provide the
formula.
4. What do you mean by Weighted Arithmetic Mean? How to calculate it? Give it’s formula also.
5. Define –
(a) Median
(b) Quartiles
(c) Lower Quartile
(d) Upper quartile
(e) Decile
(f) Percentile
6. How can we calculate (i) Median (ii) Q1 (iii) Q3 (iv) Decile (v) Percentile in Individual or Discrete Series?
Also write steps to calculate.
7. How to calculate the above said measures in a continuous series? Write steps to calculate.
8. What amendments are to be made for
(i) Inclusive Series
(ii) Cumulative series
(iii) Open End series
(iv) Unequal Interval Series.
9. What are Positional Value or Partition Measures? What are its various types?
10. Define mode
11. What is the Empirical relation between Mean, Mode and Median? Who invented it?
12. When is empirical formula used to calculate Mode? How is it used?
13. Give the precise definition of G.M. along with the formula of its calculation
14. G.M. is a calculated average. Explain.
15. Give three merits of G.M.
16. Give three demerits of G.M.
17. Give three important properties of G.M.
18. Which formula is used to show the rise in prices or the increase in population?
19. Give meaning of H.M.
20. Give three Merits of H.M.

The Institute of Cost Accountants of India 343


Fundamentals of Business Mathematics and Statistics

Answer:

 Multiple Choice Questions (MCQ)

1 2 3 4 5 6 7 8 9 10
(c) (a) (d) (a) (c) (c) (b) (c) (a) (d)
11 12 13 14 15 16 17 18 19 20
(d) (c) (a) (b) (b) (c) (c) (b) (b) (c)

 State True or False

1 2 3 4 5 6 7 8 9 10
T T F T F F T F F F
11 12 13 14 15 16 17 18 19 20
T T F F F F F F T T

344 The Institute of Cost Accountants of India


CORRELATION AND REGRESSION 6
This Module includes

6.1 Scatter Diagram


6.2 Karl Pearson’s Co-efficient of Correlation
6.3 Regression Analysis

The Institute of Cost Accountants of India 345


Fundamentals of Business Mathematics and Statistics

CORRELATION AND REGRESSION

Module Learning Objectives:


After studying this Module, the students will be able to understand –
 What is meant by Correlation and Regression Analysis in statistics
 Different methods of Computation of
● Correlation Coefficients
● Regressions Cofficients
● Regression Equations
 Relation between Correlation and Regression
 Application of correlation and regression in real life situations

346 The Institute of Cost Accountants of India


Scatter Diagram 6.1
CORRELATION ANALYSIS
Managers, generally, assess the nature and degree of relationship between variables. A marketing manager likes to
know the degree of relationship between advertising expenditure and the volume of sales. A positive relationship
is expected between these two variables.The manager is interested in finding out whether there is relationship and
if it is what the strength of the relationship is. This expectation is true between two or more variables.
Correlation analysis helps in this regard and Correlation coefficient helps the managers to know the degree of
relationship between the variables.
According to A.M. Tuttle, Correlation is defined as an analysis of the covariation between two or more variable.
Now Correlation analysis gives us a statistical tool which helps us to ascertain the degree to which one variable is
LINEARLY related to another. Remember for only linear relationship this correlation analysis offer us help to find
out degree of association. For other sorts of relationship between variables this will not. All business units in this
world make estimate of the future variables on the basis of historical happenings as follows:

What has actually happened? What is to be estimated?

1. Is there any relationship between the green and black box?


2. If there is relationship what is that?
3. How much is the strength of the relationship?

Knowledge of Regression & Correlation answers these questions.

The Institute of Cost Accountants of India 347


Fundamentals of Business Mathematics and Statistics

Types of correlation:
There are three important types of correlation which are discussed as under:
(I) Positive and Negative Correlation: Correlation between two variables, positive or negative depends on the
direction in which the variables move.
(a) Positive or direct correlation: If the two variables move in the same direction i.e. with an increase in one
variable, the other variable also increases or with a fall in one variable, the other variable also falls, the
correlation is said to be positive.
(b) Negative or Inverse Correlation: If two variables move in opposite direction i.e. with the increase in one
variable, the other variable decreases or with the fall in one variable, the other variable rises, the correlation is
said to be negative or inverse.

(II) Simple and Multiple Correlations:


(a) Simple Correlation: When there are only two variables and the relationship is studied between those two
variables, it is the case of simple correlation. Relationships between height and weight,price and demand or
income and consumption etc. are examples of simple correlation.
(b) Multiple Correlation: When there are more than two variables and we study the relationship between one
variable and all the other variables taken together then it is a case of multiple correlations. Suppose there are
three variables 1, 2, 3 we can study the multiple correlation betweenA and B & C taken together or between
B and A & C together etc. It can be denoted as R1.23 Or R3.12
(III) Partial and Total Correlation:
(a) Partial Correlation: When there are more than two variables and the relationship between any two of the
variables is studied assuming other variables as constant it is a case of partial correlation.This, in fact, is an
extension of multiple correlation. Suppose we study the relationship between rainfall and crop, without taking
into consideration the effects of other inputs like fertilizers, seeds and pesticides etc., this will be a case of
partial correlation. Symbolically if x, y, z are three variables then partial correlation between x and y excluding
z will be given by rxy. z Similarly rxz. y and ryz. x are possible.
(b) Total Correlation: When the correlation among the variables under study is worked out taking all at a time,
it is called total correlation.

Degree and Interpretation of Correlation Coefficient


According to Karl Pearson, the Coefficient of Correlation lies between – I & + 1 i.e. –1 ≤ r ≤ 1. It implies if there
is perfect positive relationship between two variables, the value of correletation coefficient would be +1. On the
contrary it would be - 1 when there is perfect negative relationship between two variables. Within these limits, the
value of correletation coefficient is interpreted as per the table below:

(i) When r = + 1 Perfect positive correlation


(ii) When 0.75 < r < 1 High degree of positive correlation
(iii) When 0.5 < r < 0.75 Moderate degree of positive correlation
(iv) When 0 < r < 0.5 Low degree of positive correlation
(v) When r = 0 No correlation at all
(vi) When 1 < r < – 0.75 High degree of negative correlation

348 The Institute of Cost Accountants of India


Correlation and Regression

(vii) When – 0.75 < r < – 0.5 Moderate degree of negative correlation
(viii) When – 0.5 < r < 0 Low degree of negative correlation
(ix) When r = – 1 Perfect negative correlation

Scatter Diagram
One of the way of getting an idea about correlation is Scatter Diagram.

Scatter Diagram Method


The scatter diagram represents graphically the relation between two variables Xand Y. For each pair of Xand Y,
one dot is put and we get as many points on the graph as the number of observations. Degree of correlation between
the variables can be estimated by examining the shape of the plotted dots.
Following are some scattered diagrams showing varied degrees of correlation.

Perfect positive correlation; r = 1 Perfect negative correlation; r = – 1

Low degree of positive correlation; r > 1 Low degree of negative correlation; r < 1

The Institute of Cost Accountants of India 349


Fundamentals of Business Mathematics and Statistics

No correlation; r – 0

Advantages
(1) It is very easy to draw a scatter diagram
(2) It is easily understood and interpreted
(3) Extreme items does not unduly affect the result as such points remain isolated in the diagram

Disadvantages
(1) It does not give precise degree of correlation
(2) It is not amenable to further mathematical treatment

350 The Institute of Cost Accountants of India


Karl Pearson’s Co-efficient of
Correlation 6.2
METHODS OF STUDY OF CORRELATION
We will study linear correlation through Karl Pearson’s method or Covariance method.Although scatter diagram
method and graphic method are there but they are very subjective and rarely used in reality

Direct method:
Σ {( X − X )(Y − Y )}
Coefficient of Correlation = r = ... ... ... (1)
Σ( X − X ) 2 Σ(Y − Y ) 2
The above formula can also be written as
Σ {( X − X )(Y − Y )} Cov ( x, y)
r= Or, r = ... ... ... (2)
N σ xσ y σ x ⋅σ y

Σ(X − X ) Σ (Y − Y )
2 2

Where σ x = and σ y = .
N N

Σ {( X − X )(Y − Y )}
is called Cov (x, y) or Co-variance between X & Y. σ x & σ y are Standard Deviations of series
N
X and Y.
Another way of writing the above formula is

Σ XY −
( Σ X )( Σ Y )
r= N … … … … (3)
 2 ( Σ X )2   2 ( Σ Y )2 
Σ X − N  Σ Y − N 
  
Indirect Method:

Σdx.dy −
( Σdx )( Σdy )
r= N … … … … (4)
 2 ( Σdx )
2
  2 ( Σdy ) 
2

Σdx − N  Σdy − N 
  

Where dx = X – A, dy = Y – B, A & B are assumed means of X and Y series


Σdx dy − N ( X − AX )(Y − AY )
… … … … (5)
σ X σY
Where AX & AY are assumd means of X and Y series

The Institute of Cost Accountants of India 351


Fundamentals of Business Mathematics and Statistics

Illustration 1. Calculate the Correlation Coefficient between the height of father and son from the given data

Height of father (In inches) 64 65 66 67 68 69 70


Height of son (Inches) 66 67 65 68 70 68 72

Answer:
Calculation for Correlation Coefficient

 Sl. No. X Y X−X Y −Y ( X − X ) (Y − Y ) ( X − X )2 (Y − Y )2


= X – 67 = Y – 68
 1 64 66 -3 -2 6 9 4
2 65 67 -2 -1 2 4 1
3 66 65 -1 -3 3 1 9
4 67 68 0 0 0 0 0
 5 68 70 1 2 2 1 4
 6 69 68 2 0 0 4 0
 7 70 72 3 4 12 9 16
Total 469 476  – – 25 28 34

ΣX 469 ΣX 476
Mean of X = X = = = 67 and Mean of Y = Y = = = 68 [n = No. of pairs of observation = 7]
n 7 n 7

Σ {( X − X )(Y − Y )} 25
r= = = 0.81
Σ( X − X ) 2
Σ(Y − Y ) 2
(28 × 34)

Using the other formula we can also calculate r as below:


25 25
Σ ( X − X )(Y − Y ) = 2 2
=
28 34
= 0.81
r= ( X − X ) (Y − Y ) 7
N σ xσ y 7 7 7
N N

Alternative Method:
Calculation for Correlation Coefficient

 Sl. No. X Y XY X2 Y2
 1 64 66 4224 4096 4356
2 65 67 4355 4225 4489
3 66 65 4290 4356 4225
4 67 68 4556 4489 4624
 5 68 70 4760 4624 4900
 6 69 68 4692 4761 4624
 7 70 72 5040 4900 5184
Total 469 476 31917 31451 32402

352 The Institute of Cost Accountants of India


Correlation and Regression

ΣXY −
( ΣX )( ΣY ) 31917 −
( 469)( 476)
r= N = 7
 2 ( Σ X )2   2 ( Σ Y )2   ( )
469
2
  ( 476)2 
 ΣX − N   ΣY − N  31451 − 7  32402 − 7 
      

25
= = 0.81
28 34

The sign of r indicate the direction of the relationship between the two variables X and Y. If an inverse relationship
exists – that is, if Y decreases as X increases – then r will fall between 0 and -1. Likewise if there is a direct
relationship i.e. if Y increases with increase in X, then r will fall between 0 and 1.

Illustration 2. From the following data compute the co-efficient of correlation between X and Y:

X series Y Series
No. of items 15 15
Arithmetic Mean 25 18
Square of deviations from mean 136 138

Summation of product of deviations of X and Y series from their respective Arithmetic Mean is 122.

Answer:

r=
Σ {( X − X )(Y − Y )} =
122
= 0.89
Σ( X − X ) 2
Σ (Y − Y ) 2
136 × 138

Illustration 3. The following table gives the soil temperature and the germination time at various places.Calculate
the co-efficient of correlation and interpret the value

Temperature 57 42 40 38 42 45 42 44 40 46 44 43
Germination Time 10 26 30 41 29 27 27 19 18 19 31 29

Take 44 and 26 as assumed means for Temperature and Germination Time respectively.

Answer:

Σdx.dy −
( Σdx )( Σdy )
N
Coefficient of Correlation = r =
 2 ( Σdx ) 2
  2 ( Σdy )2 
Σdx − N  Σdy − N 
   

The Institute of Cost Accountants of India 353


Fundamentals of Business Mathematics and Statistics

Detail computations are


Calculations for Correlation Coefficient

Temperature (X) Germination (Y) dx = X-44 dy = Y-26 dx dy dx2 dy2


57 10 13 -16 -208 169 256
42 26 -2 0 0 4 0
40 30 -4 4 -16 16 16
38 41 -6 15 -90 36 225
42 29 -2 3 -6 4 9
45 27 1 1 1 1 1
42 27 -2 1 -2 4 1
44 19 0 -7 0 0 49
40 18 -4 -8 32 16 64
46 19 2 -7 -14 4 49
44 31 0 5 0 0 25
43 29 -1 3 -3 1 9
Total  – -5 -6 -306 255 704

Here N = No. of pairs of observation = 12

(Σ dx) (Σ dy) (−5) (−6)


Σ dx.dy − (−306) −
r= N = 12 = −0.732
2
 2 (Σ dx)   2 (Σ dy) 2   2
(−5)   (−6) 2 
Σ dx − N  Σ dy − N  255 − 12  704 − 12 
     

Illustration 4. Given
No. of pairs of observations for X and Y series = 8
X series Arithmetic average = 74.5
X series assumed average = 69
X series standard deviation =13.07
Y series arithmetic average = 125.5
Y series assumed average = 112
Y series standard deviation= 15.85
Summation of product of corresponding deviations of X and Y series = 2176
Calculate coefficient of correlation.

354 The Institute of Cost Accountants of India


Correlation and Regression

Answer:
Σdx.dy − N ( X − AX )(Y − AY )
r=
Nσ X σ Y

So r = 2176 − 8 × ( 74.5 − 69) × (125.5 − 112) = 0.955


8 × 13.07 × 15.85

Properties of Karl Pearson’s coefficient:


The following are the main properties of Karl Pearson’s coefficient of correlation.
1. In case correlation is present, then coefficient of correlation would lie between ±1 i.e. –1 ≤ r ≤ 1. If correlation
is absent, then it is denoted by zero.
2. Coefficient of correlation is based on a suitable measure of variation as it takes into account all items of the
variable.
3. Coefficient of correlation measures both the direction as well as degree of change in the dependent variable.
4. If there is accidental correlation, in that case the coefficient of correlation might lead to fallacious conclusions.
It is known as non-sense or spurious correlation.
5. The coefficient of correlation does not prove causation but it is simply a measure of co-variation. It happens
because variations in X and Y series may be due to
(i) Some common cause,
(ii) Some mutual dependence,
(iii) Some change and
(iv) Some causation of the subject to be relative
6. It is independent of changes of scale and origin of the variables X and Y.
7. Coefficient of correlation is the geometric mean of two regression coefficients. Symbolically

r = ± byxbxy
8. Coefficient of correlation is independent of the unit of measurement
9. Coefficient of correlation works both ways i.e., rxy = ryx
10. If the value of x and y are linearly related with each other i.e., if we have the relation between x and y as
y = ax + b, the correlation coefficient between x and y will be positive and if the relation between x and y be
y  = – ax + b, then ‘r’ will be negative, ‘a’ being a negative constant.

Coefficient of Correlation and Probable Error:


Probable error is an instrument which measures the reliability and dependability of the value of ‘r’, the Karl
Pearson’s coefficient of correlation. The probable error of coefficient of correlation helps in interpreting its value.
The coefficient of correlation is generally computed from samples, which aresubject to errors of sampling. From
the interpretation point of view Probable Error is very useful.
According to Horace Secrist. “The probable error of ‘r’ is an amount which if added to and subtracted from
the average correlation coefficient, produces amounts within which the chances are even that a coefficient of
correlation from a series selected at random will fall”.

The Institute of Cost Accountants of India 355


Fundamentals of Business Mathematics and Statistics

According to Wheldon, “probable error defines the limit above and below the size of the coefficient determined
within which there is an equal chance that the coefficient of correlation similarly calculated from other samples
will fall”
Karl Person’s Probable error is calculated by using the following formula
1− r2
P.E = 0.6745
N
Where P.E. = Probable Error
r = Co-efficient of correlation
N = Number of pairs of observations
This is used in interpreting whether ‘r’ is significant or not. If ‘r’ is more than P.E. then there is correlation and it
is significant if ‘r’ is more than six times P.E.

Utility or Importance of Probable Error:


Following are the main points of utility of probable error:
(1) Determination of Limits: The Probable error of coefficient of correlation determines the two limits (r ± P.E.)
within which, coefficients of correlation of randomly selected samples from the same universe will fall
(2) Interpretation of ‘r’
The interpretation of r based on the probable error is as follows:
(i) If ‘r’ is less than the probable error (i.e. r < P.E.), then there is no evidence of correlation in the two
variables i.e., correlation is insignificant.
(ii) If ‘r’ is greater than probable error (i.e. r > P.E.), then the correlation is significant. If r is not more than
six times the probable error (i.e. r >/ 6 P.E.), then it is not significant.
(iii) If the probable error is small, correlation is definitely existing when r > 0.5
(iv) If r > 0.3 and probable error is relatively small, ‘r’ is not treated as marked
(v) In other cases noting can be concluded with certainty

Illustration 5: Coefficient of correlation between advertising expenditure and sales for 9 items was observed as
+ 0.69. Find out probable error of correlation coefficient and comment on the significance of r.
Answer:
1− r2 1 − 0.692
P.E = 0.6745 = 0.6745 × = 0.118
N 9
r = 0.69 < 6 × 0.118 = 0.708
So, we can say that r is less than 6 times the value of probable error. Hence the value of r is not significant.

Illustration 6: To study the correlation between the weights and heights of the students of a college, a sample of
100 is taken from the universe. The sample study gives the coefficient of correlation between two variables as 0.9.
Within what limits does it hold good for the universe.

356 The Institute of Cost Accountants of India


Correlation and Regression

Answer:
1− r2 1 − 0.92
Here r = 0.9, P.E = 0.6745 = 0.6745 × = 0.013
N 100
Upper limit = r + P.E = 0.9 + 0.013 = 0.913
Lower limit = r – P.E = 0.9 – 0.013 = 0.887

Rank Correlation (Spearman’s Method)


In 1904, Prof. Charles Edward Spearman had devised a method of computing coefficient of correlation.It is based
on the ranking of different items in the variable. This method is useful where actual item values are not given,
simply their ranks in the series are known. Thus it is a good measure in cases where abstract quantity of one group
is correlated with that of the other group. In Spearman’s coefficient of correlation, we take the differences in ranks,
squaring them and finding out the aggregate of the squared differences.
Symbolically

6ΣD 2 ... ... ... ... (6)


rk = 1 −
N ( N 2 − 1)

Here rk = Coefficient of rank correlation


D = Rank differences
N = Number of Pairs

The value of rank correlation falls between ± 1. i.e. –1 ≤ rk ≤ 1


In rank coefficient of correlation, we shall study three different cases
Case I When ranks against the actual observations are not given
Case II When ranks are given and actual observations may or may not be given
Case III When ranks are equal i.e. against some of the observations of one variable or both the variables have
observations with same rank.

Illustration 7. Compute Rank correlation coefficient from the data of the following table

X 415 434 420 430 424 428


Y 330 332 328 321 327 325

Answer:
This is a problem related to Case I mentioned above.
We will use the following formula
6ΣD 2
rk = 1 −
N ( N 2 − 1)

The Institute of Cost Accountants of India 357


Fundamentals of Business Mathematics and Statistics

Detail computations are follows:


Calculations for Rank Correlation Coefficient

X Rank (R1) Y Rank (R2) D = R1 – R2 D2


415 6 330 3 3 9
434 1 332 1 0 0
420 5 328 4 1 1
430 2 331 2 0 0
424 4 327 5 -1 1
428 3 325 6 -3 9
Total – – – – 20
Here N = No. of pairs of observatins = 6
So
6ΣD 2 6 × 20
rk = 1 − = 1− = 1 − 0.571 = 0.429
N ( N 2 − 1) 6 × (36 − 1)

Illustration 8. The ranks of students in Hindi (R1) and Economics (R2) are given

Hindi (R1) 6 1 5 2 4 3
Economics (R2) 3 1 4 2 5 6

Answer:
This is a problem related to of case II mentioned above
We will use the formula
6ΣD 2
rk = 1 −
N ( N 2 − 1)
Detail computations are as follows:

Calculations for Rank Correlation Coefficient

Rank (R1) Rank (R2) D = R1 – R2 D2


6 3 3 9
1 1 0 0
5 4 1 1
2 2 0 0
4 5 -1 1
3 6 -3 9
Total – – 20
Here N = No. of pairs of observatins = 6

358 The Institute of Cost Accountants of India


Correlation and Regression

So
6ΣD 2 6 × 20
rk = 1 − = 1− = 1 − 0.571 = 0.429
N ( N 2 − 1) 6 × (36 − 1)

Case III: When ranks are equal (Tied or repeated Ranks)


In certain cases, we may find equal ranks for two or more than two values. In that case, each individual item is
given an average rank. If two individual items are ranked equal at third place, they are each given the rank (3+4)/ 2
= 3.5, while if three are ranked at third place then (3+4+5)/3 = 4 would be the common rank for third, fourth and
fifth places.
When equal ranks are given to some entries, some adjustment in the above formula of Rank Correlation has to be
made and the modified formula is:

6  ΣD 2 +

1
12{( ) ( ) ( 
m13 − m1 + m23 − m2 +  + mn3 − mn  )}
 ... ... ... ... ... (7)
rk = 1 −
N3 − N
Where m1 = 1st Set of Number of items which have common rank, m2 = 2nd set of Number of items which have
common rank .... mn = nth set of Number of items which have common rank.

Illustration 9: Eight students have obtained the following marks in Accountancy and Economics. Calculate the
Rank Co-efficient of correlation.

Accountancy (X) 25 30 38 22 50 70 30 90
Economics (Y) 50 40 60 40 30 20 40 70

Answer:
We will use the formula

6  ΣD 2 +
1
12 {( ) ( ) 
m13 − m1 + m23 − m2 +  + mn3 − mn ( )}
rk = 1 −  
N3 − N
Where N = No. of pairs of observation = 8

Detail computations are as follows:

Calculations for Rank Correlation Coefficient


Accountancy (X) Rank (R1) Economics (Y) Rank (R2) D = R1 – R2 D2
25 7 50 3 4 16
30 5.5 40 5 0.5 0.25
38 4 60 2 2 4
22 8 40 5 3 9
50 3 30 7 -4 16
70 2 20 8 -6 36
30 5.5 40 5 0.5 0.25
90 1 70 1 0 0
Total – – – – 81.5

The Institute of Cost Accountants of India 359


Fundamentals of Business Mathematics and Statistics

[Note: While ranking the marks of Accountancy it is seen that there is a tie of 2 students securing the same marks
5+ 6
30 to get placed at the 5th spot. Thus they have given an average rank of = 5.5
2
Similarly in ranking the marks of Economics there in a tie of 3 students securing the marks 40 each to get placed at
4+5+ 6
the 4th spot. Hence they have given an average rank of = 5. Also m1 = 2 and m2 = 3
3
So

 1 3 1 3   1 3 1 3 
6 ΣD 2 +
 12 (
m1 − m1 +)12 (
m2 − m2 

) 6 81.5 +
 12 (
2 −2 +
12 )
3 −3  (
=0
)
rk = 1 − = 1−
N3 − N 83 − 8

Coefficient of Correlation (Concurrent Deviation Method)


Under concurrent deviation method only the direction of change in the concerned variables is taken into account.
For each term the change is considered with reference to the previous value, which may be either in plus (+) or in
minus (–).
Plus (+) is used for showing increase in value & minus (–) is used for showing decrease.
Formula used is

r=± {(
±
2C − N
N )}
Where
r = Correlation coefficient
C = no of concurrent deviations
N = Number of pairs of deviations compared

The meaning of ± sign both inside and outside the square root is that the value of (2C − N
N )would be positive when
both the signs are positive and it would be negative when both the signs are negative. This is significant in case
the value is negative, because under root of negative values cannot be calculated and therefore if the value of

( C N
) is negative it is multiplied by the negative (–) sign inside the square root to make it positive. In such case
the sign outside will also be minus (–).

360 The Institute of Cost Accountants of India


Correlation and Regression

Illustration 10. Compute the coefficient of correlation for the following data using Concurrent Deviation Method.

Month Output of Steel (in 000 tons) Unemployed in steel (in 000)
January 8.5 60
February 9.2 65
March 9.3 61
April 8.5 74
May 7.2 92
June 5.9 157
July 5.1 130
August 6.6 106
September 7.9 58
October 7.6 80
November 8.2 52
December 9.2 45
Answer:
Detail computations are
Deviations Deviations Non
from from
Month Output preceding Unemployed preceding Concurrencies Concurrencies
month month
January 8.5 60
February 9.2 + 65 + +
March 9.3 + 61 - -
April 8.5 - 74 + -
May 7.2 - 92 + -
June 5.9 - 157 + -
July 5.1 - 130 - +
August 6.6 + 106 - -
September 7.9 + 58 - -
October 7.6 - 80 + -
November 8.2 + 52 - -
December 9.2 + 45 - -
Total 2 9
Concurrencies mean when signs are square i.e. Case 1: deviation of output from preceding month is positive and
deviation of unemployed from preceding month is also positive (February), Case 2: deviation of output from
preceding month is negative and deviation of unemployed from preceding month is also negative (July). In other
cases non-concurrencies are there. So here C = 2 & N = 11.
So

r=± {(
±
2C − N
N )} { (
=− −
2 × 2 − 11
11 )} = − 0.798

The Institute of Cost Accountants of India 361


Regression Analysis 6.3
REGRESSION ANALYSIS
Professional Business decisions are based on predictions of future events. To have this predictions, mangers got to
rely on the relationship between what is already known and what is to be estimated. Knowledge on Regression and
Correlation helps managers in this regard. i.e.

What has actually happened? What is to be estimated?

1. Is there any relationship between the green and black box?


2. If there is relationship what is that?
3. How much is the strength of the relationship?

Knowledge of Regression & Correlation answers these questions.

Knowledge of Regression & Correlation answers these questions.


Through regression analysis we, business managers, predict variable represented in the black box, called dependent
variable on the basis of another variable represented in the green box, called independent variable.
The statistical technique of estimating or predicting the unknown value of a dependent variable from the known
value of an independent variable is called Regression Analysis.
“Regression is the measure of the average relationship between two or more variables in terms of the original units
of the data.” – is the definition of Regression according to  Morris M. Blair.

362 The Institute of Cost Accountants of India


Correlation and Regression

Sir Francis Galton introduced the concept of ‘Regression’ for the first time in 1877 when he studied the relationship
between the heights of fathers and sons.

Objectives of Regression analysis:


Regression analysis does the following
1. Explaing the variations in the dependent variable as a result of using a number of independent variables.
2. Describes the nature of relationship in a precise manner by means of regression equation.
3. It is used in prediction and forecasting problems
4. It helps in removing unwanted factors.

Basically in regression analysis we develop “Regression Equation” which relates independent variables with
dependent variables.

Classification of Regression Analysis


The regression analysis can be classified on the following basis: -
(i) Number of variables
(ii) Change in Proportion

(i) On the basis of Number of Variables: On the basis of number of variables regression analysis can be classified
as –
1. Simple regression
2. Partial Regression
3. Multiple Regression
If Regression equation relates one dependent variable with one independent variable, then Regression is called
Simple Regression.
When more than two variables are studied in a functional relationship but the relationship of only two variables
is analysed at a time, keeping other variables as constant,such a regression analysis is called Partial Regression.
If Regression equation relates one dependent variable with more than one independent variable, then Regression
is called Multiple Regression.

(ii) On the basis of Change in Proportions: On the basis of proportions the regression can be classified into the
following categories:
1. Linear regression
2. Non-linear regression
Relationship between variable can be of different types:

The Institute of Cost Accountants of India 363


Fundamentals of Business Mathematics and Statistics

Direct Linear
25

20

15
Dependent variable

10

0
0 2 4 6 8 10 12
Independent variable

Inverse Linear
35

30

25

20
Dependent variable

15

10

0
0 2 4 6 8 10 12
Independent variable

364 The Institute of Cost Accountants of India


Correlation and Regression

Direct Curvilinear
250

200
Dependent variable

150

100

50

0
0 2 4 6 8 10 12
Independent variable

Inverse Curvilinear
140

120

100
Dependent variable

80

60

40

20

0
0 2 4 6 8 10 12
Independent variable

Relationship observed by regression is only the relationship of association of variables but not necessarily of cause
and effect. Sale of Petrol may have direct relationship with the sale of petrol cars but positive growth of economy
actually causes increase in demand of petrol and car.
Linear Regression line is the equation that linearly relates the two variables mathematically. Regression line
showing linear relationship between two variables is of the form:

The Institute of Cost Accountants of India 365


Fundamentals of Business Mathematics and Statistics

Y = a+b X Independent Variable

Dependent Variable

a is called Y axis intercept or Y-intercept or simply intercept, b is the slope of the line.
A regression line is a graphic technique to show the functional relationship between the two variables X and Y. i.e.
dependent and independent variables. It is a line which shows average relationship between two variables X and
Y. Thus, this is a line of average. This is also called an estimating line as it gives the average estimated value of
dependent variable (Y) for any given value of independent variable (X).
The regression lines can be drawn by two methods as given below: -
1. Free Hand Curve Method
2. The method of Least Squares
1. Free Hand Curve Method: This method is also known as the method of Scatter Diagram. This is a very
simple method of constructing regression lines. At the same time it is a crude and very rough and rarely used
method of drawing regression lines. In this method, the value of paired observations of the variable are plotted
on the graph paper. It takes the shape of a diagram scattered over the graphic range of X axis and Y axis. The
independent variable is taken on the horizontal axis.
A straight line is drawn through the scattered points on the graph such that it conforms to the following requisites:
(a) It is as maximum as possible nearer to all the points on the graph
(b) It is as equidistant from of all the points on either sides of the line
(c) It passes through the centre of the scattered points.

Illustration 11. From the following data relating to X and Y, draw a regression line of Y on X by free hand curve
method.
X: 10, 16, 24, 36, 48; Y: 20, 12, 32, 40,55;
Answer:

Scatter Diagram
60

50

40

30
Y

Free Hand line


20

10

0
0 10 20 30 40 50 60
X

366 The Institute of Cost Accountants of India


Correlation and Regression

This method being most subjective is not commonly used in practice.

2. The method of Least Squares


The other method of drawing a line of regression is the method of least squares According to the least squares
method the line should be drawn through the plotted points in such a way that the sum of the squares of the
deviations of the actual Y values from the computed ‘Y’ values is the minimum or the least. The line which fits the
points in the best manner should have Σ (Y–Yc)2 as minimum. A line fitted by this method is called the line of best
fit. One of the characteristics of the line of best fit is that deviations above the line are equal to the deviations below
the line. It implies that the total of the positive and negative deviations is Zero, i.e. Σ(Y–Yc) = 0. The line of best
fit or the straight line passes through the overall mean of the data i.e. X , Y .
In the above diagram actual observation of Y for different values of X are plotted and represented by scattered
points. The linear regression line which we will form through least square method will follow the equation
Y = a + b X as already mentained. On development of such a line, if we put different values of X given in the
previous illustration we will get different values of Y. These value of Y will be called Computed Y values or in
short Yc. There could be differences between observed value Y and computed value . The difference could be
positive or negative. i.e. Y – Yc > 0 Or Y – Yc < 0. Through least square method we actually want to make square of
deviations i.e. square of (Y – Yc) as minimum or least.
For forming regression line of the form Y = a + b X, a and b are found out by the method of least squares and the
required formulae are:
ΣXY − n X Y
b= &a= Y –b X
ΣX 2 − nX 2

X & Y are the mean of respective series.

Illustration 12. Using the data in the table below:

X 20 11 15 10 19 21
Y 5 15 14 17 10 11
(i) Fit the regression line;
(ii) Predict Y for X = 12;
Answer:
Calculations for Regression Line
Sl. No. X Y XY X2
1 20 5 100 400
2 11 15 165 121
3 15 14 210 225
4 10 17 170 100
5 19 10 190 361
6 21 11 231 441
Total 96 72 1066 1648
Mean of X = ΣX/n = 96/6 = 16 and Mean of Y = ΣY/n = 72/6 = 12

The Institute of Cost Accountants of India 367


Fundamentals of Business Mathematics and Statistics

X 2 = (16)2 = 256
ΣXY − n X Y
So b = = (1066 – 6 × 16 × 12)/ (1648 – 6 × 256)
ΣX 2 − n X 2
= (–) 86/112 = (–) 0.768

So a = Y − b X =12 – (–) 0.768 × 16 = 24.288


So the regression line is Y = 24.288 – 0.768 X

Alternative Method of finding ‘a’ and ‘b’


Let the equation of the Regression line be
Y=a+bX
Normal equations are

ΣY = a ⋅ N + b ⋅ ΣX ... (1)
2
and ΣXY = aΣX + bΣX ... (2)

Putting the values in (1) and (2) we have

72 = a.6 + b.96 or, a +16b = 12 ..... (3) [N = No. of pairs of observation = 6]

and 1066 = a.96 + b.1648

or, 1066 = (12 – 16b) 96 + 1648b [As from (3), a = 12 – 16b]

or, 1066 = 1152 – 1536b + 1648b or, –8 = 112b or, b = – 0.768

∴ a = 12 – 16b = 12 – 16 × (– 0.768) = 24.288

If X = 12 then Y = 24.288 – 0.768 × 12 = 15.072


This way of finding regression equation is called forming Regression Equations through Normal Equations.
Basically the value of a & b above are found by simultaneously solving the following two normal equations:
ΣY = Na + bΣX ... ... ... ... (i)

ΣXY = aΣX + bΣX 2 ... ... ... ... (ii)

Equation (i) and (ii) are called normal equations

There are two types of Regression equations:


(i) Regression equation of Y on X, where Y is the dependent variable and X is the independent one. Equation is
of the form Y = a + b X discussed above.
(ii) Regression equation of X on Y, where X is the dependent variable and Y is the independent one. Equation is
of the form X = a + bY

368 The Institute of Cost Accountants of India


Correlation and Regression

For the first case (already discussed) and the second case the related formulae are given below for convenience:

Y = a + bX X = a + bY
ΣXY − nX Y ΣXY − n X Y
byx = bxy =
ΣX 2 − nX 2 ΣY 2 − nY 2

a = Y − bX a = X − bY

The two normal equations for regression line X on Y are:


ΣX = Na + bΣY ... ... ... ... (i)
ΣXY = aΣY + bΣY 2 ... ... ... ... (ii)

Regression equations can be written with the help of Regression Coefficients also:

byx = Regression coefficient Y on X bxy = Regression coefficient X on Y

Form of Regression line (Y on X) from Form of Regression line (X on Y) from


Regression Coefficient: Regression Coefficient:
Y − Y = byx ( X − X ) X − X = byx (Y − Y )

Σ ( X − X )(Y − Y ) Σ ( X − X )(Y − Y )
byx = bxy =
Σ(X − X ) Σ (Y − Y )
2 2

Illustration 13. From the following data obtain the two regression equations:

Sales 91 97 108 121 67 124 51 73 111 57


Purchase 71 75 69 97 70 91 39 61 80 47

Answer:

Calculations for Regression Coefficients

Sales (X) (X − X ) ( X − X )2 Purchase (Y) (Y − Y ) (Y − Y ) 2 ( X − X ) (Y − Y )


91 1 1 71 1 1 1
97 7 49 75 5 25 35
108 18 324 69 -1 1 18
121 31 961 97 27 729 837
67 -23 529 70 0 0 0
124 34 1156 91 21 441 714
51 -39 1521 39 -31 961 1209
73 -17 289 61 -9 81 153
111 21 441 80 10 100 210
57 -33 1089 47 -23 529 759
Σ 0 6360 0 2868 3900

The Institute of Cost Accountants of India 369


Fundamentals of Business Mathematics and Statistics

ΣX ΣX
X= = 900/10 = 90, X = = 700/10 = 70
10 10
Σ ( X − X )(Y − Y )
So byx = = 3900/6360 = 0.613
Σ(X − X )
2

Σ ( X − X )(Y − Y )
So bxy = = 3900/2868 = 1.36
Σ (Y − Y )
2

So Regression equation Y on X is:


(i) Y − Y = byx ( X − X )
Or, Y – 70 = 0.613 (X – 90)
Or, Y = 0.613X + 14.83

So Regression equation X on Y is:


(ii) X − X = byx (Y − Y )
Or, X – 90 = 1.36 (Y – 70)
Or, X = 1.36 Y – 5.2

Some important relations:


σ
1. Regression coefficient of x on y = bxy = r ⋅ x , whether r = Correlation Coefficient, σx = S.d of x and σy = S.d
σy
of y
σy
2. Regression coefficient of y on x = byx = r , where the symbols have same meaning as above
σx
3. From (1) and (2) above it can be written that r2 = byx . bxy or, r = ± bxy ⋅ bxy

4. From (3) above, conclusion can be drawn that Coefficient of determination (r2) is the product of two regression
coefficients.

5. Two Regression lines interest at the point ( x , y )

6. To get Regression Coefficient from a given Regression equation the procedure is –

(i) If the given Regression Equation is for Y on X then convert it to the form Y = a + bx and ‘b’ i.e. the
coefficient of X is the Regression Coefficient Y on X or byx. In other words byx = b

(ii) If the given Regression Equation is for X on Y then convert it to the form X = a′ + b′ Y and ‘ b′ ’ i.e. the
coefficient of Y is the Regression Coefficient X on Y or bxy. Thus bxy = b′ in this case.

Illustration 14. Given the bivariate data:

X 1 5 3 2 1 2 7 3
Y 6 1 0 0 1 2 1 5
Find regression equations by taking deviations of items from the means of X and Y respectively

370 The Institute of Cost Accountants of India


Correlation and Regression

Answer:
Regression equations are:

Y − Y = byx ( X − X ) or, y = byx ⋅ x

X − X = bxy (Y − Y ) or, y = bxy ⋅ y


Here x = X − X and y = Y − Y
Σ ( X − X )(Y − Y ) Σxy
byx = = .... (1)
Σ( X − X ) Σx 2
2

Σ ( X − X )(Y − Y ) Σxy
bxy = = .... (2)
Σ (Y − Y ) Σy 2
2

Detail computations are as follows:


Calculations for Regression Coefficients

Sl. No. X Y x = X − X = X −3 y = Y −Y = Y − 2 x2 y2 xy
1 1 6 -2 4 4 16 -8
2 5 1 2 -1 4 1 -2
3 3 0 0 -2 0 4 0
4 2 0 -1 -2 1 4 2
5 1 1 -2 -1 4 1 2
6 2 2 -1 0 1 0 0
7 7 1 4 -1 16 1 -4
8 3 5 0 3 0 9 0
Total 24 16 – – 30 36 -10

ΣX 24 ΣY 16
A. M of X = X = = = 3 and A.M of Y = Y = = = 2 [n = No. of pairs of observation = 8]
n 8 n 8
−10 −1 −10 −5
So byx = = & bxy = = [Puting the values in (1) and (2)
30 3 36 18
−1
So Y − Y = byx ( X − X ) Or, Y − 2 = × ( X − 3) Or X + 3Y − 9 = 0
3
and
5
X − X = bxy (Y − Y ) Or, X − 3 = (Y − 2) Or 18 X + 5Y − 64 = 0
18

Alternatively:

Σ ( X − X )(Y − Y ) −10
r= = = −0.304
Σ (X − X ) 2
Σ (Y − Y ) 2
30 × 36

The Institute of Cost Accountants of India 371


Fundamentals of Business Mathematics and Statistics

30 36
σx = = 1.94 and σ y = = 2.12
8 8

Regression equation of X on Y:
1.94
X − X = bxy (Y − Y ) = X − 3 = −0.304 × (Y − 2)
2.12
1.94
Or, X − 3 = −0.304 ×
2.12
(Y − 2) = −0.277 (Y − 2)
Or, X – 3 = – 0.277Y + 0.556
Or, X + 0.277Y – 3.556 = 0

Regression equation of Y on X:
2.12
Y − Y = byx ( X − X ) = Y − 2 = −0.304 × ( X − 3)
1.94
2.12
Or, Y − 2 = −0.304 ×
1.94
( X − 3) = −0.333 ( X − 3)
Or, Y – 2 = – 0.333X + 0.999
Or, 0.333X + Y – 2.999 = 0

The Standard error of Estimate:


To measure the reliability of the regression equation (i.e. degree of scattering of observed values of dependent
variable around the computed value of dependent variable from the regression line i.e. (Y – Yc) a measuring
parameter named Standard error of Estimate is to be used. The formula for finding

ΣY 2 − a ΣY − bΣXY
Se =
n−2
Larger the Standard error of estimate, the greater the scattering of points around the regression line. If Se= 0 then
all the given values of Y will lie exactly on the regression line for the given values of X.
For the observed points assuming normally distributed around the regression line we can expect
(i) 68% of the points within ±1Se ;
(ii) 95.5% of the points within ± 2 Se ;
(iii) 99.7% of the points within ± 3Se ;

Illustration 15: A study by the University, Department of Transportation on the effect of bus ticket prices upon
the number of passengers produced the following results:

Ticket price (cents) 15 20 25 30 40 50


No. of Passengers per 100 Miles 440 430 450 370 340 370
i) Develop the regression equation;
ii) Determine Standard Error of Estimate;
iii) Interpret the result;

372 The Institute of Cost Accountants of India


Correlation and Regression

Answer:
Let the Regression Equation be Y = a + bX
Calculations for Regression Equation

Sl. Ticket Price (cents) No. of Passenger/100 miles


XY X2 Y2
No. (X) (Y)
1 15 440 6600 225 193600
2 20 430 8600 400 184900
3 25 450 11250 625 202500
4 30 370 11100 900 136900
5 40 340 13600 1600 115600
6 50 370 18500 2500 136900
Total 180 2400 69650 6250 970400
ΣX 180 ΣY 2400
A. M of X = X = = = 30 and A.M of Y = Y = = = 400 [n = No. of pairs of observation = 6]
n 6 n 6
ΣXY − nXY
So b = = (69650 – 6 × 30 × 400)/ (6250 – 6 × 900)
ΣX 2 − nX 2
= – 2.765

So a = Y − b X = 400 – (– 2.765) × 30 = 482.95

Required Regression equation is Y = 482.95 – 2.765X

So Se = ΣY 2 − a ΣY − bΣXY
n−2

970400 − 482.95 × 2400 − (−2.765) ⋅ 69650


= = 31.234
6−2

Interpretation:
(i) 68% of the points are within ± Se i.e. ± 31.234;
(ii) 95.5% of the points are within ± 2Se i.e. ± 62.468;
(iii) 99.7% of the points are within ± 3Se i.e. ± 93.702

Y = a + bX X = a + bY

ΣY 2 − a ΣY − bΣXY ΣX 2 − a ΣX − bΣXY
Se = Se =
n−2 n−2

Illustration 16. For certain X and Y series which are correlated, the two lines of regression are:
5X – 6Y + 90 = 0 and 15X – 8Y – 130 = 0. Find the means of the two series and the correlation coefficient between
them.

The Institute of Cost Accountants of India 373


Fundamentals of Business Mathematics and Statistics

Answer: The two regression line intersects each other at the point ( X , Y ).
1
From first equation we have X = 5 ( 6Y − 90) = 1.2Y − 18
Putting this value in 2nd equation we have 15 (1.2Y − 18) − 8Y − 130 = 0
Or 18Y – 270 – 8Y – 130 = 0
Or 10Y = 400
Or Y = 40
So X = 1.2 × 40 – 18 = 30
Therefore Means of the two series are X = 30, Y = 40

Let us suppose 1st equation is the equation of line of regression of Y on X and the 2nd equation is that of X on Y
So from 1st equation we can write 6Y = 5X + 90

Or, Y = 5 X + 90
6 6
5
So bYX = Coefficient of X in the Regression Equation Y on X =
6
Similarly from 2nd equation we can write 15X = 8Y + 130

Or, X = 8 Y + 130
15 15
8
So bXY = Coefficient of Y in the Regression Equation X on Y =
15
2
Therefore r 2 = bYX .bXY = 5 . 8 = 40 or, r = ±
6 15 90 3

Since both the regression coefficients are positive, r must be positive and we take r = 2/3
[Note: If we had assumed that the 1st equation is the regression equation of X on Y and 2nd equation is the regression
6 15
equation of Y on X we get bXY = & bYX = 8
5
6 15 90 3
Therefore r = bYX .bXY = 5 . 8 = 40 Or, r = 2 > 1 which not a feasible solution]
2

Illustration 17. The regression equation of profit (X) on sales (Y) of a certain firm is 3Y – 5X + 108 = 0. The
average sales of the firm was `44,000 and the variance of profits is 9/16thof the variance of sales. Find the average
profit and the coefficient of correlation between sales and profit.
Answer:
The regression equation of profit (X) on sales (Y) of the firm is 3Y – 5X + 108 = 0.
Average sales Y = 44,000
The average profit can be obtained by putting Y = 44000 in the regression equation of X on Y (as X , Y satisfy both
the regression equations)
We get 3 × 44000 – 5 X + 108 = 0, Or 132000 + 108 = 5X

374 The Institute of Cost Accountants of India


Correlation and Regression

132108
Or, X = = 26421.6. So required value of Average Profit = X = ` 26421.60
5
2 9 2 σ 3
By question σ x = 16 σ y or x =
σy 4
3 108
From the equation 3Y – 5X + 108 = 0 we can write X = 5 Y + 5

bxy = Coefficient of Y in the Regression equation X on Y


3 σ
= =r x
5 σy
3 4 4
So r = . = = 0.8
5 3 5

Illustration 18. While calculating the coefficient of correlation between two variable X and Y the following results
were obtained:
The number of observations N = 25, ΣX = 125, ΣY = 100, ΣX2 = 650, ΣY2 = 460, ΣXY = 508. It was however later
discovered at the time of checking that two pairs of observations (X,Y) were copied as (6,14) and (8,6) while the
correct values were (8,12) and (6,8) respectively. Find the correct value of the coefficient of correlation.

Answer:
Corrected ΣX = 125 – 6 – 8 + 8 + 6 = 125

Corrected ΣY = 100 – 14 – 6 + 12 + 8 = 100

Corrected ΣX2 = 650 – 62 – 82 + 82 + 62 = 650

Corrected ΣY2 = 460 – 142 – 62 + 82 + 122 = 436

Corrected ΣXY = 508 – 6X14 – 8X6 + 8X12 + 6X8 = 520


Correct value of Coefficient of Correlation

(ΣXY )Cov − [(ΣX )Cov ⋅ (ΣY )Cov ] / N


= rCov =
 ( ΣX )2Cov   ( ΣY )2Cov 
( ΣX )Cov − ( ΣY )Cov −
2 2
N  N 
   

520 − [125 × 100] / 25


=
 (125) 2   (100) 2 
650 − 25  436 − 25 
  

520 − 500 20 20 20 2
= = = = =
{650 − 625}{436 − 400} 25 × 36 5 × 6 30 3

The Institute of Cost Accountants of India 375


Fundamentals of Business Mathematics and Statistics

Illustration 19. You are given the following data.

X Y
Arithmetic Mean 20 25
Standard Deviation 5 4
Coefficient of correlation between X and Y is 0.6. Form the regression equations of X on Y and Y on X

Answer:
We know that
r σy
Regression coefficient of Y on X = bXY = and
σx

r ρx
Regression coefficient of X on Y = bXY =
σy

0.6 × 4 0.6 × 5
So bXY = = 0.48 & bXY = = 0.75
5 4

Regression equation of Y on X : Y − Y = bYX ( X − X )


Y – 25 = 0.48 (X – 20)
Or, Y – 25 = 0.48X – 9.6
Or, Y = 0.48X + 15.4

Regression equation of X on Y: X − X = bXY (Y − Y )


X – 20 = 0.75 (Y – 25)
Or, X – 20 = 0.75Y – 18.75
Or, X = 0.75Y + 1.25

Illustration 20. Given the bivariate data:

X: 2, 6, 4, 3, 2, 2, 8, 4; Y: 7, 2, 1, 1, 2, 3, 2,6;

(a) Fit the regression line of Y on X and hence predict Y, if X = 20

(b) Fit the regression line of X on Y and hence predict X, if Y = 5.

Answer:
Regression equations are:

Y − Y = byx ( X − X )

X − X = bxy (Y − Y )

376 The Institute of Cost Accountants of India


Correlation and Regression

Σ( X − X )(Y − Y )
byx =
Σ( X − X ) 2

Σ( X − X )(Y − Y )
bxy =
Σ(Y − Y ) 2

Calculations for Regression Coefficients

Sl. No. X Y x = X = X − 3.875 y = Y −Y = Y − 3 xy x2 y2

1 2 7 -1.875 4 -7.5 3.515625 16


2 6 2 2.125 -1 -2.125 4.515625 1
3 4 1 0.125 -2 -0.25 0.015625 4
4 3 1 -0.875 -2 1.75 0.765625 4
5 2 2 -1.875 -1 1.875 3.515625 1
6 2 3 -1.875 0 0 3.515625 0
7 8 2 4.125 -1 -4.125 17.0156325 1
8 4 6 0.125 3 0.375 0.015625 9
Total 31 24 -10 32.875 36

ΣX 31 ΣY 24
Mean of X = X = = = 3.875 , Mean of Y = Y = = = 3 [ n = No. of pairs of observation = 8]
n 8 n 8

Σ ( X − X )(Y − Y ) 10
byx = =− = −0.304
Σ(X − X ) 32.875
2

Σ ( X − X )(Y − Y ) 10 5
bxy = =− = − = −0.278
Σ (Y − Y ) 36 18
2

So the Regression line Y on X is


Y − Y = byx ( X − X ) Or, Y – 3 = – 0.304 (X – 3.875) Or, 0.304X + Y – 4.178 = 0,

When X = 20 then 0.304 × 20 + Y – 4.178 = 0 Or, Y = – 1.902

So the Regression line X on Y is


X − X = bxy (Y − Y ) Or, X – 3.875 = – 0.278 (Y – 3) Or, X + 0.278Y – 4.709 = 0,

When Y = 5 than X + 0.278 × 5 – 4.709 = 0 Or, X = 3.319

The Institute of Cost Accountants of India 377


Fundamentals of Business Mathematics and Statistics

Properties of Regression Coefficient:


The main properties of regression coefficients are as under:
1. Both the regression coefficients bxy and byx cannot be greater than unity, i.e., either both are less than unity
or one of them must be less than unity. In other words, the square root of the product of two regression
coefficients must be less than or equal to 1 or – 1 or bxy × byx ≤ 1

2. Both the regression coefficients will have the same sign i.e.,

(i) If bxy is positive, then byx will also be positive.

(ii) If bxy is negative, then byx will also be negative.

(iii) Both bxy and byx must have same signs. If both are positive, r will be positive and vice-versa.

3. Correlation Co-efficient is the geometric mean between regression coefficients i.e., r = ± bxy × byx

bxy + byx
4. A.M of Regression Coefficients is greater than or equal to the Coefficient of correlation i.e. ≥r
2
5. Regression Coefficients are independent of change of origin but not of scale.

Illustration 21. Past data on household income and expenditure reveals that
(a) The average absolute increase in income in relation to increase in expenditure is `1.5 crore and
(b) The average absolute increase in expenditure in relation to increase in income in `50 crore
Find the coefficient of correlation between household income and expenditure.

Answer:
Let the Increase in Income = x
And the increase in expenditure = y.
The value of regression coefficient bxy = 1.5
Similarly the regression coefficient byx = 0.5

r = ± bxy × byx = + 1.5 × 0.5 = 0.75 = 0.866 [As both bxy and byx are positive, r is also positive]

Correlation and Regression:


Correlation and Regression analysis are two important statistical tools to study the functional relationship between
the variables. Coefficient of correlation is a measure of degree of covariance between x and y whereas the aim of
regression analysis is to study the nature of relationship between the variables, so that we may know the value of
one variable on the basis of another. However, we can study the relation and difference between the correlation
and regression as under.

Relation between Correlation and Regression:


The regression lines are used to predict the value of dependent variable given the different values of independent
variable. Two regression lines show the average relationship between two variables.

378 The Institute of Cost Accountants of India


Correlation and Regression

If there is perfect correlation (positive or negative, i.e., r ±1) both the lines will coincide. In this case there will be
one regression line.
In case r = 0, i.e., both the variables are independent, both the lines will interest each other at right angles i.e.,
parallel to the axes.
Higher the value of r, closer will be the regression lines to each other.
On the other hand if the value of r is low, the gap between regression lines will be wider.
It is important to note that the point, where both the regression lines intersect each other, gives us the mean values
of X and Y.

Difference between Correlation and Regression


The difference between correlation and regression can be analysed as under –
1. Nature of Relationship: Correlation analysis tests the closeness of the variables, whereas regression analysis
measures the extent of change in dependent (y) variable due to change in the independent (x) variable i.e., both
the nature and extent of functional relationship between the variables is studied.
2. Relationship: In regression analysis, the causal relationship in variables moving in the same or opposite
direction is studied while in correlation analysis, the study is made by taking into consideration the cause and
effect relationship between the variables. i.e. only closeness of the variables is studied.
3. Mutual dependence of variables: Correlation studies the mutual dependence of variables but in regression
analysis the functional relationship showing dependence of one variable upon other is analysed.
4. Spurious Correlation: In correlation chance of spurious correlation between two variables, having no
practical importance, may be observed, but there is no chance of existence of such type of relation in regression
analysis.
5. Mathematical treatment: Correlation is having no scope of further mathematical treatment where as
regression can be used for further treatment.
6. Origin & Scale: Regression analysis is independent of change of origin but not of scale, where as correlation
is both independent of change in origin and scale.
7. Relative and Absolute Measures: Regression analysis is an absolute measure showing the change in the
value of y or x for unit change in the value of x or y. whereas correlation coefficient is a relative measure of
linear relationship between x and y and is independent of the measurement. It is a number which lies between
+1 & -1.
8. Applicability: Correlation has very limited scope of application. It is limited to the linear relationship between
two variables, but the scope of applicability of regression analysis is very wide. It can be covered under linear
as well as non-linear relationship between the variables.
9. Differentiation in variables: In correlation both variables are considered at par for study purposes,whereas
in regression analysis variables are differentiated as dependent and independent variables.
10. Symmetrical or Asymmetrical formation: Correlation is symmetrical in formation, i.e. rxy = ryx . The
measurement of the co-variability of two variables is symmetrical in formation. It means ‘r’ is a both way
relationship of x on y or y on x, but in regression, the approach is one way of analysis treating one variable as
dependent and the other as independent, thus making the analysis Asymmetrical, that is bxy ≠ byx .

The Institute of Cost Accountants of India 379


Fundamentals of Business Mathematics and Statistics

EXERCISE - I
1. From the following table calculate the coefficient of correlation by Karl Pearson’s method and also fill the gap.

X 8 4 12 6 10
Y 11 13 ? 10 9
Arithmetic means of X and Y series are 8 and 10 respectively.

2. Given Number of pairs of observations of X and Y = 9


X Series Arithmetic average = 70.5,
X Series Standard deviation = 9.07,
X series Assumed average = 65.0,
Y Series Assumed average = 108.0
Y Series Arithmetic Average = 121.5
Y Series standard deviation = 11.85
Summation of product of corresponding deviation of X and Y series = 1451
Calculate coefficient of correlation.

3. Find the rank correlation for the following distribution

Marks in Economics 48 60 72 62 56 40 39 52 30
Marks in Accountancy 62 78 65 70 38 54 60 32 31

4. Calculate correlation coefficient from the following results:


N = 10, Σx = 100 Σy = 150, Σ(X–10)2 = 180, Σ(Y–15)2 = 215 Σ(X–10) (Y–15) = 60

5. If the covariance between X and Y variables is 10 and the variance of X and Y are respectively 16and 9, find
the coefficient of correlation.

6. From the data given below, find the number of items.


r = 0.5, Σxy = 120, Σx2 = 90, σ y = 8
(Where x and y are deviations from arithmetic average).

7. From the following data compute the coefficient of correlation between X and Y:
X- Series Y- Series
Arithmetic Mean 15 28
Sum of Squares of deviations from mean 144 225
Summation of product of deviations of X and Y series from their respective means = 20
8. In a question on correlation the value of r is 0.64 and its P.E. = 0.1312. What was the value of N?
9. From the marks obtained by 8 students in Accountancy and Statistics, compute rank coefficient of correlation.

Marks in Accountancy 60 15 20 28 12 40 80 20
Marks in Statistics 10 40 30 50 30 20 60 30

380 The Institute of Cost Accountants of India


Correlation and Regression

10. The coefficient of rank correlation between marks in Quantitative Mathematics and Economics obtained by a
certain group of students is 7/11. The sum of the squares of differences in ranks is 60.What is the number of
students in the group?
11. The coefficient of rank correlation of the marks obtained by 10 students in statistics and accountancy was
found to be 0.5. It was later discovered that the difference in ranks in the two subjects obtained by one of the
students was wrongly taken as 3 instead of 7. Find the correct coefficient of rank correlation.
12. Find correlation coefficient for data given below:

Age (years) 42 36 48 43 55 52 38
Blood Pressure 132 120 140 133 142 148 122
13. Calculate Karl Pearson’s coefficient of correlation between expenditure on advertisement and sales from the
data given below:

Advertisement expense in ` ‘000 39 78 65 62 90 82 75 25 98 36


Sales in lakh ` 47 84 53 58 86 62 68 60 91 51
14. Find the coefficient of correlation and probable error from the data:

X 67 69 71 75 85 93 87 73
Y 95 80 87 80 79 75 80 85
15. From the following data compute the coefficient of correlation between X and Y.

X- Series Y- Series
Arithmetic Mean 25 18
Sum of Squares of deviations from mean 136 138
Summation of product of deviations of X and Y series from their respective means = 122. Number of pairs of
value is 15.
16. Ten competitors in a beauty contest are ranked by 3 judges in the following order:

Judge A 1 6 5 10 3 2 4 9 7 8
Judge B 3 5 8 4 7 10 2 1 6 9
Judge C 6 4 9 8 1 2 3 10 5 7
Use Rank correlation coefficient to determine which pair of judges has the nearest approach to common taste
in beauty.
17. Calculate the coefficient of correlation by any suitable method:

A 115 168 170 127 118 129 135 140


B 2 6 8 11 1 1 4 3
Also calculate probable error
18. From the data given below, compute the correlation coefficient by the method of concurrent deviations.

Year 1996 1997 1998 1999 2000 2001 2002


Supply 150 154 160 172 160 165 180
Price 200 180 170 160 190 180 172

The Institute of Cost Accountants of India 381


Fundamentals of Business Mathematics and Statistics

19. Consider the data summarized below:


X 42 44 58 55 89 98 66
Y 56 49 53 58 65 76 58
The coefficient of correlation between the variables X and Y is 0.64, their covariance is 16. The variance of X
is 9. Find the standard deviation of Y.
20. Calculate the coefficient of correlation by concurrent deviation method from the following:
X 60 55 50 55 30 70 40 35 80 80 75
Y 65 40 35 75 63 80 35 20 80 60 60
21. Calculate Pearson’s coefficient of correlation from the following data using 44 and 26 as the origin of X and
Y respectively.
X 43 44 46 40 44 42 45 42 38 40 42 57
Y 29 31 19 18 19 27 27 29 41 30 26 10
22. Given the bivariate data
X 2 6 4 3 2 3 8 4
Y 7 2 1 1 2 3 2 6
Obtain regression equations taking deviations from 5 in case of X and 4 in case of Y.
23. Past 10 years data on Rainfall and Yield of wheat in a certain village offered the following results.
Av. Wheat Yield 25 Qtl
Av. Rainfall 20 Cms
Variance of wheat output 3 Qtl
Variance of Rainfall 5 Cms
Correlation Coefficient 0.65
Find the most likely yield output per acre when rainfall is 35cms.
24. In a partially destroyed laboratory records relating to legible correlation data shows, the following results
σ x = 9 , Regression equations 8X – 10 Y + 66 = 0 & 40X – 18X = 214.
2

What were (a) the mean values of X and Y, (b) σy , c) the co-efficient of correlation between X and Y?
25. Given
X- Series Y- Series
Arithmetic Mean 18 100
Standard Deviation 14 20
Coefficient of correlation between X and Y is + 0.8. Find out
(a) The most probable value of Y if X is 70 and most probable value of X if Y is 90.
(b) If the regression coefficients are 0.8 and 0.6, what would be the value of the coefficient of correlation?
26. Given that the means of X and Y are 65 and 67, their standard deviations are 2.5 and 3.5 respectively and the
coefficient of correlation between them is 0.8.
(i) Write down the equations of the regression lines
(ii) Obtain the best estimate of X when Y = 70
(iii) Using the estimated value of X as the given value of X, estimate the corresponding value of Y.

382 The Institute of Cost Accountants of India


Correlation and Regression

27. The correlation coefficient between the variables X and Y is r = 0.60. If σx = 1.50, σy = 2.00, X = 10, Y = 20,
find the equations of the regression lines (i) Y on X (ii) X on Y.
28. Find out σy and r from the following data: 3x = y, 4y = 3x and σx = 2.
29. Given that the regression equations of Y on X and X on Y are respectively Y = X and 4X-Y = 3. Find the
correlation coefficient between X and Y.
30. From the following data calculate (i) coefficient of correlation (ii) Standard deviation of Y.
X = 0.854 Y; Y = 0.89X; =3
31. If the two lines of regression are
4X – 5Y + 30 = 0 and 20X – 9Y–107 = 0
Which of these is the line of regression of X on Y. Find r and when = 3.
32. From the following regression equations, calculate
20x – 9Y = 107
4X – 5Y = – 33
33. The following table gives the age of car of a certain make and annual maintenance costs. Obtain the regression
equation for cost related to age, Estimate maintenance cost of a car whose age is10 Years.
Age of cars (X) in years 2 4 6 8
Maintenance Cost in hundreds of ` (Y) 10 20 25 30

34. Find two regression equations from the following data:


X: 10, 25, 34, 42, 37, 35, 36, 45;
Y: 56, 64, 63, 58, 73, 75, 82, 77;
35. Obtain the line of regression of Y on X for the following data:
Age (Years) X: 66, 38, 56, 42, 72, 36, 63, 47, 55, 45;
Blood Pressure Y: 145, 124, 147, 125, 160, 118, 149, 128, 150, 124;
Estimate the blood pressure of a man whose age is 50 years.
36. In trying to evaluate the effectiveness in its advertising campaign, a firm compiled the following information.
Year 1998 1999 2000 2001 2002 2003 2004 2005
Advertisement expenditure (`’000) 12 15 15 23 24 38 42 48
Sales (` lakhs) 5 5.6 5.8 7 7.2 8.8 9.2 9.5
Calculate the regression equation of sales on advertising expenditure. Estimate the probable sales when
advertisement expenditure is `60 thousand.
37. Two random variables have the least square regression lines with equations 3x +2y –26 = 0 and 6x + y –31 =  0.
Find the mean value and the correlation coefficient between X and Y.
38. The following data show the years of services (x) and average salary per month (y) with respect to 7 persons
working in a college. Fit a straight line regression of Y on X.
Sr. No 1 2 3 4 5 6 7
X (years) 1 3 7 6 2 4 5
Y (`’000) 5 7 7 8 6 4 3

The Institute of Cost Accountants of India 383


Fundamentals of Business Mathematics and Statistics

Exercise:
Theoretical Questions
 Multiple Choice Questions (MCQ)

1. If bXY and bYX are regression coefficients of series X on series Y and regression coefficients of series Y on
series X respectively then which one of the following is correct?
(a) bXY × bYX = r , where r is the correlation coefficient
(b) bXY × bYX = r 2 , where r is the correlation coefficient
(c) bXY × bYX = − r , where r is the correlation coefficient
(d) bXY × bYX = 1/ r , where r is the correlation coefficient
2. If bXY and bYX are regression coefficients of series X on series Y and regression coefficients of series Y on
series X respectively then which one of the following is correct?
(a) bXY and bYX will be either both positive or both negative
(b) bXY will be positive and bYX will be negative
(b) bXY will be negative and bYX will be positive
(d) Nothing can be said like this, it depends on X & Y values
3. Say true or false:
“If r is the correlation coefficient between two series X & Y then covariance between X & Y is given as “r
× variance of series X × variance of series Y ”
4. Match the items in column A with the items in column B

A B
i) is used to obtain the best estimates of X for
a) Y − Y given values of Y
b) X − X = bXY (Y − Y ) ii) is equal to r × σ X
1
c) bYX = iii) is equal to bYX ( X − X )
bXY

d) σY × bXY iv) is a condition for making r 2 = 1

5. Which one of the following is correct?


(a) Regression equation predicts maximum probable values of one variable for specified values of other
variable
(b) Regression equation predicts most likely values of one variable for specified values of other variable
(c) Regression equation predicts maxi-min values of one variable for specified values of other variable
(d) Regression equation predicts minimum probable values of one variable for specified values of other
variable

384 The Institute of Cost Accountants of India


Correlation and Regression

6. If = r = 0.52, σX = 4.6 & σY = 36.8 then bXY is equal to


(a) 0.24
(b) 4.16
(c) +1
(d) 0.065

7. If bXY & bYX are regression coefficients between X on Y and Y on X respectively and r is the correlation
coefficient between X and Y then
bXY + bYX
(a) ≤r
2
bXY + bYX
(b) ≤ r2
2
b +b
(c) XY YX ≥r
2
bXY + bYX
(d) ≥ r2
2

8. If bXY & bYX are regression coefficients between X on Y and Y on X respectively then

(a) bXY × bYX ≤ 1

(b) bXY × bYX ≥ 1

(c) bXY × bYX = −1

(d) bXY × bYX ≤ 1

9. In a regression equation
(a) Regression coefficient represents the increment in the value of the independent variable for a unit
change in the value of the dependent variable
(b) Regression coefficient represents the increment in the value of the dependent variable for a unit change
in the value of the independent variable
(c) Regression coefficient represents the mean value of the independent variable for a unit change in the
value of the dependent variable
(d) Regression coefficient represents the mean value of the dependent variable for a unit change in the
value of the independent variable

10. If r2 = 0.3 & bXY = – 1.5 then bYX is equal to


(a) + 1
(b) – 0.2
(c) – 1
(d) – 0.45

The Institute of Cost Accountants of India 385


Fundamentals of Business Mathematics and Statistics

11. In a bivariate regression analysis ΣXY = 1355.25, (ΣX) (ΣY) = 6396, ΣX2 = 591.50 & ΣX = 52. If there are
5 items then bYX
(a) 1
(b) 0.97
(c) 0.667
(d) 1.5
12. X = 1.36Y – 5.2 & Y = 0.61X + 1.51 are two regression equations. Correlation coefficient between
is
(a) – 0.67
(b) – 0.911
(c) 0.911
(d) 0.67

13. In a bivariate regression analysis comprising of series X & Y, if Σ ( X − X ) 2 = Σ (Y − Y ) 2 then


(a) bXY = bYX
(b) bXY > bYX
(c) bXY < bYX
(d) Correlation coefficient =1

14. Match the items in column A with the items in column B

A B
a) Coefficient of correlation i) is a functional relationship which shows dependence
of one variable on the other
b) Arithmetic mean of regression ii) is the geometric mean between regression coefficients
coefficients
c) Regression Analysis iii) is the mean values of the variables X and Y in case of
two variables case
d) Intersection point of lines of regression iv) is greater than equal to correlation coefficient

15. In a bivariate regression analysis the difference between actual value of dependent variable and the
predicted value of the dependent variable is called
(a) Outlier
(b) Slope
(c) Residual
(d) Scattered point

16. Consider the following results:


N = 12, Σdx = 0, Σdy = 4, Σdx2 = 1344, Σdy2 = 215, Σdxdy = – 4360

386 The Institute of Cost Accountants of India


Correlation and Regression

Appropriate regression coefficient is


(a) -0.821
(b) 1
(c) 5.67
(d) -3.244

17. In a bivariate regression analysis for dependent variable if d = Actual value – Predicted value then at
different values of independent variable
(a) Best fit curve occurs when d 2 + d 2 + ... ... ... + d 2 is minimum
1 2 n

(b) Best fit curve occurs when d12 + d22 + ... ... ... + dn2 is maximum
(c) Best fit curve occurs when d 2 + d 2 + ... ... ... + d 2 is zero
1 2 n

(d) Best fit curve occurs when d12 + d22 + ... ... ... + dn2 is one

18. Consider the following results:


11 1
N = 6, Σy = 42, Σy2 = 318, byx = − , Σ x 2 − (Σ x) 2 = 34
34 n
Then bxy is
(a) 11/34
(b) 11/24
(c) – 34/11
(d) – 11/24

19. In a bivariate analysis if two regression equations are mx – y + 10 = 0 & – 2x + 5y = 14. If coefficient of
correlation between x & y is 1 , then value of m is
10
(a) 10
(b) 5/2
(c) 4
(d) 1
20. In a bivariate analysis if two regression equations are 8x – 10y + 66 = 0 & 40x – 18y – 214 = 0. Then x ,
y , the mean of the series x & y care respectively
(a) 13, 17
(b) 17, 17
(c) 5/4, 20/9.
(d) 8, 18

The Institute of Cost Accountants of India 387


Fundamentals of Business Mathematics and Statistics

Answer:

 Multiple Choice Questions (MCQ)

1. 2. 3. 4. 5.
(b) (a) False (a) (iii), (b) (i), (c) (iv), (d) (ii) (b)
6. 7. 8. 9. 10.
(d) (c) (a) (b) (b)
11. 12. 13. 14. 15.
(d) (c) (a) (a) (ii), (b) (iv), (c) (i), (d) (iii) (c)
16. 17. 18. 19. 20.
(d) (a) (d) (c) (a)

388 The Institute of Cost Accountants of India


PROBABILITY 7
This Modules includes

7.1 Concepts and Terminologies


7.2 Primary applications of Probability Theorems

The Institute of Cost Accountants of India 389


Fundamentals of Business Mathematics and Statistics

PROBABILITY

Module Learning Objectives:


After studying this Module, the students will be able to understand –
 Principal concepts of Probability
 Features of Probability
 Application of Permutation and Combination in solving problems of Probability
 Sample Space, Theories of Probability
 Conditional probability, Rule of total probability
 Bayes’ Theorem
 Application of probability in real world

390 The Institute of Cost Accountants of India


Probability

Concepts and Terminologies 7.1


GENERAL CONCEPT
The concept of probability is difficult to define in precise terms. In ordinary language, the word probable means
likely or chance. The probability theory is an important branch of mathematics. Generally the word, probability,
is used to denote the likelihood of the occurrence of an event, based on past experiences. By looking at the clear
sky, one will say that there will not be any rain today. On the other hand, by looking at the cloudy sky or overcast
sky, one will say that there will be rain today. In the earlier sentence, we aim that there will not be rain and in the
latter we expect rain. On the other hand a mathematician says that the probability of rain is 0 in the first case and
that the probability of rain is 1 in the second case. In between 0 and 1, there are fractions denoting the chance of
the event occurring.
If a coin is tossed, the coin falls down. The coin has two sides ; head and tail. On tossing a coin, the coin may fall
down either with the head up or tail up. It is assumed that a coin, on reaching the ground, will not stand on its edge.
So the probability of the coin coming down with either Head or Tail up is 1. The probability of the head coming up
is 50% and the tail coming up is 50%, In other words we can say the probability of the head or the tail coming up
is  12 , i.e ‘head’ and ‘tail’ share equal chances. The probability that it will come down head or tail is unity.

SOME USEFUL TERMS


Before discussing the theory of probability, let us have an understanding of the following terms :
Random Experiment or Trial
If an experiment or trial can be repeated under the same conditions, any number of times and it is possible to count
the total number of outcomes, but individual result i.e. individual outcome is not predictable. Suppose we toss a
coin. It is not possible to predict exactly the outcomes. The outcome may be either head up or tail up. Thus an
action or an operation which can produce any result or outcome is called a random experiment or a trial.
Outcome
This means the possible result of a Random Experiment. If we consider the Random Experiment of “Tossing
of a Coin” then there are two possible results – appearence of Head or H and appreance of Tail or T. Hence the
outcomes in this case are H & T.
Event
Any possible outcome of a random experiment is called an event. Performing an experiment is called trial and
outcomes are termed as events.
An event whose occurrence is inevitable when a certain random experiment is performed, is called a sure event or
certain event. At the same time, an event which can never occur when a certain random experiment is performed is
called an impossible event. The events may be simple or composite. An event is called simple if it corresponds to a

The Institute of Cost Accountants of India 391


Fundamentals of Business Mathematics and Statistics

single possible outcome. For example, in rolling a die, the chance of getting 2 is a simple event. Further in tossing
a die, chance of getting event numbers (1, 3, 5) are compound event.
Sample space
The set or aggregate of all possible outcomes is known as sample space. For example, when we roll a die, the
possible outcomes are 1, 2, 3, 4, 5, and 6 ; one and only one face come upwards. Thus, all the outcomes—1, 2, 3, 4,
5 and 6 are sample space. And each possible outcome or element in a sample space called sample point.
Mutually exclusive events or cases
Two events are said to be mutually exclusive if the occurrence of one of them excludes the possibility of the
occurrence of the other in a single observation. The occurrence of one event prevents the occurrence of the other
event. As such, mutually exclusive events are those events, the occurrence of which prevents the possibility of the
other to occur. All simple events are mutually exclusive. Thus, if a coin is tossed, either the head can be up or tail
can be up; but both cannot be up at the same time.
Similarly, in one throw of a die, an even and odd number cannot come up at the same time. Thus two or more
events are considered mutually exclusive if the events cannot occur together.
Equally likely events
The outcomes are said to be equally likely when one does not occur more often than the others.
That is, two or more events are said to be equally likely if the chance of their happening is equal. Thus, in a throw
of a die the coming up of 1, 2, 3, 4, 5 and 6 is equally likely. For example, head and tail are equally likely events
in tossing an unbiased coin.
Exhaustive events
The total number of possible outcomes of a random experiment is called exhaustive events. The group of events
is exhaustive, as there is no other possible outcome. Thus tossing a coin, the possible outcome are head or tail;
exhaustive events are two. Similarly throwing a die, the outcomes are 1, 2, 3, 4, 5 and 6. In case of two coins, the
possible number of outcomes are 4 i.e. (22), i.e., HH, HT TH and TT. In case of 3 coins, the possible outcomes are
23=8 and so on. Thus, in a throw of n” coin, the exhaustive number of case is 2n.
Independent Events
A set of events is said to be independent, if the occurrence of any one of them does not, in any way, affect the
Occurrence of any other in the set. For instance, when we toss a coin twice, the result of the second toss will in no
way be affected by the result of the first toss.
Dependent Events
Two events are said to be dependent, if the occurrence or non-occurrence of one event in any trial affects the
probability of the other subsequent trials. If the occurrence of one event affects the happening of the other events,
then they are said to be dependent events. For example, the probability of drawing a king from a pack of 52 cards
is 4/52, ; the card is not put back; then the probability of drawing a king again is 3/51. Thus the outcome of the
first event affects the outcome of the second event and they are dependent. But if the card is put back, then the
probability of drawing a king is 4/52 and is an independent event.
Simple and Compound Events
When a single event take place, the probability of its happening or not happening is known as simple event.
When two or more events take place simultaneously, their occurrence is known as compound event (compound
probability); for instance, throwing a die.

392 The Institute of Cost Accountants of India


Probability

Complementary Events
The complement of an events, means non-occurrence of A and is denoted by . contains those points of the sample
space which do not belong to A. For instance let there be two events A and B. A is called the complementary event
of B and vice verse, if A and B are mutually exclusive and exhaustive.
Favourable Cases
The number of outcomes which result in the happening of a desired event are called favourable cases to the event.
For example, in drawing a card from a pack of cards, the cases favourable to “getting a diamond” are 13 and to
“getting an ace of spade” is only one. Take another example, in a single throw of a dice the number of favourable
cases of getting an odd number are three -1,3 and 5.

MEASUREMENT OF PROBABILITY
The origin and development of the theory of probability dates back to the seventeenth century. Ordinarily speaking
the probability of an event denotes the likelihood of its happening. A value of the probability is a number ranges
between 0 and 1. Different schools of thought have defined the term probability differently. The various schools of
thought which have defined probability are discussed briefly.
Classical Approach (Priori Probability)
The classical approach is the oldest method of measuring probabilities and has its origin in gambling games.
According to this approach, the probability of an event is the ratio of favourable outcomes to the total number of
mutually exclusive, exhaustive and equally likely outcomes. If we toss a coin we are certain that the head or tail
will come up. The probability of the coin coming down is 1, of the head coming up is 1 and of the tail coming up
2
is 1 . It is customary to describe the probability of one event as ‘p’ (success) and of the other event as ‘q’ (failure)
2
as there is no third event.

Number of outcomes favourable to the event


Probability of an event = –
Total number of mutually exclusive, exhaustive likely outcomes
If an event can occur in ‘a’ ways and fail to occur in ‘b’ ways and these are equally to occur, then the probability
a
of the event occurring, is denoted by P. Such probabilities are also known as unitary or theoretical or
a+b
mathematical probability. If p is the probability of the event happening and q is the probability of its not happening,
then
a b
p= and q =
a+b a+b
a b a+b
Hence p + q = + + =1
( a + b) ( a + b) a + b
Therefore
p + q = 1 or, 1 – p = q & 1 – q = p
1
Probabilities can be expressed either as ratio, fraction or percentage, such as or 0.5 or 50%
2
Limitations of Classical Approach
1. This definition is confined to the problems of games of chance only and cannot explain the problem other than
the games of chance.
2. We cannot apply this method, when the total number of outcomes cannot be calculated.

The Institute of Cost Accountants of India 393


Fundamentals of Business Mathematics and Statistics

3. When the outcomes of a random experiment are not equally likely, this method cannot be applied.
4. It is difficult to subdivide the possible outcomes of a random experiment into mutually exclusive, exhaustive
and equally likely in most cases.
5. The definition says that the outcomes are equally likely i.e equiprobable. But before finding the probability
how the outcomes can be taken as equiprobable.

Example 1:
What is the chance of getting a king in a draw from a pack of 52 cards?
Solution:
The total number of cases that can happen = 52 (52 cards are there).
4 1
Total number of kings are 4 ; hence favourable cases = 4 Therefore probability of drawing a king = = .
52 13
Example 2:
Two coins are tossed simultaneously. What is the probability of getting a head and a tail?
Solution:
The possible combinations of the outcomes when two coins are tossed are HH, HT, TH, TT. The favourable ways
are two out of these four possible ways and all these are equally likely to happen.
2 1
Hence the probability of getting a head and a tail is = .
4 2
Example 3:
One card is drawn at random from a well-shuffled pack of 52 cards. What is the probability that it will be (a) a
diamond (b) a queen?
Solution:
(a) There are 13 diamond cards in a pack of 52 cards. The number of ways in which a card can be drawn from that
pack is 52. The number of cases favourable to the event of getting a diamond is 13.
Hence probability of drawing a diamond
13 1
=
52 4
(b) There are 4 queens in the pack. So the number of cases favourable to the event of getting a queen = 4
4 1
The required probability =
52 13
Example 4:
Two cards are drawn from a pack of cards at random. What is the probability that it will be (a) a diamond and a
heart (b) a king and a queen (c) two kings?
Solution:
(a) The number of ways of drawing 2 cards from a pack of 52 cards

52 × 51
= 52C2 = = 26 × 51
1× 2
There are 13 each of Diamonds and Hearts in a pack of 52 Cards.

394 The Institute of Cost Accountants of India


Probability

The number of ways of drawing a diamond and a heart = 13C1 × 13C1 = 13 × 13


13 × 13 13
The required probability = =
26 × 51 102
Again there are 4 each of Kings and Queens in a pack of 52 Cards.

(b) The number of ways of drawing a king and a queen = 4C × 4C = 4 × 4


1 1

The required probability of drawing a King and a Queen.


4×4 6
= =
26 × 51 663
4 4 4 × 3 × 2 ×1
(c) Two kings can be drawn out of 4 kings in 4C2 = = = = 6 ways
4 − 2⋅ 2 2⋅ 2 2 ×1× 2 ×1
The probability of drawing 2 kings
6 1
= 26 × 51 = 221

Example 5:
A bag contains 7 red, 12 white and 4 green balls. What is the probability that :
(a) 3 balls drawn are all white and
(b) 3 balls drawn are one of each colour ?
Solution:
(a) Total number of balls
= 7 +12 + 4 = 23
Total number of possible ways of drawing 3 out of 23 balls
23 23 × 22 × 21 × 20
= 23C3 = 3 ⋅ 23 − 3 = 3 × 2 × 1 × 20 = 1771

Number of possible ways of drawing 3 out of 12 white balls


12 12 × 11 × 10 × 9
= 12C3 = 3 ⋅ 12 − 3 = 3 × 2 × 1 × 10 = 220
12
C3 220
Therefore, probability of drawing 3 white balls = = = 0.1242 = 0.1242
23
C3 1771
(b) Number of possible ways of drawing 1 out of 7 red balls = 7C1
Number of possible ways of drawing 1 out of 12 white balls = 12C1
Number of possible ways of drawing 1 out of 4 green balls = 4C1
Therefore the probability of drawing 3 balls of each colours
7
C1 × 12C1 × 4C1 7 × 12 × 4
= =
23
C3 1771

= 0.1897

The Institute of Cost Accountants of India 395


Fundamentals of Business Mathematics and Statistics

Relative Frequency Theory of probability


Classical approach is useful for solving problems involving game of chances—throwing dice, coins, etc. but if
applied to other types of problems it does not provide answers. For instance, if a man jumps from a height of 300
feet, the probability of his survival will, not be 50%, since survival and death are not equally likely in this case.
Similarly, the prices of shares of a Joint Stock Company have three alternatives i.e. the prices may remain constant
or prices may go up or prices may go down. Thus, the classical approach fails to answe questions of these type.
If we toss a coin 20 times, the classical probability suggests that we; should have heads ten times. But in practice
it may not be so. These empirical approach suggests, that if a coin is tossed a large number of times, say, 1,000
times, we can expect 50% heads and 50% tails. Vor Miscs explained, “If the experiment be repeated a large number
of times under essentially identical conditions, the limiting value of the ratio of the number of times the event A
happens to the total, number of trials.of the experiments as the number of trials increases indefinitely, is called the
probability of the occurrence of A”.
m
Thus, P( A) = lim
n→∞ n

The happening of an event is determined on the basis of past experience or on the basis of relative frequency
of success in the past. For instance, a machine produces 10% unacceptable articles of the total output. On the
basis of such experience or experiments, we may arrive at that (i) the relative frequency obtained on the basis
of past experience can be shown to come very close to the classical probability. For example, as said earlier, a
coin is tossed for 6 times, we may not get exactly 3 heads and 3 tails. But, the coin is tossed for larger number of
times, say 10,000 times, we can expect heads and tails very close to 50% (ii) There are certain laws, according to
which the ‘occurrence’ or ‘non-occurrence of the events take place. Posterior probabilities, also called Empirical
Probabilities are based on experiences of the past and on experiments conducted. Thus, relative frequency can be
termed as a measure of probability and it is calculated on the basis of empirical or statistical findings. For instance
if a machine produces 100 articles in the past, 2 particles were found to be defective, then the probability of the
defective articles is 2/100 or 2%.
Limitations of Relative Frequency Theory of Probability
1. The experimental conditions may not remain essentially homogeneous and identical in a large number of
repetitions of the experiment.
2. The relative frequency —, may not attain a unique value no matter however large N may be.
3. Probability P(A) defined can never be obtained in practice. We can only attempt to get a close estimate of P(A)
by making N sufficiently large.
Example 6:
An urn contains 8 white and 3 red balls. If two balls are drawn at random, find the probability that (a) both are
white, (b) both are red and (c) one is of each colour.
Solution:
Total number of balls in the urn = 8 + 3 =11
Two balls can be drawn out of 11 balls in 11C2 ways.
11 × 10
Exhaustive number of outcomes = 11C2 = = 55.
2
8×7
(a) Two white balls to be drawn out of 8 white. It can be done in 8C = = 28 ways.
2 2
28
The probability that both the balls drawn are white =
55

396 The Institute of Cost Accountants of India


Probability

(b) Two red balls to be drawn out of 3 red balls. It can be done in 3C2 = 3 ways.
3
Hence, the probability that both the balls drawn are red =
55
(c) The number of favourable cases for the event-drawing one white and one red ball is
8
C1× 3C1 = 8 × 3 = 24.
24
Therefore, the probability of drawing one red and one white ball =
55

Example 7:
Tickets are numbered from 1 to 100. They are well shuffled and a ticket is drawn at random. What is the probability
that the drawn ticket has:
(a) an even number,
(b) a number 5 or a multiple of 5,
(c) a number which is greater than 75,
(d) a number which is a square?

Solution:
(a) The total number of exhaustive, mutually exclusive and equally likely outcomes is 100. There are 50 even
numbered tickets.

Therefore, number of casesfavourable to the event is 50.


50 1
Therefore, the probability that the drawn ticket bears an even number = =
100 2
(b) Suppose A denotes the event that the drawn ticket has a number 5 or a multiple of 5. These are 5, 10, 15,
20,...100. That is 20 such number are possible which are favourable to the event.
20 1
Therefore, P(A) = =
100 5

(c) Between 1 & 100 there are 25 numbers, which are greater than 75. Say B denote the event that the number on
the drawn ticket is greater than 75.
25 1
Therefore, P( B ) = =
100 4

(d) Between 1 & 100 there are 10 numbers which are perfect squares i.e., 1, 4, 9, 16, 25, 36, 49, 64, 81, 100.
Let C denotes to event that the number on the drawn ticket is a perfect square.
10 1
Therefore, P(C ) = =
100 4

Example 8:
Four cards are drawn one by one from a pack of 52 cards without replacement. What is the probability that they
are all of different suits?

The Institute of Cost Accountants of India 397


Fundamentals of Business Mathematics and Statistics

Solution:

There are 52 cards in a full pack which belong to 4 suits @13 each. The first card can be drawn in 52C ways.
1
As this is not replaced, there remains 51 cards before drawing teh second one.

So the second card can be drawn in 51C ways.


1

This way 4 cards can be drawn in 52C × 51C × 50C × 49C = 52 × 51 × 50 × 49 ways.
1 1 1 1
Now the drawn cards should be of 4 different suits.

( )
Hence the first card can be of a particular suit in 4C1 × 13C1 = 4 × 13 = 52 ways
Before drawing the second done, there remains 3 different suits. Hence the second card can be of a suit other than

( )
the first are in 3C1 × 13C1 = 3 ×13 = 39

( )
Similarly the third one can be a suit other than the first two in 2C1 × 13C1 = 2 ×13 = 16 ways

( )
The last one can be drawn in 1C1 × 13C1 = 1 × 13 = 13 ways

13 13
50 × 39 × 26 × 13 2197
So the required probability is =
52 × 51 × 50 × 49 20825
17 25

398 The Institute of Cost Accountants of India


Probability

Primary applications of
Probability Theorems 7.2
Theorems of Probability
We have studied what probability is and how it can be measured. We dealt with simple problems. Now we shall
consider some of the laws of probability to tackle complex situation. There are two important theorems, viz., (1)
the Addition Theorem and (2) the Multiplication Theorem.
Addition Theorem
The simplest and most important rule used in the calculation is the addition rule, it states, “If two events are
mutually exclusive, then the probability of the occurrence of either A or B is the sum of the probabilities of A and
B. Thus,
P(A or B) = (P + B) = P(A) + P(B), when A and B are mutually exclusive.

Example 9:
A bag contains 4 white, 3 black and 5 red balls. What is the probability of getting a white or a red ball at random
in a single random draw?

Solution:
4
The probability of getting a white ball =
12
5
The probability of getting a red ball =
12
4 5 9 9
The probability of getting a white or a red = + = = × 100 = 75%
12 12 10 12
When events are not mutually exclusive
The addition theorem studied above is not applicable when the events are not mutually exclusive. In such cases
where the events are not mutually exclusive, the probability is :
P(At least one of A or B) = P (A + B) P(A) + P(B) – P(A and B), when A and B are not mutually exclusive

Example 10:
Two students X and Y work independently on a problem. The probability that A will solve it is 3/4 and the
probability that Y will solve it is 2/3. What is the probability that the problem will be solved?
Solution:
The problem will be solves if at least one of X and Y solve it
P(A least one of A or B) = P(A) + P(B) – P (A and B)

The Institute of Cost Accountants of India 399


Fundamentals of Business Mathematics and Statistics

The probability that X will solve the problem is 3/4 = P(A)


The probability that Y will solve the problem is 2/3 = P(B)
The events are not mutually exclusive as both of them may solve the problem.

Therefore, the probability =


3 2 3 2
+ − ×
4 3 4 3 ( )
17 6 11
= − =
12 12 12
Alternatively:
The probability that X will solve it and Y fail to solve it = 3/4 × 1/3 = 3/12

∴ Probability that the problem will be solved = 2 + 3 = 11


3 12 12
Alternatively
The probability that X will fail to solve and will Y solve it
= 1/4 × 2/3 = 2/12
3 2 9 + 2 11
∴ Probability that the problem will be solved = + = =
4 12 12 12
Alternatively
1 1 1
The probability that neither X nor Y will solve it × =
4 3 12
Hence, the probability that the problem will be solved
1 11
= 1− =
12 12

Multiplication Theorem
When it is desired to estimate the chances of the happening of successive events, the separate probabilities of these
successive events are multiplied. If two events A and B are independent, then the probability that both will occur
is equal to the product of the respective probabilities. We find the probability of the happening of two or more
events in succession.
P(A and B) = P(AB) = P(A) × P(B) when A and B are independent
Example 11:
In two tosses of a fair coin, what are the chances of head in both ?
Solution:
Probability of head in first toss = 1/2
Probability of head in the second toss = 1/2
Probability of head in both tosses = 1/2 ×1/2 = 1/4
Example 12:
The probability that X and Y will be alive ten years hence is 0.5 and 0.8 respectively. What is the probability that
both of them will be alive ten years hence?

400 The Institute of Cost Accountants of India


Probability

Solution:
Probability of X being alive ten years hence = 0.5
Probability of Y being alive ten years hence = 0.8
Probability of X and Y both being alive ten years hence = 0.5 × 0.8 = 0.4

When events are dependent:


If the events are dependent, the probability is conditional. Two events A and B are dependent ; B occurs only when
A is known to have occurred.
P (B /A) means the probability of occurance of B given that A has occurred.

P( AB )  A   P( AB ) 
P( B / A) = ; P  =  
P ( A)  B   P( B ) 

B  A
P (A & B) = P (AB) = P( A) ⋅ P  A  = P( B ) ⋅ P  B  when A and B are not independent
   

Example 13:
A man want to marry a girl having qualities: White complexion the probability of getting such girl is 1 in 20.
Professionally qualified - the probabihty of getting is 1 in 50. Westernised style - the probability is 1 in 100.
Find out the probability of his getting married to such a girl, who has all the three qualities.

Solution:

1
The probability of a girl with white complexion = or 0.05. The probability of a girl with professional qualification
20
1 1
= or 0.02. The probabihty of a girl with westernised style = or 0.01. Since the events are independent, the
50 100
1 1 1
probabihty of getting married to a girl having all three qualities = 20 × 50 × 100 = 0.05 × 0.02 × 0.01 = 0.00001

Example 14:
A university has to select an examiner from a list of 50 persons, 20 of them are women and 30 men. 10 of them
knows Hindi and 40 not. 15 of them are teachers and the remaining 35 are not. What is the probability of the
University selecting a Hindi-knowing woman teacher?
Solution:
20
Probability of selecting a woman =
50
15
Probability of selecting a teacher =
50
10
Probability of selecting a Hindi-knowing candidate =
50
Since the events are independent the probabihty of the University selecting a Hindi-knowing woman teacher is:
20 15 10 3
× × = or 0.024.
50 50 50 125

The Institute of Cost Accountants of India 401


Fundamentals of Business Mathematics and Statistics

Example 15:
A ball is drawn at random from a box containing 6 red balls, 4 white balls and 5 blue balls. Determine the probability
that it is :
(i) Red (ii) white, (iii) Blue, (iv) Not Red and (v) Red or White.
Solution:
There are 6 Red, 4 White and 5 Blue balls i.e 6 + 4 + 5 = 15 balls in the box.
6
(i) Probability of Red = or 0.40
15
4
(ii) Probability of white = or 0.267
15
5
(iii) Probability of Blue = or 0.333
15
9
(iv) Probability of not Red = or 0.60
15
6 4 10
(v) Probability of Red or White + = = 0.667
15 15 15

BAYES’ THEOREM
This theorem is associated with the name of Reverend Thomas Bayes. It is also known as the inverse probability.
Probabilities can be revised when new information pertaining to a random experiment is obtained. One of the
important applications of the conditional probability is in the computation of unknown probabilities, on the basis
of the information supplied by the experiment or past records. That is, the applications of the results of probability
theory involves estimating unknown probabilities and making decisions on the basis of new sample information.
This concept is referred to as Bayes’ Theorem. Quite often the businessman has the extra information on a particular
event, either through a personal belief or from the past history of the events. Revision of probability arises from a
need to make better use of experimental information. Probabilities assigned on the basis of personal experience,
before observing the outcomes of the experiment are called prior probabilities. For example, probabilities assigned
to past sales records, to past number of defectives produced by a machine, are examples of prior probabilities.
When the probabilities are revised with the use of Bayes’ rule, they are called posterior probabilities. Bayes’
theorem is useful in solving practical business problems in the light of additional information. Thus popularity of
the theorem has been mainly because of its usefulness in revising a set of old probability (Prior Probability) in the
light of additional information made available and to derive a set of new probabilily (i.e. Posterior Probability)
Bayes’ Theorem: An event A can occur only if one of the mutually exclusive and exhaustive set of events B1, B2,
..... Bn occurs. Suppse that the unconditional probabilities P(B1), P(B2), .... P(Bn) and the conditional probabilities
P(A/B1), P(A/B2), .... P(A/Bn).
are known. Then the conditional probability P(Bi /A) of a specific event Bi, when A is stated to have actually
accquared, is given by
P(B1) ⋅ P( A / Bi )
P(B1 / A) = n

∑ P( B ) ⋅ P( A / B )
i =i
1 i

This is known as Bayes’ Theorem.


The following examples illustrate the application of Bayes’ Theorem.

402 The Institute of Cost Accountants of India


Probability

Example 16:
You note that your officer is happy on 60% of your calls, so you assign a probability of his being happy on your
visit as 0.6 or 6/10. You have noticed also that if he is happy, he accedes to your request with a probability of 0.4
1
or 4/10 whereas if he is not happy, he accedes to the request with a probability of 0.1 or . You call one day, and
10
he accedes to your request. What is the probability of his being happy ?
Solution:
Let- H be the event that the officer is happy and H be the event that the officer is not happy

6 (Given) P( H ) = 1 − P( H ) = 1 − 6 = 4
P( H ) = 10 10
10
Let A be the event that he accedes to your request
4 , 1 (As per given in formation)
P( A / H ) = P( A / H ) =
10 10
To find P(H/A), we use Bayes’ Theorem as follows–

P(H / A) =
( )
P( H ) × P A H
=
6 4
10
×
10
A( )
P( H ) × P H + P( H ) × P
A
H ( )
6 4 4 1
⋅ + ⋅
10 10 10 10

24
100 24 6
= = = = 0.857
24 4 28 7
+
100 100

Example 17:
A company has two plants to manufacture scooters. Plant I manufactures 80% of the scooters and plant II
manufactures 20%. At Plant I, 85 out of 100 scooters are rated standard quality or better. At Plant II, only 65 out of
100 scooters are rated standard quality or better. What is the probability that the scooter selected at random came
from Plant I if it is known that the scooter is of standard quality?
What is the probability that the scooter came from Plant II if it is known that the scooter is of standard quality.

Solution:
Let A1 be the event of drawing a scooter produced by Plant I and A2 be the event of drawing a scooter produced by
Plant II. B be the event of drawing a standard quality scooter produced by either Plant I or Plant II
Then, from the first information :
80 20
P( A1) = = 80% = 0.80 and P( A2 ) = = 20% = 0.20
100 100

From the additional information :


 B  85  B  65
P  = = 85% and P   = = 65%
 A1  100  A2  100

The Institute of Cost Accountants of India 403


Fundamentals of Business Mathematics and Statistics

The required values are computed in the following table :

Event Prior Conditional Joint Posterior Probability

Probability Probability Probability (Revised) (5) [4 ÷ P (B)]

(1) (2) (3) (4) = (2) × (3) (5) = (4) ÷ P (B)

0.68 68
A1 0.80 0.85 0.68 =
0.81 81

0.13 13
A2 0.20 0.65 0.13 =
0.81 81

Total 1 – P (B) = 0.81 1

The above calculation can be verified as follows :


If 1,000 scooters were produced by the two plants in a particular week, the number of scooters produced by Plant
I & Plant II are respectively :
l,000 × 80% = 800 scooters
1,000 × 20% = 200 scooters

The number of standard quality scooters produced by Plant I :


800 × 85/100 = 680 scooters

The number of standard quality scooters produced by Plant II :


200 × 65/100 = 130 Scooters.

The probability that a standard quality scooter was produced by Plant I is :


680 680 68
= = =
680 + 130 810 81

The probability that a standard quality scooter was produced by Plant II is :

130 130 13
= = =
680 + 130 810 81

From the first information we may say that the standard scooter is drawn from Plant I since P(A1) = 80% which is
greater than P(A2) = 20%,
From the additional information i.e. at Plant I, 85 out of 100 and at Plant II 65 out of 100 are rated standard quality,
we can give better answer, Thus we may conclude that the standard quality of scooter is more likely drawn from
the output by Plant I.
The same process i.e. revision can be repeated if more information is made available. Thus it is a good theorem in
improving the quality of probability in decision making under uncertainty.

404 The Institute of Cost Accountants of India


Probability

Example 18:

Box I contains three defective and seven non-defective balls, and Box II contains one defective and nine non-
defective balls. We select a box at random and then draw one ball at random from the box.

(a) What is the probability of drawing a non-defective ball?

(b) What is the probability of drawing a defective ball?

(c) What is the probability that box I was chosen, given a defective ball is drawn?

Solution:
As no specific information about the look of the boxes are provided, it is assumed that they ae identical in apperance.
1
So P(B1) or Probability that Box I is chosen =
2
1
P(B2) or Probability that Box II is chosen =
2

Let P(D) - Probability that a defective Ball is drawn and P(ND) = Probability that a non-defective Ball is drawn

Joint Probability

1 3 3
(i) Probability of selecting Box I and getting a defective ball from it = × =
2 10 20

1 1 1
(ii) Probability of selecting Box II and getting a defective ball from it = × =
2 10 20

1 7 7
(iii) Probability of selecting Box I and getting a non defective ball from it = × =
2 10 20

1 9 9
(iv) Probability of selecting Box II and getting a non defective ball from it = 2 × 10 = 20

(a) P(ND) = P (Box I and non-defective) + P (Box II and non-defective) = P [Case (iii)] + P [Case (iv)]


= ( 1 7
×
2 10)(1 9
+ ×
2 10
= )
16
20

(b) P(D) = P (Box I and defective) + P (Box II and defective)


= ( 1 3
×
2 10)(1 1
+ ×
2 10
=
4
)
20

(c) Bayes’ Theorem :

P( B1 and D) 3 / 20 3
P( B1 / D) = = =
P( D) 4 / 20 4

The Institute of Cost Accountants of India 405


Fundamentals of Business Mathematics and Statistics

B  B 
P(B1) and P(B2) are called prior probabilities and P  1  and P  2  are called posterior probabilities. The above
 D  D
information is summarised in the following table :

Event Prior Probability Conditional Probability Joint Probability Posterior Probability


(1) (2) (3) (4) = (2) × (3) (5) = (4) ÷ P(D)

B1 1 3/10 3/20 3/4


2

B2 1 1/10 1/20 1/4


2
Total 1 – P (D) = 4/20 1

ODDS
We must know the concept of odds. The word “Odds” is frequently used in statistics. Odds relate the chances in
favour of an event to the chances against it. For instance, the odds are 2 : 1 that A will get a job, means that there are
2 chances that he will get the job and 1 chance against his getting the job. This can also be converted into probability
as getting the job = 2/3. Therefore, if the odds are a : b in favour of an event then P(A) = a/(a + b). Further, it may
be noted that the odds are a : b in favour of an event is the same as to say that the odds are b : a against the event.
If the probability of an event is p, then the odds in favour of its occurrence are p to (1-p) and the odds against its
occurrence are 1-p to p.
Example 19:
Suppose it is 11 to 5 against a person A who is now 38 years of age living till he is 73 and 5 to 3 against B who is
43 living till he is 78. Find the chance that at least one of these persons will be alive 35 years hence.
Solution:
11 11
The probability that A will die within 35 years = =
11 + 5 16
5 5
The probability that B will die within 35 years = =
5+3 8
The probability that both of them will die within 35 years
11 5 55
= × =
16 8 128

The probability that both of them will not die i.e. atleast one of them will be alive
55 73 73
= 1− = or × 100 = 57%
128 128 128

Example 20:
Two cards are drawn at random one by one without replacement from a well-shuffled pack of 52 cards. What is
the probability that :
(a) both are aces,

406 The Institute of Cost Accountants of India


Probability

(b) both are red,


(c) at least one is an ace?

Solution:
(a) Let A indicate the event of drawing an ace. So drawing of aces on both occassions can be denoted as AA.
 A
P( AA) = P( A) × P  
 A

P(A) : Probability of drawing of an ace first

P ()
A
A
: Probability of an ace at the second draw, given that the first was an ace.

4
Therefore, P(A) = [As those are 4 aces in a pack of 52 cards]
52
 A 3
P  = [After drawing an ace for the first time there remains 51 cards in the pack of which 3 are aces.
 A  51
4 5 1
P( AA) = × =
52 51 221

(b) Let R indicate the event of drawing a red card. So the event of drawing red cards on both occassions can be
denoted as RR
 R
P( RR) = P( R) × P  
 R

26 25
= × [As there are 26 red cards in a pack of 52 cards and also the remains 51 cards after the first drawing
52 51
of which 25 are red]
25
=
102
(c) Let E indicate the event of drawing not an ace. Then the probability that no aces are drawn on both occassions
is indicated as P(EE). Probability of not drawing an ace :
 E
P( EE ) = P( E ) × P  
 E

48 47 188
= × =
52 51 221

Therefor, probability of drawing at least one ace


188 33
= 1− =
221 221

Example 21 :
The odds in favour of a certain event are 2 to 5 and the odds against another event independent of the former are 5
to 6. Find the chance that at least one of the events will happen.

The Institute of Cost Accountants of India 407


Fundamentals of Business Mathematics and Statistics

Solution:
Odds in favour of the 1st event (A, let) = 2:5
2 2
So, P(A) = =
2+5 7
Odds against the 2nd event (B, let) = 5:6

6 6
So, P(A) = =
5 + 6 11
Probability that at least are of the events will happen = P(A+B) = P(A) + P (B) – P(AB)

2 6 2 6
or, P(A+B) = + − × [As A & B are independent, P(AB) = P(A) × P(B)]
7 11 7 11
2 6 12
or, P(A+B) = + −
7 11 77
22 + 44 − 12
or, P(A+B) =
77
52
or, P(A+B) =
77

Alternatively:
Probability that the first event does not happen
2 5
= 1− =
7 7
Probability that the second event does not happen
6 5
= 1− =
11 11
The chance that both do not happen
5 5 25
= × = [As the events are independent]
7 11 77
25 52
The chance that one at least will happen = 1 − =
77 77

Example 22:
What is the chance that a leap year, selected at random will contain 53 Sundays?

Solution:
As a leap year consists of 366 days, it contains 52 complete weeks (i.e 52 Sunday for sure) and two more days.
The two consecutive days may appear in either of the following combinations :
(a) Monday and Tuesday

408 The Institute of Cost Accountants of India


Probability

(b) Tuesday and Wednesday


(c) Wednesday and Thursday
(d) Thursday and Friday
(e) Friday and Saturday
(f) Saturday and Sunday, and
(g) Sunday and Monday

If (f) or (g) occur, then the year consists of 52 + 1 = 53 Sundays.


Therefore the number of favourable cases = 2
Total number of cases = 7
2
The required probability =
7

Example 23:
A problem in statistics is given to three students A, B, C whose chances of solving it are 1/2, 1/3, 1/4 respectively.
What is the probability that the problem will be solved?

Solution:
1 1
The probability that A fails to soive the problem = 1 − =
2 2
1 2
The probability that B fails to solve the problem = − =
3 3
1 3
The probability that C fails to solve the problem = 1 − =
4 4
1 2 3 1
The probability that the problem is not solved by A, B and C = 2 × 3 × 4 = 4 (As the events are independent)

1 3
Therefore, the probability that the problem is solved = 1 − =
4 4

Example 24:
An ordinary die is tossed twice and the difference between the number of spots turned up is noted. Find the
probability of a difference of 3.

Solution:
The sample space consists of 36 values.
The event space has the following 6 cases : (1, 4), (2, 5), (3, 6), (4, 1), (5, 2), (6, 3)
6
The required probability =
36

The Institute of Cost Accountants of India 409


Fundamentals of Business Mathematics and Statistics

Example 25:
From a pack of 52 cards, two cards are drawn at random ; find the chance that one is a knave and the other a queen.

Solution:
No. of elements in the Sample space = 52C2

No. of elements in the Event space = 4C1 × 4C1

(as there are 4 queens and 4 knaves in the pack)


4
C1 × 4C1 8
Required Probability = =
52
C2 663

Example 26:
A bag contains 7 red balls and 5 white balls. 4 balls are drawn at random. What is the probability that (i) all of them
are red ; (ii) two of them are red and two white?

Solution:
(i) Favourable cases 7C4, Exhaustive cases 12C4
7
C4 105 7
Probability = = =
12
C4 495 33

(ii) Favourable cases = 7C2 × 5C2

Exhaustive cases = 12C4


7
C2 × 5C2 12 × 10 14
Probability = = =
12
C4 495 33

Example 27:
A petrol pump proprietor sells on an average ` 80,000 worth of petrol on rainy days and an average of ` 95,000 on
clear days. Statistics from the Metereological Department show that the probability is 0.76 for clear weather and
0.24 for rainy weather on coming Monday. Find the expected value of petrol sale on coming Monday.

Solution:

X1 = ` 80,000; P1 = 0.24

X2 = ` 95,000 P2 = 0.76

Mathematical expectation = ΣPX = P1 X1 + P2 X2

= 0.24 X 80,000 + 0.76 X 95,000

= 19,200 + 72,200 = ` 91,400.


The expected value of petrol sale on coming Monday = ` 91,400

410 The Institute of Cost Accountants of India


Probability

Example 28:
A bag contains 6 white and 9 black balls. Two drawings of 4 balls are made such that (a) the Balls are replaced
before the second trial (b) the balls are not replaced before the second trial. Find the probability that the first
drawing will give 4 white and the second 4 black balls in each case.

Solution:

(a) When the balls are replaced before the second trial, the number of ways in which 4 balls may be drawn is 15C4
in each drawing

The number of ways in which 4 white balls can be drawn = 6C4

The number of ways in which 4 black balls can be drawn = 9C4

Therefore, the probability of drawing 4 white balls at first trial


6
C4 1
= =
15
C4 91

The Second trial of drawing 4 black balls.
9
C4 9 × 8 × 7 × 6 4! 6
= = × =
15
C4 4! 15 × 14 × 13 × 12 65

1 6 6
Therefore the chance of the Compound event = × =
91 65 5915
(b) When the balls are not replaced :
At the first trial, 4 balls may be drawn in 15C4 ways and 4 white balls may be drawn in 6C4 ways.
6
C4 1
Therefore the chance of 4 white balls at first trial = = (as above)
C4 91
15

When 4 white balls have been drawn and removed, the bag contains 2 white and 9 black balls.
Therefore at the second trial, 4 balls can be drawn in 11C ways and 4 black balls can be drawn in 9C4 ways So, the
4
chance of 4 black balls at the second trial
9
C4
= 11
C4

1 9×8×7×6 4! 21
= = × =
91 4! 11 × 10 × 9 × 8 55

1 21 3
Therefore the chance of the compound event = 91 × 55 = 715

Example 29:
A salesman is known to sell a product in 3 out of 5 attempts while another salesman in 2 out of 5 attempts. Find
the probability that (i) No sale will be effected when they both try to sell the product and (ii) Either of them will
succeed in selling the product.

The Institute of Cost Accountants of India 411


Fundamentals of Business Mathematics and Statistics

Solution:
Let the two salesmen be A and B.
3
P (A) = The probability that the salesman A is able to sell the product =
5
2
P (B) = The probability that the salesman B is able to sell the product =
5
 3  2  6
(i) probability that no sale will be effected = 1 −  1 −  =
 5   5  25
(ii) probability that either of them will succeed in selling the product = P(A + B) = P(A) + P(B) – P(AB)
3 2 3 2 19
= + − × =
5 5 5 5 25

Example 30:
A class consists of 100 students, 25 of them are girls and 75 boys, 20 of them are rich and remaining poor, 40 of
them are fair complexioned. What is the probability of selecting a fair complexsioned rich girl?

Solution:
40 2
Probability of selecting a fair complexioned student = =
100 5
20 1
Probability of selecting a rich student = =
100 5
25 1
Probability of selecting a girl = =
100 4
Since the events are independent, by multiplication rule of probability, the
2 1 1 2
probability of selecting a fair complexioned rich girl = × × = = 0.02
5 5 4 100

Example 31:
Three groups of workers contain 3 men and one woman, 2 man and 2 women, and 1 man and 3 women respectively.
One worker is selected at random from each group. What is the probability that the group selected consists of 1
man and 2 women?
Solution:
There are three possibilities :
(i) Man is selected from the first group and women from second and third group
(ii) Man is selected from the second group and women from first and third groups
(iii) Man is selected from the third group and women from first and second groups.
∴ the probability of selecting a group of one man & two woman


= ( )(
3 2 3
4 4 4
2 1 3
4 4 4
1 1 2
× × + × × + × ×
4 4 4 )( )
18 6 2 13
= + + =
64 64 64 32

412 The Institute of Cost Accountants of India


Probability

Exercise:
Theoretical Questions
 Multiple Choice Questions (MCQ)

1. Probability theory is often referred to as


(a) Science of prediction
(b) Science of uncertainty
(c) Science of chance
(d) Science of decision making

2. In general probability is
(a) A numerical value between 0 and 1, exclusive, describing the absolute possibility an event will occur
(b) A numerical value between 0 and 1, inclusive, describing the absolute possibility an event will occur
(c) A numerical value between 0 and 1, exclusive, describing the relative possibility an event will occur
(d) A numerical value between 0 and 1, inclusive, describing the relative possibility an event will occur

3. The probability of two events A and B are 0.05 and 0.95 respectively. We can infer that
(a) Event A is more probable to happen
(b) Event B is more improbable to happen
(c) Event B is more probable to happen
(d) Event A & B are sure to happen

4. “Sun will disappear from blue sky today forever”. With our available information & belief which one of
the following value is most appropriate as probability to this event?
(a) 0.2
(b) 0.8
(c) 1
(d) 0

5. For an event Odds in favour are “five to two” This means that
(a) In a total of seven trials the event will occur five times
(b) In a total of seven trials the event will occur two times
(c) In a total of five trials the event will occur two times
(d) In a total of seven trials the event will not occur five times

The Institute of Cost Accountants of India 413


Fundamentals of Business Mathematics and Statistics

6. Classical probability is based on the assumption that


(a) The outcomes of an experiment are already known
(b) The probability of an outcome of an experiment is always 0.5
(c) The probability of all outcomes in an experiment is always 1
(d) The outcomes of an experiment are equally likely

7. If an experiment has a set of events that includes every possible outcomes, then the set is called
(a) Mutually Exclusive set
(b) Mutually Exhaustive set
(c) Collectively Exhaustive set
(d) Exhaustive & Exclusive set

8. Classical probability is
Number of favourable outcome
(a) Probability of an event = Total number of possible outcome

Number of favourable outcome


(b) Probability of an event = Total number of trails

Number of possible outcome


(c) Probability of an event = Total number of trails

Number of trials
(d) Probability of an event = Number of possible outcomes

9. Addition rule for mutually exclusive events A & B is

(a) P ( A or B ) = P ( A) + P ( B )

(b) P ( A or B ) = P ( A + B )

(c) P ( A or B ) = P ( A) + P ( B ) − P ( AB )

(d) P ( A or B ) = P ( A + B − AB )

10. If two unbiased coins are tossed once, the probability of getting both the heads is
(a) 0.25
(b) 0.50
(c) 0.75
(d) 1

414 The Institute of Cost Accountants of India


Probability

11. The probability that a leap year selected at random contain 53 Sundays is
(a) 0.143
(b) 1
(c) 0.286
(d) 0.48

12. A lot contains 10 items of which 3 are defective. Three items are chosen from the lot at random one after
another without replacement. The probability that all the three are defective is
(a) 0.008
(b) 0.992
(c) 0.067
(d) 0.05

13. If P ( A) = 0.3, P ( B ) = 0.2 and P (C ) = 0.1 , then assuming A,B and C are independent events, the
probability of occurrence of at least one of the three events is
(a) 0.7
(b) 0.8
(c) 0.006
(d) 0.496

14. In IPL Kolkata Knight Riders plays 70% of their games at night (8 O’clock slot) and 30% during the
day (4 O clock slot). The team wins 50% of their night games and 90% of their day games. According to
today’s newspaper they own yesterday. The probability that the game was played at night is
(a) 0.4667
(b) 0.5645
(c) 0.35
(d) 0.5

15. When two events happen simultaneously which of the following is true?
(a) The outcome of the first event always have an effect on the outcome of the second event
(b) The outcome of the first event may or may not have an effect on the outcome of the second event
(c) The outcome of the first event does not not have any effect on the outcome of the second event
(d) The outcome of the first event have always a 50% effect on the outcome of the second event

The Institute of Cost Accountants of India 415


Fundamentals of Business Mathematics and Statistics

16. A survey by Air travelers’ association revealed that 60% of its member made airline reservations last year.
Two members are selected at random. The probability that both the members made airline reservations last
year is
(a) 0.60
(b) 0.40
(c) 0.36
(d) 0.16

17. If an unbiased coin is tossed once, then the two events head and tall are
(a) Mutually exclusive
(b) Exhaustive
(c) Equally likely
(d) All these

18. Sum of probability of an event A and its complement is __


(a) 1
(b) 0
1
(c)
2
1
(d) −
2

19. If p: q are the odds in favour of an event, then the probability of that event is
(a) p/q
(b) p/(p + q)
(c) q/(p + q)
(d) None of these

20. A bag contains 30 balls numbered from 1 to 30. One ball is drawn at random. The probability that the
number of the drawn ball will be multiple of 3 or 7 is
(a) 7/15
(b) 13/30
(c) 1/2
(d) None of these

416 The Institute of Cost Accountants of India


Probability

21. Three coins are tossed together. The probability of getting exactly two heads is
(a) 5/8
(b) 3/8
(c) 1/8
(d) None

22. Two dice are thrown together. The probability of the event that the sum of numbers shown is greater than
5 is
(a) 13/18
(b) 15/18
(c) 1
(d) None

23. Probability of throwing an even number with an ordinary six faced die is
(a) 1/2
(b) 1
(c) 0
(d) -1/2

24. 4 coins are tossed. The probability that there are 2 heads is
(a) 1/2
(b) 3/8
(c) 1/8
(d) None of these

25. A bag contains 10 red and 10 green balls. A ball is drawn from it. The probability that it will be green is
(a) 1/10
(b) 1/3
(c) 1/2
(d) None of these

The Institute of Cost Accountants of India 417


Fundamentals of Business Mathematics and Statistics

Answer:

 Multiple Choice Questions (MCQ)

1. 2. 3. 4. 5. 6. 7. 8. 9 10.
(b) (d) (c) (d) (a) (d) (c) (a) (a) (a)
11. 12. 13. 14. 15. 16. 17. 18. 19 20.
(c) (a) (d) (b) (b) (c) (d) (a) (b) (b)
21. 22. 23. 24. 25.
(b) (a) (a) (b) (c)

418 The Institute of Cost Accountants of India


INDEX NUMBERS AND TIME SERIES 8
This Module includes

8.1 Uses of Index Numbers, Methods of Construction of Index Number


8.2 Components of Time Series and Calculation of Trend by Moving Average Method

The Institute of Cost Accountants of India 419


Fundamentals of Business Mathematics and Statistics

INDEX NUMBERS AND TIME SERIES

Module Learning Objectives:


After studying this Module, the students will be able to understand –
 Concept of Index Numbers and Time Series
 Different computation methods and interpretation of results
 Practical Application of Index Number and Time Series

420 The Institute of Cost Accountants of India


Uses of Index Numbers, Methods of
construction of index number 8.1
INDEX NUMBER
Index number is a single ratio (usually in percentage) which measures the combined (i.e. averaged) change of
several variables between two different times, places or situations.
The statement that the index number of wholesale prices in an economy for year 2020 was 250 compared to 2010
means that there was a net increase in the price of wholesale commodities in that economy to the extent of 150
percent.
In comparing the prices of 2020 with the price of 2010, Year 2010 is taken as the Base year. The price at the base
year is taken as 100 and the price at 2020, called Current year, is expressed as a percentage of the price of the base
year. Thus if p1 is the price of 2020 and p0 is the price of 2010 then we can form an index number, named Price
Index as
p1 Current Price
× 100 = × 100
p2 Base Price

There are various kinds of index numbers. These are broadly


(a) Price Index
(b) Quantity Index
(c) Value Index
(d) Diffusion Index
(e) Others
Methods of computation of Index Nuumbers:
Methods

Aggregative or Average Average of Relatives

Simple Weighted Simple Weighted

1. Laspeyres’ Method Fixed Base


Chain Base
2. Paasche’s Method
3. Fisher’s Method
4. Bowley’s Method

The Institute of Cost Accountants of India 421


Fundamentals of Business Mathematics and Statistics

PRICE INDEX:
Aggregative Method:
(a) Simple average or simple aggregative method
∑ P1
P01 = × 100
∑ P0
Where
P01 = Price index Number for the current year 1 w.r.t the base year 0.

P1 = Price for the current period

P0 = Price for the base period

Illustration 1. From the following details calculate price index using simple aggregative method:

Commodities Units Base Year Price (P0 in `) Current Year Price Current Year Price (P2 in `)
(P1 in `)
Butter kg 20 23 36
Biscuits kg 10 14 19.5
Eggs Dozen 3.6 4.25 6
Vegetable oil kg 8 10.75 18
Bread 400 gms 0.7 0.95 1.25
Total — 42.3 52.95 80.75

∑ P1 52.95
P01 = × 100 = × 100 = 125.177
∑ P0 42.3

ΣP2 80.75
P02 = × 100 = × 100 = 190.898
ΣP0 42.3

This method though simple has the following limitations:


� The prices of various commodities may be quoted in different units. Thus the index is influenced very much by
the units in which commodities are quoted and accordingly some of the commodities may get more importance
because they are quoted in a particular unit.
� In this method various commodities are weighted according to the magnitudes of their prices and accordingly
commodities which are highly priced exert a greater influence on the value of the index than the commodities
which are low priced
To overcome above difficulties appropriate weights are assigned to various commodities to reflect the relative
importance in the group. Usually the quantities consumed, sold or marketed in the base year or in current year or
in some typical year are used as weights. So we do weighted aggregative method as ­—

422 The Institute of Cost Accountants of India


Index Numbers and Time Series

(b) Weighted aggregative method:

P01 =
∑PW × 100
1

∑PW
0

Where
P01 = Price index Number for the current year 1 w.r.t the base year 0

P1 = Price for the current period

P0 = Price for the base period

W = Weight attached to different commodities

Illustration 2. From the following details calculate price index using weighted aggregative method

Quantity Units Prices


Commodities P0W P1W
(W) P0 P1
A 7 0.321 0.581 2.247 4.067
B 8 0.054 0.067 0.432 0.536
C 4 0.224 0.305 0.896 1.22
Total – – – 3.575 5.823

P01 =
∑PW × 100 = 5.823 × 100 = 162.88
1

∑PW0
3.575

(b.1) Laspeyres’ Method:

P01 =
∑Pq
1 0
× 100
∑P q
0 0

Where
P01 = Price index Number for the current year 1 w.r.t the base year 0
P1 = Prices for the current period
P0 = Prices for the base period, q0 = Quantity of base period

Illustration 3. From the following details calculate price index using Laspeyres’ Method:

Commodity Price (`) Quantity


Base (P0) Current (P1) Base (Q0) Current (Q1)
A 2 2.5 10 12
B 1.8 3.2 16 10
C 3.5 4.8 8 8
D 2.8 4 12 10

The Institute of Cost Accountants of India 423


Fundamentals of Business Mathematics and Statistics

Answer:
Calculations for Laspeyres’ Price Index Number

Commodity P1 P0 Q1 Q0 P1×Q0 P0×Q0


A 2.5 2 12 10 25 20
B 3.2 1.8 10 16 51.2 28.8
C 4.8 3.5 8 8 38.4 28
D 4 2.8 10 12 48 33.6
Total 162.6 110.4

Laspeyres’ Index = P01 =


∑Pq 1 0
× 100 =
162.6
× 100 = 147.28
∑P q 0 0
110.4
(b.2) Paasche’s Method:

P01 =
∑Pq 1 1
× 100
∑P q 0 1

Illustration 4. From the data of the above illustration calculate price index using Paasche’s Method:
Answer:
Calculations for Paasche’s Price Index Number

Commodity P1 P0 Q1 Q0 P1*Q1 P0*Q1


A 2.5 2 12 10 30 24
B 3.2 1.8 10 16 32 18
C 4.8 3.5 8 8 38.4 28
D 4 2.8 10 12 40 28
Total 140.4 98

Paasche’s Index = P01 =


∑Pq 1 1
× 100 =
140.4
× 100 = 143.26
∑P q 0 1
98
(b.3) Fisher’s Method:

P01 = L × P =
∑Pq × ∑Pq
1 0 1 1
× 100
∑P q ∑P q
0 0 0 1
Where:
L = Price index by Laspeyres’ Method
P = Price index by Paasche’s Method

Illustration 5. From the data of the above illustration calculate price index using Fisher’s Method:
Answer:

Fisher’s Price Index = P01 = L × P =


∑Pq × ∑Pq
1 0 1 1
× 100 = 1.4728 × 1.4326 × 100 = 145.26
∑P q ∑P q
0 0 0 1

424 The Institute of Cost Accountants of India


Index Numbers and Time Series

(b.4) Bowley’s Method:

∑Pq + ∑Pq1 0 1 1

P01 =
L+P
× 100 =
∑P q ∑P q0 0 0 1
× 100
2 2

Illustration 6. From the data of the above illustration calculate price index using Bowley’s Method:
Answer:

∑Pq + ∑Pq
1 0 1 1

Bowley’s Price Index = P01 =


L+P
× 100 =
∑P q ∑P q
0 0 0 1
× 100 =
1.4728 + 1.4326
× 100 = 145.27
2 2 2

Difference between Laspeyres’ and Pasche’s Method:

Although both the methods follow Weighted Aggregative method but there are differences:

� In Laspeyres’ method since the quantities of the base year are used as weights, the influence of price changes
on quantity demanded would not get reflected in index numbers.

� But this would be reflected in Pasche’s method where the quantities of the current year are used as weights.

� Laspeyres’ method represents the cost of maintaining the same rate of consumption or production as in the base
year but at current year’s price, whereas Pasche’s method represents the cost of consumption or production as a
whole in the current year as compared with that in the base year. i.e. effects of changes in prices on the quantity
consumed in te current period do not get reflected in Laspeyres’ method but get reflected in Pasche’s method.

� Laspeyres’ method is basically upward biased whereas Pasche’s method is downward biased

To overcome these difficulties Fisher’s and Bowley’s methods are applied to take into account the influence of
current as well as the base year.
Fisher’s method is known as Ideal due to the following reasons:
� It is free from bias, since the upward bias of Laspeyres’ method is balanced to a great extent by the downward
bias of Pasche’s method
� This method takes into account the influence of the current as well as the base year
� It is based on the geometric mean, theoretically which is considered to be the best average for constructing
index numbers

Relatives Method:
Under this method the price of each commodity in the current year is expressed as a percentage of the price of the
base year. This is known as “Price Relative” and is given by the formula

Price of the commodity in the current year


Price Relative of a commodity = × 100
Price of the commodity in the base year

The Institute of Cost Accountants of India 425


Fundamentals of Business Mathematics and Statistics

(a) Simple average of Relatives index:


1  P1 
n ∑  P0
� Using AM: P01 = = × 100

� Using GM: P01 = Antilog [∑ log ( P ) / n]

Where
P1 = Price of a commodity in the current year
P0 = Price of a commodity in the Base year
P1
P = Price Relative = × 100
P0
n = number of commodities

(b) Weighted A.M of Relatives index:


 P  
∑  1 × 100 w
∑ Pw  P  
= 
0
P01 =
∑w ∑w

Similarly we can construct different quantity index as follows:

QUANTITY INDEX:
(a) Simple aggregative quantity index:

Q01 =
∑q 1
× 100
∑q 0

(b) Laspeyres’ Quantity Index:

Q01 =
∑q P × 100
1 0

∑q P
0 0

(c) Paasche’s Quantity Index:

Q01 =
∑q P × 10
1 1

∑q P
0 1

(d) Fisher’s Quantity Index:

Q01 = L × P =
∑q P × ∑q P × 100
1 0 1 1

∑q P ∑q P
0 0 0 1

(e) Simple average of Relatives index:

∑Q 1  q1 
� Using AM: Q01 = n = n = ∑  q × 100
0

Q
� Using GM: Q01 = Antilog  ∑ log 
 n

426 The Institute of Cost Accountants of India


Index Numbers and Time Series

(f) Weighted A.M of Relatives index:


 q1  
∑ Qw
∑  q
0
× 100 w
 
Q10 = =
w ∑w
Where
q1 = Quantity of a commodity in current year
q0 = Quantity of a commodity in Base year
q
Q = Quantity Relative = 1 × 100
q0
n = number of commodities

General Index from group indices:


The items (or commodities) included in constructing an index Number are divided into some groups – similar or
related items falling in the same group. Index number for each group (called Group index) is then determined. The
General Index is the Arithmetic Mean of the group indices and is given as
ΣwI
General Index = Σw , where I = Group Index and w = group weight

EXERCISE:
1. From the following compute Laspeyres’ & Paasche’s Price index.

Base Year Current year


Item Unit Price Quantity Unit Price Quantity

A (dozen) 3.6 20 5.2 15


B (litre) 1.0 30 2.4 25
C (kg) 8.00 50 12.00 40
D (kg) 2.5 100 5.00 60

Answer:
Laspeyres’ Price Index:
Calculations for Laspeyres’ Price Index
Commodity P1 P0 q0 P1 q0 P0 q0
A 5.2 3.6 20 104 72
B 2.4 1 30 72 30
C 12 8 50 600 400
D 5 2.5 100 500 250
Total – – – 1276 752

P01 =
∑Pq1 0
× 100 =
1276
× 100 = 169.68
∑P q
0 0
752

The Institute of Cost Accountants of India 427


Fundamentals of Business Mathematics and Statistics

Paasche’s Method:
Calculations for Paasche’s Price Index

Commodity P1 P0 q1 P1 q1 P0 q1
A 5.2 3.6 15 78 54
B 2.4 1 25 60 25
C 12 8 40 480 320
D 5 2.5 60 300 150
Total 918 549

P01 =
∑Pq
1 1
× 100 =
918
× 100 = 167.21
∑P q
0 1
549

2. Refer Q1. Fisher’s price index is


(a) 186.23
(b) 168.44
(c) 203.15
(d) 114.98

Answer: (b)

Fisher’s Price Index = P01 = L × P = ∑Pq × ∑Pq


1 0 1 1
× 100 =
1276 918
× × 100 = 168.44
∑P q ∑P q
0 0 0 1
752 549

3. Refer Q1. Bowley’s price index is


(a) 186.23
(b) 203.15
(c) 168.44
(d) 114.98

Answer: (c)

∑Pq + ∑Pq
1 0 1 1
1276 918
Bowley’s Price Index: = P01 =
L+P
× 100 =
∑P q ∑P q
0 0 0 1
+
× 100 = 752 549 × 100 = 168.44
2 2 2

428 The Institute of Cost Accountants of India


Index Numbers and Time Series

4. Calculate Laspeyres’ and the Paasche’s Quantity index from the following data.
Base Year Current Year
Item
Unit Price Quantity Unit Price Quantity
A 4 10 6.8 11
B 1 8 7.4 10
C 4.5 5 7.5 8
Answer:
Calculations for Laspeyres’ Quantity Index
Commodity P0 q1 q0 q1 P0 q0 P0
A 4 11 10 44 40
B 1 10 8 10 8
C 4.5 8 5 36 22.5
Total – – – 90 70.5

Laspeyres’ Quantity Index = Q01 =


∑q P × 100 = 90 × 100 = 127.66
1 0

∑q P0 0
70.5

Calculations for Paasche’s Quantity Index


Commodity P1 q1 q0 q1 P1 q0 P1
A 6.8 11 10 74.8 68
B 2.4 10 8 24 19.2
C 7.5 8 5 60 37.5
Total – – – 158.8 124.7

Paasche’s Quantity Index = Q01 =


∑q P × 100 = 158.8 × 100 = 127.34
1 1

∑q P0 1
124.7

5. S limited sells three types of product A, B and C. Company’s records show the prices and quantities sold of each
type are as follows:
Base Year Current year
Item
Unit Price Quantity Unit Price Quantity
A 30 22 40 30
B 50 31 60 40
C 120 8 99 12

(i) Calculate an un-weighted price index for the current year


(ii) Calculate a weighted Lapeyres’ aggregate price index for the current year

The Institute of Cost Accountants of India 429


Fundamentals of Business Mathematics and Statistics

Answer:
(a) Un-weighted price index:
ΣP1 199
P01 = × 100 = × 100 = 99.5
ΣP0 200

Calculations for Unweighted Price Index & Laspeyres’ Aggregative Price Index

Commodity P1 P0 q1 q0 P1 q0 P0 q0
A 40 30 30 22 880 660
B 60 50 40 31 1860 1550
C 99 120 12 8 792 960
Total 199 200 – – 3532 3170

(b) Lapeyres’ aggregative price index = P01 =


∑Pq
1 0
× 100 =
3532
× 100 = 111.41
111.42
∑P q
0 0
3170

6. On the basis of the following information calculate the Fisher’s Ideal Index Number:

Base Year Current year


Item Unit Price Quantity Unit Price Quantity
A 2 40 6 50
B 4 50 8 40
C 6 20 9 30
D 8 10 6 20
E 10 10 5 20
Answer:

Fisher’s Ideal Index No. = P01 = L × P =


∑Pq × ∑Pq
1 0 1 1
× 100
∑P q ∑P q
0 0 0 1

Calculations for Fisher’s Ideal Index Number

Commodity P1 P0 q1 q0 P1 q0 P0 q0 P1 q1 P0 q1
A 6 2 50 40 240 80 300 100
B 8 4 40 50 400 200 320 160
C 9 6 30 20 180 120 270 180
D 6 8 20 10 60 80 120 160
E 5 10 20 10 50 100 100 200
Total – – – – 930 580 1110 800
So

P01 = L × P =
∑Pq × ∑Pq
1 0 1 1
× 100 =
930 1110
× × 100 = 149.16
∑P q ∑P q
0 0 0 1
580 800

430 The Institute of Cost Accountants of India


Index Numbers and Time Series

7. Compute a price index from the following data by (a) simple aggregative method and (b) average of price
relatives method (using both arithmetic mean and geometric mean):

Commodity A B C D E F
Price in Base Year 20 30 10 25 40 50
Price in Current Year 25 30 15 35 45 55

Answer:
Calculations for Simple Aggregative Index & Simple Average of Relatives Index

Commodity P0 P1 P1 Log P
Price Relative = × 100
P0

A 20 25 125 2.09691
B 30 30 100 2
C 10 15 150 2.176091
D 25 35 140 2.146128
E 40 45 112.5 2.051153
F 50 55 110 2.041393
Total 175 205 737.5 12.511675

(a) Simple Aggregative Method:

Index Price = P01 =


∑P × 100 = 205 × 100 = 117.14
1

∑P 0
175

(b) Simple Average of Relatives Index:

Here n = No. of commodities = 6

1  P1  1
 Using AM: P01 = n ∑  P × 100 = 6 × 737.5 = 122.92
 0 
12.511675 
 Using GM: P01 = Antilog ( ∑ log P ) / n  = Antilog   = 121.7
 6

The Institute of Cost Accountants of India 431


Fundamentals of Business Mathematics and Statistics

8. From the following data prepare Laspeyres’, Paasche’s and Fisher’s quantity index numbers for the year 2 taking
year 1 as the base

Year 1 Year 2
Item Unit Price Quantity Unit Price Quantity

I 5 10 4 12
II 8 6 7 7
III 6 3 5 4

Answer:

Commodity P1 P0 q1 q0 q1 P0 q0 P0 q1 P1 q0 P1
I 4 5 12 10 60 50 48 40
II 7 8 7 6 56 48 49 42
III 5 6 4 3 24 18 20 15
Total – – – – 140 116 117 97

For the sake of simplicity computations are made assuming Year 1 as Yer 0 and Year 2 as Year 1

Laspeyres’ Quantity Index:

Q01 =
∑q P × 100 = 140 × 100 = 120.69
1 0

∑q P
0 0
116

Paasche’s Quantity Index:

Q01 =
∑q P × 100 = 117 × 100 = 120.68
1 1
120.62
∑q P
0 1
97

Fisher’s Quantity Index:

Q01 = L × P =
∑q P × ∑q P × 100 =
1 0 1 1
1.2069 × 1.2068 × 100 = 120.65
∑q P ∑q P
0 0 0 1

432 The Institute of Cost Accountants of India


Index Numbers and Time Series

9. Compute the index number of business activity from the following data:

Industrial Activity Index Weight


Industrial Production 250 36
Mineral Production 135 7
Internal Trade 200 24
Financial Activity 135 20
Exports & Imports 325 7
Shipping Activity 300 6

Answer:
Detail computations are:

Industrial Activity Index (I) Weight (W) IW


Industrial Production 250 36 9000
Mineral Production 135 7 945
Internal Trade 200 24 4800
Financial Activity 135 20 2700
Exports & Imports 325 7 2275
Sipping Activity 300 6 1800
Total – 100 21520

So required index number of business activities is given by arithmetic mean method as

∑ IW 21250
= = = 212.50
∑W 100

Problems in the construction of Index Numbers:


(a) Determination of the exact purpose for which proposed index is constructed
(b) Selection of items which are to be included in the group of items/commodities
(c) Collection of data regarding price and quantities and selection of sources from which such data are to be
collected
(d) Choice of the base period
(e) Determination of the weights to be employed
(f) Selection between AM and GM

The Institute of Cost Accountants of India 433


Components of Time Series and
Calculation of Trend by Moving 8.2
Agerege Method
A time series is a collection of data recorded over a period of time--- like data collected daily, data collected
quarterly, data collected monthly or data collected yearly.
e.g. daily stock price movements in national stock exchange (given as follows) form a time series:

Symbol Series Date Prev. Close Open Price High Price Low Price
ITC EQ 3-Feb-22 232.15 232.15 235.2 231.8
ITC EQ 4-Feb-22 234.45 237.25 238.5 231.55
ITC EQ 7-Feb-22 234.3 234 234.75 228.5
ITC EQ 8-Feb-22 230.2 230.3 232.3 229.05
ITC EQ 9-Feb-22 231.25 232.3 233 228.75
ITC EQ 10-Feb-22 230.15 228.7 233.4 227.75
ITC EQ 11-Feb-22 232.25 232 233.3 229.8
ITC EQ 14-Feb-22 232.45 226 226 218.5
ITC EQ 15-Feb-22 219.45 221 223.5 218.55
ITC EQ 16-Feb-22 223 223.8 224.7 221.85
ITC EQ 17-Feb-22 222.75 223.05 224.35 220.85
ITC EQ 18-Feb-22 222.65 222.65 223.45 221.5
ITC EQ 21-Feb-22 221.9 220.3 221.3 217.15
ITC EQ 22-Feb-22 218 215 216.9 213
ITC EQ 23-Feb-22 214.85 216.8 217.5 215.05

Analysis of time series means an analysis of history and is used by management to make current decisions and
plans based on long term forecasting.

Components of a Time series:


There are four components in time series:
� The trend (T)
� The cyclical fluctuation (C)
� The seasonal variation (S)
� The irregular movement (I)

434 The Institute of Cost Accountants of India


Index Numbers and Time Series

The four components of a time series referred above can be considered to be interacting in additive or multiplicative
fashion to produce the observed values in a time series (here daily closing prices of ITC shares)
 Under additive model Observed Values (Y) = Trend (T) + Cyclical Fluctuation (C) + Seasonal Variation (S) +
Irregular Movement (I)
 Here S, C and I are absolute quantitative deviations about trend
 Under multiplicative model Observed Values (Y) = Trend (T) × Cyclical Fluctuation (C) × Seasonal Variation
(S) × Irregular Movement (I)
 Here S, C and I are rates, percentages or index numbers
 Under additive model it is assumed that all the four components of the time series operate independently of one
another whereas under multiplicative model it is assumed that the four components although due to different
causes are not necessarily independent and they can affect one another.
 Additive model is not appropriate for extending its application to future events but the relative values of factors
under multiplicative model are appropriate for forecasting future behaviour in a time series.
Most of the time series relating to economic and business phenomena conform to the multiplicative model and in
practice additive model occurs rarely. So in this module we discuss time series with multiplicative model.
By now we understood what is called time series, what are its components etc. Now we will learn decomposition
of a time series. Decomposition of a time series refers to the segregation of various components. In the process of
decomposition often the trend has to be found out first. This is because most of the other components especially
short term Seasonal, Cyclical and Irregular can be separated only with reference to Trend
Observed Value (TSCI )
= SCI
Trend (T )
If we plot the closing price of ITC shares with price on the Y-axis and date on X-axis, we get a plot of time series
as follows:
Date vs High Price
240

235
High Price

230

225

220

215
29-Jan-22 3-Feb-22 8-Feb-22 13-Feb-22 18-Feb-22 23-Feb-22 28-Feb-22
Dates
Chart-I

The green part of the chart-I is the time series plot. As seen there are random fluctuations date wise. Sometimes
price goes above the black line and sometimes goes below that line. The black line is called Trend or Secular trend
and it is obtained by smoothening the fluctuations in time series.

The Institute of Cost Accountants of India 435


Fundamentals of Business Mathematics and Statistics

Secular Trend: The smoothed long term direction of a time series

Although in Chart-I only a small time period is considered actually long term means more than 1 year.
India’s year wise GDP growth (%) is given in the following table and corresponding time series graph is shown in
Chart II with the trend line

Date GDP Growth (%)


12/31/2010 8.4976
12/31/2011 5.2413
12/31/2012 5.4564
12/31/2013 6.3861
12/31/2014 7.4102
12/31/2015 7.9963
12/31/2016 8.2563
12/31/2017 6.7954
12/31/2018 6.533
12/31/2019 4.0416
12/31/2020 -7.9646

Chart II
Again we observed that there are fluctuations in time series and smoothing out all the fluctuations we get a secular
trend –the black line

436 The Institute of Cost Accountants of India


Index Numbers and Time Series

Moving Average:
A Moving Average is used in smoothing a time series to see its trend. Usually Moving average will incorporate
cyclical movements also and therefore through moving average we basically do the following:
Observed Value (TSCI)
= SI
Moving Average (Trend T & Cyclical Fluctuation C)
The moving average merely smoothen the fluctuations in the data. This is accomplished by moving the arithmetic
mean values through the time series.
This is illustrated with the following example.

Year Sales (Rs. in Million) Year Sales (Rs. in Million)


1988 1 2001 4
1989 2 2002 3
1990 3 2003 4
1991 4 2004 5
1992 5 2005 6
1993 4 2006 7
1994 3 2007 6
1995 2 2008 5
1996 3 2009 4
1997 4 2010 5
1998 5 2011 6
1999 6 2012 7
2000 5 2013 8
Our objective is to smooth out these fluctuations. The method we use is “n-years Moving Average” and the resultant
smoothed out figure we usually call Secular trend which indicates the long term direction of the variable (here
sales). Here n we have taken as 7. But value of n depends on the data collected. If the data are quarterly then 4
quarters moving average is typical as there are 4 quarters in a year. If the data are daily then seven days moving
average is appropriate because there are 7 days in a week. We can also fix through use of trial and error the value
of n that best levels out the chance fluctuations.
� The first step in finding out 7 years moving average is to find 7 years moving total
 The total sales for the first 7 years is 1+2+3+4+5+4+3 = 22
 The average of this total = 22/7 = 3.143
 The 7 years total and its average are positioned against the middle year of the group i.e.1991 under the
columns seven years moving total and seven years moving average respectively
� Then we form second group of 7 years starting from 1989, i.e. omitting 1988 & including 1995
 The total sales for the second 7 years is 2+3+4+5+4+3+2 = 23
 The average of this total = 23/7 = 3.286
 This 7 years total and its average are positioned against the middle year of the group i.e.1992 under the
columns seven years moving total and seven years moving average respectively.

The Institute of Cost Accountants of India 437


Fundamentals of Business Mathematics and Statistics

� In this way we repeat the process till we form our last 7 years group which starts from 2007 and ends on 2013
 The total sales for the last 7 years is 6+5+4+5+6+7+8 = 41
 The average of this total = 41/7 = 5.857
 This 7 years total and its average are positioned against the middle year of the group i.e.2010 under column
seven years moving total and seven years moving average
The detail computations are in the following table. Given data of time series and 7 years moving average data are
plotted in a graph shown as Chart III.

Seven years Seven years


Year Sales (` in Million)
Moving Total Moving Average
1988 1 – –
1989 2 – –
1990 3 – –
1991 4 22 3.143
1992 5 23 3.286
1993 4 24 3.429
1994 3 25 3.571
1995 2 26 3.714
1996 3 27 3.857
1997 4 28 4.000
1998 5 29 4.143
1999 6 30 4.286
2000 5 31 4.429
2001 4 32 4.571
2002 3 33 4.714
2003 4 34 4.857
2004 5 35 5.000
2005 6 36 5.143
2006 7 37 5.286
2007 6 38 5.429
2008 5 39 5.571
2009 4 40 5.714
2010 5 41 5.857
2011 6 – –
2012 7 – –
2013 8 – –

438 The Institute of Cost Accountants of India


Index Numbers and Time Series

Chart III

The black line indicates the 7 years moving average data as per computation table and it is observed that fluctuations
in time series are smoothed out.
As already mentioned in “n years moving average” the value of n depends on the data collected or we can use trial
and error. Let us take the following example:

Yearly production data of a business unit are as follows:

Year Production Year Production


1995 5 2005 9
1996 6 2006 13
1997 8 2007 15
1998 10 2008 18
1999 5 2009 15
2000 2 2010 11
2001 7 2011 14
2002 10 2012 17
2003 12 2013 22
2004 11

The Institute of Cost Accountants of India 439


Fundamentals of Business Mathematics and Statistics

If we smooth out the fluctuation with “n years moving average” where n = 3 or 5 i.e. if we do 3 or 5 years moving
average then the detail computation s and corresponding plot will be as follows:

3-year 3-year 5-year 5-year


Year Production
Moving Total Moving Average Moving Total Moving Average
1995 5 – – – –
1996 6 19 6.333 – –
1997 8 24 8.000 34 6.8
1998 10 23 7.667 31 6.2
1999 5 17 5.667 32 6.4
2000 2 14 4.667 34 6.8
2001 7 19 6.333 36 7.2
2002 10 29 9.667 42 8.4
2003 12 33 11.000 49 9.8
2004 11 32 10.667 55 11
2005 9 33 11.000 60 12
2006 13 37 12.333 66 13.2
2007 15 46 15.333 70 14
2008 18 48 16.000 72 14.4
2009 15 44 14.667 73 14.6
2010 11 40 13.333 75 15
2011 14 42 14.000 79 15.8
2012 17 53 17.667 – –
2013 22 – – – –

Chart IV

440 The Institute of Cost Accountants of India


Index Numbers and Time Series

Since 5 years moving average reduces the fluctuation in a much better way than that done by 3 years moving
average so with the given production data we should take 5 years moving average.
In chart III perfectly linear trend is observed but in chart V moving average does not result precisely in a strainght
line. This is because (i) periods of oscillation do not have equal length (ii) oscillation do not have identical
amplitudes. Sales, production and other economic and business time series data usually generate not precise linear
trend through moving average but trend can always give long term direction of movement like upward, downward
etc.
Four year, six-year and other even-numbered-year moving averages present one minor problem regarding
positioning the moving totals and moving averages. Let us take the following example:

Sales figures of a business unit are as follows:

2005 2006 2007 2008 2009 2010 2011 2012 2013


8 11 9 14 9 10 10 8 12

Let us compute 4 years moving average. Details are in the following table:

Year Sales 4 year 4 year 4 year Centered


Moving Total Moving Average Moving Average
2005 8 – – –

2006 11 – – –
42 10.5
2007 9 10.625
43 10.75
2008 14 10.625
42 10.5
2009 9 10.625
43 10.75
2010 10 10
37 9.25 –
2011 10 9.625
40 10
2012 8 – – –

2013 12 – – –

� 1st Moving total comprises of the data for 2005, 2006, 2007 and 2008 i.e. 8, 11, 9 and 14 respectively
� 1st Moving total = (8+11+9+14) = 42
� Since there is no particular year which can be taken as center time period, 42 is positioned between two central
time periods i.e. 2006 and 2007

The Institute of Cost Accountants of India 441


Fundamentals of Business Mathematics and Statistics

� Same procedure is applied for 2nd, 3rd, 4th, 5th and 6th moving total i.e. with 43 (11+9+14+9), 42 (9+14+9+10),
43 (14+9+10+10), 37 (9+10+10+8), 40 (10+10+8+12)
� 4 years moving average i.e. 10.5 (42/4), 10.75 (43/4), 10.5 (42), 10.75 (43/4), 9.25 (37/4) & 10 (40/4) are
accordingly placed between two time periods
� Now average of 1st four years i.e.10.5 and average of 2nd four years i.e.10.75 are averaged and this average
10.625 [(10.5+10.75)/2] is centered on 2007.
� Similarly we will get centered moving average against year 2008, 2009, 2010, and 2011.

Chart V
Weighted Moving Average: A moving average uses the same weight for each observation. But weighted moving
average involves selecting a different weight for each data value and then computing a weighted average of the
most recent “n” values as the smoothed value. Most recent observation receives the most weight and the weight
decreases for older data values. Sum of weights must be equal to 1.
Let us take one example. Yearly sales of a business unit are given below in ` ’000:

Year Sales Year Sales


1993 5761 2003 12181
1994 6148 2004 12557
1995 6783 2005 12700
1996 7445 2006 19300
1997 7405 2007 22100
1998 11450 2008 22720
1999 11224 2009 21136
2000 11703 2010 22785
2001 11890 2011 23377
2002 12380 2012 23300

442 The Institute of Cost Accountants of India


Index Numbers and Time Series

We are going to find 3 years weighted moving average with weights 0.2, 0.3 and 0.5 respectively.
1st one of the 3 years weighted moving total 0.2 ×5761 + 0.3 × 6148 + 0.5 ×6388.1 Most recent values are given
more weight and oldest value is given the least weight. Sum of the weights must be equal to 1. For the given case
the detail computations are:

Year Sales 3 years weighted 3 years weighted


(` ’000) Moving Total Moving Average **
2003 5761 – –
2004 6148 6388.1 6388.1
2005 6783 6987 6987
2006 7445 7292.6 7292.6
2007 7405 9435.5 9435.5
2008 11450 10528 10528
2009 11224 11508.7 11508.7
2010 11703 11700.7 11700.7
2011 11890 12097.6 12097.6
2012 12380 12182.5 12182.5
2013 12181 12408.8 12408.8
2014 12557 12553.3 12553.3
2015 12700 15971.4 15971.4
2016 19300 19380 19380
2017 22100 21850 21850
2018 22720 21804 21804
2019 21136 22277.3 22277.3
2020 22785 22751.2 22751.2
2021 23377 23220.1 23220.1
2022 23300 – –

** since sum of weights is equal to 1 we will always get weighted moving total and weighted moving average
same value

1st weighted moving total 0.2 × 5761 + 0.3 × 6148 + 0.5 × 6783 = 6388.1

6388.1 6388.1
So weighted moving average = = = 6388.1
0.2 + 0.3 + 0.5 1

The Institute of Cost Accountants of India 443


Fundamentals of Business Mathematics and Statistics

Chart VI

Illustrations
1. Determine a three year moving average for the sales of a Maruti Udyog limited.
Answer:

Year Number sold 3 years 3 years


(Thousand) Moving Total Moving Average
2015 2
2016 6 12 4
2017 4 15 5
2018 5 12 4
2019 3 18 6
2020 10

444 The Institute of Cost Accountants of India


Index Numbers and Time Series

2. Calculate a four-quarter weighted moving average for the number of shares outstanding for a company for the
nine quarters of data. The data are reported in thousands. Apply weights of 0.1, 0.2, 0.3 and 0.4 respectively for
Q1, Q2, Q3 & Q4.
Year Quarter No of shares
Outstanding
2015 Q1 28766
Q2 30057
Q3 31336
Q4 33240
2016 Q1 34610
Q1 35102
Q1 35308
Q1 35203
2017 Q1 34386

Answer:
1st four quarter weighted moving average is 31584.8
It is found by ( 28766 × 0.1 + 30057 × 0.2 + 31336 × 0.3 + 33240 × 0.4 ) / ( 0.1 + 0.2 + 0.3 + 0.4 )

The Institute of Cost Accountants of India 445


Fundamentals of Business Mathematics and Statistics

Similarly for other values. Details are as follows:

Year Quarter No of shares 4 Quarter weighted 4 Quarter weighted


Outstanding Moving Average Moving Average
2015 Q1 28766 – –

Q2 30057 – –
31584.8
Q3 31336 32336.85
33088.9
Q4 33240 33647.15
34205.4
2016 Q1 34610 34552.6
34899.8
Q2 35102 35027.4
35155
Q3 35308 35021.05
34887.1
Q4 35203 – –

2017 Q1 34386 – –
3. Using three year moving average determine the trend and short term fluctuations. Plot the original and trend
values.
Year Productions Units
(Thousand)
2010 21
2011 22
2012 23
2013 25
2014 24
2015 22
2016 25
2017 26
2018 27
2019 26
Answer:
In this problem 3 years moving averages are calculated as usual. And these three years moving averages are the
Trend values.
For finding out short term fluctuations we usually use Multiplicative model. As discussed earlier Observed values
contain TSCI (i.e. Trend, Seasonal, Cyclical and Irregular components). Moving average values contain Trend (T)

446 The Institute of Cost Accountants of India


Index Numbers and Time Series

Observed Values
So short term fluctuations under multiplicative model = & short term fluctuations under additive
Moving average
model = Observe Values – Moving average. Detail computations are:

Year Productions Unit 3 years 3 years Moving Short Term Short Term
(Thousand) Moving Total Average Fluctuations Fluctuations
Multiplicative Model Additive Model
2010 21 – – – –
2011 22 66 22.000 1 0.000
2012 23 70 23.333 0.986 -0.333
2013 25 72 24.000 1.042 1.000
2014 24 71 23.667 1.014 0.333
2015 22 71 23.667 0.930 -1.667
2016 25 73 24.333 1.027 0.667
2017 26 78 26.000 1.000 0.000
2018 27 79 26.333 1.025 0.667
2019 26 – – – –

Observed Values, Trends & ST Fluctuations


30

25

20
Productions

15

10

0
2009 2010 2011 2012 2013 2014 2015 2016 2017 2018 2019 2020

Year
Production Unit (Thousand)

3 years Moving Average

Short Term Fluctuations Multiplicative


Model

4. The following data give daily sales of a shop observing a five day week, over four successive weeks. Determine
the period of the moving average and calculate the moving average accordingly:
Day 1 2 3 4 5 6 7 8 9 10
Sales 26 29 35 47 51 26 32 37 46 53
Day 11 12 13 14 15 16 17 18 19 20
Sales 28 30 36 46 54 28 31 36 46 54

The Institute of Cost Accountants of India 447


Fundamentals of Business Mathematics and Statistics

Answer:
Determination of the period of moving average: We discussed earlier in finding out “n-years Moving Average”
value of n depends on the data collected. If the data are quarterly then 4 quarters moving average is typical as there
are 4 quarters in a year. If the data are daily then seven days moving average is appropriate because there are 7 days
in a week. We can also fix by trial and error the value of n that best levels out the chance fluctuations.
In the given case data is collected daily in a week and week is composed of 5 days. So 5 days moving average is
appropriate and the detail computations are as follows:

Sales 5 day 5 day


Days
(Thousand `) Moving Total Moving Average
1 26 – –
2 29 – –
3 35 188 37.600
4 47 188 37.600
5 51 191 38.200
6 26 193 38.600
7 32 192 38.400
8 37 194 38.800
9 46 196 39.200
10 53 194 38.800
11 28 193 38.600
12 30 193 38.600
13 36 194 38.800
14 46 194 38.800
15 54 195 39.000
16 28 195 39.000
17 31 195 39.000
18 36 195 39.000
19 46 – –
20 54 – –

448 The Institute of Cost Accountants of India


Index Numbers and Time Series

Exercise:
Theoretical Questions
 Multiple Choice Questions (MCQ)
1. By using arithmetic mean method the index number from the following data is

Commodity Base price Current price Weight


Rice 30 52 8
Wheat 25 30 6
Fish 130 150 3
Potato 35 49 5
Oil 70 105 7

(a) 144.92
(b) 202.34
(c) 161.87
(d) 115.22

2. From the data given below the wholesale price index number for the year 1 taking year 0 as base using
simple arithmetic average of relatives method is

Commodity Price year 0 Price year 1


A 80 120
B 120 150
C 40 80
D 100 150
E 200 240
(a) 180
(b) 112
(c) 134
(d) 149

3. Consider the following:

Year 1 2 3 4 5 6 7 8
Annual Sales (` ’0000) 3.6 4.3 4.3 3.4 4.4 5.4 3.4 2.4
5 year moving average against year 4 is
(a) 3.80
(b) 4.00

The Institute of Cost Accountants of India 449


Fundamentals of Business Mathematics and Statistics

(c) 4.36
(d) 4.18

4. Consider the following:

Year 1 2 3 4 5 6 7 8
Annual Sales (`’0000) 3.6 4.3 4.3 3.4 4.4 5.4 3.4 2.4
4 year centered moving average against year 6 is
(a) 4.00
(b) 4.24
(c) 4.26
(d) 4.03

5. Consider the following table:

Year 1 Year 2
Item Unit Price Quantity Unit Price Quantity
I 1 16 3 15
II 3 15 8 20
III 5 18 10 21
Pasche’s price index of year 2 with respect to year 1 is
(a) 189.13
(b) 230.56
(c) 245.12
(d) 256.78

6. Fisher’s ideal index for prices from the following data is

Base Year Current Year


Item
Unit Price Quantity Unit Price Quantity
A 8 6 12 5
B 10 5 11 6
C 15 8 10 5
(a) 97.72
(b) 80.15
(c) 95.67
(d) 89.14

450 The Institute of Cost Accountants of India


Index Numbers and Time Series

7. Consider the following series of observation.

Year 1 2 3 4 5 6 7 8 9 10 11
Sales (`) 2 6 1 5 3 7 2 6 4 8 3
4 year centered moving average against year 6 is
(a) 5.125
(b) 3.875
(c) 3.625
(d) 4.375

8. Consider the following series of observation.

Year 1 2 3 4 5 6 7 8 9 10 11
Sales (`) 2 6 1 5 3 7 2 6 4 8 3
5 year weighted moving average with weights 1, 2, 2, 2, 1 against year 6 is
(a) 5.125
(b) 3.875
(c) 3.625
(d) 4.375

9. Consider the following series of observation.

Year 1 2 3 4 5 6 7 8 9 10 11
Sales (`) 2 6 1 5 3 7 2 6 4 8 3
5 year moving average against year 6 is
(a) 3.6
(b) 4.6
(c) 4.4
(d) 5.4

10. Consider the following:

Commodity Base Price (`) Current price (`) Weight


A 22 40 8
B 15 15 6
C 80 90 7
D 110 130 3
E 25 30 5

The Institute of Cost Accountants of India 451


Fundamentals of Business Mathematics and Statistics

Weighted aggregative index number is


(a) 123.34
(b) 156.11
(c) 176.52
(d) 142.89

11. Consider the following:


Commodity Base Price (`) Current price (`) Weight
A 22 40 8
B 15 15 6
C 80 90 7
D 110 130 3
E 25 30 5
Weighted A.M of price relative index number is
(a) 123.34
(b) 128.79
(c) 130.92
(d) 182.13

12. From the following find the Fisher’s Quantity index


Item Base Year Current Year
Unit Price Quantity Unit Price Quantity
A 8 6 12 5
B 10 5 11 6
C 17 8 8 5
(a) 32.76
(b) 72.34
(c) 78.12
(d) 12.74

13. From the following find the Simple average (AM) of Relative Quantity index
Item Base Year Quantity Current Year Quantity
A 8 12
B 10 11
C 15 10
(a) 111.45
(b) 108.89
(c) 32.45
(d) 115.46

452 The Institute of Cost Accountants of India


Index Numbers and Time Series

14. From the following find the Simple average (GM) of Relative Quantity index

Base Year Current Year


Item Quantity Quantity
A 8 12
B 10 11
C 15 10
(a) 100.23
(b) 111.45
(c) 190.15
(d) 103.23

15. From the following data the five year moving average against year 5

Year 1 2 3 4 5 6 7 8 9
Sales (`) 36 43 43 34 44 54 34 24 14
(a) 40
(b) 43.6
(c) 34
(d) 41.8

16. From the following data the four year centered moving average against year 6

Year 1 2 3 4 5 6 7 8 9
Sales (`) 36 43 43 34 44 54 34 24 14
(a) 40.25
(b) 40.625
(c) 35.25
(d) 40

17. From the following series find out a three year moving average against year 4 is
Year 1 2 3 4 5 6 7
Values 12 14 15 17 18 20 23
(a) 20.33
(b) 18.33
(c) 16.67
(d) 15.33

The Institute of Cost Accountants of India 453


Fundamentals of Business Mathematics and Statistics

18. From the following series find out a three year weighted moving average against year 4 with weights 1, 4,
1 is

Year 1 2 3 4 5 6 7
Values 12 14 15 17 18 20 23
(a) 20.17
(b) 16.83
(c) 18.17
(d) 15.17
19. Consider the following table:

Commodity Weights Base price p.u. (`) Current price p.u (`)
A 40 16 30
B 25 40 70
C 5 0.5 1.5
D 20 5.12 7.25
E 10 2 2.5
Weighted A.M price relative index is
(a) 146.98
(b) 174.57
(c) 124.33
(d) 156.01
20. From the following four year centered moving average against year 4 is

Year 1 2 3 4 5 6 7
Import (` m) 229 231 206 191 195 184 193
(a) 190.671
(b) 199.875
(c) 192.375
(d) 210
Answer:

 Multiple Choice Questions (MCQ)

1. 2. 3. 4. 5. 6. 7. 8. 9 10.
(a) (d) (c) (d) (b) (a) (d) (d) (b) (a)
11. 12. 13. 14. 15. 16. 17. 18. 19 20.
(c) (c) (b) (d) (d) (a) (c) (b) (b) (b)

454 The Institute of Cost Accountants of India


The Institute of Cost Accountants of India
Statutory Body under an Act of Parliament
www.icmai.in

Headquarters:
CMA Bhawan; 12, Sudder Street; Kolkata - 700016
Ph: +91-33-2252-1031/34/35/1602/1492/1619/7373/7143
Delhi Office:
CMA Bhawan; 3, Institutional Area; Lodhi Road; New Delhi - 110003
Ph: +91-11-24666100/24622156/57/58; 24666124/129
Toll Free: 1800 346 0092 / 1800 110 910
E-mail: studies@icmai.in

ISBN: 978-93-95303-02-6

Follow us on

Behind Every Successful Business Decision, there is always a CMA

You might also like